You are on page 1of 330

AY 2018 - 2019

Saint Louis University


School of Law

CASE DIGESTS
COMPENDIUM INSURANCE

Alfiler, Victor Benedict D.


Alvaro, Michelle V.
Aquipel, Jeric S.
Bunquin, Ma. Angelica Z.
Faclangen, Sharene F.
Montereal, King Anthony M.
Paroy, Kurdel Johansen C.
Quindara, Charlene A.

Mendoza, Bernadette C.
Table of Contents

INTRODUCTION
Origin and Growth of Insurance Companies
PHILIPPINE SHIPPING COMPANY, ET. AL v. GARCIA .......................................................................................... 1
ABUEG, ET AL., v. BARTOLOME SAN DIEGO............................................................................................................ 2
YANGCO, ETC. v. LASERNA, ET AL. .............................................................................................................................. 3
MANILA STEAMSHIP CO., INC., v. INSA ABDULHAMAN and LIM HONG TO .............................................. 4
CHUA YEK HONG, v. IAC, MARIANO GUNO, and DOMINADOR OLIT............................................................. 6
HEIRS OF DE LOS SANTOS, et al. v. CA and MARITIMA ...................................................................................... 7
Growth of Insurance in the Philippines
ABOITIZ SHIPPING CORPORATION v. NEW INDIA ASSURANCE COMPANY, LTD. ................................. 8
Laws Governing Insurance
INSULAR LIFE ASSURANCE CO., LTD. v. FELICIANO, ET AL. ........................................................................... 9
GERCIO v. SUN LIFE ASSURANCE OF CANADA, ET AL. ..................................................................................... 10
ANG GIOK CHIP v. SPRINGFIELD................................................................................................................................ 11
PHILIPPINE HEALTH CARE v. CIR ............................................................................................................................. 12
FORTUNE MEDICARE, INC. v. AMORIN ................................................................................................................... 13
The Insurance Commission
ALMENDRAS MINING CORPORATION, v. OFFICE OF THE INSURANCE COMMISSION....................... 14
PHILAM v. ARNALDO ...................................................................................................................................................... 16
REPUBLIC v. DEL MONTE ............................................................................................................................................. 18
MALAYAN v. CONCEPCION ........................................................................................................................................... 20
FINMAN v. INOCENCIO................................................................................................................................................... 22
THE CONTRACT OF INSURANCE
Definitions
MAYER STEEL v. CA ......................................................................................................................................................... 23
PHILHEALTH v. CIR ......................................................................................................................................................... 24
Characteristics of an Insurance Contract
UCPB v. MASAGANA TELAMART ............................................................................................................................... 25
Other Cases
SMB v. LAW UNION .......................................................................................................................................................... 26
PALILEO v. COSIO ............................................................................................................................................................. 28
SAURA IMPORT v. PHIL INTERNATIONAL ............................................................................................................ 29
PINEDA v. CA ...................................................................................................................................................................... 30
PHILAM LIFE v. ANSALDO ............................................................................................................................................ 32
WHITEGOLD v. PIONEER INSURANCE .................................................................................................................... 33
GULF RESORT v. PCIC ..................................................................................................................................................... 34
HEIRS OF MARAMAG v. MARAMAG .......................................................................................................................... 35
MANILA BANKERS v. ABAN ......................................................................................................................................... 36
ALPHA INSURANCE v. CASTOR................................................................................................................................... 37
JAIME T. GAISANO v. DEVELOPMENT INSURANCE AND SURETY CORPORATION .............................. 38
Health Care Agreements
PHILAMCARE HEALTH SYSTEMS, INC. v. COURT OF APPEALS AND JULITA TRINOS ........................ 41
BLUE CROSS HEALTH CARE INC. v. NEOMI AND DANILO OLIVARES ........................................................ 43
PHILIPPINE HEALTH CARE PROVIDERS, INC. v. COMMISSIONER OF INTERNAL ................................ 45
Protection and Indemnity Club
PANDIMAN PHILIPPINES, INC. v. MARINE MANNING MANAGEMENT CORPORATION ................... 47
HYOPSUNG MARITIME CO. v. COURT OF APPEALS ........................................................................................... 49
Construction and Interpretation of Insurance Contracts
MISAMIS LUMBER CORPORATION v. CAPITAL INSURANCE AND SURETY CO., INC. .......................... 50
SUN INSURANCE OFFICE, LTD. v. COURT OF APPEALS AND EMILIO TAN ............................................. 51
NEW LIFE ENTERPRISES, et al. v. CA, et al............................................................................................................. 52
FIRST QUEZON CITY INSURANCE COMPANY, INC. v. COURT OF APPEALS ............................................. 54
FORTUNE INSURANCE AND SURETY CO., INC. v. COURT OF APPEALS AND PRODUCERS .............. 56
CEBU SHIPYARD v. WILLIAM LINES, INC. ............................................................................................................. 58
GULF RESORTS, INC. v. PHILIPPINE CHARTER INSURANCE CORPORATION......................................... 60
QUA CHEE GAN v. LAW UNION AND ROCK INSURANCE CO., LTD............................................................... 62
FIELDMEN'S INSURANCE CO., INC. v. MERCEDES VARGAS VDA. DE SONGCO, ET AL. ....................... 64
WESTERN GUARANTY CORPORATION v. HONORABLE COURT OF APPEALS, ...................................... 66
RIZAL SURETY & INSURANCE COMPANY v. COURT OF APPEALS and TRANSWORLD ...................... 67
MALAYAN INSURANCE v. COURT OF APPEALS and TKC MARKETING ..................................................... 68
INSURABLE INTEREST
Definition and Purpose
EL ORIENTE, FABRICA DE TABACOS, INC., v. JUAN POSADAS, COLLECTOR OF .................................... 70
LINCOLN NATIONAL LIFE v. SAN JUAN LIFE INSURANCE.............................................................................. 71
VIOLETA R. LALICAN v. THE INSULAR LIFE ASSURANCE COMPANY LIMITED .................................... 72
MALAYAN INSURANCE COMPANY, INC. v. PAP CO., LTD. (PHIL. BRANCH) ............................................ 73
Insurable Interest in Property
ANTONINA LAMPANO v. PLACIDA A. JOSE, ET AL. ............................................................................................ 74
MRS. HENRY E. HARDING v. COMMERCIAL UNION ASSURANCE COMPANY.......................................... 75
GARCIA & PNB v. THE HONGKONG FIRE & MARINE INSURANCE CO., LTD. ........................................... 76
NILO CHA and STELLA UY CHA, and UNITED INSURANCE CO., INC v. COURT OF APPEALS and
CKS DEVELOPMENT CORPORATION ....................................................................................................................... 77
GAISANO CAGAYAN, INC. vs INSURANCE COMPANY OF NORTH AMERICA ........................................... 78
MALAYAN INSURANCE CO., INC. vs PHILIPPINES FIRST INSURANCE CO., INC. and REPUTABLE
FORWARDER SERVICES, INC. ...................................................................................................................................... 79
Double Insurance and Over Insurance
ULPIANO SANTA ANA v. COMMERCIAL UNION ASSURANCE COMPANY, LTD. ..................................... 80
PIONEER INSURANCE & SURETY CORPORATION v. COURT OF APPEALS .............................................. 82
Special Provisions on Mortgagor and Mortgagee
ARMANDO GEAGONIA v. COURT OF APPEALS .................................................................................................... 83
PERFECTION OF THE CONTRACT OF INSURANCE
Consensuality of Contracts of Insurance
BONIFACIO BROS. INC. v. MORA ................................................................................................................................ 84
MANILA MAHOGANY v. COURT OF APPEALS ....................................................................................................... 85
MALAYAN INSURANCE v. COURT OF APPEALS ................................................................................................... 86
EASTERN SHIPPING LINES v. PRUDENTIAL GUARANTEE AND ASSURANCE ........................................ 87
ASIAN TERMINALS, INC. v. MALAYAN INSURANCE, CO., INC., ...................................................................... 89
Form of Insurance Contract
VIOLA BADGER v. NEW YORK LIFE INSURANCE COMPANY ......................................................................... 90
Payment of Premium
GULF RESORTS, INC., v. PCIC ....................................................................................................................................... 91
UCPB GENERAL INSURANCE CO., INC. v. MASAGANA TELEMART, INC .................................................... 92
PEDRO ARCE v. THE CAPITAL INSURANCE & SURETY CO., INC................................................................... 94
ACME SHOE, RUBBER & PLASTIC CORPORATION AND CHUA PAC v. CA................................................. 95
CAPITAL INSURANCE v. PLASTIC ERA CO., INC. .................................................................................................. 96
PHILIPPINE PHOENIX SURETY & INSURANCE COMPANY v. WOODWORKS, INC. ............................... 97
VELASCO v. APOSTOL ..................................................................................................................................................... 98
VALENZUELA v. CA ....................................................................................................................................................... 100
MAKATI TUSCANY v. CA ............................................................................................................................................. 103
TIBAY v. COURT OF APPEALS .................................................................................................................................. 104
SOUTH SEA SURETY & INSURANCE CO. v. CA ................................................................................................... 106
SERVICEWIDE SPECIALISTS INCORPORATED v. CA ...................................................................................... 108
JOSE MARQUES AND MAXILITE TECHNOLOGIES, INC. v. FAR EAST BANK, ET. AL. ......................... 110
Refund of Premiums
GREAT PACIFIC LIFE INSURANCE CORPORATION v. COURT OF APPEALS.......................................... 112
Delivery of Policy
VDA DE SINDAYEN v. INSULAR LIFE..................................................................................................................... 114
PEREZ v. COURT OF APPEALS .................................................................................................................................. 115
THE POLICY OF INSURANCE, PARTIES AND THEIR INTEREST
Cover Notes/Temporary or Provisional Insurance
GREAT PACIFIC LIFE v. CA ......................................................................................................................................... 116
Types of Policies
DEVELOPMENT INSURANCE v. IAC, et al. ........................................................................................................... 117
Parties
VILLANUEVA v. ORO .................................................................................................................................................... 118
FILIPINAS COMPAÑIA DE SEGUROS v. CHRISTERN, HUENEFELD and CO., INC. ............................... 119
DELFIN NARIO and ALEJANDRA SANTOS-NARIO v. THE PHILIPPINE AMERICAN LIFE
INSURANCE COMPANY ............................................................................................................................................... 120
RE: CLAIMS FOR BENEFITS OF THE HEIRS OF THE LATE MARIO v. CHANLIONGCO ...................... 121
THE PHILIPPINE AMERICAN INSURANCE COMPANY v. HONORABLE GREGORIO G. PINEDA and
RODOLFO C. DIMAYUGA ............................................................................................................................................. 122
Other Cases
PILAR C. DE LIM v. SUN LIFE ASSURANCE COMPANY OF CANADA ......................................................... 123
RAFAEL ENRIQUEZ v. SUN LIFE ASSURANCE COMPANY OF CANADA .................................................. 124
CONCORDIA GO v. ANGELA REDFERN and THE INTERNATIONAL ASSURANCE CO., LTD ............ 125
PAULO ANG and SALLY C. ANG v. FULTON FIRE INSURANCE CO., ET AL ............................................. 126
MELECIO COQUIA, MARIA ESPANUEVA and MANILA YELLOW TAXICAB CO., INC v FIELDMEN'S
INSURANCE CO. INC. .................................................................................................................................................... 127
PACIFIC TIMBER EXPORT CORPORATION v. THE HONORABLE COURT OF APPEALS and
WORKMEN'S INSURANCE COMPANY, INC. ........................................................................................................ 129
DEVELOPMENT INSURANCE v. INTERMEDIATE APPELLATE COURT ................................................... 131
FINMAN GENERAL ASSURANCE CORP. v. WILLIAM INOCENCIO, ET AL. .............................................. 132
FORTUNE INSURANCE AND SURETY CO., INC. v. COURT OF APPEALS and PRODUCERS BANK OF
THE PHILIPPINES .......................................................................................................................................................... 133
COUNTRY BANKERS INSURANCE CORPORATION v. LIANGA BAY AND COMMUNITY MULTI-
PURPOSE COOPERATIVE, INC. ................................................................................................................................. 135
PARAMOUNT INSURANCE CORPORATION v. SPOUSES YVES and MARIA TERESA REMONDEULAZ
............................................................................................................................................................................................... 136
RESCISSION OF INSURANCE CONTRACTS
Concealment, Misrepresentation and Incontestable Clause
FRANCISCA EGUARAS vs. THE GREAT EASTERN LIFE ASSURANCE COMPANY ................................ 137
MRS. HENRI HARDING v. COMMERCIAL UNION ASSURANCE COMPANY............................................. 139
TAN CHAY HENG v. THE WEST COAST LIFE INSURANCE COMPANY, .................................................... 140
ARGENTE v. WEST COAST ......................................................................................................................................... 141
MUSÑGI v. WEST COAST LIFE ASSURANCE CO. ............................................................................................... 142
INSULAR LIFE ASSURANCE CO., LTD. v. SERAFIN FELICIANO ................................................................... 143
YU PANG CHENG alias YU PANG CHING v. CA ................................................................................................... 144
ESTEFANIA SATURNINO v. THE PHILIPPINE AMERICAN LIFE INSURANCE COMPANY ................ 145
GREAT PACIFIC v. CA ................................................................................................................................................... 146
REGINA EDILLON v. MANILA BANKER LIFE ASSURANCE CORPORATION .......................................... 148
NG GAN ZEE v. ASIAN CRUSADER LIFE ASSURANCE CORPORATION .................................................... 149
PACIFIC BANKING CORP. v. CA ................................................................................................................................ 150
TAN v. CA and THE PHILAM LIFE INSURANCE COMPANY .......................................................................... 152
THELMA VDA. DE CANILANG v. CA ........................................................................................................................ 153
GREAT PACIFIC LIFE ASSURANCE CORPORATION v. CA ............................................................................. 154
MA. LOURDES S. FLORENDO v. PHILAM PLANS, INC. .................................................................................... 155
MANILA BANKERS LIFE INSURANCE CORPORATION v. ABAN ................................................................. 156
SUN LIFE v. SIBYA.......................................................................................................................................................... 157
MANULIFE PHILIPPINES, INC., v. HERMENEGILDA YBANEZ ..................................................................... 158
Warranty
BACHRACH v. BRITISH AMERICAN........................................................................................................................ 159
YOUNG v. MIDLAND...................................................................................................................................................... 160
AMERICAN HOME v. TANTUCO ............................................................................................................................... 161
PRUDENTIAL v. TRANS ASIA .................................................................................................................................... 162
Action to Rescind
MALAYAN v. PHIL NAILS AND WIRES CORP ..................................................................................................... 163
MALAYAN v. PAP ........................................................................................................................................................... 165
Waiver of the Rights to Rescind
AMERICAN HOME v. CHUA ........................................................................................................................................ 166
QUA CHEE GAN v. LAW UNION ................................................................................................................................ 168
RISKS AND COVERAGES
Concept of Insurable Risk and Causation
CHOA TIEK SENG v. CA ................................................................................................................................................ 169
Exceptions and Exclusions
COUNTRY BANKERS v. LIANGA BAY ..................................................................................................................... 170
UNITED MERCHANTS v. COUNTRY BANKERS .................................................................................................. 172
DBP v. RADIO MINDANAO ......................................................................................................................................... 173
Principles of Causation
FGU INSURANCE v. CA ................................................................................................................................................. 174
Life Insurance | Section 181
GREAT PACIFIC v. CA ................................................................................................................................................... 175
Life Insurance | Accidental Death and Death by Accidental Means
FINMAN GENERAL ASSURANCE CORPORATION v. COURT OF APPEALS ............................................. 176
VIRGINIA CALANOC v. COURT OF APPEALS ..................................................................................................... 178
BIAGTAN v. THE INSULAR LIFE ASSURANCE COMPANY, LTD .................................................................. 179
Life Insurance | Other Cases
MANUFACTURERS LIFE INSURANCE CO. v. MEER.......................................................................................... 181
JAMES MCGUIRE v. THE MANUFACTURERS LIFE INSURANCE CO. ......................................................... 182
RUFINO D. ANDRES v. CROWN LIFE INSURANCE COMPANY ..................................................................... 183
SIMON DE LA CRUZ v. THE CAPITAL INSURANCE AND SURETY CO., INC. ........................................... 185
THE INSULAR LIFE ASSURANCE COMPANY, LTD v. EBRADO .................................................................... 186
SUN INSURANCE OFFICE LTD. v. COURT OF APPEALS ................................................................................. 188
THE INSULAR LIFE ASSURANCE COMPANY, LTD. v. KHU ........................................................................... 189
Fire Insurance | Effect of alteration in the thing insured
MALAYAN INSURANCE CO. v. PAP CO., LTD. ...................................................................................................... 191
Fire Insurance | Other Cases
TAN CHUCO v. YORKSHIRE FIRE AND LIFE INSURANCE ............................................................................. 193
BACHRACH v. BRITISH AMERICAN INSURANCE ............................................................................................. 194
DEVELOPMENT INSURANCE v. INTERMEDIATE APPELLATE COURT, et al. ....................................... 196
NEW LIFE ENTERPRISES, et al. v. CA, et al.......................................................................................................... 198
PHILIPPINE HOME ASSURANCE CORPORATION v. COURT OF APPEALS ............................................. 200
Casualty Insurance | Other Cases
PAN MALAYAN INSURANCE CORPORATION v. COURT OF APPEALS ..................................................... 202
SIMON DE LA CRUZ v. THE CAPITAL INSURANCE and SURETY CO., INC., ............................................ 203
SUN INSURANCE OFFICE, LTD., v. THE HON. COURT OF APPEALS .......................................................... 204
FORTUNE INSURANCE AND SURETY CO., INC., v. COURT OF APPEALS................................................. 205
ALPHA INSURANCE AND SURETY CO. v. ARSENIA SONIA CASTOR......................................................... 206
PHIL-NIPPON KYOEI, CORP. v. ROSALIA T. GUDELOSAO ............................................................................. 207
Suretyship
MELECIO ARRANZ v. MANILA FIDELITY AND SURETY CO., INC., ............................................................. 208
REPARATIONS COM. v. UNIVERSAL DEEP SEA FISHING .............................................................................. 209
CAPITAL INSURANCE v. RONQUILLO TRADING .............................................................................................. 210
ZARAGOZA v. FIDELINO.............................................................................................................................................. 211
EASTERN ASSURANCE & SURETY CORPORATION v. INTERMEDIATE APPELLATE COURT ........ 212
STRONGHOLD INSURANCE v. COURT APPEALS .............................................................................................. 213
NATIONAL POWER CORP. v. COURT APPEALS ................................................................................................. 214
PHILIPPINE PRYCE ASSURANCE v. COURT OF APPEALS ............................................................................. 215
PRUDENTIAL GUARANTEE AND ASSURANCE v. EQUINOX LAND CORP. .............................................. 216
AFP GENERAL INSURANCE v. MOLINA ................................................................................................................ 217
INTRA-STRATA ASSURANCE CORPORATION v. REPUBLIC OF THE PHILIPPINES ........................... 218
FIRST LEPANTO-TAISHO INSURANCE CORPORATION v. CHEVRON PHILIPPINES, INC. ............... 219
No Fault Indemnity Clause
PERLA COMPANIA DE SEGUROS v. HONORABLE ANCHETA ...................................................................... 221
Comprehensive Motor Vehicle Liability Insurance
FAR EASTERN SURETY v. MISA ............................................................................................................................... 223
SHAFER v. JUDGE ........................................................................................................................................................... 225
PEZA v. ALIKPALA ......................................................................................................................................................... 227
WESTERN GUARANTY v. CA ..................................................................................................................................... 229
PERLA COMPANIA DE SEGUROS v. CA & LIM .................................................................................................... 231
VDA. DE MAGLANA v. CONSOLACION................................................................................................................... 233
FIRST QUEZON CITY INSURANCE v. CA ............................................................................................................... 235
FIRST INTEGRATED BONDING AND INSURANCE v. HONORABLE HERNANDO ................................ 237
PARAMOUNT INSURANCE CORP v. SPOUSES ROMUALDEZ ....................................................................... 238
Reinsurance
EQUITABLE INS. AND CASUALTY CO. v. RURAL INSURANCE AND SURETY COMPANY, ................ 239
THE PHILIPPINE AMERICAN LIFE INSURANCE COMPANY v. THE AUDITOR GENERAL................ 241
IVOR ROBERT DAYTON GIBSON v. HON. PEDRO A. REVILLA .................................................................... 243
PIONEER INSURANCE & SURETY CORPORATION v. COURT OF APPEALS ........................................... 244
MARINE INSURANCE
Definition and Scope
LA RAZON SOCIAL "GO TIAOCO Y HERMANOS" v. UNION INSURANCE SOCIETY OF ...................... 246
General Average
MAGSAYSAY, INC. v. ANASTACIO AGAN .............................................................................................................. 247
FRANCISCO JARQUE v. SMITH, BELL & CO., LTD., ET AL. and UNION FIRE INSURANCE ................ 248
COMPAGNIE DE COMMERCE ET DE NAVIGATION D'EXTREME ORIENT v. THE HAMBURG
AMERIKA PACKETFACHT ACTIEN GESELLSCHAFT ....................................................................................... 250
Particular Average
A. MAGSAYSAY INC. v. ANASTACIO AGAN .......................................................................................................... 252
PHILIPPINE HOME ASSURANCE CORP v. CA AND EASTERN SHIPPING LINES .................................. 254
Risk Covered
LA RAZON SOCIAL "GO TIAOCO Y HERMANOS," v. UNION INSURANCE SOCIETY OF CANTON, . 256
ISABELA ROQUE and ONG CHIONG v. HON. INTERMEDIATE APPELATE COURT and PIONEER
INSURANCE AND SURETY CORPORATION ......................................................................................................... 257
CATHAY INSURANCE CO. v. HON. COURT OF APPEALS and REMINGTON INDUSTRIAL SALES 259
Insurable Interest in Marine Insurance | Charterer
PUROMINES, INC. v. COURT OF APPEAL and PHILIPP BROTHERS OCEANIC, INC ............................ 260
COASTWISE LIGHTERAGE CORPORATION v. COURT OF APPEALS and the PHILIPPINE GENERAL
INSURANCE COMPANY ............................................................................................................................................... 262
LEA MER INDUSTRIES, INC., v. MALAYAN INSURANCE CO., INC., ............................................................ 264
LOADSTAR SHIPPING CO. v. COURT OF APPEALS ........................................................................................... 266
PLANTERS PRODUCTS, INC. v. COURT OF APPEALS, SORIAMONT STEAMSHIP AGENCIES AND
KYOSEI KISEN KABUSHIKI KAISHA ....................................................................................................................... 268
Actual Total Loss
PAN MALAYAN INSURANCE CORP. v. COURT OF APPEALS ........................................................................ 269
PHILIPPINE MANUFACTURING CO. v. UNION INSURANCE SOCIETY OF CANTON, LTD................. 271
Constructive Total Loss
ORIENTAL ASSURANCE CORPORATION v. COURT OF APPEALS AND PANAMA SAW MILL CO.,
INC........................................................................................................................................................................................ 273
Partial Loss
ABOITIZ SHIPPING CORPORATION v. COURT OF APPEALS ........................................................................ 275
Measure of Indemnity
FILIPINO MERCHANTS INSURANCE CO., INC. v. COURT OF APPEALS and CHOA TIEK SENG..... 276
CALTEX (PHILIPPINES), INC. v. SULPICIO LINES, INC., et al. ...................................................................... 277
DELSAN TRANSPORT LINES, INC. v. THE HON. COURT OF APPEALS and AMERICAN HOME
ASSURANCE CORPORATION ..................................................................................................................................... 278
CEBU SALVAGE CORPORATION v. PHILIPPINE HOME ASSURANCE CORPORATION ...................... 280
KEPPEL CEBU SHIPYARD, INC. v. PIONEER INSURANCE AND SURETY CORPORATION ................ 281
CLAIM SETTLEMENT AND SUBROGATION
Conditions before the insured may receive on the policy after the loss
PHILIPPINE AMERICAN LIFE INSURANCE COMPANY v. COURT OF APPEALS and ELIZA............. 283
PRUDENTIAL GUARANTEE and ASSURANCE, INC. v. EQUINOX LAND CORPORATION .................. 284
ABOITIZ SHIPPING CORPORATION v. INSURANCE COMPANY OF NORTH AMERICA ..................... 285
MALAYAN INSURANCE CO., INC. v. RODELIO ALBERTO and ENRICO ALBERTO REYES ................ 287
UNITED MERCHANTS CORPORATION v. COUNTRY BANKERS INSURANCE CORPORATION ...... 288
Notice and Proof of Loss
FINMAN GENERAL ASSURANCE CORPORATION v. COURT OF APPEALS and USIPHIL
INCORPORATED............................................................................................................................................................. 291
COUNTRY BANKERS INSURANCE CORPORATION v. LIANGA BAY AND COMMUNITY MULTI-
PURPOSE COOPERATIVE, INC. ................................................................................................................................. 292
FGU INSURANCE CORPORATION v. THE COURT OF APPEALS, et al........................................................ 293
PHILIPPINE CHARTER INSURANCE CORPORATION v. CHEMOIL LIGHTERAGE CORPORATION295
ABOITIZ SHIPPING CORPORATION v. COURT OF APPEALS, et al. ............................................................ 296
MITSUBISHI MOTORS PHILIPPINES SALARIED EMPLOYEES UNION (MMPSEU) v. MITSUBISHI
MOTORS PHILIPPINES ................................................................................................................................................ 298
Action by the Insurer
CATHAY INSURANCE CO., INC., et al v. COURT OF APPEALS and EMILIA CHAN LUGAY ................ 300
Delay in the Payment of Insurance Proceeds
ZENITH INSURANCE CORPORATION v. COURT OF APPEALS and LAWRENCE FERNANDEZ ...... 301
FINMAN GENERAL ASSURANCE CORPORATION v. COURT OF APPEALS and USIPHIL .................. 302
Fraudulent Claims by the Insured
UY HU & CO. v. THE PRUDENTIAL ASSURANCE CO., LTD. ........................................................................... 303
Bringing and Action and Prescription of Action
ALFONSO G. LOPEZ v. FILIPINAS COMPAÑIA DE SEGUROS ........................................................................ 305
SUN INSURANCE OFFICE, LTD v. COURT OF APPEALS and EMILIO TAN .............................................. 306
JACQUELINE JIMENEZ VDA. DE GABRIEL v. COURT OF APPEALS and FORTUNE INSURANCE &
SURETY COMPANY, INC .............................................................................................................................................. 308
TRAVELLERS INSURANCE & SURETY CORPORATION v. COURT OF APPEALS and VICENTE
MENDOZA ......................................................................................................................................................................... 310
Subrogation
FIREMAN'S FUND INSURANCE COMPANY and FIRESTONE TIRE AND RUBBER COMPANY OF THE
PHILIPPINES v. JAMILA & COMPANY, INC. and FIRST QUEZON CITY ..................................................... 311
MALAYAN INSURANCE CO., INC v. COURT OF APPEALS, MARTIN C. VALLEJOS, SIO CHOY, SAN
LEON RICE MILL, INC. and PANGASINAN TRANSPORTATION CO., INC, ................................................ 313
PAN MALAYAN INSURANCE CORPORATION v. COURT OF APPEALS, ERLINDA FABIE AND ....... 315
THE PHILIPPINE AMERICAN GENERAL INSURANCE COMPANY, INC. v. COURT OF APPEALS and
FELMAN SHIPPING LINES.......................................................................................................................................... 317
WALLEM PHILIPPINES SHIPPING INC. and SEACOAST MARITIME CORPORATION v.
PRUDENTIAL GUARANTEE & ASSURANCE INC. and COURT OF APPEAL ............................................. 319
R ea l a nd h yp ot h ec a ry n a tu r e o f ma ri ti m e l a w

PHILIPPINE SHIPPING COMPANY, ET. AL v. GARCIA


G.R. No. 1600
June 1, 1906

Arellano, C.J.

Facts:

The Philippine Shipping Company, the owner of the steamship Nuestra Sra. De Lourdes,
claims an indemnification of 44,000 pesos for the loss of the said ship as a result of a collision.
Ynchusti & Co. also claimed 24,705.64 pesos as an indemnification for the loss of the cargo of hemp
and coprax carried by the said ship on her last trip. The defendant, Francisco Garcia Vergara, was the
owner of the steamship Navarra, which collided with the Lourdes.

The court below found as a matter of fact that the steamship Lourdes was sailing in
accordance with law, but that the Navarra was not, and was therefore responsible for the collision.

Issue:

Are the owners and agents of a vessel causing the loss of another vessel by collision liable
beyond the vessel itself causing the collision and other things appertain thereto?

Ruling:

No. The owners and agents of a vessel causing the loss of another vessel by collision are not
liable beyond the vessel itself causing the collision and other things appertaining thereto.

That which distinguishes the maritime from the civil law and even from the mercantile law
in general is the ’real’ and hypothecary nature of the former, and the many securities of a ’real’ nature
that maritime customs from time immemorial, the laws, the codes, and the later jurisprudence, have
provided for the protection of the various and conflicting interests which are ventured and risked in
maritime expeditions. This repeals the civil law to such an extent that, in certain cases, where the
mortgaged property is lost no personal action lies against the owner or agent of the vessels.

In view of the above stated principles, the defendant is liable for the indemnification to which
the plaintiff is entitle by reason of the collision, but he is not required to pay such indemnification for
the reason that the obligation thus incurred has been extinguished on account of the loss of the thing
bound for the payment thereof.

1
R ea l a nd h yp ot h ec a ry n a tu r e o f ma ri ti m e l a w

ABUEG, ET AL., v. BARTOLOME SAN DIEGO


G.R. No. L-773
December 17, 1946

Padilla, J.

Facts:

Defendant’s ships, the M/S San Diego II and the M/S Bartolome, while engaged in fishing
operations were caught by a typhoon as a consequence of which they were sunk and totally lost.
Amado Nuñez, Victoriano Salvacion and Francisco Oching while acting in their capacities perished in
the shipwreck. The vessels were not covered by any insurance.

Defendant cited Article 587 of the Code of Commerce which provides that if the vessel
together with all her tackle and freight money earned during the voyage are abandoned, the agent's
liability to third persons for tortious acts of the captain in the care of the goods which the ship carried
is extinguished, Article 837 which provides that in cases of collision, the ship owners' liability is
limited to the value of the vessel with all her equipment and freight earned during the voyage, and
Article 643 which provides that if the vessel and freight are totally lost, the agent's liability for wages
of the crew is extinguished.

Issue:

As owner of the two ships which sunk, is his liability extinguished?

Ruling:

No. He is still liable under the Workmen’s Compensation Act.

The real and hypothecary nature of the liability of the shipowner or agent embodied in the
provisions of the Code of Commerce, had its origin in the prevailing continues of the maritime trade
and sea voyages during the medieval ages, attended by innumerable hazards and perils. To offset
against these adverse conditions and encourage shipbuilding and maritime commerce, it was
deemed necessary to confine the liability of the owner or agent arising from the operation of a ship
to the vessel, equipment, and freight, or insurance, if any, so that if the shipowner or agent abandoned
the ship, equipment, and freight, his liability was extinguished.

However, the provisions of the Code of Commerce invoked by appellant have no room in the
application of the Workmen's Compensation Act which seeks to improve, and aims at the
amelioration of, the condition of laborers and employees. It is not the liability for the damage or loss
of the cargo or injury to, or death of, a passenger by or through the misconduct of the captain or
master of the ship; nor the liability for the loss of the ship as result of collision; nor the responsibility
for wages of the crew, but a liability created by a statute to compensate employees and laborers in
cases of injury received by or inflicted upon them, while engaged in the performance of their work or
employment, or the heirs and dependents and laborers and employees in the event of death caused
by their employment. Such compensation has nothing to do with the provisions of the Code of
Commerce regarding maritime commerce. It is an item in the cost of production which must be
included in the budget of any well-managed industry.

2
R ea l a nd h yp ot h ec a ry n a tu r e o f ma ri ti m e l a w

YANGCO, ETC. v. LASERNA, ET AL.


G.R. No. L-47447-47449
October 29, 1941

Moran, J.

Facts:

At about one o'clock in the afternoon of May 26, 1927, the steamer S.S. Negros, belonging to
petitioner here, Teodoro R. Yangco, left the port of Romblon on its return trip to Manila. Typhoon
signal No. 2 was then up, of which fact the captain was duly advised and his attention thereto called
by the passengers themselves before the vessel set sail. The boat was overloaded as indicated by the
loadline which was 6 to 7 inches below the surface of the water. Baggage, trunks and other
equipments were heaped on the upper deck, the hold being packed to capacity. In addition, the vessel
carried thirty sacks of crushed marble and about one hundred sacks of copra and some lumber. The
passengers, numbering about 180, were overcrowded, the vessel's capacity being limited to only 123
passengers. After two hours of sailing, the boat encountered strong winds and rough seas between
the islands of Banton and Simara, and as the waves splashed the ladies' dresses, the awnings were
lowered. As the sea became increasingly violent, the captain ordered the vessel to turn left, evidently
to return to port, but in the manuever, the vessel was caught sidewise by a big wave which caused it
to capsize and sink. Many of the passengers died in the mishap

Issue:

May the shipowner or agent, notwithstanding the total loss of the vessel as a result of the
negligence of its captain, be properly held liable in damages for the consequent death of its
passengers?

Ruling:

Article 587 of the Code of Commerce provides that the agent shall also be civilly liable for the
indemnities in favor of third persons which arise from the conduct of the captain in the care of the
goods which the vessel carried; but he may exempt himself therefrom by abandoning the vessel with
all her equipments and the freight he may have earned during the voyage.

If the shipowner or agent may in any way be held civilly liable at all for injury to or death of
passengers arising from the negligence of the captain in cases of collisions or shipwrecks, his liability
is merely co-extensive with his interest in the vessel such that a total loss thereof results in its
extinction. In arriving at this conclusion, The courts have not been unmindful of the fact that the ill-
fated steamship Negros, as a vessel engaged in interisland trade, is a common carrier, and that the as
a vessel engaged in interisland trade, is a common carrier, and that the relationship between the
petitioner and the passengers who died in the mishap rests on a contract of carriage. But assuming
that petitioner is liable for a breach of contract of carriage, the exclusively "real and hypothecary
nature" of maritime law operates to limit such liability to the value of the vessel, or to the insurance
thereon, if any. In the instant case it does not appear that the vessel was insured.

Whether the abandonment of the vessel sought by the petitioner in the instant case was in
accordance with law of not, is immaterial. The vessel having totally perished, any act of abandonment
would be an idle ceremony.

3
R ea l a nd h yp ot h ec a ry n a tu r e o f ma ri ti m e l a w

MANILA STEAMSHIP CO., INC., v. INSA ABDULHAMAN and LIM HONG TO


G.R. No. L-9534
September 29, 1956

Reyes, J. B. L., J.

Facts:

In the evening of May 4, 1948, the M/L “Consuelo V”, laden with cargoes and passengers left
the port of Zamboanga City. The weather was good and fair. Among her passengers were
the plaintiff, Insa Abdulhaman, his wife Carimla Mora and their five children. The plaintiff and his
wife paid their fare before the voyage started.

On that same night the M/S “Bowline Knot” was navigating.

When some of the passengers of the M/L “Consuelo V” were then sleeping, they felt the
shocking collision of the M/L “Consuelo V” and a big motorship, which later on was identified as the
M/V “Bowline Knot”.

Because the M/L “Consuelo V” capsized, nine (9) passengers were dead and missing and all
the cargoes carried on said boat, including those of the Plaintiff as appear in the list, were lost.

Petitioner Manila Steamship Co. pleads that it is exempt from any liability to Plaintiff under
Article 1903 of the Civil Code because it had exercised the diligence of a good father of a family in the
selection of its employees.

Issue:

Is the petitioner exempt from any liability to plaintiff invoking Article 1903 of the Civil Code?

Ruling:

No, the defense is untenable.

While it is true that Plaintiff’s action against Petitioner is based on a tort or quasi-delict, the
tort in question is not a civil tort under the Civil Code but a maritime tort resulting in a collision at
sea, governed by Articles 826-939 of the Code of Commerce. Under Article 827 of the Code of
Commerce, in case of collision between two vessels imputable to both of them, each vessel shall suffer
her own damage and both shall be solidarily liable for the damages occasioned to their cargoes. The
characteristic language of the law in making the “vessels” solidarily liable for the damages due to the
maritime collision emphasizes the direct nature of the responsibilities on account of the collision
incurred by the shipowner under maritime law, as distinguished from the civil law and mercantile
law in general. This direct responsibility is recognized in Article 618 of the Code of Commerce under
which the captain shall be civilly liable to the ship agent, and the latter is the one liable to third
persons.

In fact, it is a general principle, well established in maritime law and custom, that shipowners
and ship agents are civilly liable for the acts of the captain (Code of Commerce, Article 586) and for
the indemnities due the third persons (Article 587); so that injured parties may immediately look for

4
R ea l a nd h yp ot h ec a ry n a tu r e o f ma ri ti m e l a w

reimbursement to the owner of the ship, it being universally recognized that the ship master or
captain is primarily the representative of the owner.

5
R ea l a nd h yp ot h ec a ry n a tu r e o f ma ri ti m e l a w

CHUA YEK HONG, v. IAC, MARIANO GUNO, and DOMINADOR OLIT


G.R. No. L-74811
September 30, 1988

Melencio-Herrera, J.

Facts:

Petitioner is a duly licensed copra dealer, while private respondents are the owners of the
vessel, "M/V Luzviminda I," a common carrier engaged in coastwise trade from the different ports of
Oriental Mindoro to the Port of Manila.

In October 1977, petitioner loaded sacks of copra on board the vessel "M/V Luzviminda I" for
shipment from to Manila. Said cargo, however, did not reach Manila because somewhere between
Cape Santiago and Calatagan, Batangas, the vessel capsized and sank with all its cargo.

The appellate court held that private respondents’ liability, as shipowners, for the loss of the
cargo is merely co-extensive with their interest in the vessel such that a total loss thereof results in
its extinction.

Issue:

Are the defendants absolved from any and all liabilities arising from the loss of 1,000 sacks
of copra belonging to plaintiff-appellee?

Ruling:

Under Article 587, this direct liability is moderated and limited by the shipagent’s or
shipowner’s right of abandonment of the vessel and earned freight. This expresses the universal
principle of limited liability under maritime law. The most fundamental effect of abandonment is the
cessation of the responsibility of the ship agent/owner. It has thus been held that by necessary
implication, the ship agent’s or shipowner’s liability is confined to that which he is entitled as of right
to abandon — "the vessel with all her equipment and the freight it may have earned during the
voyage," and "to the insurance thereof if any". In other words, the shipowner’s or agent’s liability is
merely co-extensive with his interest in the vessel such that a total loss thereof results in its
extinction. "No vessel, no liability" expresses in a nutshell the limited liability rule. The total
destruction of the vessel extinguishes maritime liens as there is no longer any res to which it can
attach

The rationale therefor has been explained as follows: "The real and hypothecary nature of the
liability of the shipowner or agent embodied in the provisions of the Maritime Law, Book III, Code of
Commerce, had its origin in the prevailing conditions of the maritime trade and sea voyages during
the medieval ages, attended by innumerable hazards and perils. To offset against these adverse
conditions and to encourage shipbuilding and maritime commerce, it was deemed necessary to
confine the liability of the owner or agent arising from the operation of a ship to the vessel,
equipment, and freight, or insurance, if any, so that if the shipowner or agent abandoned the ship,
equipment, and freight, his liability was extinguished."

6
R ea l a nd h yp ot h ec a ry n a tu r e o f ma ri ti m e l a w

HEIRS OF DE LOS SANTOS, et al. v. CA and MARITIMA


G.R. No. L-51165
June 21, 1990

Medialdea. J.

Facts:

The M/V 'Mindoro', vessel of the defendant, sailed on November 2, 1967 at about 2:00 p.m.
which should have been 6:00 p.m. with many passengers aboard.

The boat met typhoon 'Welming' and due to the strong waves it sank causing the drowning
of many passengers among whom were Amparo delos Santos and all the aforesaid children. It
appears also that Teresa Pamatian and Diego Salim, who were also passengers also drowned.

It appears that in a decision of the Board of Marine Inquiry, it was found that the captain and
some officers of the crew were negligent in operating the vessel. However, the defendant alleges that
no negligence was ever established and, in fact, the shipowners and their officers took all the
necessary precautions in operating the vessel. Furthermore, the loss of lives as a result of the
drowning of some passengers, including the relatives of the herein plaintiff, was due to force majeure.

Issue:

Is Marimita liable upon the negligence of its captain and its crew?

Ruling:

Yes. Article 587 of the Code of Commerce states that the ship agent shall also be civilly liable
for indemnities in favor of third persons which may arise from the conduct of the captain in the care
of the goods which he loaded on the vessel, but he may exempt himself therefrom by abandoning the
vessel with all her equipments and the freight it may have earned during the voyage.

It must be stressed at this point that Article 587 speaks only of situations where the fault or
negligence is committed solely by the captain. In cases where the shipowner is likewise to be blamed,
Article 587 does not apply.

In this case, Maritima is negligent. If the captain knew of the typhoon beforehand, it is
inconceivable for Maritima to be totally in the dark of 'Welming.' In allowing the ship to depart late
from Manila despite the typhoon advisories, Maritima displayed lack of foresight and minimum
concern for the safety of its passengers taking into account the surrounding circumstances of the
case.

7
R ea l a nd h yp ot h ec a ry n a tu r e o f ma ri ti m e l a w

ABOITIZ SHIPPING CORPORATION v. NEW INDIA ASSURANCE COMPANY, LTD.


G.R. No. 156978
August 24, 2007

Quisumbing, J.

Facts:

On October 31, 1980, M/V P. Aboitiz, a vessel owned by petitioner, sank on her voyage from
Hong Kong to Malaysia. Respondent is the insurer of the lost cargoes loaded on board and consigned
to General Textile, Inc. After respondent indemnified General Textile, Inc., it was subrogated to its
rights, interests and actions against petitioner.

Respondent filed an action before the RTC for recovery against petitioner. The trial court held
petitioner liable for the total value of the lost cargoes instead of applying the doctrine of limited
liability.

Issue:

Will the doctrine of limited liability apply in this case?

Ruling:

No.

We need only to stress that from the nature of their business and for reasons of public policy,
common carriers are bound to observe extraordinary diligence over the goods they transport
according to all the circumstances of each case. In the event of loss, destruction or deterioration of
the insured goods, common carriers are responsible, unless they can prove that the loss, destruction
or deterioration was brought about by the causes specified in Article 1734 of the Civil Code.12 In all
other cases, common carriers are presumed to have been at fault or to have acted negligently, unless
they prove that they observed extraordinary diligence.

The weather was moderate when M/V P. Aboitiz sank. Both the trial and appellate courts also
ruled that the M/V P. Aboitiz sank due to its unseaworthiness and not due to typhoon. To limit
petitioner's liability to the amount of the insurance proceeds, it has the burden of showing that the
unseaworthiness of the vessel was not due to its fault or negligence. But it failed to do so. Where the
shipowner fails to overcome the presumption of negligence, the doctrine of limited liability cannot
be applied.

8
Gr ow th of I ns ura nc e i n t h e Ph il i pp i n es

INSULAR LIFE ASSURANCE CO., LTD. v. FELICIANO, ET AL.


G.R. No. L-47593
December 29, 1943

Ozaeta, J.

Facts:

Evaristo Feliciano was issued an insurance policy by Insular Life. In September 1935, he died.
His heirs filed an insurance claim but Insular Life denied the application as it averred that Feliciano’s
application was attended by fraud. It was later found in court that the insurance agent and the
medical examiner of Insular Life who assisted Feliciano in signing the application knew that Feliciano
was already suffering from tuberculosis; that they were aware of the true medical condition of
Feliciano yet they still made it appear that he was healthy in the insurance application form; that
Feliciano signed the application in blank and the agent filled the information for him.

Issue:

Can Insular Life avoid the insurance policy by reason of the fact that its agent knowingly and
intentionally wrote down the answers in the application differing from those made by Feliciano
hence instead of serving the interests of his principal, acts in his own or another’s interest and
adversely to that of his principal?

Ruling:

No. Insular Life must pay the insurance policy. The weight of authority is that if an agent of
the insurer, after obtaining from an applicant for insurance a correct and truthful answer to
interrogatories contained in the application for insurance, without knowledge of the applicant fills in
false answers, either fraudulently or otherwise, the insurer cannot assert the falsity of such answers
as a defense to liability on the policy, and this is true generally without regard to the subject matter
of the answers or the nature of the agent’s duties or limitations on his authority, at least if not brought
to the attention of the applicant.

The fact that the insured did not read the application which he signed, is not indicative of bad
faith. It has been held that it is not negligence for the insured to sign an application without first
reading it if the insurer by its conduct in appointing the agent influenced the insured to place trust
and confidence in the agent.

9
Ju r isd ict io n of t h e In su ra nc e Co m miss io n ; A d mi ni st ra tiv e a nd A dj ud ica t or y P o w er s

GERCIO v. SUN LIFE ASSURANCE OF CANADA, ET AL.


G.R. No. 23703
September 28, 1925

Malcolm, J.

Facts:

Sun Life Assurance Co. of Canada issued a 20-year endowment insurance policy on the life of
Hilario Gercio. The insurance company agreed to insure the life of Gercio for the sum of P2,000, to be
paid him on February 1, 1930, or if the insured should die before said date, then to his wife, Mrs.
Andrea Zialcita, should she survive him; otherwise, to the executors, administrators, or assigns of the
insured. The policy did not include any provision reserving to the insured the right to change the
beneficiary

In 1919, she was convicted of the crime of adultery and thereafter, a decree of divorce was
issued.

Gercio formally notified the Sun Life that he had revoked his donation in favor of Andrea
Zialcita, and that he had designated in her stead his present wife, Adela Garcia de Gercio, as the
beneficiary of the policy due to the divorce decree. However, Sun Life refused.

Issue:

Does Gercio have the right to change the beneficiary of the policy?

Ruling:

No.

Under the law, the wife has an insurable interest in the life of her husband. The beneficiary has
an absolute vested interest in the policy from the date of its issuance and delivery. So when a policy
of life insurance is taken out by the husband in which the wife is named as beneficiary, she has a
subsisting interest in the policy. The rule applies to a policy to which there are attached the incidents
of a loan value, cash surrender value, an automatic extension by premiums paid, and to an
endowment policy, as well as to an ordinary life insurance policy.

If the husband wishes to retain to himself the control and ownership of the policy he may so
provide in the policy. But if the policy contains no provision authorizing a change of beneficiary
without the beneficiary's consent, the insured cannot make such change.

Accordingly, it is held that a life insurance policy of a husband made payable to the wife as
beneficiary, is the separate property of the beneficiary and beyond the control of the husband.

The effect produced by the divorce, the Philippine Divorce Law, Act No. 2710, merely provides in
section 9 that the decree of divorce shall dissolve the community property as soon as such decree
becomes final. Absence of a statute to the contrary, that if a policy is taken out upon a husband's life
the wife is named as beneficiary therein, a subsequent divorce does not destroy her rights under the
policy.

10
Ju r isd ict io n of t h e In su ra nc e Co m miss io n ; A d mi ni st ra tiv e a nd A dj ud ica t or y P o w er s

ANG GIOK CHIP v. SPRINGFIELD


G.R. No. L-33637
December 31, 1931

Malcolm, J.

Facts:

Ang insured his warehouse for the total value of Php 60,000. One of these, amounting to
10,000, was with Springfield Insurance Company. His warehouse burned down, then he attempted
to recover 8,000 from Springfield for the indemnity. The insurance company interposed its defense
on a rider in the policy in the form of Warranty F, fixing the amount of hazardous good that can be
stored in a building to be covered by the insurance. They claimed that Ang violated the 3 percent limit
by placing hazardous goods to as high as 39 percent of all the goods stored in the building. His suit to
recover was granted by the trial court.

Issue:

Is the warranty referred to in the policy forming part of the contract of insurance and in the
form of a rider to the insurance policy null and void?

Ruling:

No. The warranty is valid.

The Insurance Act, Section 65, taken from California law, states: "Every express warranty,
made at or before the execution of a policy, must be contained in the policy itself, or in another
instrument signed by the insured and referred to in the policy, as making a part of it."

The court concluded that Warranty F is contained in the policy itself, because by the contract
of insurance agreed to by the parties it was made to be a part. It wasn’t a separate instrument agreed
to by the parties.

The receipt of the policy by the insured without objection binds him. It was his duty to read
the policy and know its terms. He also never chose to accept a different policy by considering the
earlier one as a mistake. Hence, the rider is valid.

11
Ju r isd ict io n of t h e In su ra nc e Co m miss io n ; A d mi ni st ra tiv e a nd A dj ud ica t or y P o w er s

PHILIPPINE HEALTH CARE v. CIR


G.R. No. 167330
September 18, 2009

Corona, J.

Facts:

Petitioner is a domestic corporation whose primary purpose is to establish, maintain, conduct


and operate a prepaid group practice health care delivery system or a health maintenance
organization to take care of the sick and disabled persons enrolled in the health care plan and to
provide for the administrative, legal, and financial responsibilities of the organization. In 2000, CIR
sent petitioner a formal demand letter and the corresponding assessment notices demanding the
payment of deficiency taxes. The deficiency assessment was imposed on petitioner’s health care
agreement with the members of its health care program pursuant to Section 185 of the 1997 Tax
Code.

Petitioner protested the assessment. However, CIR claimed that petitioner’s health care
agreement was a contract of insurance subject to DST under Section 185 of the 1997 Tax Code.

Issue:

Is Philippine Health Care Providers, Inc. engaged in insurance business?

Ruling:

No. Health Maintenance Organizations are not engaged in the insurance business.

Under RA 7878 an HMO is “an entity that provides, offers or arranges for coverage of
designated health services needed by plan members for a fixed prepaid premium. The payments do
not vary with the extent, frequency or type of services provided. Section 2 (2) of PD 1460 enumerates
what constitutes “doing an insurance business” or “transacting an insurance business” which are
making or proposing to make, as insurer, any insurance contract; making or proposing to make, as
surety, any contract of suretyship as a vocation and not as merely incidental to any other legitimate
business or activity of the surety; doing any kind of business, including a reinsurance business,
specifically recognized as constituting the doing of an insurance business within the meaning of this
Code; doing or proposing to do any business in substance equivalent to any of the foregoing in a
manner designed to evade the provisions of this Code.

Overall, petitioner appears to provide insurance-type benefits to its members (with respect
to its curative medical services), but these are incidental to the principal activity of providing them
medical care. The “insurance-like” aspect of petitioner’s business is miniscule compared to its
noninsurance activities. Therefore, since it substantially provides health care services rather than
insurance services, it cannot be considered as being in the insurance business.

12
Ju r isd ict io n of t h e In su ra nc e Co m miss io n ; A d mi ni st ra tiv e a nd A dj ud ica t or y P o w er s

FORTUNE MEDICARE, INC. v. AMORIN


G.R. No. 195872
March 12, 2014

Facts:

While Amorin was on vacation in Hawaii, he underwent an emergency surgery, specifically


appendectomy, causing him to incur professional and hospitalization expenses of US$7,242.35 and
US$1,777.79, respectively. Being a cardholder of Fortune Medicare, Inc. (Fortune Care), a corporation
engaged in providing health maintenance services to its members, he attempted to recover the full
amount upon his return to Manila. However, the company merely approved a reimbursement of
P12,151.36, an amount that was based on the average cost of appendectomy, net of Medicare
deduction, if the procedure were performed in an accredited hospital in Metro Manila. Amorin
received under protest the approved amount, but asked for its adjustment to cover the total amount
of professional fees which he had paid, and eighty percent (80%) of the approved standard charges
based on “American standard”, considering that the emergency procedure occurred in the U.S.A.,
citing provisions of the contract.

Issue:

Can a member of a health care provider recover to the extent agreed in the contract?

Ruling:

Yes. In the case at bar, the Supreme Court said that for purposes of determining the liability
of a health care provider to its members, jurisprudence holds that a health care agreement is in the
nature of non-life insurance, which is primarily a contract of indemnity. Once the member incurs
hospital, medical or any other expense arising from sickness, injury or other stipulated contingent,
the health care provider must pay for the same to the extent agreed upon under the contract.

With regard the ambiguities in the contract, settled is the rule that they should be interpreted
against the party that caused the ambiguity. “Any ambiguity in a contract whose terms are susceptible
of different interpretations must be read against the party who drafted it.” Furthermore, it affirmed
the CA’s finding that Fortune Care’s liability to Amorin under the subject Health Care Contract should
be based on the expenses for hospital and professional fees which he actually incurred, and should
not be limited by the amount that he would have incurred had his emergency treatment been
performed in an accredited hospital in the Philippines.

13
Ju r isd ict io n of t h e In su ra nc e Co m miss io n ; A d mi ni st ra tiv e a nd A dj ud ica t or y P o w er s

ALMENDRAS MINING CORPORATION, v. OFFICE OF THE INSURANCE COMMISSION


G.R. No. L-72878
April 15, 1988

Feliciano, J.

Facts:

In the morning of 3 September 1984, the marine cargo vessel LCT "Don Paulo," while on a
voyage from Davao to Mariveles, Bataan, was forced aground somewhere in the vicinity of Sogod,
Tablas Island, Romblon after having been hit by strong winds and tidal waves brought about by
tropical typhoon "Nitang" Later that same day, petitioner Almendras Mining Corporation
("Almendras"), owner of the vessel, executed and filed the corresponding Marine Protest. Public
respondent Commission, through the Insurance Commissioner, issued a Resolution 6 ordering the
dismissal of petitioner Almendras’ complaint. Hence, a Certiorari was filed after its Motion for
Consideration was denied.

Public respondent Commission raised the argument that the present Petition for Certiorari is
improperly filed, that appeal to the Secretary of Finance from public respondent Commission’s
disputed Resolution and Order is the proper recourse for petitioner under the facts and
circumstances of this case

Issue:

Does this Court have jurisdiction over the subject matter?

Ruling:

No. The Court has no jurisdiction to try and decide the instant Petition.

The provisions of the Insurance Code (Presidential Decree No. 1460), as amended, clearly
indicate that the Office of the Insurance Commission is an administrative agency vested with
regulatory power as well as with adjudicatory authority. Among the several regulatory or non-quasi
judicial duties of the Insurance Commissioner under the Insurance Code is the authority to issue, or
refuse issuance of, a Certificate of Authority to a person or entity desirous of engaging in insurance
business in the Philippines, and to revoke or suspend such Certificate of Authority upon a finding of
the existence of statutory grounds for such revocation or suspension.

Section 414 of the Insurance Code specifies the Secretary of Finance as to the authority to
which a decision of the Insurance Commissioner rendered in the exercise of its regulatory function
may be appealed. The adjudicatory authority of the Insurance Commissioner as generally described
in Section 416 of the Insurance Code, as amended, covers all claims and complaints involving any
loss, damage or liability for which an insurer may be answerable under any kind of policy or contract
of insurance.

Section 416 (as amended by B.P. Blg. 874) also specifies the authority to which appeal may
be taken from a final order or decision of the Commissioner given in the exercise of his adjudicatory
or quasi-judicial power. It may be noted that under Section 9 (3) of B.P. Blg. 129, appeals from a final
decision of the Insurance Commissioner rendered in the exercise of his adjudicatory authority now
fall within the exclusive appellate jurisdiction of the Court of Appeals.

14
Ju r isd ict io n of t h e In su ra nc e Co m miss io n ; A d mi ni st ra tiv e a nd A dj ud ica t or y P o w er s

The Court observes, however, that both parties had agreed at the 23 August 1985 hearing
before the Insurance Commissioner to submit the case for resolution on the sole issue of whether or
not revocation or suspension of private respondent Bankers’ Certificate of Authority to engage in
insurance business was justified. The scope of the issues involved having been so limited, the
Insurance Commissioner was left with the task of determining whether or not private respondent
Bankers was guilty of an act or acts constituting a statutory ground for revocation or suspension of
its Certificate of Authority. Clearly, therefore, the Insurance Commissioner’s disputed Resolution and
Order was issued in the performance of administrative and regulatory duties and functions and
should have been appealed by petitioner to the Office of the Secretary of Finance.

15
Ju r isd ict io n of t h e In su ra nc e Co m miss io n ; A d mi ni st ra tiv e a nd A dj ud ica t or y P o w er s

PHILAM v. ARNALDO
G.R. No. 76452
July 26, 1994

Quiason, J.

Facts:

One Ramon Paterno complained about the unfair practices committed by the company
against its agents, employees and consumers. The Commissioner called for a hearing where Paterno
was required to specify which acts were illegal. Paterno then specified that the fees and charges
stated in the Contract of Agency between Philam and its agents be declared void. Philam, on the other
hand, averred that there Paterno must submit a verified formal complaint and that his letter didn’t
contain information Philam was seeking from him. Philam then questioned the Insurance
Commission’s jurisdiction over the matter and submitted a motion to quash. The commissioner
denied this. Hence this petition.

Issue:

Does the resolution of the legality of the Contract of Agency falls within the jurisdiction of the
Insurance Commissioner?

Ruling:

No. According to the Insurance code, the Insurance Commissioner was authorized to suspend,
directors, officers, and agents of insurance companies. In general, he was tasked to regulate the
insurance business, which includes: (2) The term "doing an insurance business" or "transacting an
insurance business," within the meaning of this Code, shall include (a) making or proposing to make,
as insurer, any insurance contract; (b) making, or proposing to make, as surety, any contract of
suretyship as a vocation and not as merely incidental to any other legitimate business or activity of
the surety; (c) doing any kind of business, including a reinsurance business, specifically recognized
as constituting the doing of an insurance business within the meaning of this Code; (d) doing or
proposing to do any business in substance equivalent to any of the foregoing in a manner designed
to evade the provisions of this Code.

The contract of agency between Philamlife and its agents wasn’t included with the
Commissoner’s power to regulate the business. Hence, the Insurance commissioner wasn’t vested
with jurisidiction under the rule “expresio unius est exclusion alterius”.

The respondent contended that the commissioner had the quasi-judicial power to adjudicate
under Section 416 of the Code. This was, however, regarding complaints filed by the insured against
the Insurance Company. Also, the insurance code only discusses the licensing requirements for
agents and brokers. The Insurance Code does not have provisions governing the relations between
insurance companies and their agents.

Last, in the case of Investment Planning Corporation of the Philippines v. Social Security
Commission, the Court held “that an insurance company may have two classes of agents who sell its
insurance policies: (1) salaried employees who keep definite hours and work under the control and
supervision of the company; and (2) registered representatives, who work on commission basis.”The

16
Ju r isd ict io n of t h e In su ra nc e Co m miss io n ; A d mi ni st ra tiv e a nd A dj ud ica t or y P o w er s

agents under the 2nd sentence are governed by the Civil Code laws on agency. This means that the
regular courts have jurisdiction over this category.

17
Ju r isd ict io n of t h e In su ra nc e Co m miss io n ; A d mi ni st ra tiv e a nd A dj ud ica t or y P o w er s

REPUBLIC v. DEL MONTE


G.R. No. 156956
October 9, 2006

Panganiban, C.J.

Facts:

Vilfran Liner lost in a case against Del Monte Motors. They were made to pay 11 million pesos
for service contracts with Del Monte, and such was sourced from the counterbond posted by Vilfran.
CISCO issued the counterbond. CISCO opposed but was rebuffed. The RTC released a motion for
execution commanding the sheriff to levy the amount on the property of CISCO. To completely satisfy
the amount, the Insurance Commissioner was also commanded to withdraw the security deposit filed
by CISCO with the Commission according to Sec 203 of the Insurance Code.

Insurance Commissioner Malinis was ordered by the RTC to withdraw the security bond of
CISCO for the payment of the insurance indemnity won by Del Monte Motor against Vilfran Liner, the
insured. Malinis didn’t obey the order, so the respondent moved to cite him in contempt of Court. The
RTC ruled against Malinis because he didn’t have legal basis.

Issues:

1. May the security deposit held by the Insurance Commissioner pursuant to Section 203 of the
Insurance Code be levied or garnished in favor of only one insured?
2. Does Insurance Commissioner have power to withhold the release of the security deposit?

Ruling:

1. No. Section 203 of the Insurance Code provides that no judgment creditor or other
claimant shall have the right to levy upon any of the securities of the insurer held on deposit pursuant
to the requirement of the Commissioner.

The court also claimed that the security deposit shall be (1) answerable for all the obligations
of the depositing insurer under its insurance contracts; (2) at all times free from any liens or
encumbrance; and (3) exempt from levy by any claimant.

To allow the garnishment of that deposit would impair the fund by decreasing it to less than
the percentage of paid-up capital that the law requires to be maintained. Further, this move would
create, in favor of respondent, a preference of credit over the other policy holders and beneficiaries.
Also, the securities are held as a contingency fund to answer for the claims against the insurance
company by all its policy holders and their beneficiaries. This step is taken in the event that the
company becomes insolvent or otherwise unable to satisfy the claims against it. Thus, a single
claimant may not lay stake on the securities to the exclusion of all others. The other parties may have
their own claims against the insurance company under other insurance contracts it has entered into.

2. Yes. The Insurance Code has vested the Office of the Insurance Commission with both
regulatory and adjudicatory authority over insurance matters.

18
Ju r isd ict io n of t h e In su ra nc e Co m miss io n ; A d mi ni st ra tiv e a nd A dj ud ica t or y P o w er s

Under Section 414 of the Insurance Code, the Commissioner may issue such rulings,
instructions, circulars, orders and decisions as he may deem necessary to secure the enforcement of
the provisions of this Code.

Included here is the duty to hold security deposits under Sections 191 and 202 of the Code
for the benefit of policy holders. Section 192, on the other hand, states “the securities deposited as
aforesaid shall be returned upon the company's making application therefor and proving to the
satisfaction of the Commissioner that it has no further liability under any of its policies in the
Philippines. He has been given great discretion to regulate the business to protect the public. Lastly,
an implied trust is created by the law for the benefit of all claimants under subsisting insurance
contracts issued by the insurance company. He believed that the security deposit was exempt from
execution to protect the policy holders.

19
Ju r isd ict io n of t h e In su ra nc e Co m miss io n ; A d mi ni st ra tiv e a nd A dj ud ica t or y P o w er s

MALAYAN v. CONCEPCION
G.R. No. 207277
January 16, 2017

Del Castillo, J.

Facts:

Lin filed a complaint for Collection of Sum of Money with damages against Malayan. She
alleged that she obtain various loans from RCBC secured by 6 clustered warehouses at Bulacan. The
5 warehouses were insured with Malayan for Php 56 Million and the other house for Php 2 Million.
Later on, the warehouses were burned. The BFP issued a Fire Clearance Certificate, after certifying
that the fire was accidental. Despite such, Malayan denied the claim of Lin stating that the forensic
investigators hired by Malayan concluded that the fire was caused by arson and not accidental.

In this administrative case, Lin claimed that since it had been conclusively found that the
cause of the fire was "accidental," the only issue left to be resolved is whether Malayan should be held
liable for unfair claim settlement practice under Section 241 in relation to Section 247 of the
Insurance Code due to its unjustified refusal to settle her claim; and that in consequence of the
foregoing failings, Malayan's license to operate as a non-life insurance company should be revoked
or suspended, until such time that it fully complies with the IC Resolution ordering it to accord more
weight to the BFP's findings.

On August 17, 2010, Malayan filed a motion to dismiss Civil Case No. 10-122738 based on
forum shopping. It argued that the administrative case was instituted to prompt or incite IC into
ordering Malayan to pay her insurance claim; that the elements of forum shopping are present in
these two cases because there exists identity of parties since Malayan's individual officers who were
impleaded in the civil case are also involved in the administrative case; that the same interests are
shared and represented in both the civil and administrative cases; that there is identity of causes of
action and reliefs sought in the two cases since the administrative case is merely disguised as an
unfair claim settlement charge, although its real purpose is to allow Lin to recover her insurance
claim from Malayan; that Lin sought to obtain the same reliefs in the administrative case as in the
civil case; that Lin did not comply with her sworn undertaking in the Certification on Non-Forum
Shopping which she attached to the civil case, because she deliberately failed to notify the RTC about
the pending administrative case within five days from the filing thereof.

Issue:

Can claimants file both civil and administrative case against the insurer?

Ruling:

Yes. The settled rule is that criminal and civil cases are altogether different from
administrative matters, such that the disposition in the first two will not inevitably govern the third
and vice versa.

In the context of the case at bar, matters handled by the IC are delineated as either regulatory
or adjudicatory, both of which have distinct characteristics. The provisions of the Insurance Code, as
amended, clearly indicate that the Office of the Insurance Commission is an administrative agency
vested with regulatory power as well as with adjudicatory authority. Go v. Office of the Ombudsman

20
Ju r isd ict io n of t h e In su ra nc e Co m miss io n ; A d mi ni st ra tiv e a nd A dj ud ica t or y P o w er s

reiterated the above-stated distinctions vis-a-vis the principles enunciating that a civil case before
the trial court involving recovery of payment of the insured's insurance claim plus damages, can
proceed simultaneously with an administrative case before the IC.

21
Po w e rs o f th e I ns u ra n c e Co m mis si on

FINMAN v. INOCENCIO
G.R. No. 90273-75
November 15, 1989

Feliciano, J.

Facts:

Pan Pacific Overseas is a recruitment agency which offers jobs abroad duly registered with
the POEA. Finman General is acting as Pan Pacific’s surety, as required by POEA rules and Art. 31 of
the Labor Code. Pan Pacific was sued by William Inocencio and 3 others for alleged violation of Article
32 and 34 of the Labor Code. Inocencio alleged that Pan Pacific charged and collected fees but failed
to provide employment abroad.

POEA ruled in favor of Inocencio et al and had impleaded Finman in the complaint as well.
The Labor Secretary affirmed POEA’s ruling. Finman General asserts that it should not be impleaded
in the case because it is not a party to the contract between Pan Pacific and Inocencio et al.

Issue:

Is Finman General solidarily liable?

Ruling:

Yes. Since Pan Pacific had thoughtfully refrained from notifying the POEA of its new address
and from responding to the complaints, petitioner Finman may well be regarded as an indispensable
party to the proceedings before the POEA. Whether Finman was an indispensable or merely a proper
party to the proceedings, the SC held that the POEA could properly implead it as party respondent
either upon the request of Inocencio et al or motu propio. Such is the situation under the Revised
Rules of Court.

Finman General is solidarily liable. Under Section 176 of the Insurance Code, as amended, the
liability of a surety in a surety bond, in the case, Finman is joint and several with the principal obligor,
which is Pan Pacific.

22
Co nt ra c t of I ns u ra n c e

MAYER STEEL v. CA
G.R. No. 124050
June 19, 1997

Puno, J.

Facts:

Hong Kong Government Supplies Department contracted Mayer Steel Pipe Corporation to
manufacture and supply various steel pipes and fittings. Prior to the shipping, Mayer insured these
pipes and fittings against all risks with South Sea Surety and Insurance Co., Inc. and Charter Insurance
Corp., with Industrial Inspection Inc. appointed as third-party inspector. After examining the pipes
and fittings, Industrial Inspection certified that they are in good order condition. However, when the
goods reached Hong Kong, it was discovered that a substantial portion thereof was damaged. The
trial court found in favor of the insured. However, when the case was elevated to the CA, it set aside
the decision of the trial court and dismissed the complaint on the ground of prescription. It held that
the action was barred under Sec. 3(6) of the COGSA since it was filed only on April 17, 1986, more
than two years from the time the goods were unloaded from the vessel.

Issue:

Is the action barred by prescription?

Ruling:

Sec. 3(6) of the COGSA states that the carrier and the ship shall be discharged from all liability
for loss or damage to the goods if no suit is filed within one year after delivery of the goods or the
date when they should have been delivered.

Under this provision, only the carrier’s liability is extinguished if no suit is brought within one
year. But the liability of the insurer is not extinguished because the insurer’s liability is based not on
the contract of carriage but on the contract of insurance.

An insurance contract is a contract whereby one party, for a consideration known as the
premium, agrees to indemnify another for loss or damage which he may suffer from a specified peril.
An “all risks” insurance policy covers all kinds of loss other than those due to willful and fraudulent
act of the insured. Thus, when private respondents issued the “all risks” policies to Mayer, they bound
themselves to indemnify the latter in case of loss or damage to the goods insured. Such obligation
prescribes in ten years, in accordance with Article 1144 of the New Civil Code.

23
Do in g a n I ns u ra nc e b us in es s o r t ra nsa ct i ng a n Ins u ra nc e bu si n ess

PHILHEALTH v. CIR
G.R. No. 167330
June 12, 2008

Corona, J.

Facts:

The petitioner, a prepaid health-care organization offering benefits to its members. The CIR
found that the organization had a deficiency in the payment of the DST under Section 185 of the 1997
Tax Code which stipulated its implementation:
“On all policies of insurance or bonds or obligations of the nature of indemnity for
loss, damage, or liability made or renewed by any person, association or company or
corporation transacting the business of accident, fidelity, employer's liability, plate,
glass, steam boiler, burglar, elevator, automatic sprinkler, or other branch of
insurance”.

The CIR sent a demand for the payment of deficiency taxes, including surcharges and interest,
for 1996-1997. The petitioner protested to the CIR, but it didn’t act on the appeal. Hence, the company
had to go to the CTA. The latter declared judgment against them and reduced the taxes. It ordered
them to pay 22 million pesos for deficiency VAT for 1997 and 31 million deficiency VAT for 1996. CA
denied the company’s appeal and increased taxes to 55 and 68 million for 1996 to 1997.

Issues:

Is Philhealth, a health maintenance organization, engaged in the business of insurance?

Ruling:

No. The basic distinction between medical service corporations and ordinary health and
accident insurers is that the former undertake to provide prepaid medical services through
participating physicians, thus relieving subscribers of any further financial burden, while the latter
only undertake to indemnify an insured for medical expenses up to, but not beyond, the schedule of
rates contained in the policy. A participating provider of health care services is one who agrees in
writing to render health care services to or for persons covered by a contract issued by health service
corporation in return for which the health service corporation agrees to make payment directly to
the participating provider.

Any indemnification resulting from the payment for services rendered in case of emergency
by non-participating health providers would still be incidental to petitioner’s purpose of providing
and arranging for health care services and does not transform it into an insurer.

Its obligation to maintain the good health of its members its undertaking under its
agreements is not to indemnify its members against any loss or damage arising from a medical
condition but, on the contrary, to provide the health and medical services needed to prevent such
loss or damage.

24
Sy na l l a g ma tic C on t ra c t

UCPB v. MASAGANA TELAMART


G.R. No. 137172
April 4, 2001

David, Jr., J.

Facts:

Masagana obtained from UCPB 5 insurance policies on its properties. On June 13, 1992,
plaintiffs’ properties were razed by fire. On July 13, 1992, plaintiff tendered, and defendant accepted,
5 Equitable Bank Manager's Checks in the total amount of P225,753.45 as renewal premium
payments for which Official Receipt Direct Premium No. was issued by defendant. On the same day,
defendant returned the 5 manager's checks stating in its letter that it was rejecting Masagana's claim
on the following grounds: a) Said policies expired last May 22, 1992 and were not renewed for
another term; b) Defendant had put plaintiff and its alleged broker on notice of non-renewal earlier;
and c) The properties covered by the said policies were burned in a fire that took place last June 13,
1992, or before tender of premium payment. Both the Court of Appeals and the trial court found that
sufficient proof exists that Respondent, which had procured insurance coverage from Petitioner for
a number of years, had been granted a 60 to 90-day credit term for the renewal of the policies. As
stated in the insurance policy: 26. Renewal Clause. — Unless the company at least forty five days in
advance of the end of the policy period mails or delivers to the assured at the address shown in the
policy notice of its intention not to renew the policy or to condition its renewal upon reduction of
limits or elimination of coverages, the assured shall be entitled to renew the policy upon payment of
the premium due on the effective date of renewal.

Issue:

Are the fire insurance policies issued by petitioner to the respondent extended or renewed
by an implied credit arrangement though actual payment of premium was tendered on a later date
and after the occurrence of the risk insured against?

Ruling:

Yes. Tuscany has provided a fourth exception to Section 77, namely, that the insurer may
grant credit extension for the payment of the premium. This simply means that if the insurer has
granted the insured a credit term for the payment of the premium and loss occurs before the
expiration of the term, recovery on the policy should be allowed even though the premium is paid
after the loss but within the credit term. Moreover, there is nothing in Section 77 which prohibits the
parties in an insurance contract to provide a credit term within which to pay the premiums. That
agreement is not against the law, morals, good customs, public order or public policy. The agreement
binds the parties. Article 1306 of the Civil Code provides: ARTICLE 1306. The contracting parties may
establish such stipulations clauses, terms and conditions as they may deem convenient, provided
they are not contrary to law, morals, good customs, public order, or public policy. Finally, in the
instant case, it would be unjust and inequitable if recovery on the policy would not be permitted
against Petitioner, which had consistently granted a 60- to 90-day credit term for the payment of
premiums despite its full awareness of Section 77. Estoppel bars it from taking refuge under said
Section, since Respondent relied in good faith on such practice. Estoppel then is the fifth exception to
Section 77.

25
Ki n ds o f I nsu ra nc e Co n tra c t

SMB v. LAW UNION


G.R. No. L-14300
January 19, 1920

Street, J.

Facts:

On January 12, 1916, D. P. Dunn, then the owner of the property to which the insurance
relates, mortgaged the same to the San Miguel Brewery to secure a debt. In the contract of mortgage
Dunn agreed to keep the property insured at his expense to the full amount of its value in companies
to be selected by the Brewery Company and authorized the latter in case of loss to receive the
proceeds of the insurance and to retain such part as might be necessary to cover the mortgage debt.
At the same time, Dunn authorized and requested the Brewery Company to effect said insurance
itself. Accordingly, on the same date Antonio Brias, general manager of the Brewery, made a verbal
application to the Law Union and Rock Insurance Company for insurance to said property. In reply
to a question of the company's agent as to whether the Brewery was the owner of the property, he
stated that the company was interested only as a mortgage creditor. No information was asked as to
who was the owner of the property, and no information upon this point was given. Insurance
company issued its own policy and procured a policy in a like amount to be issued by the "Filipinas"
Compania de Seguros. Both policies were issued in the name of the San Miguel Brewery as the
assured, and contained no reference to any other interest in the property. Both policies contain the
usual clause requiring assignments to be approved and noted on the policy. The premiums were paid
by the Brewery and charged to Dunn. A year later the policies were renewed, without change. Dunn
sold the insured property to the defendant Henry Harding, but not assignment of the insurance, or of
the insurance policies, was at any time made to him. When the property was destroyed, Law Union
and Rock Insurance Co., and Filipinas Compania settled with San Miguel the extent of the mortgage
credit. The lower court ruled that Law Union and Rock Insurance Co., and Filipinas Compania are
absolved from the difference; and that no cause of action by Henry Harding against the insurance
companies was shown. He is not a party to the contracts of insurance and cannot directly maintain
an action thereon.

Issues:

1. Does San Miguel possess insurable interest as mortgage creditor to the extent of the mortgage
credit?
2. Whether Henry Harding has also insurable interest over the subject matter.

Rulings:

As to San Miguel Brewery, it has insurable interest. The two insurance companies admitted
in effect their liability to the San Miguel Brewery to the extent of its mortgage credit, but denying
liability to Harding on the ground that under the contracts of insurance the liability of the insurance
companies was limited to the insurable interest of San Miguel.
As to Henry Harding, none. These conclusions are not only deducible from the principles governing
the operation and effect of insurance contracts in general but the point is clearly covered by the
express provisions of sections 16 and 50 of the Insurance Act (Act No. 2427). In the first of the
sections cited, it is declared that "the measure of an insurable interest in property is the extent to
which the insured might be damnified by loss or injury thereof" (sec. 16); while in the other it is
stated that "the insurance shall be applied exclusively to the proper interest of the person in whose

26
Ki n ds o f I nsu ra nc e Co n tra c t

name it is made unless otherwise specified in the policy" These provisions would have been fatal to
any attempt at recovery even by D. P. Dunn, if the ownership of the property had continued in him
up to the time of the loss; and as regards Harding, an additional insuperable obstacle is found in the
fact that the ownership of the property had been charged, prior to the loss, without any
corresponding change having been effected in the policy of insurance. In section 19 of the Insurance
Act we find it stated that "a change of interest in any part of a thing insured unaccompanied by a
corresponding change of interest in the insurance, suspends the insurance to an equivalent extent,
until the interest in the thing and the interest in the insurance are vested in the same person." Again
in section 55 it is declared that "the mere transfer of a thing insured does not transfer the policy, but
suspends it until the same person becomes the owner of both the policy and the thing insured."

27
Ki n ds o f I nsu ra nc e Co n tra c t

PALILEO v. COSIO
G.R. No. L-7667
November 28, 1955

Bautista Angelo, J.

Facts:

Plaintiff obtained from defendant a loan. To secure the payment of the aforesaid loan,
defendant required plaintiff to sign a document known as “Conditional Sale of Residential Building”,
purporting to convey to defendant, with right to repurchase, a two-story building of strong materials
belonging to plaintiff. This document did not express the true intention of the parties which was
merely to place said property as security for the payment of the loan. After the execution of the
aforesaid document, defendant insured the building against fire, the insurance policy having been
issued in the name of defendant. The building was partly destroyed by fire and, after proper demand,
defendant collected from the insurance company an indemnity. Plaintiff demanded from defendant
that she be credited with the necessary amount to pay her obligation out of the insurance proceeds
but defendant refused to do so.

Issue:

Is a mortgagor entitled to the insurance proceeds of the mortgaged property independently


insured by the mortgagee? What is the effect of the insurance?

Ruling:

No. The rule is that “where a mortgagee, independently of the mortgagor, insures the
mortgaged property in his own name and for his own interest, he is entitled to the insurance proceeds
in case of loss, but in such case, he is not allowed to retain his claim against the mortgagor, but is
passed by subrogation to the insurer to the extent of the money paid.” Or, stated in another way, “the
mortgagee may insure his interest in the property independently of the mortgagor. In that event,
upon the destruction of the property the insurance money paid to the mortgagee will not inure to the
benefit of the mortgagor, and the amount due under the mortgage debt remains unchanged. The
mortgagee, however, is not allowed to retain his claim against the mortgagor, but it passes by
subrogation to the insurer, to the extent of the insurance money paid.”

28
Ki n ds o f I nsu ra nc e Co n tra c t

SAURA IMPORT v. PHIL INTERNATIONAL


G.R. No. L-15184
May 31, 1963

Paredes, J.

Facts:

The petitioner Saura Import & Export Co., Inc. mortgaged to PNB on December 26, 1952 its
registered parcel of land in Davao to secure the payment of a promissory note. A building owned by
petitioner was erected on the parcel of land and the building had always been covered by insurance
even before the execution of the mortgage contract. The mortgage agreement required petitioner to
insure the mortgaged property at all times against fire and earthquake for an amount and with such
company satisfactory to the mortgagee. It obtained a fire insurance from respondent, PISC for a
period of 1 year starting October 2, 1954. It also required petitioner to endorse the insurance policy
to PNB in a memo which states that “loss if any, shall be payable to PNB as their interest may appear,
subject to the terms and conditions and warranties of this policy”. The policy was delivered to PNB
by petitioner.

On October 15, 1954, about 13 days after the issuance of the policy, PISC canceled the same,
effective as of the date of issue. Notice of cancellation was sent to PNB in writing through registered
mail. On April 6, 1955 the building and its contents were burned. On April 11, 1985, petitioner filed
a claim with PISC and mortgagee bank. Upon presentation of notice of loss with PNB, petitioner
learned for the first time that the policy had been previously cancelled by PISC, when petitioner’s
folder in the bank’s file was opened and the notice of the cancellation by PISC was found.

Issue:

Whether or not there was proper cancellation of the policy

Ruling:

No. The court held that the policy in question does not provide for the notice of cancellation,
its form or period. The insurance law, Act No. 2427, does not likewise provide for such notice. It
devotes upon the court to apply the generally accepted principles of insurance, regarding
cancellation of the insurance policy by the insurer. Actual notice of cancellation in a clear and
unequivocal manner, preferably in writing should be given by the insurer to the insured so that the
later might be given an opportunity to obtain other insurance for his own protection. The notice
should be personal to the insured and not to and/or through any unauthorized person by the policy.
Both PISC and the PNB failed, wittingly or unwittingly to notify petitioner of the cancellation made.
Notice to the bank, as far as petitioner herein is concerned, is not effective notice.

29
Ki n ds o f I nsu ra nc e Co n tra c t

PINEDA v. CA
G.R. No. 105562
September 27, 1993

Davide, Jr., J.

Facts:

Prime Marine Services procured Group PoIicy to provide life insurance coverage to its sea-
based employees enrolled under the plan. On 17 February 1986, during the effectivity of the policy,
6 covered employees of the PMSI perished at sea when their vessel sunk somewhere in El Jadida,
Morocco. They were survived by complainants, the beneficiaries under the policy. Complainants
sought to claim death benefits due them and they approached the President and General Manager of
PMSI, Capt. Nuval. They were thus made to execute, with the exception of the spouses Alarcon, special
powers of attorney authorizing Capt. Nuval to, among others, "follow up, ask, demand, collect and
receive" for their benefit indemnities of sums of money due them relative to the sinking of M/V
Nemos. By virtue of these written powers of attorney, complainants were able to receive their
respective death benefits. Unknown to them, the PMSI, in its capacity as employer and policyholder
of the life insurance of its deceased workers, filed with respondent formal claims for and in behalf of
the beneficiaries, through its President, Capt. Nuval. Among the documents submitted by the latter
for the processing of the claims were five SPAs executed by complainants. On the basis of these and
other documents duly submitted, respondent drew against its account with the BPI six checks, four
for P200,00.00 each, one for P50,000.00 and another for P40,00.00, payable to the order of
complainants. These checks were released to the treasurer of PMSI upon instructions of Nuval over
the phone to Mr. Urbano. Nuval, upon receipt of these checks from the treasurer, who happened to
be his son-in-law, endorsed and deposited them in his account with the Commercial Bank of Manila.
After complainants learned that they were entitled, as beneficiaries, to life insurance benefits under
a group policy with respondent-appellant, they sought to recover these benefits from Insular Life but
the latter denied their claim on the ground that the liability to complainants was already extinguished
upon delivery to and receipt by PMSI of the 6 checks issued in their names. An administrative case
was filed against Insular Life. The Insurance Commission ruled in their favor but the appellate court
reversed the same. Hence, petition for certiorari.

Issue:

Is a Special Power of Attorney to the employer necessary for the latter to claim proceeds in
behalf of the beneficiaries?

Ruling:

Yes. The coverage terms for group insurance are usually stated in a master agreement or
policy that is issued by the insurer to a representative of the group or to an administrator of the
insurance program, such as an employer. Although the employer may be the titular or named insured,
the insurance is actually related to the life and health of the employee. Indeed, the employee is in the
position of a real party to the master policy, and even in a non-contributory plan, the payment by the
employer of the entire premium is a part of the total compensation paid for the services of the
employee. Put differently, the labor of the employees is the true source of the benefits, which are a
form of additional compensation to them. The employer is the agent of the insurer in performing the
duties of administering group insurance policies. It cannot be said that the employer acts entirely for
its benefit or for the benefit of its employees in undertaking administrative functions. While a

30
Ki n ds o f I nsu ra nc e Co n tra c t

reduced premium may result if the employer relieves the insurer of these tasks, and this, of course,
is advantageous to both the employer and the employees, the insurer also enjoys significant
advantages from the arrangement. The reduction in the premium which results from employer-
administration permits the insurer to realize a larger volume of sales, and at the same time the
insurer's own administrative costs are markedly reduced. Insular Life, however, likewise recognized
Capt. Nuval as the attorney-in-fact of the petitioners. Unfortunately, through its official, Mr. Urbano,
it acted imprudently and negligently in the premises by relying without question on the special
power of attorney.

31
Ki n ds o f I nsu ra nc e Co n tra c t

PHILAM LIFE v. ANSALDO


G.R. No. 76452
July 26, 1994

Quiason, J.

Facts:

One Ramon Paterno complained about the unfair practices committed by the company
against its agents, employees and consumers. The Commissioner called for a hearing where Paterno
was required to specify which acts were illegal. Paterno then specified that the fees and charges
stated in the Contract of Agency between Philam and its agents be declared void. Philam, on the other
hand, averred that there Paterno must submit a verified formal complaint and that his letter didn’t
contain information Philam was seeking from him. Philam then questioned the Insurance
Commission’s jurisdiction over the matter and submitted a motion to quash. The commissioner
denied this. Hence this petition.

Issue:

Does the resolution of the legality of the Contract of Agency fall within the jurisdiction of the
Insurance Commissioner?

Ruling:

No. According to the Insurance code, the Insurance Commissioner was authorized to suspend,
directors, officers, and agents of insurance companies. In general, he was tasked to regulate the
insurance business. The contract of agency between Philamlife and its agents wasn’t included with
the Commissoner’s power to regulate the business. Hence, the Insurance commissioner wasn’t vested
with jurisidiction under the rule “expresio unius est exclusion alterius”. The insurance code only
discusses the licensing requirements for agents and brokers. The Insurance Code does not have
provisions governing the relations between insurance companies and their agents.

32
Ki n ds o f I nsu ra nc e Co n tra c t

WHITEGOLD v. PIONEER INSURANCE


G.R. No. 154514
July 28, 2005

Quisumbing, J.

Facts:

White Gold Marine Services, Inc. owns several shipping vessels. Steamship Mutual
Underwriting Association, Ltd. (based in Bermuda) is a protection and indemnity club which is an
association composed of shipowners in general who band together for the specific purpose of
providing insurance cover on a mutual basis against liabilities incidental to shipowning that the
members incur in favor of third parties. White Gold, through Pioneer Insurance (agent of Steamship
Mutual here), procured a protection and indemnity coverage from Steamship Mutual. Steamship
Mutual does not have authority from the Insurance Commission to conduct insurance business in the
Philippines but its collection agent here (Pioneer Insurance) has been licensed to conduct insurance
business.

Later, Steamship Mutual filed a case for collection of sum of money against White Gold due to
the latter’s failure to pay its balance with the former. White Gold averred that Steamship Mutual has
no license [hence it cannot collect]. Nor can it collect through Pioneer Insurance because, though
Pioneer Insurance is licensed as an insurance company, it is not licensed to be an insurance
broker/agent. Steamship Mutual insisted it is not conducting insurance business here and is merely
a protection and indemnity club. The Insurance Commission as well as the Court of Appeals ruled
against White Gold.

Issue:

Is Steamship required to obtain a license to operate business?

Ruling:

Yes. The test to determine if a contract is an insurance contract or not, depends on the nature
of the promise, the act required to be performed, and the exact nature of the agreement in the light
of the occurrence, contingency, or circumstances under which the performance becomes requisite. It
is not by what it is called. If it is a contract of indemnity, it must be a contract of insurance. In fact, a
protection and indemnity club is a form of insurance where the members are both the insurers and
the insured. It is a mutual insurance company. The club indemnifies the member for whatever risks
it may incur against a third party where the third party is other than the club and the members.
Hence, Steamship Mutual needs to procure a license from the Insurance Commission in order to
continue operating here.

Pioneer Insurance also needs to secure another license as an insurance broker/agent of


Steamship Mutual pursuant to Section 299 of the Insurance Code.

33
Ki n ds o f I nsu ra nc e Co n tra c t

GULF RESORT v. PCIC


G.R. No. 156167
May 16, 2005

Puno, J.

Facts:

Gulf Resorts, Inc at Agoo, La Union was insured with American Home Assurance Company
which includes loss or damage to shock to any of the property insured by this Policy occasioned by
or through or in consequence of earthquake. On July 16, 1990, an earthquake struck Central Luzon
and Northern Luzon so the properties and 2 swimming pools in its Agoo Playa Resort were damaged.
On August 23, 1990, Gulf's claim was denied on the ground that its insurance policy only afforded
earthquake shock coverage to the two swimming pools of the resort. Petitioner contends that
pursuant to this rider, no qualifications were placed on the scope of the earthquake shock
coverage. Thus, the policy extended earthquake shock coverage to all of the insured properties. The
RTC favored American Home stating that endorsement rider means that only the two swimming
pools were insured against earthquake shock. The CA later affirmed the RTC.

Issue:

Can petitioner claim for its properties?

Ruling:

Yes. It is basic that all the provisions of the insurance policy should be examined and
interpreted in consonance with each other. All its parts are reflective of the true intent of the parties.
An insurance premium is the consideration paid an insurer for undertaking to indemnify the insured
against a specified peril. In the subject policy, no premium payments were made with regard to
earthquake shock coverage, except on the two swimming pools.

34
Ki n ds o f I nsu ra nc e Co n tra c t

HEIRS OF MARAMAG v. MARAMAG


G.R. No. 181132
5 June 2009
Nachura, J.

Facts:

Loreto Maramag designated as beneficiary his concubine Eva de Guzman Maramag. Vicenta
Maramag and Odessa, Karl Brian, and Trisha Angelie, the heirs of Loreto Maramag, and his
concubine Eva de Guzman Maramag, who was a suspect in the killing of Loreto and his illegitimate
children are claiming for his insurance. Vicenta alleges that Eva is disqualified from claiming. Further,
it was claimed that the illegitimate children of Loreto—Odessa, Karl Brian, and Trisha Angelie—were
entitled only to one-half of the legitime of the legitimate children, thus, the proceeds released to
Odessa and those to be released to Karl Brian and Trisha Angelie were inofficious and should be
reduced; and petitioners could not be deprived of their legitimes, which should be satisfied first.

Issue:

Can illegitimate children be beneficiaries in an insurance contract?

Ruling:

Yes. Section 53 of the Insurance Code states that the insurance proceeds shall be applied
exclusively to the proper interest of the person in whose name or for whose benefit it is made unless
otherwise specified in the policy. Pursuant thereto, it is obvious that the only persons entitled to
claim the insurance proceeds are either the insured, if still alive; or the beneficiary, if the insured is
already deceased, upon the maturation of the policy. The exception to this rule is a situation where
the insurance contract was intended to benefit third persons who are not parties to the same in the
form of favorable stipulations or indemnity. In such a case, third parties may directly sue and claim
from the insurer.

Petitioners are third parties to the insurance contracts with Insular and Grepalife and, thus,
are not entitled to the proceeds thereof. Accordingly, respondents Insular and Grepalife have no legal
obligation to turn over the insurance proceeds to petitioners. The revocation of Eva as a beneficiary
in one policy and her disqualification as such in another are of no moment considering that the
designation of the illegitimate children as beneficiaries in Loreto’s insurance policies remains valid.
Because no legal proscription exists in naming as beneficiaries the children of illicit relationships by
the insured, the shares of Eva in the insurance proceeds, whether forfeited by the court in view of the
prohibition on donations under Article 739 of the Civil Code or by the insurers themselves for reasons
based on the insurance contracts, must be awarded to the said illegitimate children, the designated
beneficiaries, to the exclusion of petitioners. It is only in cases where the insured has not designated
any beneficiary, or when the designated beneficiary is disqualified by law to receive the proceeds,
that the insurance policy proceeds shall redound to the benefit of the estate of the insured.

35
Ki n ds o f I nsu ra nc e Co n tra c t

MANILA BANKERS v. ABAN


G.R. No. 175666
July 29, 2013

Del Castillo, J.

Facts:

Delia Sotero, designated the Respondent, her niece, as her beneficiary for a life insurance
policy from Petitioner, which was issued on August 30, 1993, with a face value of ₱100,000.00. On
April 10, 1996, Sotero died and respondent filed a claim for the insurance proceeds on July, 1996.
Upon investigation, petitioner found that Sotero was illiterate, that she did not sign the application,
nor did she have the financial capability to pay the premiums and she was sickly since 1990.
Respondent was the one who filed the application and designated herself as beneficiary. Petitioner
denied respondent’s claims and refunded the premiums paid. Petitioner filed a civil case for
rescission and/or annulment of the policy on the ground of fraud or misrepresentation rendering it
voidable. Respondent filed a motion to dismiss claiming that petitioner’s cause of action was barred
by prescription.

Issue:

Is the insurance policy is void by reason of fraudulent concealment?

Ruling:

No. Section 48 of the Insurance Code regulates both the actions of the insurers and
prospective takers of life insurance. It gives insurers enough time to inquire whether the policy was
obtained by fraud, concealment, or misrepresentation; on the other hand, it forewarns scheming
individuals that their attempts at insurance fraud would be timely uncovered – thus deterring them
from venturing into such nefarious enterprise. At the same time, legitimate policy holders are
absolutely protected from unwarranted denial of their claims or delay in the collection of insurance
proceeds occasioned by allegations of fraud, concealment, or misrepresentation by insurers, claims
which may no longer be set up after the two-year period expires as ordained under the law. The
"incontestability clause" is a provision in law that after a policy of life insurance made payable on the
death of the insured shall have been in force during the lifetime of the insured for a period of two (2)
years from the date of its issue or of its last reinstatement, the insurer cannot prove that the policy is
void ab initio or is rescissible by reason of fraudulent concealment or misrepresentation of the
insured or his agent. As borne by the records, the policy was issued on August 30, 1993, the insured
died on April 10, 1996, and the claim was denied on April 16, 1997. The insurance policy was thus in
force for a period of 3 years, 7 months, and 24 days. Considering that the insured died after the two-
year period, the plaintiff-appellant is, therefore, barred from proving that the policy is void ab initio
by reason of the insured’s fraudulent concealment or misrepresentation or want of insurable interest
on the part of the beneficiary, herein defendant-appellee.

36
Ki n ds o f I nsu ra nc e Co n tra c t

ALPHA INSURANCE v. CASTOR


G.R. No. 198174
September 2, 2013

Peralta, J.
Facts:

Castor obtained a Motor Car Policy for her Toyota Revo DLX DSL with Alpha Insurance. The
contract of insurance obligates the petitioner to pay the respondent the amount of P630,000 in case
of loss or damage to said vehicle during the period covered. On April 16, 2007, respondent instructed
her driver, Jose Joel Salazar Lanuza to bring the vehicle to nearby auto-shop for a tune up. However,
Lanuza no longer returned the motor vehicle and despite diligent efforts to locate the same, said
efforts proved futile. Resultantly, respondent promptly reported the incident to the police and
concomitantly notified petitioner of the said loss and demanded payment of the insurance proceeds.
Alpha, however, denied the demand of Castor claiming that they are not liable since the culprit who
stole the vehicle is employed with Castor. Under the Exceptions to Section III of the Policy, the
Company shall not be liable for (4) any malicious damage caused by the insured, any member of his
family or by “A PERSON IN THE INSURED’S SERVICE”. Castor filed a Complaint for Sum of Money with
Damages against Alpha before the Regional Trial Court of Quezon City. The trial court rendered its
decision in favor of Castor which decision is affirmed in toto by the Court of Appeals. Hence, this
Petition for Review on Certiorari.

Issue:

Was the loss of respondent’s vehicle excluded under the insurance policy?

Ruling:

No. The words “loss” and “damage” mean different things in common ordinary usage. The
word “loss” refers to the act or fact of losing, or failure to keep possession, while the word “damage”
means deterioration or injury to property. Therefore, petitioner cannot exclude the loss of Castor’s
vehicle under the insurance policy under paragraph 4 of “Exceptions to Section III”, since the same
refers only to “malicious damage”, or more specifically, “injury” to the motor vehicle caused by a
person under the insured’s service. Paragraph 4 clearly does not contemplate “loss of property”. A
contract of insurance is a contract of adhesion. So, when the terms of the insurance contract contain
limitations on liability, courts should construe them in such a way as to preclude the insurer from
non-compliance with his obligation. Thus, in Eternal Gardens Memorial Park Corporation v.
Philippine American Life Insurance Company, this Court ruled that it must be remembered that an
insurance contract is a contract of adhesion which must be construed liberally in favor of the insured
and strictly against the insurer in order to safeguard the latter’s interest.

37
Ki n ds o f I nsu ra nc e Co n tra c t

JAIME T. GAISANO v. DEVELOPMENT INSURANCE AND SURETY CORPORATION


G.R. No. 190702
February 27, 2017

Jardeleza, J.

Facts:

Petitioner was the registered owner of a 1992 Mitsubishi Montero with plate number GTJ-
777, while respondent is a domestic corporation engaged in the insurance business. On September
27, 1996, respondent issued a comprehensive commercial vehicle policy to petitioner in the amount
of P1,500,000.00 over the vehicle for a period of one year commencing on September 27, 1996 up to
September 27, 1997. Respondent also issued two other commercial vehicle policies to petitioner
covering two other motor vehicles for the same period.

To collect the premiums and other charges on the policies, respondent's agent, Trans-Pacific
Underwriters Agency (Trans-Pacific), issued a statement of account to petitioner's company, Noah's
Ark Merchandising (Noah's Ark). Noah's Ark immediately processed the payments and issued a Far
East Bank check dated September 27, 1996 payable to Trans-Pacific on the same day. The check
bearing the amount of P140,893.50 represents payment for the three insurance policies, with
P55,620.60 for the premium and other charges over the vehicle. However, nobody from Trans-Pacific
picked up the check that day because its president and general manager, Rolando Herradura, was
celebrating his birthday. Trans-Pacific informed Noah's Ark that its messenger would get the check
the next day, September 28.13

In the evening of September 27, 1996, while under the official custody of Noah's Ark
marketing manager Achilles Pacquing (Pacquing) as a service company vehicle, the vehicle was
stolen in the vicinity of SM Megamall at Ortigas, Mandaluyong City. Pacquing reported the loss to the
PNP Traffic Management Command at Camp Crame in Quezon City. Despite search and retrieval
efforts, the vehicle was not recovered.

Oblivious of the incident, Trans-Pacific picked up the check. It issued an official receipt
numbered 124713 dated September 28, 1996, acknowledging the receipt of P55,620.60 for the
premium and other charges over the vehicle. The check issued to Trans--Pacific for P140,893.50 was
deposited with Metrobank for encashment on October 1, 1996.

On October 1, 1996, Pacquing informed petitioner of the vehicle's loss. Thereafter, petitioner
reported the loss and filed a claim with respondent for the insurance proceeds of P1,500,000.00.
After investigation, respondent denied petitioner's claim on the ground that there was no insurance
contract. Petitioner, through counsel, sent a final demand on July 7, 1997. Respondent, however,
refused to pay the insurance proceeds or return the premium paid on the vehicle.

On October 9, 1997, petitioner filed a complaint for collection of sum of money and damages
with the RTC where it sought to collect the insurance proceeds from respondent. In its answer,
respondent asserted that the non-payment of the premium rendered the policy ineffective. The
premium was received by the respondent only on October 2, 1996, and there was no known loss
covered by the policy to which the payment could be applied.

Issue:

38
Ki n ds o f I nsu ra nc e Co n tra c t

Whether there is a binding insurance contract between petitioner and respondent.

Ruling:

The petition was denied. Insurance is a contract whereby one undertakes for a consideration
to indemnify another against loss, damage or liability arising from an unknown or contingent event.
Just like any other contract, it requires a cause or consideration. The consideration is the premium,
which must be paid at the time and in the way and manner specified in the policy. If not so paid, the
policy will lapse and be forfeited by its own terms. The law, however, limits the parties' autonomy as
to when payment of premium may be made for the contract to take effect. The general rule in
insurance laws is that unless the premium is paid, the insurance policy is not valid and binding.
Section 77 of the Insurance Code, applicable at the time of the issuance of the policy.

Here, there is no dispute that the check was delivered to and was accepted by respondent's
agent, Trans-Pacific, only on September 28, 1996. No payment of premium had thus been made at
the time of the loss of the vehicle on September 27, 1996. While petitioner claims that Trans-Pacific
was informed that the check was ready for pick-up on September 27, 1996, the notice of the
availability of the check, by itself, does not produce the effect of payment of the premium. Trans-
Pacific could not be considered in delay in accepting the check because when it informed petitioner
that it will only be able to pick-up the check the next day, petitioner did not protest to this, but instead
allowed Trans-Pacific to do so. Thus, at the time of loss, there was no payment of premium yet to
make the insurance policy effective.

In UCPB General Insurance Co., Inc., we summarized the exceptions as follows: (1) in case of
life or industrial life policy, whenever the grace period provision applies, as expressly provided by
Section 77 itself; (2) where the insurer acknowledged in the policy or contract of insurance itself the
receipt of premium, even if premium has not been actually paid, as expressly provided by Section 78
itself; (3) where the parties agreed that premium payment shall be in installments and partial
payment has been made at the time of loss, as held in Makati Tuscany Condominium Corp. v. Court of
Appeals;53 (4) where the insurer granted the insured a credit term for the payment of the premium,
and loss occurs before the expiration of the term, as held in Makati Tuscany Condominium Corp.; and
(5) where the insurer is in estoppel as when it has consistently granted a 60 to 90-day credit term
for the payment of premiums.

Petitioner argues that his case falls under the fourth and fifth exceptions because the parties
intended the contract of insurance to be immediately effective upon issuance, despite non-payment
of the premium. This waiver to a pre-payment in full of the premium places respondent in estoppel.
We do not agree with petitioner.

Petitioner failed to establish the fact of a grant by respondent of a credit term in his favor, or
that the grant has been consistent. While there was mention of a credit agreement between Trans--
Pacific and respondent, such arrangement was not proven and was internal between agent and
principal. Under the principle of relativity of contracts, contracts bind the parties who entered into
it. It cannot favor or prejudice a third person, even if he is aware of the contract and has acted with
knowledge.

We cannot sustain petitioner's claim that the parties agreed that the insurance contract is
immediately effective upon issuance despite non- payment of the premiums. Even if there is a waiver
of pre-payment of premiums, that in itself does not become an exception to Section 77, unless the
insured clearly gave a credit term or extension. This is the clear import of the fourth exception in the

39
Ki n ds o f I nsu ra nc e Co n tra c t

UCPB General Insurance Co., Inc. To rule otherwise would render nugatory the requirement in
Section 77 that "[n]otwithstanding any agreement to the contrary, no policy or contract of insurance
issued by an insurance company is valid and binding unless and until the premium thereof has been
paid, x x x."

The policy states that the insured's application for the insurance is subject to the payment of
the premium. There is no waiver of pre-payment, in full or in installment, of the premiums under the
policy. Consequently, respondent cannot be placed in estoppel.Thus, we find that petitioner is not
entitled to the insurance proceeds because no insurance policy became effective for lack of premium
payment.

40
H ea l thca r e a g r e e m e nts

PHILAMCARE HEALTH SYSTEMS, INC. v. COURT OF APPEALS AND JULITA TRINOS


G.R. No. 125678
March 18, 2002

Ynares-Santiago, J.

Facts:

Ernani Trinos, deceased husband of respondent Julita Trinos, applied for a health care
coverage with petitioner Philamcare Health Systems, Inc. In the standard application form, he
answered no to the following question: He answered in the negative when asked, “Have you or any
of your family members ever consulted or been treated for high blood pressure, heart trouble,
diabetes, cancer, liver disease, asthma or peptic ulcer? (If Yes, give details) “.

The application was approved for a period of one year. Accordingly, he was issued Health
Care Agreement No. P010194. Under the agreement, respondents’ husband was entitled to avail of
hospitalization benefits, whether ordinary or emergency, listed therein. He was also entitled to avail
of out-patient benefits such as annual physical examinations, preventive health care and other out-
patient services. Upon the termination of the agreement, the same was extended for another
year. The amount of coverage was increased to a maximum sum of P75,000.00 per disability.

During the period of his coverage, Ernani suffered a heart attack and was confined at the
Manila Medical Center (MMC) for one month. Respondent tried to claim the benefits under the health
care agreement. However, petitioner denied her claim saying that the Health Care Agreement was
void. According to petitioner, there was a concealment regarding Ernanis medical history. Doctors at
the MMC allegedly discovered at the time of Ernanis confinement that he was hypertensive, diabetic
and asthmatic, contrary to his answer in the application form. Thus, respondent paid the
hospitalization expenses herself, amounting to about P76,000.00.

After her husband was discharged from the MMC, he was attended by a physical therapist at
home. Later, he was admitted at the Chinese General Hospital. Due to financial difficulties, however,
respondent brought her husband home again. Respondent was constrained to bring him back to the
Chinese General Hospital where he died on the same day.

Respondent filed a case with the RTC for reimbursement of her expenses plus moral damages
and attorney’s fees. She was awarded 76,000 for the bills and 40,000 for damages. The CA affirmed
but deleted awards for damages. Hence, this appeal.

Issue:

Whether a health care agreement is not an insurance contract; hence the incontestability
clause under the Insurance Code does not apply.

Ruling:

In the case at bar, the insurable interest of respondents husband in obtaining the health care
agreement was his own health. The health care agreement was in the nature of non-life insurance,
which is primarily a contract of indemnity. Once the member incurs hospital, medical or any other

41
H ea l thca r e a g r e e m e nts

expense arising from sickness, injury or other stipulated contingent, the health care provider must
pay for the same to the extent agreed upon under the contract.

Petitioner cannot rely on the stipulation regarding Invalidation of agreement where failure
of the insured to disclose information was a ground for revocation simply because the answer to
thequestion asked was based on the opinion of the insured. He is not a medical doctor, so he cannot
accurately gauge his condition.

The fraudulent intent on the part of the insured must be established to warrant rescission of
the insurance contract. Concealment as a defense for the health care provider or insurer to avoid
liability is an affirmative defense and the duty to establish such defense by satisfactory and
convincing evidence rests upon the provider or insurer. In any case, with or without the authority to
investigate, petitioner is liable for claims made under the contract. Having assumed a responsibility
under the agreement, petitioner is bound to answer the same to the extent agreed upon. In the end,
the liability of the health care provider attaches once the member is hospitalized for the disease or
injury covered by the agreement or whenever he avails of the covered benefits which he has prepaid.

Under Section 27 of the Insurance Code, a concealment entitles the injured party to rescind a
contract of insurance. The right to rescind should be exercised previous to the commencement of an
action on the contract. In this case, no rescission was made. Besides, the cancellation of health care
agreements as in insurance policies require the concurrence of the following conditions: 1. Prior
notice of cancellation to insured; 2. Notice must be based on the occurrence after effective date of the
policy of one or more of the grounds mentioned; 3. Must be in writing, mailed or delivered to the
insured at the address shown in the policy; 4. Must state the grounds relied upon provided in Section
64 of the Insurance Code and upon request of insured, to furnish facts on which cancellation is based.

None of the above pre-conditions was fulfilled in this case.

Anent the incontestability of the membership of respondent’s husband, we quote with


approval the following findings of the trial court:

Under the title Claim procedures of expenses, the defendant Philamcare Health
Systems Inc. had twelve months from the date of issuance of the Agreement within
which to contest the membership of the patient if he had previous ailment of asthma,
and six months from the issuance of the agreement if the patient was sick of diabetes
or hypertension. The periods having expired, the defense of concealment or
misrepresentation no longer lie.

Finally, petitioner alleges that respondent was not the legal wife of the deceased member
considering that at the time of their marriage, the deceased was previously married to another
woman who was still alive. The health care agreement is in the nature of a contract of
indemnity. Hence, payment should be made to the party who incurred the expenses. It is not
controverted that respondent paid all the hospital and medical expenses. She is therefore entitled to
reimbursement. The records adequately prove the expenses incurred by respondent for the
deceaseds hospitalization, medication and the professional fees of the attending physicians.

42
H ea l thca r e a g r e e m e nts

BLUE CROSS HEALTH CARE INC. v. NEOMI AND DANILO OLIVARES


G.R. No. 169737
February 12, 2008

Leonardo-De Castro, J.

Facts:

Neomi T. Olivares applied for a health care program with petitioner Blue Cross Health Care,
Inc., a health maintenance firm. For the period October 16, 2002 to October 15, 2003, she paid the
amount of P11,117. For the same period, she also availed of the additional service of limitless
consultations for an additional amount of P1,000.She paid these amounts in full on October 17, 2002.
The application was approved on October 22, 2002. In the health care agreement, ailments due to
pre-existing conditions were excluded from the coverage.

On November 30, 2002, or barely 38 days from the effectivity of her health insurance,
respondent Neomi suffered a stroke and was admitted at the Medical City which was one of the
hospitals accredited by petitioner. During her confinement, she underwent several laboratory tests.
On December 2, 2002, her attending physician informed her that she could be discharged from the
hospital. She incurred hospital expenses amounting to P34,217.20. Consequently, she requested from
the representative of petitioner at Medical City a letter of authorization in order to settle her medical
bills. But petitioner refused to issue the letter and suspended payment pending the submission of a
certification from her attending physician that the stroke she suffered was not caused by a pre-
existing condition.

She was discharged from the hospital on December 3, 2002. On December 5, 2002, she
demanded that petitioner pay her medical bill. When petitioner still refused, she and her husband,
respondent Danilo Olivares, were constrained to settle the bill. They thereafter filed a complaint for
collection of sum of money against petitioner in the MeTC. In its answer, petitioner maintained that
it had not yet denied respondents' claim as it was still awaiting the attending physicians report.

Issue:

Whether petitioner was able to prove that respondent Neomi's stroke was caused by a pre-
existing condition and therefore was excluded from the coverage of the health care agreement.

Ruling:

In Philamcare Health Systems, Inc. v. CA, we ruled that a health care agreement is in the
nature of a non-life insurance. It is an established rule in insurance contracts that when their terms
contain limitations on liability, they should be construed strictly against the insurer. These are
contracts of adhesion the terms of which must be interpreted and enforced stringently against the
insurer which prepared the contract. This doctrine is equally applicable to health care agreements.

Petitioner never presented any evidence to prove that respondent Neomi's stroke was due to
a pre-existing condition. It merely speculated that Dr. Saniel's report would be adverse to Neomi,
based on her invocation of the doctor-patient privilege. This was a disputable presumption at best.

43
H ea l thca r e a g r e e m e nts

Suffice it to say that this presumption does not apply if (a) the evidence is at the disposal of
both parties; (b) the suppression was not willful; (c) it is merely corroborative or cumulative and (d)
the suppression is an exercise of a privilege. Here, respondents' refusal to present or allow the
presentation of Dr. Saniel's report was justified. It was privileged communication between physician
and patient.

Furthermore, as already stated, limitations of liability on the part of the insurer or health care
provider must be construed in such a way as to preclude it from evading its obligations. Accordingly,
they should be scrutinized by the courts with extreme jealousy and care and with a jaundiced
eye. Since petitioner had the burden of proving exception to liability, it should have made its own
assessment of whether respondent Neomi had a pre-existing condition when it failed to obtain the
attending physician's report. It could not just passively wait for Dr. Saniel's report to bail it out. The
mere reliance on a disputable presumption does not meet the strict standard required under our
jurisprudence.

44
H ea l thca r e a g r e e m e nts

PHILIPPINE HEALTH CARE PROVIDERS, INC. v. COMMISSIONER OF INTERNAL REVENUE


G.R. No. 167330
June 12, 2008

Corona, J.

Facts:

Petitioner is a domestic corporation whose primary purpose is "to establish, maintain,


conduct and operate a prepaid group practice health care delivery system or a health maintenance
organization to take care of the sick and disabled persons enrolled in the health care plan and to
provide for the administrative, legal, and financial responsibilities of the organization." Individuals
enrolled in its health care programs pay an annual membership fee and are entitled to various
preventive, diagnostic and curative medical services provided by its duly licensed physicians,
specialists and other professional technical staff participating in the group practice health delivery
system at a hospital or clinic owned, operated or accredited by it.

Commissioner of Internal Revenue sent petitioner a formal demand letter and the
corresponding assessment notices demanding the payment of deficiency taxes, including surcharges
and interest, for the taxable years 1996 and 1997 in the total amount of P224,702,641.18.

Petitioner protested the assessment. As respondent did not act on the protest, petitioner filed
a petition for review in the Court of Tax Appeals (CTA) seeking the cancellation of the deficiency VAT
and DST assessments. Respondent claimed that petitioner's health care agreement was a contract of
insurance subject to DST under Section 185 of the 1997 Tax Code. The CA held that petitioner's health
care agreement was in the nature of a non-life insurance contract subject to DST. Petitioner moved
for reconsideration but the CA denied it. Hence, this petition.

Issue:

Whether or not a health care agreement in the nature of an insurance contract.

Ruling:

Under the law, a contract of insurance is an agreement whereby one undertakes for a
consideration to indemnify another against loss, damage or liability arising from an unknown or
contingent event. The event insured against must be designated in the contract and must either be
unknown or contingent.

Petitioner's health care agreement is primarily a contract of indemnity. The Court ruled that
a health care agreement is in the nature of a non-life insurance policy. Contrary to petitioner's claim,
its health care agreement is not a contract for the provision of medical services. Petitioner does not
actually provide medical or hospital services but merely arranges for the same and pays for them up
to the stipulated maximum amount of coverage. It is also incorrect to say that the health care
agreement is not based on loss or damage because, under the said agreement, petitioner assumes the
liability and indemnifies its member for hospital, medical and related expenses (such as professional
fees of physicians). The term "loss or damage" is broad enough to cover the monetary expense or
liability a member will incur in case of illness or injury.

45
H ea l thca r e a g r e e m e nts

Under the health care agreement, the rendition of hospital, medical and professional services
to the member in case of sickness, injury or emergency or his availment of so-called "out-patient
services" (including physical examination, x-ray and laboratory tests, medical consultations, vaccine
administration and family planning counseling) is the contingent event which gives rise to liability
on the part of the member. In case of exposure of the member to liability, he would be entitled to
indemnification by petitioner. Furthermore, the fact that petitioner must relieve its member from
liability by paying for expenses arising from the stipulated contingencies belies its claim that its
services are prepaid. The expenses to be incurred by each member cannot be predicted beforehand,
if they can be predicted at all. Petitioner assumes the risk of paying for the costs of the services even
if they are significantly and substantially more than what the member has "prepaid." Petitioner does
not bear the costs alone but distributes or spreads them out among a large group of persons bearing
a similar risk, that is, among all the other members of the health care program. This is insurance.

The insurable interest of every member of petitioner's health care program in obtaining the
health care agreement is his own health. Under the agreement, petitioner is bound to indemnify any
member who incurs hospital, medical or any other expense arising from sickness, injury or other
stipulated contingency to the extent agreed upon under the contract. Petitioner's contention that it
is a health maintenance organization and not an insurance company is irrelevant. Contracts between
companies like petitioner and the beneficiaries under their plans are treated as insurance contracts.

46
Pr o t ect io n a nd I n d em n ity Cl u b

PANDIMAN PHILIPPINES, INC. v. MARINE MANNING MANAGEMENT CORPORATION AND


ROSITA D.R. SINGHID
G.R. No. 143313
June 21, 2005

Garcia, J.

Facts:

Respondent Rosita Singhids deceased husband Benito Singhid (Benito) was hired by Fullwin
Maritime Limited (Fullwin), through its local agent, respondent Marine Manning and Management
Corporation (MMMC), as chief cook on board the vessel MV Sun Richie Five for a term of twelve (12)
months. The vessel and its crew were insured with Ocean Marine Mutual Insurance Association
Limited (OMMIAL), a Protection and Indemnity Club (P&I Club) of which the Sun Richie Five Bulkers
S.A., owner of the vessel Sun Richie Five, is a member. OMMIAL transacted business in the Philippines
through its local correspondent, herein petitioner Pandiman Philippines, Inc. (PPI).

While the vessel was on its way to Shanghai, China from Ho Chih Minh City, Vietnam Benito
suffered a heart attack, and subsequently died on June 24, 1997. His remains were flown back to the
Philippines.

After Benitos remains were interred, his widow Rosita filed a claim for death benefits with
MMMC, which, however, referred her to herein petitioner PPI. Upon Rositas submission of all the
required documents, petitioner approved the claim and recommended payment thereof in the
amount of US$79,000.00. But, despite said recommendation, Rositas death claims remained unpaid.

Hence, Rosita filed with the Labor Arbiter a complaint for recovery of death benefits, moral and
exemplary damages and attorneys fees. Named respondents in the complaint are MMMC, Fullwin,
petitioner PPI and OMMIAL. The Labor Arbiter dismissed the complaint insofar as petitioner is
concerned.

On MMMCs appeal to the NLRC, the latter, set aside that of the Labor Arbiter, absolved
respondent MMMC from any liability and instead held petitioner and OMMIAL liable for Rositas
claim. Therefrom, petitioner went to the Court of Appeals on a petition for certiorari. The appellate
court dismissed the petition for lack of merit and accordingly affirmed the challenged decision of the
NLRC.

Issue:

Whether or not petitioner PPI may be held liable for Rositas claim for death benefits as Benitos
widow.

Ruling:

MV Sun Richie Five Bulkers S.A., owner of the vessel Sun Richie Five, was a member of a P&I Club,
which is an association composed of shipowners in general who band together for the specific
purpose of providing insurance cover on a mutual basis against liabilities incidental to shipowning
that the members incur in favor of third parties. The vessel and its crew were covered by a Class 1
Protection and Indemnity agreement beginning noon of February 20, 1997 up to February 20, 1998
as embodied in the Certificate of Entry[11] issued by OMMIAL.

47
Pr o t ect io n a nd I n d em n ity Cl u b

In this protection and indemnity agreement, which is actually an insurance contract, the
provisions of the Insurance Code (P.D. 1460, as amended) is the governing law. In the subject
insurance contract, the P&I Club (OMMIAL) is the insurer, the shipowner (Sun Richie Five Bulkers
S.A.) is the insured, and herein respondent Rosita Singhid as widow and heir of a crew on board the
insured vessel like Benito, is a beneficiary.

There is nothing therein to show that an insurance contract in this case was in fact negotiated
between the insured Sun Richie Five and the insurer OMMIAL, through petitioner as insurance agent
which will make petitioner an insurance agent under Section 300 of the Insurance Code. As it is, the
NLRC, in its decision, merely relied on petitioners reference to OMMIAL as its principal instead of its
client. Such reference, however, will not and cannot vary the definition of what an insurance agent
actually is under the aforecited law, nor can it automatically turn petitioner into one, thereby
becoming correspondingly liable to all the duties, requirements, liabilities and penalties to which an
insurance agent is subject to. We, therefore, hold that petitioner PPI is not an insurance agent under
the obtaining circumstances.

In any event, payment for claims arising from the peril insured against, to which the insurer is
liable, is definitely not one of the liabilities of an insurance agent. Thus, there is no legal basis
whatsoever for holding petitioner solidarily liable with insurer OMMIAL for Rositas claim for death
benefits on account of her husbands demise while under the employ of MMMCs principal, Fullwin.

The insurance contract between the insurer and the insured, under Article 1311 of the Civil
Code, is binding only upon the parties (and their assigns and heirs) who execute the same. With the
reality, as borne by the records, that petitioner PPI is not a party to the insurance contract in question,
no liability or obligation arising therefrom, may be imposed upon it.

48
Pr o t ect io n a nd I n d em n ity Cl u b

HYOPSUNG MARITIME CO. v. COURT OF APPEALS


G.R. No. 77369
August 3l, 1988

Sarmiento, J.

Facts:

An admiralty case was filed by Pioneer Insurance & Surety Corp., as plaintiff (private
respondent herein), against Hyopsung Maritime Co., Ltd. (petitioner herein), Aurelio Navigation
Corp. S.A., and Litonjua Shipping Co., as parties defendants. Pioneer Insurance & Surety Corp., as
subrogee of the consignee, sought the recovery of the value (P5,000,537.48) of the lost or undelivered
cargo-consisting of steel billets allegedly shipped on board the vessel MV "Don Aurelio" plus interest,
attorney's fees, litigation expenses, exemplary damages, and costs of the suit.

The vessel was a member of a Protection & Indemnity Club (P & I Club, for short), which is
"an association composed of ship owners in general who band together for the specific purpose of
providing insurance cover on a mutual basis against liabilities incidental to ship owning that the
members incur in favor of third parties. Thus, the law firm of Teves, Campos, Hernandez & Lim, as
one of the designated legal representatives of the P & I Club concerned, filed an Answer supposedly
on behalf of all the defendants. Subsequently, however, the new counsel, Ferrer, Valte Mariano &
Sangalang law firm, for the defendant Litonjua Shipping Co., alleged ship agent of the two other
defendants, entered its appearance in substitution of Teves, Campos, Hernandez & Lim. Likewise,
Atty. Eulalio A. Ventura filed a special appearance as counsel for the herein petitioner, alleged
charterer of the vessel, for the sole purpose of questioning the jurisdiction of the court.

Issue:

Whether or not there was a voluntary appearance by the petitioner's counsel such that
jurisdiction over the petitioner has been acquired by the trial court.

Ruling:

The answer with counterclaim filed by the law firm of Teves, Campos, Hernandez & Lim dated
August 24, 1981, was never authorized by appellee Hyopsung Maritime Co. Ltd., evidenced by the
admission of the law firm itself through Atty. Jaime Vibar. We likewise hold that, the civil case below
being a personal action, personal or substituted service of summons on the petitioner, pursuant to
Sec. 14, Rule 14 (then Sec. 14, Rule 7) of the Rules of Court, is necessary to confer jurisdiction on the
court; 7 however, considering that the respondent Court of Appeals accepted the explanation of the
president of the petitioner company that it is not doing business in the Philippines, and no proof to
the contrary having been adduced below by the private respondent, ergo, the petitioner is not
amenable to process and the jurisdiction of the local courts.

On the other hand, the present suit is for the recovery of damages based on a breach of
contract which appears to have been entirely entered into, executed, and consummated in Korea.
Indisputably, the Pohang Korea, by a Korean firm with offices at Seoul, Korea; the corresponding bill
of lading was issued in Seoul, Korea and the freight was prepaid also at Seoul; the above vessel with
its cargo never even docked at Manila or at any other port of entry in the Philippines; lastly, the
petitioner did not appoint any ship agent in the Philippines. Simply put, the petitioner is beyond the
reach of our courts.

49
Co nst ru ct io n a nd I n t er pr e ta ti on of I ns ur a nc e Co nt ra c ts

MISAMIS LUMBER CORPORATION v. CAPITAL INSURANCE AND SURETY CO., INC.


G.R. No. L-21380
May 20, 1966

Reyes, J.B.L., J.

Facts:

Plaintiff-appellee Misamis Lumber Corporation, under its former name, Lanao Timber Mills,
Inc., insured its Ford Falcon motor car for the amount of P14,000 with the defendant-appellant,
Capital Insurance & Surety Company, Inc.

At around eleven o'clock in the evening of 25 November 1961, and while the above-
mentioned insurance policy was in force, the insured car, while traveling along in Aurora Boulevard
in front of the Pepsi-Cola plant in Quezon City, passed over a water hole which the driver did not see
because an oncoming car did not dim its light. The crankcase and flywheel housing of the car broke
when it hit a hollow block lying alongside the water hole. At the instance of the plaintiff-appellee, the
car was towed and repaired by Morosi Motors at its shop at 1906 Taft Avenue Extension at a total
cost of P302.27.

When the repairs on the car had already been made, the plaintiff-appellee made a report of
the accident to the defendant-appellant Capital Insurance & Surety Company. Since the defendant-
appellant refused to pay for the total cost of to wage and repairs, suit was filed in the municipal court
originally.

Issue:

Whether or not the insurance company is liable for more than the amount of the repair limit.

Ruling:

The insurance policy stipulated in paragraph 4 that if the insured authorizes the repair the
liability of the insurer, per its sub-paragraph (a), is limited to P150.00. The literal meaning of this
stipulation must control, it being the actual contract, expressly and plainly provided for in the policy.
It will be observed that the policy drew out not only the limits of the insurer's liability but also the
mechanics that the insured had to follow to be entitled to full indemnity of repairs. The option to
undertake the repairs is accorded to the insurance company per paragraph 2. The said company was
deprived of the option because the insured took it upon itself to have the repairs made, and only
notified the insurer when the repairs were done. As a consequence, paragraph 4, which limits the
company's liability to P150.00, applies.

The insurance contract may be rather onerous ("one-sided", as the lower court put it), but
that in itself does not justify the abrogation of its express terms, terms which the insured accepted or
adhered to and which is the law between the contracting parties.

Finally, to require the insurer to prove that the cost of the repairs ordered by the insured is
unreasonable, as the appealed decision does, when the insurer was not given an opportunity to
inspect and assess the damage before the repairs were made, strikes Us as contrary to elementary
justice and equity.

50
Co nst ru ct io n a nd I n t er pr e ta ti on of I ns ur a nc e Co nt ra c ts

SUN INSURANCE OFFICE, LTD. v. COURT OF APPEALS AND EMILIO TAN


G.R. No. 89741
March 13, 1991

Paras, J.

Facts:

Emilio Tan took from herein petitioner a P300,000.00 property insurance policy to cover his
interest in the electrical supply store of his brother housed in a building in Iloilo City. Four (4) days
after the issuance of the policy, the building was burned including the insured store. Tan filed his
claim for fire loss with petitioner, but petitioner wrote Tan denying the latter's claim. Tan wrote
petitioner, seeking reconsideration of the denial of his claim. Tan's counsel wrote the Insurer
inquiring about the status of his request for reconsideration. Petitioner answered the letter advising
Tan's counsel that the Insurer's denial of Tan's claim remained unchanged, enclosing copies of
petitioners' letters of February 29, 1984 and May 17, 1985 (response to petition for reconsideration).

Tan filed a case but petitioner filed a motion to dismiss on the alleged ground that the action
had already prescribed. Said motion was denied; and petitioner's motion for reconsideration was
also denied in an order. Petitioner went to the Court of Appeals and sought the nullification of the
said orders, but the Court of Appeals, in its decision denied the petition and held that the court a
quo may continue until its final termination. A motion for reconsideration was filed, but the same
was denied by the Court of Appeals. Hence, the instant petition.

Issue:

Whether or not the filing of a motion for reconsideration interrupts the twelve (12) months
prescriptive period to contest the denial of the insurance claim.

Ruling:

To allow the filing of a motion for reconsideration to suspend the running of the prescriptive
period of twelve months, a whole new body of rules on the matter should be promulgated so as to
avoid any conflict that may be brought by it.

While it is a cardinal principle of insurance law that a policy or contract of insurance is to be


construed liberally in favor of the insured and strictly against the insurer company, yet, contracts of
insurance, like other contracts, are to be construed according to the sense and meaning of the terms
which the parties themselves have used. If such terms are clear and unambiguous, they must be taken
and understood in their plain, ordinary and popular sense.

Under condition 27, If a claim be made and rejected and an action or suit be not commenced
either in the Insurance Commission or in any court of competent jurisdiction within twelve (12)
months from receipt of notice of such rejection, or in case of arbitration taking place as provided
herein, within twelve (12) months after due notice of the award made by the arbitrator or arbitrators
or umpire, then the claim shall for all purposes be deemed to have been abandoned and shall not
thereafter be recoverable hereunder.

The 12-month prescriptive period started to run from the said date of April 2, 1984, for such
is the plain meaning and intention of Section 27 of the insurance policy.

51
Co nst ru ct io n a nd I n t er pr e ta ti on of I ns ur a nc e Co nt ra c ts

NEW LIFE ENTERPRISES, et al. v. CA, et al.


G.R. No. 94071
March 31, 1992

Regalado, J.

Facts:

Julian Sy and Jose Sy Bang have formed a business partnership under the business name of
New Life Enterprises, the partnership engaged in the sale of construction materials at its place of
business, a two storey building. Julian Sy insured the stocks in trade of New Life Enterprises with
Western Guaranty Corporation, Reliance Surety and Insurance. Co., Inc., and Equitable Insurance
Corporation.
Western Guaranty Corporation issued Fire Insurance Policy No. 37201 in the amount of
P350,000.00. This policy was renewed on May, 13, 1982. On July 30,1981, Reliance Surety and
Insurance Co., Inc. issued Fire Insurance Policy No. 69135 in the amount of P300,000.00 (Renewed
under Renewal Certificate No. 41997) An additional insurance was issued by the same company on
November 12, 1981 under Fire Insurance Policy No. 71547 in the amount of P700,000.00. On
February 8, 1982, Equitable Insurance Corporation issued Fire Insurance Policy No. 39328 in the
amount of P200,000.00.
When the building occupied by the New Life Enterprises was gutted by fire, the stocks in the
trade inside said building were insured against fire in the total amount of P1,550,000.00. The cause
of fire was electrical in nature.
After the fire, Julian Sy went to the agent of Reliance Insurance whom he asked to accompany
him to the office of the company so that he can file his claim. He averred that in support of his claim,
he submitted the fire clearance, the insurance policies and inventory of stocks. The three insurance
companies denied Julian's claim for payment.
In relation to the case against Reliance Surety and Insurance Company, a certain Atty. Serafin
D. Dator, acting in behalf of the plaintiff, sent a letter dated February 13, 1983 to Executive Vice-
President Mary Dee Co asking that he be informed as to the specific policy conditions allegedly
violated by the plaintiff. She replied stating that Sy violated Condition No. 3 of said insurance policies,
otherwise known as the "Other Insurance Clause," is uniformly contained in all the aforestated
insurance contracts of herein petitioners, as follows:
Because of the denial of their claims for payment by the three (3) insurance companies,
petitioner filed separate civil actions against the former before the Regional Trial Court of Lucena
City, which cases were consolidated for trial, and thereafter the court below rendered its decision on
December 19, l986 with the following disposition:
Issue:
Whether the petitioner violated the insurance policies.
Ruling:
New Life and Julian admitted that the insurance policies issued by the companies did not state
or endorse the other insurance coverage obtained or subsequently effected on the same stocks-in-
trade. The policy issued by Western did not declare Reliance and Equitable as co-insurers on the
same stocks, while Reliance's policies covering the same stocks did not likewise declare Western and
Equitable as co-insurers. Equitable's policy stated "nil" in the space thereon requiring indication of
any co-insurance although there were three policies subsisting on the same stocks-in-trade at the
time of the loss. The insured is specifically required to disclose to the insurer any other insurance
and its particulars which he may have effected on the same subject matter. The knowledge of such
insurance by the insurer's agents, even assuming the acquisition thereof by the former, is not the
"notice" that would estopped the insurers from denying the claim. The obvious purpose of the

52
Co nst ru ct io n a nd I n t er pr e ta ti on of I ns ur a nc e Co nt ra c ts

disclosure requirement is to prevent over-insurance and thus avert the perpetration of fraud. The
public, as well as the insurer, is interested in preventing the situation in which a fire would be
profitable to the insured.
While it is a cardinal principle of insurance law that a policy or contract of insurance is to be
construed liberally in favor of the insured and strictly against the insurer company, yet contracts of
insurance, like other contracts, are to be construed according to the sense and meaning of the terms
which the parties themselves have used. If such terms are clear and unambiguous, they must be taken
and understood in their plain, ordinary and popular sense. Moreover, obligations arising from
contracts have the force of law between the contracting parties and should be complied with in good
faith.
As the insurance policy against fire expressly required that notice should be given by the
insured of other insurance upon the same property, the total absence of such notice nullifies the
policy.

53
Co nst ru ct io n a nd I n t er pr e ta ti on of I ns ur a nc e Co nt ra c ts

FIRST QUEZON CITY INSURANCE COMPANY, INC. v. COURT OF APPEALS


G.R. No. 98414
February 8, 1993

Griño-Aquino, J.

Facts:

On June 10, 1984, at about 3:00 p.m., after sending off certain seamen at the departure area
of then known as Manila International Airport (MIA), Plaintiff Jose V. del Rosario proceeded to the
loading and unloading zone for public utility bus stop to wait for a passenger bus bound for Quezon
City. While at the bus stop, the plaintiff saw a DMTC bus bearing body No. 236 and plate No. NVU-798
and which, per its signboard, was plying the Pasay to Quezon City (passing España) route. As it
approach the bus stop, the bus slowed down with all its doors wide open: while moving at a crawling
pace, i.e., as slow as an "ordinary walk," it was taking several passengers, about five or seven of them
including the plaintiff, all of whom managed to board the bus while it was already at the bus stop;
plaintiff was the last one to board the bus.

While the plaintiff was still on the bus' running board with his hand on the bus door's handle
bar, the slowly moving bus sped forward at a high speed, as a result of which, the plaintiff lost his
balance and fell from the bus. As plaintiff clung instinctively to the handle bar, he was dragged by the
bus along the asphalted road for about two (2) seconds. Plaintiff screamed of pain and anguished
even as the other passengers shouted and the bus' driver, Gil Agpalo, an employee of defendant and
third-party plaintiff DMTC, abruptly stopped the bus. Then, Gil forthwith fled from the scene, leaving
the bus and the injured plaintiff behind.

Thereafter, the plaintiff was brought to the hospital where he was given immediate medical
treatment. The doctors performed a major surgical operation on plaintiff's right leg. While at the
hospital, plaintiff was febrile or feverish for about forty (40) days. A second major surgical
operation, i.e., a skin grafting operation, was performed on plaintiff's right leg.

The trial court dismissed defendant De Dios Marikina Transportation Co. Inc.'s counterclaim
for lack of merit and ordering said defendant to pay plaintiff. The Court of Appeals modified the
dispositive part of the decision. The insurance company (now the petitioner) filed a motion for
reconsideration which was denied in a resolution dated April 22, 1991. Hence, this petition for
review.

Issue:

Can the amount of the insurance company’s liability be limited to Php 12, 000 as clearly
provided in the insurance policy?

Ruling:

Yes. The insurance company clearly passed the maximum limit of the petitioner's liability for
damages arising from death or bodily injury at P12,000.00 per passenger and its maximum liability
per accident at P50,000.00. Since only one passenger was injured in the accident, the insurer's
liability for the damages suffered by said passenger is pegged to the amount of P12,000.00 only. What
does the limit of P50,000.00 per accident mean? It means that the insurer's liability for any single
accident will not exceed P50,000.00 regardless of the number of passengers killed or injured therein.

54
Co nst ru ct io n a nd I n t er pr e ta ti on of I ns ur a nc e Co nt ra c ts

For example, if ten (10) passengers had been injured by the operation of the insured bus, the insurer's
liability for the accident would not be P120,000.00 (at the rate of P12,000.00 per passenger) but
would be limited to only P50,000.00 for the entire accident, as provided in the insurance contract.

The bus company may not recover from the insurance company (herein petitioner) more
than P 12,000.00 per passenger killed or injured, or fifty thousand (P50,000.00) pesos per accident
even if under the judgment of the court, the erring bus operator will have to pay more than
P12,000.00 to each injured passenger. The trial court's interpretation of the insurance contract was
the correct interpretation.

55
Co nst ru ct io n a nd I n t er pr e ta ti on of I ns ur a nc e Co nt ra c ts

FORTUNE INSURANCE AND SURETY CO., INC. v. COURT OF APPEALS AND PRODUCERS BANK
OF THE PHILIPPINES
G.R. No. 115278
May 23, 1995

Davide, Jr., J.

Facts:

Producers Bank of the Philippines insured with Fortune Insurance and Surety Co. P725, 000.
An armored car of the plaintiff, while in the process of transferring cash in the sum of P725,000.00
under the custody of its teller, Maribeth Alampay, from its Pasay Branch to its Head Office at 8737
Paseo de Roxas, Makati, Metro Manila on June 29, 1987, was robbed of the said cash. The robbery
took place while the armored car was traveling along Taft Avenue in Pasay City. The armored car was
driven by Benjamin Magalong Y de Vera, escorted by Security Guard Saturnino Atiga Y Rosete. After
an investigation conducted by the Pasay police authorities, the driver Magalong and guard Atiga were
charged, together with Edelmer Bantigue Y Eulalio, Reynaldo Aquino and John Doe, with violation of
P.D. 532 (Anti-Highway Robbery Law)

Upon claiming, Fortune refused stating that it is not liable since under the general exceptions
of the policy: any loss caused by any dishonest, fraudulent or criminal act of the insured or any officer,
employee, partner, director, trustee or authorized representative of the Insured whether acting alone
or in conjunction with others. The insurance agency contended that the guards automatically became
the authorized representatives of the bank when they cited International Timber Corp. v. NLRC
where a contractor is a "labor-only" contractor in the sense that there is an employer-employee
relationship between the owner of the project and the employees of the "labor-only" contractor. They
cited Art. 106. Of the Labor Code which said: Contractor or subcontractor. — There is "labor-only"
contracting where the person supplying workers to an employer does not have substantial capital or
investment I the form of tools, equipment, machineries, work premises, among others, and the
workers recruited and placed by such persons are performing activities which are directly related to
the principal business of such employer. In such cases, the person or intermediary shall be
considered merely as an agent of the employer who shall be responsible to the workers in the same
manner and extent as if the latter were directly employed by him. The bank asserted that the guards
were not its employees since it had nothing to do with their selection and engagement, the payment
of their wages, their dismissal, and the control of their conduct.

Issue:

Whether the driver and security guard are employees under the general exception.

Ruling:

Yes. Fortune is exempt from liability under the general exceptions clause of the insurance
policy. It is clear to us that insofar as Fortune is concerned, it was its intention to exclude and exempt
from protection and coverage losses arising from dishonest, fraudulent, or criminal acts of persons
granted or having unrestricted access to Producers' money or payroll. When it used then the term
"employee," it must have had in mind any person who qualifies as such as generally and universally
understood, or jurisprudentially established in the light of the four standards in the determination of
the employer-employee relationship, or as statutorily declared even in a limited sense as in the case
of Article 106 of the Labor Code which considers the employees under a "labor-only" contract as

56
Co nst ru ct io n a nd I n t er pr e ta ti on of I ns ur a nc e Co nt ra c ts

employees of the party employing them and not of the party who supplied them to the employer. But
even granting for the sake of argument that these contracts were not "labor-only" contracts, and PRC
Management Systems and Unicorn Security Services were truly independent contractors, we are
satisfied that Magalong and Atiga were, in respect of the transfer of Producer's money from its Pasay
City branch to its head office in Makati, its "authorized representatives" who served as such with its
teller Maribeth Alampay. Howsoever viewed, Producers entrusted the three with the specific duty to
safely transfer the money to its head office, with Alampay to be responsible for its custody in transit;
Magalong to drive the armored vehicle which would carry the money; and Atiga to provide the
needed security for the money, the vehicle, and his two other companions. In short, for these
particular tasks, the three acted as agents of Producers. A "representative" is defined as one who
represents or stands in the place of another; one who represents others or another in a special
capacity, as an agent, and is interchangeable with "agent."

57
Co nst ru ct io n a nd I n t er pr e ta ti on of I ns ur a nc e Co nt ra c ts

CEBU SHIPYARD v. WILLIAM LINES, INC.


G.R. No. 132607
May 5, 1999

Purisima, J.

Facts:

Cebu Shipyard and Engineering Works, Inc. (CSEW) is a domestic corporation engaged in the
business of dry-docking and repairing of marine vessels while the private respondent, Prudential
Guarantee and Assurance, Inc. (Prudential), also a domestic corporation is in the non-life insurance
business.

William Lines, Inc. (plaintiff below) is in the shipping business. It was the owner of M/V Manila
City, a luxury passenger-cargo vessel, which caught fire and sank on February 16, 1991. At the time
of the unfortunate occurrence sued upon, subject vessel was insured with Prudential for
P45,000,000.00 pesos for hull and machinery. The Hull Policy included an Additional Perils
(INCHMAREE) Clause covering loss of or damage to the vessel through the negligence of, among
others, ship repairmen. Provided such loss or damage has not resulted from want of due diligence by
the Assured, the Owners or Managers of the Vessel, of any of them. Masters, Officers, Crew or Pilots
are not to be considered Owners within the meaning of this Clause should they hold shares in the
Vessel. CSEW was also insured by Prudential for third party liability under a Shiprepairers Legal
Liability Insurance Policy. William Lines, Inc. brought its vessel, M/V Manila City, to the Cebu
Shipyard in Lapulapu City for annual dry-docking and repair. The contracts, denominated as Work
Orders, were signed thereafter stipulating that the Contractor shall replace at its own work and at its
own cost any work or material which can be shown to be defective and that the Contractor shall not
be under any liability to the Customer either in contract or for delict or quasi-delict or otherwise
except for negligence, with limited liability of up to P1 million only. Also, the insurance on the vessel
should be maintained by the customer and/or owner of the vessel during the period the contract is
in effect.

While the M/V Manila City was undergoing dry-docking and repairs within the premises of
CSEW, the master, officers and crew of M/V Manila City stayed in the vessel, using their cabins as
living quarters. Other employees hired by William Lines to do repairs and maintenance work on the
vessel were also present during the dry-docking. After subject vessel was transferred to the docking
quay, it caught fire and sank, resulting to its eventual total loss. William Lines, Inc. filed a complaint
for damages against CSEW, alleging that the fire which broke out in M/V Manila City was caused by
CSEWs negligence and lack of care. An amended Complaint impleading Prudential as co-plaintiff,
after the latter had paid William Lines, Inc. the value of the hull and machinery insurance on the M/V
Manila City. As a result of such payment Prudential was subrogated to the claim of P45 million,
representing the value of the said insurance it paid.

The trial court brought judgment against CSEW 45 million for the ship indemnity, 65 million for
loss of income, and more than 13 million in other damages. The court of appeals affirmed the trial
court’s decision, finding negligence on the part of CSEW.

Issues:

Whether or not the insurance on the vessel should be maintained by the customer and/or
owner of the vessel while under drydock or repair.

58
Co nst ru ct io n a nd I n t er pr e ta ti on of I ns ur a nc e Co nt ra c ts

Ruling:

Yes. Petitioner contends that Prudential is not entitled to be subrogated to the rights of
William Lines, Inc., theorizing that (1) the fire which gutted M/V Manila City was an excluded risk
and (2) it is a co-assured under the Marine Hull Insurance Policy.

It is petitioners submission that the loss of M/V Manila City or damage thereto is expressly
excluded from the coverage of the insurance because the same resulted from want of due diligence
by the Assured, Owners or Managers which is not included in the risks insured against. Again, this
theory of petitioner is bereft of any factual or legal basis. It proceeds from a wrong premise that the
fire which gutted subject vessel was caused by the negligence of the employees of William Lines,
Inc. To repeat, the issue of who between the parties was negligent has already been resolved against
Cebu Shipyard and Engineering Works, Inc. Petitioner theorizes further that there can be no right of
subrogation as it is deemed a co-assured under the subject insurance policy. To buttress its stance
that it is a co-assured, petitioner placed reliance on Clause 20 of of the Work Order states that the
insurance on the vessel should be maintained by the customer and/or owner of the vessel during the
period the contract is in effect. According to petitioner, under the aforecited clause, William Lines,
Inc., agreed to assume the risk of loss of the vessel while under drydock or repair and to such extent,
it is benefited and effectively constituted as a co-assured under the policy.

This theory of petitioner is devoid of sustainable merit. Clause 20 of the Work Order in
question is clear in the sense that it requires William Lines to maintain insurance on the vessel during
the period of dry-docking or repair. Concededly, such a stipulation works to the benefit of CSEW as
the ship repairer. However, the fact that CSEW benefits from the said stipulation does not
automatically make it as a co-assured of William Lines. The intention of the parties to make each
other a co-assured under an insurance policy is to be gleaned principally from the insurance contract
or policy itself and not from any other contract or agreement because the insurance policy
denominates the assured and the beneficiaries of the insurance. The hull and machinery insurance
procured by William Lines, Inc. from Prudential named only William Lines, Inc. as the assured. There
was no manifestation of any intention of William Lines, Inc. to constitute CSEW as a co-assured under
subject policy. It is axiomatic that when the terms of a contract are clear its stipulations control. Thus,
when the insurance policy involved named only William Lines, Inc. as the assured thereunder, the
claim of CSEW that it is a co-assured is unfounded.

59
Co nst ru ct io n a nd I n t er pr e ta ti on of I ns ur a nc e Co nt ra c ts

GULF RESORTS, INC. v. PHILIPPINE CHARTER INSURANCE CORPORATION


G.R. No. 156167,
May 16, 2005

Puno, J.

Facts:

Plaintiff is the owner of the Plaza Resort situated at Agoo, La Union and had its properties in
said resort insured originally with the American Home Assurance Company (AHAC-AIU). In the first
four insurance policies issued by AHAC-AIU, the risk of loss from earthquake shock was extended
only to plaintiffs two swimming pools.

In consideration of the payment by the insured to the company of the sum included additional
premium the Company agrees, notwithstanding what is stated in the printed conditions of this policy
due to the contrary, that this insurance covers loss or damage to shock to any of the property insured
by this Policy occasioned by or through or in consequence of earthquake.

On July 16, 1990 an earthquake struck Central Luzon and Northern Luzon and plaintiffs
properties covered by Policy No. 31944 issued by defendant, including the two swimming pools in
its Agoo Playa Resort were damaged.

After the earthquake, petitioner advised respondent that it would be making a claim under
its Insurance Policy for damages on its properties. On August 11, 1990, petitioner filed its formal
demand for settlement of the damage to all its properties in the Agoo Playa Resort. On August 23,
1990, respondent denied petitioners claim on the ground that its insurance policy only afforded
earthquake shock coverage to the two swimming pools of the resort. Petitioner and respondent failed
to arrive at a settlement. Thus, on January 24, 1991, petitioner filed a complaint with the regional
trial court which rendered a decision in favour of the respondent. The appellate court affirmed the
decision of the trial court.

Issue:

Whether or not all the properties covered thereunder, are insured against the risk of
earthquake shock.

Ruling:

An insurance premium is the consideration paid an insurer for undertaking to indemnify the
insured against a specified peril. In fire, casualty, and marine insurance, the premium payable
becomes a debt as soon as the risk attaches. In the subject policy, no premium payments were made
with regard to earthquake shock coverage, except on the two swimming pools. There is no mention
of any premium payable for the other resort properties with regard to earthquake shock. This is
consistent with the history of petitioner’s previous insurance policies from AHAC-AIU. As borne out
by petitioner’s witnesses:

In sum, there is no ambiguity in the terms of the contract and its riders. Petitioner cannot rely
on the general rule that insurance contracts are contracts of adhesion which should be liberally
construed in favor of the insured and strictly against the insurer company which usually prepares
it. A contract of adhesion is one wherein a party, usually a corporation, prepares the stipulations in

60
Co nst ru ct io n a nd I n t er pr e ta ti on of I ns ur a nc e Co nt ra c ts

the contract, while the other party merely affixes his signature or his "adhesion" thereto. Through
the years, the courts have held that in these type of contracts, the parties do not bargain on equal
footing, the weaker party's participation being reduced to the alternative to take it or leave it. Thus,
these contracts are viewed as traps for the weaker party whom the courts of justice must
protect. Consequently, any ambiguity therein is resolved against the insurer, or construed liberally
in favor of the insured.

We cannot apply the general rule on contracts of adhesion to the case at bar. Petitioner cannot
claim it did not know the provisions of the policy. From the inception of the policy, petitioner had
required the respondent to copy verbatim the provisions and terms of its latest insurance policy from
AHAC-AIU. The testimony of Mr. Leopoldo Mantohac, a direct participant in securing the insurance
policy of petitioner, is reflective of petitioners knowledge.

61
Co nst ru ct io n a nd I n t er pr e ta ti on of I ns ur a nc e Co nt ra c ts

QUA CHEE GAN v. LAW UNION AND ROCK INSURANCE CO., LTD.
G.R. No. L-4611
December 17, 1955

Reyes, J. B. L., J.

Facts:

Qua Chee Gan, a merchant of Albay, owned owned four warehouses or bodegas in the
municipality of Tabaco, Albay, used for the storage of stocks of copra and of hemp, baled and loose,
in which the appellee dealth extensively. They had been, with their contents, insured with the
defendant Company since 1937. Fire of undetermined origin that broke out and lasted almost one
week, gutted and completely destroyed Bodegas Nos. 1, 2 and 4, with the merchandise stored theren.
The plaintiff having submitted the corresponding fire claims, totalling P398,562.81 (but reduced to
the full amount of the insurance, P370,000), the Insurance Company resisted payment, claiming
violation of warranties and conditions, filing of fraudulent claims, and that the fire had been
deliberately caused by the insured or by other persons in connivance with him. Qua Chee Gan was
acquitted in the arson case. He then demanded that Law Union pay up. This time, Law Union averred
that the insurance contract is void because Qua Chee Gan failed to install 11 hydrants; and that
gasoline was found in one of the warehouses.

Issue:

Whether or not the policies should be avoided for breach of warranty, specifically the one
appearing on a rider pasted (with other similar riders) on the face of the policies.

Ruling:

No. The appellant is barred by waiver (or rather estoppel) to claim violation of the so-called
fire hydrants warranty, for the reason that knowing fully all that the number of hydrants demanded
therein never existed from the very beginning, the appellant neverthless issued the policies in
question subject to such warranty, and received the corresponding premiums. It would be perilously
close to conniving at fraud upon the insured to allow appellant to claims now as void ab initio the
policies that it had issued to the plaintiff without warning of their fatal defect, of which it was
informed, and after it had misled the defendant into believing that the policies were effective.

The law expressed by American held that where the insurer, at the time of the issuance of a
policy of insurance, has knowledge of existing facts which, if insisted on, would invalidate the contract
from its very inception, such knowledge constitutes a waiver of conditions in the contract
inconsistent with the facts, and the insurer is stopped thereafter from asserting the breach of such
conditions. The law is charitable enough to assume, in the absence of any showing to the contrary,
that an insurance company intends to executed a valid contract in return for the premium received;
and when the policy contains a condition which renders it voidable at its inception, and this result is
known to the insurer, it will be presumed to have intended to waive the conditions and to execute a
binding contract, rather than to have deceived the insured into thinking he is insured when in fact he
is not, and to have taken his money without consideration.

Gasoline is not specifically mentioned among the prohibited articles listed in the so-called
"hemp warranty." The cause relied upon by the insurer speaks of "oils (animal and/or vegetable
and/or mineral and/or their liquid products having a flash point below 300o Fahrenheit", and is

62
Co nst ru ct io n a nd I n t er pr e ta ti on of I ns ur a nc e Co nt ra c ts

decidedly ambiguous and uncertain; for in ordinary parlance, "Oils" mean "lubricants" and not
gasoline or kerosene. Here, again, by reason of the exclusive control of the insurance company over
the terms and phraseology of the contract, the ambiguity must be held strictly against the insurer and
liberraly in favor of the insured, specially to avoid a forfeiture. Further, oil is incidental to Qua Chee
Gan’s business, it being used for motor fuel.

63
Co nst ru ct io n a nd I n t er pr e ta ti on of I ns ur a nc e Co nt ra c ts

FIELDMEN'S INSURANCE CO., INC. v. MERCEDES VARGAS VDA. DE SONGCO, ET AL.


G.R. No. L-24833
September 23, 1968

Fernando, J.:

Facts:

Federico Songco of Floridablanca, Pampanga, a man of scant education being only a first
grader owned a private jeepney with Plate No. 41-289 for the year 1960. He was induced by
Fieldmen's Insurance Company Pampanga agent Benjamin Sambat to apply for a Common Carrier's
Liability Insurance Policy covering his motor vehicle. Upon paying an annual premium of P16.50,
defendant Fieldmen's Insurance Company, Inc. issued a Common Carriers Accident Insurance Policy
No. 45-HO- 4254 the duration of which will be for one (1) year, effective September 15, 1960 to
September 15, 1961. On September 22, 1961, the defendant company, upon payment of the
corresponding premium, renewed the policy by extending the coverage from October 15, 1961 to
October 15, 1962. This time Federico Songco's private jeepney carried Plate No. J-68136-Pampanga-
1961. On October 29, 1961, during the effectivity of the renewed policy, the insured vehicle while
being driven by Rodolfo Songco, a duly licensed driver and son of Federico (the vehicle owner)
collided with a car in the municipality of Calumpit, province of Bulacan, as a result of which mishap
Federico Songco (father) and Rodolfo Songco (son) died, Carlos Songco (another son), the latter's
wife, Angelita Songco, and a family friend by the name of Jose Manuel sustained physical injuries of
varying degree."

Issue:

Whether or not the Songcos’ can claim the insurance proceeds despite the fact that the vehicle
concerned was an owner and not a common carrier.

Ruling:

Fieldmens’ Insurance Company is "estopped from enforcing forfeitures in its favor, in order
to forestall fraud or imposition on the insured." The doctrine of estoppel undeniably calls for
application. After petitioner Fieldmen's Insurance Co., Inc. had led the insured Federico Songco to
believe that he could qualify under the common carrier liability insurance policy, and to enter into
contract of insurance paying the premiums due, it could not, thereafter, in any litigation arising out
of such representation, be permitted to change its stand to the detriment of the heirs of the insured.
As estoppel is primarily based on the doctrine of good faith and the avoidance of harm that will befall
the innocent party due to its injurious reliance, the failure to apply it in this case would result in a
gross travesty of justice.

"Since some of the conditions contained in the policy issued by the defendant-appellant were
impossible to comply with under the existing conditions at the time and 'inconsistent with the known
facts,' the insurer 'is estopped from asserting breach of such conditions.'

Even if it be assumed that there was an ambiguity, an excerpt from the Qua Chee Gan decision
would reveal anew the weakness of petitioner's contention. Thus: "Moreover, taking into account the
well known rule that ambiguities or obscurities must be strictly interpreted against the party that
caused them, the 'memo of warranty' invoked by appellant bars the latter from questioning the
existence of the appliances called for in the insured premises, since its initial expression, 'the

64
Co nst ru ct io n a nd I n t er pr e ta ti on of I ns ur a nc e Co nt ra c ts

undernoted appliances for the extinction of fire being kept on the premises insured hereby, ... it is
hereby warranted ...,' admits of interpretation as an admission of the existence of such appliances
which appellant cannot now contradict, should the parol evidence rule apply."

65
Co nst ru ct io n a nd I n t er pr e ta ti on of I ns ur a nc e Co nt ra c ts

WESTERN GUARANTY CORPORATION v. HONORABLE COURT OF APPEALS,


G.R. No. 91666
July 20, 1990

Feliciano, J.

Facts:

Priscilla E. Rodriguez was struck by a De Dios passenger bus owned by De Dios


Transportation Co., Inc., then driven by Walter Saga y Aspero. The bus driver disregarded the stop
signal given by a traffic policeman to allow pedestrians to cross the road. Priscilla was thrown to
the ground, hitting her forehead. She was treated at the Protacio Emergency Hospital and later
on hospitalized at the San Juan De Dios Hospital. Her face was permanently disfigured, causing
her serious anxiety and moral distress. De Dios was insured then with Western Guaranty
Corporation. Because of the incident, Priscilla Rodriguez filed a complaint for damages before the
Regional Trial Court against De Dios Transportation Co. and Walter A. Saga. De Dios
Transportation Co., in turn, filed a third-party complaint against its insurance carrier, Western
Guaranty. The trial court ruled in favor of Priscilla Rodriguez. On appeal, the Court of Appeals
affirmed in toto the decision of the trial court.

Issue:

Is Western Guaranty liable to pay beyond the limits set forth in the Schedule of
Indemnities and in finding Western liable for loss of earnings, moral damages and attorney's
fees?

Ruling:

Yes. Schedule of Indemnities does not purport to restrict the kinds of damages that may
be awarded against Western once liability has arisen. It appears to us self-evident that the
Schedule of Indemnities was not intended to be an enumeration, much less a closed enumeration,
of the specific kinds of damages which may be awarded under the Master Policy Western has
issued. The reading urged by Western of the Schedule of Indemnities comes too close to working
fraud upon both the insured and the third party beneficiary of Section 1, quoted above. For
Western's reading would drastically and without warning limit the otherwise unlimited (save for
the over-all quantitative limit of liability of P50,000.00 per person per accident) and
comprehensive scope of liability assumed by the insurer Western under Section 1: "all sums
necessary to discharge liability of the insured in respect of [bodily injury to a third party]".

Western would have the Court construe the Schedule of Indemnities as comprising
contractual limitations of liability which, as already noted, is comprehensively defined in Section
1 — Liability to the Public" — of the Master Policy. It is well settled, however, that contractual
limitations of liability found in insurance contracts should be regarded by courts with a jaundiced
eye and extreme care and should be so construed as to preclude the insurer from evading
compliance with its just obligations.

66
Co nst ru ct io n a nd I n t er pr e ta ti on of I ns ur a nc e Co nt ra c ts

RIZAL SURETY & INSURANCE COMPANY v. COURT OF APPEALS and TRANSWORLD


KNITTING MILLS, INC.
G.R. No. 112360
July 18, 2000

Purisima, J.

Facts:

Rizal Surety & Insurance Company (Rizal Surety) issued a 1-million-peso fire insurance
policy with Transworld Knitting Mills (Transworld) against the four-span building the latter
owned. The coverage was then increased to 1.5 million. A four-span building was part of the
policy. Thereafter, a fire broke out and gutted the building, together with a two storey building
behind it were gaming machines were stored. Because of the fire, Transworld filed its claims but
to no avail. Hence, it brought a suit in court. It aimed to make Rizal pay for almost 3 million
including legal interest and damages. Rizal claimed that the policy only covered damage on the
four-span building and not the two storey building. The trial court ruled in Transworld’s favor
and ordered Rizal to pay actual damages only. On appeal, the Court of Appeals on the other hand
increased the damages. The insurance company sought reconsideration and CA answered by
modifying the imposition of interest.

Issue:

Is the fire insurance policy limited only to the coverage of what were stored in the four-
span building, thereby making Rizal Surety not liable herein?

Ruling:

No. The policy had clauses on the building coverage which provides that “contained
and/or stored during the currency of this Policy in the premises occupied by them forming part
of the buildings situated within own Compound" "First, said properties must be contained and/or
stored in the areas occupied by Transworld and second, said areas must form part of the building
described in the policy. This generally means that the policy didn’t limit its coverage to what was
stored in the four-span building. As to questions of fact, both the trial court and the Court of
Appeals found that the so called "annex” was not an annex building but an integral part of the
four-span building described in the policy and consequently, the machines and spare parts stored
were covered by the fire insurance. A report said: "Two-storey building constructed of partly
timber and partly concrete hollow blocks under roof which is adjoining and intercommunicating
with the repair of the first right span of the lofty storey building and thence by property fence
wall "Art.1377.

The interpretation of obscure words or stipulations in a contract shall not favor the party
who caused the obscurity". Citing Landicho v. GSIS- the 'terms in an insurance policy, which are
ambiguous, equivocal, or uncertain are to be construed strictly and most strongly against the
insurer, and liberally in favor of the insured so as to effect the dominant purpose of indemnity or
payment to the insured’.

67
Co nst ru ct io n a nd I n t er pr e ta ti on of I ns ur a nc e Co nt ra c ts

MALAYAN INSURANCE v. COURT OF APPEALS and TKC MARKETING CORPORATION


G.R. No. 119599
March 20, 1997

Romero, J.

Facts:

TKC Marketing Corporation (TMC) was the owner/consignee of soya bean meal which
was loaded on board the ship MV Al Kaziemah for carriage from the port of Rio del Grande, Brazil,
to the port of Manila. Said cargo was insured against the risk of loss by Malayan Insurance
Corporation for which it issued two (2) Marine Cargo Policy Nos. M/LP 97800305 and M/LP
97800306. While the vessel was docketed, it was arrested and detained by civil authorities
somewhere in Durban, South Africa. As a result, TMC notified Malayan of the arrest of the vessel
and made a formal claim, representing the dollar equivalent on the policies, for non-delivery of
the cargo. TMC likewise sought the assistance of Malayan on what to do with the cargo. Malayan,
replied that the arrest of the vessel by civil authority was not a peril covered by the policies.
Meanwhile, the insurance coverage was extended under the same terms and conditions
embodied in the original policies while in the process of making arrangements for the
transhipment of the cargo from Durban to Manila. the cargo was sold, therefrom TMC forthwith
reduced its claim representing it's loss after the proceeds of the sale were deducted from the
original claim of $916,886.66 or P20,184,159.55. Malayan maintained its position that the arrest
of the vessel by civil authorities on a question of ownership was an excepted risk under the
marine insurance policies. This prompted TMC to file a complaint for damages praying that aside
from its claim.

The trial court ruled in TMC’s favour and required Malayan to pay, aside from the
insurance claim. On appeal, the appellate court ruled that restraint or detainment within the
meaning of Clause 12 rules out detention by ordinary legal processes. Hence, arrests by civil
authorities, such as what happened in the instant case, is an excepted risk under Clause 12 of the
Institute Cargo Clause or the F.C. & S. Clause. However, with the deletion of Clause 12 of the
Institute Cargo Clause and the consequent adoption or institution of the Institute War Clauses
(Cargo), the arrest and seizure by judicial processes which were excluded under the former policy
became one of the covered risks.

Issue:

Is Malayan overly straining its interpretation of the provisions of the policy in order to
avoid being liable for TMC's claim?

Ruling:

Yes. Court finds it pointless for Malayan to maintain its position that it only insures risks
of "arrest" occasioned by executive or political acts of government which is interpreted as not
referring to those caused by ordinary legal processes as contained in the "Perils" Clause; deletes
the F.C. & S. Clause which excludes risks of arrest occasioned by executive or political acts of the
government and naturally, also those caused by ordinary legal processes; and, thereafter
incorporates subsection 1.1 of Section 1 of the Institute War Clauses which now includes in the
coverage risks of arrest due to executive or political acts of a government but then still excludes
"arrests" occasioned by ordinary legal processes when subsection 1.1 of Section 1 of said Clauses
should also have included "arrests" previously excluded from the coverage of the F.C. & S. Clause.

It has been held that a strained interpretation which is unnatural and forced, as to lead to
an absurd conclusion or to render the policy nonsensical, should, by all means, be avoided.

68
Co nst ru ct io n a nd I n t er pr e ta ti on of I ns ur a nc e Co nt ra c ts

Likewise, it must be borne in mind that such contracts are invariably prepared by the companies
and must be accepted by the insured in the form in which they are written. Any construction of a
marine policy rendering it void should be avoided. Such policies will, therefore, be construed
strictly against the company in order to avoid a forfeiture, unless no other result is possible from
the language used.

69
Ins u ra bl e I n t e r est i n P ro p e rt y

EL ORIENTE, FABRICA DE TABACOS, INC., v. JUAN POSADAS, COLLECTOR OF INTERNAL


REVENUE
G. R. No. 34774
September 21, 1931

Malcolm, J.

Facts:

El Oriente, Fabrica de Tabacos, Inc. (El Oriente) in order to protect itself against the loss
that it might suffer by reason of the death of its manager, A. Velhagen, procured from the
Manufacturers Life Insurance Co., of Toronto, Canada, thru its local agent E.E. Elser, an insurance
policy on the life of the said A. Velhagen for the sum of $50,000. El Oriente designated itself as the
sole beneficiary of said policy. During the time the life insurance policy was in force and effect, El
Oriente paid from its funds all the insurance premiums due thereon. It charged as expenses of its
business all the said premiums and deducted the same from its gross incomes as reported in its
annual income tax returns, which deductions were allowed by the Juan Posadas upon a showing
made by the El Oriente that such premiums were legitimate expenses of its business. Upon the
death of said A. Velhagen, the El Oriente received all the proceeds of the said life insurance policy,
together with the interests and the dividends accruing thereon. Because of that, EL Oriente
claimed exemption under section 4 of the Income Tax Law. On the other hand, the Collector of
Internal Revenue assessed and levied the sum of P3,148.74 as income tax on the proceeds of the
insurance policy, which tax the plaintiff paid under instant protest.

Issue:

Are the proceeds of insurance taken by a corporation on the life of an important official
to indemnify it against loss in case of his death taxable as income under the Philippine Income
Tax Law?

Ruling:

No. It will be recalled that El Oriente, Fabrica de Tabacos, Inc., took out the insurance on
the life of its manager, who had had more than thirty-five years' experience in the manufacture
of cigars in the Philippines, to protect itself against the loss it might suffer by reason of the death
of its manager. The Court do not believe that this fact signifies that when the El Oriente received
P104,957.88 from the insurance on the life of its manager, it thereby realized a net profit in this
amount. It is true that the Income Tax Law, in exempting individual beneficiaries, speaks of the
proceeds of life insurance policies as income, but this is a very slight indication of legislative
intention. In reality, what the plaintiff received was in the nature of an indemnity for the loss
which it actually suffered because of the death of its manager.

Considering the uncertainty of Philippine law, and considering the lack of express
legislative intention to tax the proceeds of life insurance policies paid to corporate beneficiaries,
particularly when in the exemption in favor of individual beneficiaries in the chapter on this
subject, the clause is inserted "exempt from the provisions of this law," the court deems it
reasonable to hold the proceeds of the life insurance policy in question as representing an
indemnity and not taxable income.

70
Ins u ra bl e I n t e r est i n P ro p e rt y

LINCOLN NATIONAL LIFE v. SAN JUAN LIFE INSURANCE


G.R. No. 34588-88
Nov. 27, 1971

Melo, J.

Facts:

Luis Parco insured five insurance policies from Lincoln National Life, which totalled at
P200,000.00. Parco insured one Misterioso San Juan. He falsely represented in the policies that:
1) Misterioso was a proprietor and a fish merchant for ten years; 2) That he had no employer but
himself; 3) That his income exceeds P5,000.00 a year; and 4) That he had no pending applications
for life insurance. However, it was established that Misterioso was not in fact a merchant but a
tenant in a coconut land owned by Luis, his employer. Moreover, a number of applications for
insurance have been filed by Luis, several of which have been declined. Subsequently, a severed
human head in an advanced state of decomposition was found in a jeepney and allegedly, the
unidentified passenger was said to be Misterioso. Upon collection of the proceeds of the policies,
Lincoln refuses to pay stating that there were false misrepresentations and concealment of
material facts made by Misterioso and Luis.

Issue:

Can Luis collect on the insurance policies?

Ruling:

No. It appears that, based on the circumstances and evidence, the insurance was really
taken out by the employer. The policies are in effect wagering and highly speculative which are
void for reasons of public policy. They lack the element of insurable interest. Here, the insured
was a tenant in a coconut land owned by the employer and his earnings were barely that of a farm
laborer. It was established that the insured could not have afforded the insurance policies drawn
on his life. As to the false misrepresentations, there was no shred of evidence that Lincoln had
prior knowledge of said false misrepresentations when it approved the life insurance. As such,
the insurance contracts should be rescinded.

71
Ins u ra bl e I n t e r est i n P ro p e rt y

VIOLETA R. LALICAN v. THE INSULAR LIFE ASSURANCE COMPANY LIMITED


G.R. No. 183526
August 25, 2009

Chico-Nazario, J.:

FACTS:

Violeta Lalican is the widow of the deceased Eulogio C. Lalican. During his lifetime, Eulogio
applied for an insurance policy with Insular Life. Insular Life issued in favor of Eulogio Policy No.
9011992, which contained a 20-Year Endowment Variable Income Package Flexi Plan worth
P500,000.00, with two riders valued at P500,000.00 each. Violeta was named as the primary
beneficiary. Eulogio paid the premiums, however, he failed to pay the subsequent premium due.
Policy No. 9011992, therefore, lapsed and became void pursuant to the policy. Eulogio submitted
an Application for Reinstatement of Policy No. 9011992. Insular Life notified Eulogio that his
Application for Reinstatement could not be fully processed because he left unpaid the overdue
interest thereon. Eulogio submitted a second Application for Reinstatement of Policy. The
husband of the agent of Insular Life received Eulogio’s second Application for Reinstatement and
issued a receipt for the amount Eulogio deposited. A while later, Eulogio died. Without knowing
of Eulogios death, the agent forwarded to the Insular Life Regional Office Eulogio’s second
Application for Reinstatement of Policy No. 9011992 and P17,500.00 deposit. However, Insular
Life no longer acted upon Eulogios second Application for Reinstatement, as the former was
informed that Eulogio had already passed away. Violeta filed with Insular Life a claim for payment
of the full proceeds of the policy. Insular Life informed Violeta that her claim could not be granted
since, at the time of Eulogios death, the Policy had already lapsed, and Eulogio failed to reinstate
the same. This prompted Violeta to file for a Complaint for Death Claim Benefit.

Issue:

Do Eulogio have an existing insurable interest in his own life until the day of his death?

Ruling:

Yes. An insurable interest is one of the most basic and essential requirements in an
insurance contract. In general, an insurable interest is that interest which a person is deemed to
have in the subject matter insured, where he has a relation or connection with or concern in it,
such that the person will derive pecuniary benefit or advantage from the preservation of the
subject matter insured and will suffer pecuniary loss or damage from its destruction, termination,
or injury by the happening of the event insured against. The existence of an insurable interest
gives a person the legal right to insure the subject matter of the policy of insurance. Section 10 of
the Insurance Code indeed provides that every person has an insurable interest in his own life.
Section 19 of the same code also states that an interest in the life or health of a person insured
must exist when the insurance takes effect, but need not exist thereafter or when the loss occurs.
While Eulogio was still alive, he had an insurable interest in his own life, which he did insure
under Policy No. 9011992.

72
Ins u ra bl e I n t e r est i n P ro p e rt y

MALAYAN INSURANCE COMPANY, INC. v. PAP CO., LTD. (PHIL. BRANCH)


G.R. No. 200784
August 7, 2013

Mendoza, J.:

Facts:

Malayan Insurance Company issued Fire Insurance Policy No. F00227-000073 to PAP Co.,
Ltd. for the latter’s machineries and equipment located at Sanyo Precision Phils. Building. The
insurance, was procured by PAP Co. for Rizal Commercial Banking Corporation (RCBC), the
mortgagee of the insured machineries and equipment. After the passage of almost a year but prior
to the expiration of the insurance coverage, PAP Co. renewed the policy on an "as is" basis.
Pursuant thereto, a renewal policy, Fire Insurance Policy No. F-00227-000079, was issued.
During the subsistence of the renewal policy, the insured machineries and equipment were totally
lost by fire. Hence, PAP Co. filed a fire insurance claim with Malayan Insurance Company, Inc.
(Malayan) in the amount insured. Malayan denied the claim upon the ground that, at the time of
the loss, the insured machineries and equipment were transferred by PAP Co. to a location
different from that indicated in the policy. Specifically, that the insured machineries were
transferred from the Sanyo Building to the Pace Pacific Bldg., Lot 14, Block 14, Phase III, PEZA,
Rosario, Cavite (Pace Pacific). Contesting the denial, PAP Co. argued that Malayan cannot avoid
liability as it was informed of the transfer by RCBC, the party duty-bound to relay such
information. However, Malayan reiterated its denial of PAP Co.’s claim. Distraught, PAP Co. filed
the complaint against Malayan.

Issue:

Is Malayan liable for the loss of the insured properties under the fire insurance policy?

Ruling:

No. The policy forbade the removal of the insured properties unless sanctioned by
Malayan. Evidently, by the clear and express condition in the renewal policy, the removal of the
insured property to any building or place required the consent of Malayan. Any transfer effected
by the insured, without the insurer’s consent, would free the latter from any liability. The
respondent failed to notify, and to obtain the consent of, Malayan regarding the removal. The
records are bereft of any convincing and concrete evidence that Malayan was notified of the
transfer of the insured properties from the Sanyo factory to the Pace factory. Considering that the
original policy was renewed on an "as is basis," it follows that the renewal policy carried with it
the same stipulations and limitations. There being an unconsented removal, the transfer was at
PAP’s own risk. Consequently, it must suffer the consequences of the fire. It can also be said that
with the transfer of the location of the subject properties, without notice and without Malayan’s
consent, after the renewal of the policy, PAP clearly committed concealment, misrepresentation
and a breach of a material warranty.

73
Ins u ra bl e I n t e r est i n P ro p e rt y

ANTONINA LAMPANO v. PLACIDA A. JOSE, ET AL.


G.R. No. L-9401
March 30, 1915

Trent, J.:

Facts:

Mariano R. Barretto, constructed a house for Placida A. Jose, on No. 72, plot F. Estate of
Nagtahan for P6,000. Subsequent thereto Jose sold the house to the Antonina Lampano.
Subsequently, the house was destroyed by fire. At the time of the fire Lampano still owed Jose,
evidenced by a promissory note, and the latter still owed Barretto on the cost of the construction.
After the completion of the house and sometime before it was destroyed, Barretto took out an
insurance policy upon it in his own name, with the consent of Jose. After its destruction, Barretto
from the insurance company. According to Lampano there was a verbal agreement between her
and Jose that the latter agreed to deliver to her the insurance policy on the building, however, she
did not learn that the policy was in the name of Barretto until after the fire. On the other hand,
Jose, denied that she agreed to transfer the policy of insurance to the Lampano claiming that the
insurance was taken out and paid for by Barretto before the sale of the house to the Lampano.
The insurance was entirely for the personal account and in the exclusive interest of Barretto. The
trial court, pursuant to the case filed before it, found that there was no privity of contract between
the Lampano and Barretto. The court decided the respective rights to the insurance money and
entered judgment against Barretto and in favor of Jose. This was done upon the theory that the
insurance policy was held in trust for Jose, and that any balance, resulting after deducting the
amount owing upon the construction contract and paid for premiums, belonged to her.

Issue:

Does the plaintiff have any right to recover from Barretto any portion of the insurance
money?

Ruling:

No. If Barretto had an insurable interest in the house, he could insure this interest for his
sole protection. The policy was in the name of Barretto alone. It was, therefore, a personal
contract between him and the company and not a contract which ran with the property.
According to this personal contract the insurance policy was payable to the insured without
regard to the nature and extent of his interest in the property, provided that he had, as we have
said, an insurable interest at the time of the making of the contract, and also at the time of the fire.
Where different persons have different interests in the same property, the insurance taken by
one in his own right and in his own interest does not in any way insure to the benefit of another.
In this case, Barretto assumed the responsibility for the insurance. The premiums were paid by
him without any agreement or right to recoup the amount paid therefor should no loss result to
the property. It would not, therefore, be in accordance with the law and his contractual
obligations to compel him to account for the insurance money, or any par thereof, to the plaintiff,
who assumed no risk whatever.

74
Ins u ra bl e I n t e r est i n P ro p e rt y

MRS. HENRY E. HARDING v. COMMERCIAL UNION ASSURANCE COMPANY


G.R. No. L-12707
August 10, 1918

Fisher, J.

Facts:

Mrs. Henry E. Harding was the owner of a Studebaker automobile. In consideration of the
payment to the Commercial Union Assurance Company (Commercial) of the premium by Mrs. Harding,
with the consent of her husband, Commercial by its duly authorized agent, Smith, Bell & Company, made
its policy of insurance in writing upon said automobile. The said automobile was totally destroyed by fire.
Thereafter, within the period mentioned in the said policy of insurance, Mrs. Harding, furnished the
Commercial the proofs of her said loss and interest, and otherwise performed all the conditions of said
policy on her part. Commercial however, has not paid said loss nor any part thereof, although due demand
was made upon it by Mrs. Harding. According to Commercial, the warranties, statements, and
representations were false. Pursuant to this, the trial court held that there was no proof of fraud on the
part of Mrs. Harding in her statement of the value of the automobile, or with respect to its ownership; that
she had an insurable interest therein; and that Commecial, having agreed to the estimated value and
having insured the automobile for that amount, upon the basis of which the premium was paid, is bound
by it and must pay the loss in accordance with the stipulated insured value.

Issue:

Is there an insurable interest on the part of Mrs. Harding over the automobile at the time of the
issuance of the policy?

Ruling:

Yes. The court below found, and the evidence shows, that the automobile was bought by Mrs.
Harding's husband a few weeks before the issuance of the policy in question for the sum of P2,800, and
that between that time and the issuance of the policy some P900 was spent upon it in repairs and
repainting. It is true that the printed form calls for a statement of the "price paid by the proposer," but the
court is of the opinion that it would be unfair to hold the policy void simply because the outlay represented
by the automobile was made by the Mr. Harding and not by his wife, to whom he had given the automobile.
It cannot be assumed that Commercial should not have issued the policy unless it were strictly true that
the price representing the cost of the machine had been paid by the insured and by no other person — that
it would no event insure an automobile acquired by gift, inheritance, exchange, or any other title not
requiring the owner to make a specific cash outlay for its acquisition.

By the terms of section 149 of the Act cited, the valuation in a policy of marine insurance is
conclusive if the insured had an insurable interest and was not guilty of fraud. The Supreme Court,
therefore, holds that plaintiff was the owner of the automobile in question and had an insurable interest
therein; that there was no fraud on her part in procuring the insurance; and that the valuation of the
automobile, for the purposes of the insurance, is binding upon the defendant corporation.

75
Ins u ra bl e I n t e r est i n P ro p e rt y

GARCIA & PNB v. THE HONGKONG FIRE & MARINE INSURANCE CO., LTD.
G.R. No. 20341
September 1, 1923

Johns, J.
Facts:
Domingo Garcia, then a merchant and owner of a bazaar known as "Las Novedades" entered into
a contract with the Hongkong Fire & Marine Insurance Co., LTD. (HFMI) whereby it insured his
merchandise in the sum of P15,000 at a premium of P300 per annum; HFMI issued its fire insurance policy
No. 1951 in favor of the Garcia, not on the merchandise in the building, but on the building which contained
the merchandise. Because of this, the policy does not contain the true agreement and intent of the parties.
Then, Garcia executed a mortgage to Philippine National Bank on the merchandise insured by the HFMI,
and that with the consent of the latter, the he endorsed the policy to the Bank. While the policy was in force
and effect, a fire took place which destroyed the merchandise in the building together with the building
itself. A demand was made upon the HFMI for the payment of P15,000, as provided for in the policy, and
that payment was refused.
Issue:
Did the HFMI knew that it was the merchandise which was insured and not the building, thereby
making Garcia entitled to recover from the merchandise?
Ruling:
No. Garcia had his dealings with the officials of the branch Bank at Legaspi where he was doing
business as a merchant, of which the officials of that Bank had knowledge. Under such facts, the
presumption of knowledge, if any, on the part of the Bank would be that the policy was on the merchandise.
Be that as it may, when the HFMI received the letter from the Bank, it knew from its own records that the
policy was issued on the building, and, as a matter of fair dealing, it should have notified the Bank that the
policy was on the building. It will be noted that the letters in question were all written several months
before the fire. Garcia wanted insurance upon a stock of goods, which he owned, and he received and paid
for a policy on a building, which he did not own, and while the policy was in force and effect, both the
building, which he did not own, and the stock of merchandise, which he did own, were completely
destroyed by fire. Garcia was a well-known merchant, and his merchandise was in the building described
in the policy.
Under these circumstances it seems clear and manifest that the insured, as well as the manager of
the National Bank at Legaspi, who was interested in the policy, because the same secured a loan of P6,000
made to Domingo Garcia, and the corporation of Wise & Co., Ltd., which represented the insurance
company, have been in the belief that it was not the building but the merchandise that was insured, for the
reason that none of them paid attention to the context of the policy.

76
Ins u ra bl e I n t e r est i n P ro p e rt y

NILO CHA and STELLA UY CHA, and UNITED INSURANCE CO., INC v. COURT OF APPEALS
and CKS DEVELOPMENT CORPORATION
G.R. No. 124520
August 18, 1997

Padilla, J.

Facts:

Spouses Nilo Cha and Stella Uy-Cha and CKS Development Corporation entered a 1-year lease
contract with a stipulation not to insure against fire the chattels, merchandise, textiles, goods and effects
placed at any stall or store or space in the leased premises without first obtaining the written consent and
approval of the lessor. Without due regard to the agreement, Sps. Cha insured against loss by fire their
merchandise inside the leased premises for P500,000 with the United Insurance Co., Inc. without the
written consent of CKS. On the day the lease contract was to expire, fire broke out inside the leased
premises and CKS, upon learning that the spouses procured an insurance, wrote to United to have the
proceeds be paid directly to them. But United refused so CKS filed against Spouses Cha and United.

Issue:

Do the Spouses Cha have insurable interest despite violation of the lease agreement against
insurance over the leased premised without the consent and approval of the lessor?

Ruling:

Yes, this notwithstanding the spouses’ separate liability for breach of lease. Sec. 18. No contract or
policy of insurance on property shall be enforceable except for the benefit of some person having an
insurable interest in the property insured A non-life insurance policy such as the fire insurance policy
taken by petitioner-spouses over their merchandise is primarily a contract of indemnity. Insurable interest
in the property insured must exist a t the time the insurance takes effect and at the time the loss occurs.
The basis of such requirement of insurable interest in property insured is based on sound public policy: to
prevent a person from taking out an insurance policy on property upon which he has no insurable interest
and collecting the proceeds of said policy in case of loss of the property. In such a case, the contract of
insurance is a mere wager which is void under Section 25 of the Insurance Code. SECTION 25. Every
stipulation in a policy of Insurance for the payment of loss, whether the person insured has or has not any
interest in the property insured, or that the policy shall be received as proof of such interest, and every
policy executed by way of gaming or wagering, is void Section 17. The measure of an insurable interest in
property is the extent to which the insured might be damnified by loss of injury thereof. The automatic
assignment of the policy to CKS under the provision of the lease contract previously quoted is void for
being contrary to law and/or public policy. The proceeds of the fire insurance policy thus rightfully belong
to the spouses. The liability of the Cha spouses to CKS for violating their lease contract in that Cha spouses
obtained a fire insurance policy over their own merchandise, without the consent of CKS, is a separate and
distinct issue which we do not resolve in this case.

77
Ins u ra bl e I n t e r est i n P ro p e rt y

GAISANO CAGAYAN, INC. vs INSURANCE COMPANY OF NORTH AMERICA


G.R. NO. 147839
June 8, 2006

Austria-Martinez, J.

Facts:

Gaisano Cagayan, Inc. (Gaisano) is a customer and dealer of the products of Intercapitol Marketing
Corporation (IMC), maker of Wrangler Jeans, and Levi Strauss (Phils) Inc, (LSPI). IMC and LSPI obtained
from Insurance Company of North America (ICNA) fire insurance policies with book debt endorsements.
The insurance policies provide for coverage on “book debts in connection with ready-made clothing
materials which have been sold or delivered to various customers and dealers of the Insured anywhere in
the Philippines”. Thereafter, Gaisano Superstore Complex was consumed by fire. Included in the items lost
or destroyed in the fire were stocks of ready-made clothing materials sold and delivered by IMC and LSPI.
Pursuant to this, ICNA filed a complaint for damages against Gaisano, with respect to the claims of IMC and
LSPI under the respective fire insurance policies. The Regional Trial Court dismissed ICNA’s complaint
ratiocinating that the fire was purely accidental and the cause of fire is not attributable to the negligence of
Gaisano. On appeal, the Court of Appeals reversed RTC’s decision.

Issue:

Do IMC and LSPI have an insurable interest?

Ruling:

Yes. The insurable interest of IMC and LSPI pertain to the unpaid accounts appearing in their
books of account 45 days after the time of the loss covered by the policies When the words of a contract
are plain and readily understood, there is no room for construction. The questioned insurance policies
provide coverage for “book debts in connection with ready-made clothing materials which have been sold
or delivered to various customers and dealers of the Insured anywhere in the Philippines”.

Nowhere is it provided in the questioned insurance policies that the subject of the insurance is the
goods sold and delivered to the customers and dealers of the insured. Thus, what were insured against
were the accounts of IMC and LSPI with petitioner w/c remained unpaid 45 days after the loss through
fire, and not the loss or destruction of the goods delivered. The present case falls under par (1), Art. 1504
CC: Unless otherwise agreed, the goods remain at the seller's risk until the ownership therein is transferred
to the buyer, but when the ownership therein is transferred to the buyer the goods are at the buyer's risk
whether actual delivery has been made or not, except that: Where delivery of the goods has been made to
the buyer or to a bailee for the buyer, in pursuance of the contract and the ownership in the goods has been
retained by the seller merely to secure performance by the buyer of his obligations under the contract, the
goods are at the buyer's risk from the time of such delivery; IMC and LSPI did not lose complete interest
over the goods. They have an insurable interest until full payment of the value of the delivered goods. In
property insurance, one’s interest is not determined by concept of title, but whether insured has
substantial economic interest in the property. In this case, the insurable interest of IMC and LSPI pertain
to the unpaid accounts appearing in their books of account 45 days after the time of the loss covered by
the policies.

78
Ins u ra bl e I n t e r est i n P ro p e rt y

MALAYAN INSURANCE CO., INC. vs PHILIPPINES FIRST INSURANCE CO., INC. and
REPUTABLE FORWARDER SERVICES, INC.
G.R. NO. 184300
July 11, 2012

Reyes, J.
Facts:

Wyeth Philippines, Inc. (Wyeth) forged a contract of carriage with Reputable Forwarder Services,
Inc. (Reputable), a common carrier for the transport of its goods and product. Wyeth insured the goods
with Philippine First Insurance Co., Inc., while Republic insured the same goods with Malayan Insurance
Co, Inc. (Malayan). During transit, certain goods were lost due to hijacking of 10 armed men. Philippine
first paid the proceeds to Wyeth, subrogating the rights of Wyeth to Philippine first which filed a claim
against Reputable and Malayan as a 3rd party defendant. Reputable and Malayan refused the claim of
Philippine first. Malayan contended that there was double insurance and that the first insurer, Philippine
First, should bear all the loss.

Issue:

Is there double insurance in this case thereby justifying Malayan and Reputable’s refusal to the
claim?

Ruling:

No. The interest of Wyeth over the property subject matter of both insurance contracts is also
different and distinct from that of Reputable s. The policy issued by Philippines First was in consideration
of the legal and/or equitable interest of Wyeth over its own goods. On the other hand, what was issued by
Malayan to Reputable was over the latter s insurable interest over the safety of the goods, which may
become the basis of the latter s liability in case of loss or damage to the property and falls within the
contemplation of Section 15 of the Insurance Code. Therefore, even though the two concerned insurance
policies were issued over the same goods and cover the same risk, there arises no double insurance since
they were issued to two different persons/entities having distinct insurable interests. Necessarily, over
insurance by double insurance cannot likewise exist. Hence, as correctly ruled by the RTC and CA, neither
Section 5 nor Section 12 of the SR Policy can be applied.

79
Dou bl e I ns ur a nc e a n d Ov e r I nsu ra nc e

ULPIANO SANTA ANA v. COMMERCIAL UNION ASSURANCE COMPANY, LTD.


G.R. No. L-32889
November 20, 1930

Villa-Real, J

Facts:

In 1923, Ulpiano Sta. Ana built a house of strong materials with a galvanized iron roof on A. Luna
Street, Pasig, Rizal. On October 1, 1925, the plaintiff Ulpiano Santa Ana took out a three-thousand peso fire
insurance policy on the house in the Phoenix Assurance Company, and six thousand-peso policy in the
Guardian Assurance Company, Limited, for a period of one year from that date until October 1, 1926,
paying the respective premiums of P97.50 and P196 to said companies through their duly authorized
Philippine agent, Kerr & Company.

On November 19, 1925, the plaintiff Ulpiano Santa Ana mortgaged said house to the plaintiff
Rafael Garcia, and the policies issued by the Phoenix Assurance Company and the Guardian Assurance
Company, Limited, were endorsed to the mortgagee, Rafael Garcia. On December 16, 1925, the plaintiff
Urpiano Santa Ana reinsured said house with the defendant companies, the Globe and Rutgers Fire
Insurance Company of New York, and the Commercial Union Assurance Company, Limited of London,
through their common agent duly authorized to represent them in the Philippine Islands, the Pacific
Commercial Company, for the amount of P3,000 each, paying the 90-peso premium due upon each policy,
which was to be effective for one year from the aforementioned date until December 16, 1926

On September 20, 1926, Ulpiano Santa Ana took out another insurance policy on the house in
question for P6,000 in the "Filipinas, Compania de Seguros, which issued the oneyear policy, upon
receiving from said plaintiff the amount of P195 as premium thereon.

In the morning of October 1, 1926, that is, twelve hours before the expiration of the policies issued
by the Phoenix Assurance Company and the Guardian Assurance Company, Limited for P3,000 and P6,000
respectively, a fire broke out in the insured house. Ulpiano Santa Ana gave notice in due time of the loss to
each and every one of the companies in which he had insured the house and demanded payment of the
respective policies; the assurance companies refused payment on the ground that the claim of P21,000
filed by him was fraudulent, being in excess of the real value of the insured property; that none of said
companies had been informed of the existence of the other policies in the other companies, and that the
fire was intentional. Ulpiano Santa Ana therefore brought the actions which gave rise to these cases.

Issue:

Is Sta. Ana entitled to claim from the insurers?

Ruling:

No. It has been provided in English and Spanish in notices attached to the insurance policies issued
by the Phoenix Assurance Co., Ltd., and the Guardian Assurance Company upon the first of October, 1925
, and by the Globe and Rutgers Fire Insurance Company of New York, and the Commercial Union,
Company, upon the 19th of December of the same year, that no other insurance should be admitted upon
the property thereby assured without the consent of said companies duly given by endorsement, and
whereas the first two policies were taken out on one and the same date, namely, October 1st, and the last
two likewise upon one and the same date, that is, December 16, 1925, the insured plaintiff, Ulpiano Sta.
Ana, has not stated in the last two policies that his property had previously been insured with the Phoenix
and the Guardian Assurance Company, and still less in the insurance policy of the "Compania Filipinas"
taken out on September 20 of the following year, 1926, nor has he obtained upon the first two policies the
necessary endorsements for the three subsequent insurance policies. It should be noted that clause three

80
Dou bl e I ns ur a nc e a n d Ov e r I nsu ra nc e

of the "Filipinas" policy drawn up in Spanish, and the english policies issued by the four other companies,
provided that any outstanding insurance upon the whole or a portion of the objects thereby assured must
be declared by the insured in writing and he must cause the company to add or insert it in the policy,
without which such policy shall be null and avoid, and the insured will not be entitled to indemnity in case
of loss.

81
Dou bl e I ns ur a nc e a n d Ov e r I nsu ra nc e

PIONEER INSURANCE & SURETY CORPORATION v. COURT OF APPEALS


G.R. No. 84197
July 28, 1989

Gutierrez, Jr., J.

Facts:

On May 17, 1965, at Tokyo, Japan, Japan Domestic Airlines (JDA) and Lim entered into and
executed a sales contract for the sale and purchase of two aircrafts and one set of necessary spare parts for
the total agreed price of US $109,000.00 to be paid in instalments. One DC-3 Aircraft with Registry No. PIC-
718, arrived in Manila on June 7,1965 while the other aircraft, arrived in Manila on July 18,1965.

On May 22, 1965, Pioneer Insurance and Surety Corporation as surety executed and issued its
Surety Bond No. 6639in favor of JDA, in behalf of its principal, Lim, for the balance price of the aircrafts and
spare parts.

It appears that Border Machinery and Heavy Equipment Company, Inc., Francisco and Modesto
Cervantes and Constancio Maglana contributed some funds used in the purchase of the above aircrafts
and spare parts. The funds were supposed to be their contributions to a new corporation proposed by Lim
to expand his airline business. They executed two (2) separate indemnity agreements in favor of Pioneer,
one signed by Maglana and the other jointly signed by Lim for SAL, Bormaheco and the Cervanteses. The
indemnity agreements stipulated that the indemnitors principally agree and bind themselves jointly and
severally to indemnify and hold and save harmless Pioneer from and against any/all damages, losses,
costs, damages, taxes, penalties, charges and expenses of whatever kind and nature which Pioneer may
incur in consequence of having become surety upon the bond/note and to pay, reimburse and make good
to Pioneer, its successors and assigns, all sums and amounts of money which it or its representatives
should or may pay or cause to be paid or become liable to pay on them of whatever kind and nature.

On June 10, 1965, Lim doing business under the name and style of SAL executed in favor of Pioneer
as deed of chattel mortgage as security for the latter's suretyship in favor of the former. It was stipulated
therein that Lim transfer and convey to the surety the two aircrafts.
Lim defaulted on his subsequent installment payments prompting JDA to request payments from the
surety. Pioneer paid a total sum of P298,626.12.

Issue:

Do the respondents have interest in the reinsurance?

Ruling:

In the case of Phil. Air Lines, Inc. v. Heald Lumber Co., it was held that if a property is insured and
the owner receives the indemnity from the insurer, it is provided in said article that the insurer is deemed
subrogated to the rights of the insured against the wrongdoer and if the amount paid by the insurer does
not fully cover the loss, then the aggrieved party is the one entitled to recover the deficiency. Evidently,
under this legal provision, the real party in interest with regard to the portion of the indemnity paid is the
insurer and not the insured.
It is clear from the records that Pioneer sued in its own name and not as an attorneyin-fact of the reinsurer.
Accordingly, the appellate court did not commit a reversible error in dismissing the petitioner's complaint
as against the respondents for the reason that the petitioner was not the real party in interest in the
complaint and, therefore, has no cause of action against the respondents.

82
Sp ec ia l P r ovi si on s o n M o rt ga g o r a n d Mo r tga g e e

ARMANDO GEAGONIA v. COURT OF APPEALS


G.R. No. 114427
February 6, 1995

Davide, Jr., J

Facts:

Geagonia is the owner of Norman’s Mart located in the public market of San Francisco, Agusan Del
Sur. On December 22, 1989, he obtained from Country Bankers Insurance Corporation a fire insurance
policy for P100,000.00. The period of the policy was from December 22, 1989 to December 22, 1990 and
covered the following: “Stock-in-Trade consisting principally of dry goods such as RTW’s for men and
women wear and other usual to assured’s business.”

The petitioner declared in the policy under the subheading entitled CO-INSURANCE that
Mercantile Insurance Co., Inc was the co-insurer for P50,000.00.

On May 27,1990, fire of accidental origin broke out around 7:30 p.m. at the public market of San
Francisco, Agusan del Sur. The petitioner’s insured stock-in-trade were completely destroyed prompting
him to file with the private respondent a claim under the policy in which respondent denied because it
found that at the time of the loss the petitioner’s stock-and-trade were covered by another fire insurance
policy. The basis of the private respondent’s denial was the petitioner’s alleged violation of a condition in
the policy.

Issue:

Is the petitioner precluded from recovering from the insurer.

Ruling:

Yes. Separate insurances covering different insurable interests may be obtained by the mortgagor
and the mortgagee. The fire insurance policies issued by the PFIC name the petitioner as the assured and
contain a mortgage clause.

In the policy obtained by the mortgagor with loss payable clause in favour of the mortgagee as his
interest may appear, the mortgagee is only a beneficiary under the contract, and recognized as such by the
insurer but not made a party to the contract himself. Hence, any acts of the mortgagor which defeats his
right will also defeat the right of the mortgagee. This kind of policy covers only such interest as the
morgagee has at the issuing of the policy.

A mortgagee may also procure a policy as a contracting party in accordance with the terms of an
agreement by which the mortgagor is to pay the premiums upon such insurance. It has been noted, that
although the mortgagee is himself the insured, as where he applies for a policy, fully informs the authorized
agent of his interests, pays the premiums, and obtains on the assurance that it insures him, the policy is in
fact in the form used to insure a mortgagor with loss payable clause.

83
P er f ec ti o n of C o nt ra c t: Co ns e nsua l ity of C o nt r a cts o f In su ra nc e

BONIFACIO BROS. INC. v. MORA


G.R. L – 20853
May 29, 1967

Castro, J.

Facts:

Enrique Mora, owner of Oldsmobile sedan model 1956, mortgaged the same to H.S. Reyes Inc.,
with the condition that the former would insure the automobile with the latter as beneficiary. The
automobile was thereafter insured with the State Bonding & Insurance Co., Inc. and motor car insurance
policy was issued to him. During the effectivity of the insurance contract, the car met with an accident. The
insurance company then assigned the accident to the Bayne Adjustment Co. for investigation and appraisal
of the damage. Mora without the knowledge and consent of HS Reyes, authorized Bonifacio Bros to fix the
car, using materials supplied by the Ayala Auto Parts Company. For the cost of Labor and materials,
respondent was billed P2,102.73. The bill was sent to the insurer’s appraiser. The insurance company
drew a check in the amount of the insurance proceeds and entrusted the check to its appraiser for delivery
to the proper party. The car was delivered to respondent without the consent of HS Reyes, and without
payment to Bonifacio Bros and Ayala. Upon the theory that the insurance proceeds should be directly paid
to them, Bonifacio and Ayala filed a complaint against respondent and the insurer with the MTC for
collection of P2,102.73.

Issue:

Whether or not there is privity of contract between Bonifacio and Ayala on one hand and State
Insurance on the other

Ruling:

No. The court held that it is fundamental that contracts take effect only between the parties
thereto, except in some specific instance provided by law where the contract contains some stipulation in
favor of a third person. Such stipulation is known as stipulation pour autri or a stipulation in favor of a
third person not a party to the contract. Under this doctrine, a third person is able to avail himself of a
benefit granted to him by the terms of the contract, provided that the contracting parties have clearly and
deliberately conferred a favor upon such person. A third person not a party to the contract has no action
against the parties thereto, and cannot generally demand the enforcement of the same. The trial court
correctly held that no cause of action exists in favor of the appellants in so far as the proceeds of insurance
are concerned. The appellant’s claim, if at all, is merely equitable in nature and must be made effective
through Enrique Mora who entered into a contract with Bonifacio Bros Inc. This conclusion is deducible
not only from the principle governing the operation and effect of insurance contracts in general, but is
clearly covered by the express provision of Section 50 of the Insurance Act.

84
P er f ec ti o n of C o nt ra c t: Co ns e nsua l ity of C o nt r a cts o f In su ra nc e

MANILA MAHOGANY v. COURT OF APPEALS


G.R. No. L-52756
October 12, 1987

Padilla, J

Facts:

From 6 March 1970 to 6 March 1971, petitioner insured its Mercedes Benz sedan with
respondent insurance company. On 4 May 1970 the insured vehicle was bumped and damaged by a truck
owned by San Miguel Corporation. For the damage caused, respondent company paid petitioner five
thousand pesos (P5, 000.00) in amicable settlement. On 11 December 1972, respondent company wrote
Insurance Adjusters, Inc. to demand reimbursement from San Miguel Corporation of the amount it had
paid petitioner. Insurance Adjusters, Inc. refused reimbursement, alleging that San Miguel Corporation
had already paid petitioner P4, 500.00 for the damages to petitioner's motor vehicle. Respondent
insurance company thus demanded from petitioner reimbursement of the sum of P4, 500.00 paid by San
Miguel Corporation however petitioner refused. Respondent then filed suit for the recovery of the P4,
500.00. Petitioner forwards since total damages were valued by petitioner at P9, 486.43 and only P5,
000.00 was received by petitioner from respondent, petitioner argues that it was entitled to go after San
Miguel Corporation to claim the additional P4, 500.00.

Issue:

Can the sum paid by SMC to petitioner be recovered by respondent?

Ruling:

Yes. The Court held that since petitioner by its own acts released San Miguel Corporation, thereby
defeating private respondents’ right of subrogation, the right of action of petitioner against the insurer was
also nullified. Petitioner is entitled to keep the sum of P4, 500.00 paid by San Miguel Corporation under its
clear right to file a deficiency claim for damages incurred, against the wrongdoer, should the insurance
company not fully pay for the injury caused. However, when petitioner released San Miguel Corporation
from any liability, petitioner's right to retain the sum of P5, 000.00 no longer existed, thereby entitling
private respondent to recover the same.
Since the insurer can be subrogated to only such rights as the insured may have, should the
insured, after receiving payment from the insurer, release the wrongdoer who caused the loss, the insurer
loses his rights against the latter. But in such a case, the insurer will be entitled to recover from the insured
whatever it has paid to the latter, unless the release was made with the consent of the insurer.

85
P er f ec ti o n of C o nt ra c t: Co ns e nsua l ity of C o nt r a cts o f In su ra nc e

MALAYAN INSURANCE v. COURT OF APPEALS


G.R. No. L-36413
September 26, 1988

Padilla, J

Facts:

On 29 March 1967 Malayan Insurance, issued in favor of private respondent Sio Choy a Private
Car Comprehensive Policy. The insured jeep then, while being driven by one Juan P. Campollo an employee
of the respondent San Leon Rice Mill, collided with a passenger bus belonging to the respondent
Pangasinan Transportation causing damage to the jeep and to respondent Martin C. Vallejos, who was
riding in the jeep.

Martin C. Vallejos then filed an action for damages against Sio Choy, Malayan Insurance and the
PANTRANCO for damages. Sio Choy later filed a separate answer with a cross-claim against petitioner
wherein he alleged that he paid for the hospitalization and other fees of the plaintiff. Herein petitioner also
filed third-party complaint against the San Leon Rice Mill for the reason that the person driving the jeep of
Sio Choy, at the time of the accident, was its employee.

Issue:

Should Petitioner be reimbursed by respondent?

Ruling:

Yes, there was subrogation. The Court held that the respondents Sio Choy and San Leon Rice Mill
are the principal tortfeasors who are primarily liable to respondent Vallejos. Respondent Sio Choy is made
liable to said plaintiff as owner of the jeep. San Leon Rice Mill, Inc. is liable to Vallejos, for it being the
employer of the driver of the jeep at the time of the motor vehicle mishap.

While it is true that where the insurance contract provides for indemnity against liability to third
persons, such third persons can directly sue the insurer, however, the direct liability of the insurer under
indemnity contracts against third party liability does not mean that the insurer can be held solidarily liable
with the insured. The right of subrogation is of the highest equity. The loss in the first instance is that of
the insured but after reimbursement or compensation, it becomes the loss of the insurer. It follows,
therefore, that petitioner, upon paying respondent Vallejos the amount of riot exceeding P20,000.00, shall
become the subrogee of the insured, the respondent Sio Choy; as such, it is subrogated to whatever rights
the latter has against respondent San Leon Rice Mill, Inc. as such petitioner is entitled to reimbursement
from respondent San Leone Rice Mill.

86
P er f ec ti o n of C o nt ra c t: Co ns e nsua l ity of C o nt r a cts o f In su ra nc e

EASTERN SHIPPING LINES v. PRUDENTIAL GUARANTEE AND ASSURANCE


G.R. No. 174116
September 11, 2009

Peralta, J

Facts:

On November 8, 1995, fifty-six cases of Nissan auto parts were loaded on board M/V Apollo Tujuh
at Nagoya, Japan, to be shipped to Manila. The shipment was consigned to Nissan Motor Philippines. The
carrier was owned and operated by petitioner Eastern Shipping Lines. The carrier then arrived at Manila
and the shipment was then discharged from the vessel onto the custody of the arrastre operator, Asian
Terminals.

On November 24 to 28, 1995, the shipment was delivered to the warehouse of Nissan in Quezon
City. A survey of the shipment was then conducted. On the survey it was found that there were missing
and damaged items the surveyor opined that the shortage and damage sustained by the shipment were
due to pilferage and improper handling, respectively while in the custody of the vessel and/or Arrastre
Contractors."

Nissan demanded the sum of ₱1,047,298.34 representing the cost of the damages sustained by
the shipment from petitioner, the owner of the vessel, and ATI, the arrastre operator. As insurer of the
shipment against all risks, respondent Prudential Guarantee and Assurance paid Nissan the sum of
₱1,047,298.34. Respondent then sued petitioner and ATI for reimbursement of the amount it paid to
Nissan before the Regional Trial Court. The RTC held Eastern Shipping Lines, Inc. and ATI liable. On appeal,
The CA exonerated ATI and ruled that petitioner was solely responsible for the damages caused to the
cargoes.

Issue:

Should Petitioner pay out the reimbursement?

Ruling:

No for a marine risk note is not an insurance policy. It is only an acknowledgment or declaration
of the insurer confirming the specific shipment covered by its marine open policy, the evaluation of the
cargo and the chargeable premium.

The Marine Risk Note on its face does not specify when the insurance was constituted. It is
significant that the date when the alleged insurance contract was constituted cannot be established with
certainty without the contract itself. Said point is crucial because there can be no insurance on a risk that
had already occurred by the time the contract was executed. The Court rules that based on the applicable
jurisprudence, because of the inadequacy of the Marine Cargo Risk Note, it was incumbent on respondent
to present in evidence the Marine Insurance Policy, and having failed in doing so, its claim of subrogation
must necessarily fail.

On November 14, 1995, Shandong Weifang Soda Ash Plant shipped on board the vessel MV
"Jinlian I" 60,000 plastic bags of soda ash dense from China to Manila. The shipment was insured with
respondent Malayan Insurance Company, Inc.. On November 21, 1995, upon arrival of the vessel in Manila,
the stevedores of petitioner Asian Terminals, Inc., unloaded the 60,000 bags of soda ash dense from the
vessel and brought them to the open storage area of petitioner for temporary storage and safekeeping.
When the unloading of the bags was completed on November 28, 1995, 2,702 bags were found to be in
bad order condition. On November 29, 1995, the stevedores began loading the bags in the trucks of MEC
Customs Brokerage for transport and delivery to the consignee. After all the bags were unloaded in the

87
P er f ec ti o n of C o nt ra c t: Co ns e nsua l ity of C o nt r a cts o f In su ra nc e

warehouses of the consignee, a total of 2,881 bags were in bad order condition due to spillage, caking, and
hardening of the contents. Insurer paid the value of the lost/ damaged cargoes to the consignee in the
amount of P643,600.25. Respondent, as subrogee of the consignee, filed before the RTC of Manila a
complaint for damages against petitioner (Asian Terminals, Inc.), the shipper (Inchcape Shipping
Services), and the cargo broker (MEC Customs Brokerage). The RTC rendered a decision finding petitioner
liable for the damage/loss sustained by the shipment but absolving the other defendants - Inchcape
Shipping Services and MEC Customs Brokerage.

88
P er f ec ti o n of C o nt ra c t: Co ns e nsua l ity of C o nt r a cts o f In su ra nc e

ASIAN TERMINALS, INC. v. MALAYAN INSURANCE, CO., INC.,


G.R. No. 171406
April 4, 2011

Del Castillo, J.

Facts:

On November 14, 1995, Shandong Weifang Soda Ash Plant shipped on board the vessel MV
"Jinlian I" 60,000 plastic bags of soda ash dense from China to Manila. The shipment was insured with
respondent Malayan Insurance Company, Inc.. On November 21, 1995, upon arrival of the vessel in Manila,
the stevedores of petitioner Asian Terminals, Inc., unloaded the 60,000 bags of soda ash dense from the
vessel and brought them to the open storage area of petitioner for temporary storage and safekeeping.
When the unloading of the bags was completed on November 28, 1995, 2,702 bags were found to be in
bad order condition. On November 29, 1995, the stevedores began loading the bags in the trucks of MEC
Customs Brokerage for transport and delivery to the consignee. After all the bags were unloaded in the
warehouses of the consignee, a total of 2,881 bags were in bad order condition due to spillage, caking, and
hardening of the contents. Insurer paid the value of the lost/ damaged cargoes to the consignee in the
amount of P643,600.25. Respondent, as subrogee of the consignee, filed before the RTC of Manila a
complaint for damages against petitioner (Asian Terminals, Inc.), the shipper (Inchcape Shipping
Services), and the cargo broker (MEC Customs Brokerage). The RTC rendered a decision finding petitioner
liable for the damage/loss sustained by the shipment but absolving the other defendants - Inchcape
Shipping Services and MEC Customs Brokerage.

Issue:

Whether the non-presentation of the insurance contract or policy is fatal to respondent’s cause of
action.

Ruling:

No. The court ruled that non-presentation of the insurance contract or policy is not necessarily
fatal. Citing Delsan Transport Lines, Inc. v. Court of Appeals, the presentation in evidence of the marine
insurance policy is not indispensable before the insurer may recover from the common carrier the insured
value of the lost cargo in the exercise of its subrogatory right. The subrogation receipt, by itself, is sufficient
to establish not only the relationship of the insurer and the assured shipper of the lost cargo of industrial
fuel oil, but also the amount paid to settle the insurance claim. The right of subrogation accrues simply
upon payment by the insurance company of the insurance claim.

89
P er f ec ti o n of C o nt ra c t: F or m o f I nsu ra nc e C on t ra ct s

VIOLA BADGER v. NEW YORK LIFE INSURANCE COMPANY


G.R. No. 3069
January 23, 1907

Willard, J

Facts:

On July 5, 1902, William Badger made out a written application for a policy of insurance upon his
life for $5,000.00 in favour of his wife Harriet Viola Badger. Herdman sent the papers on July 24 to the
office of the defendant company in Shanghai, where they were received on August 11. Badger executed a
promissory note for $14.90, the balance of the first premium, which was sent Herdman on July 17, 1902.
On July 31, Mrs. Badger, acting for her husband, sent to Herdman cash on payment of the note. Badger died
on August 1, 1902, of cholera.

No policy was ever issued upon his application.

The plaintiff brought this action to recover the sum of $5,000, alleging that a contract of insurance
had been made by the company with Badger. The lower court ruled in favour of the defendant.

Issue:

Was there a perfected contract of insurance between the parties?

Ruling:

No. The evidence shows conclusively that there was no parol agreement between the parties that
the insurance had commenced on July 5, 1902. Moreover, there is evidence showing that the company
expressly refused to be bound until the application had been accepted either by its office in New York.

The court found that the receipt was sent upon the blank which contained a reference to the
Shanghai office. Whether it was upon this form of receipt or upon the other one is of no consequence. In
one of the receipt, it is stated that the company shall incur no liability under the application until it has been
received, approved by the resident board of the company at Shanghai, and a policy issued thereon by the
resident board, and the full premium has actually been paid to and accepted by the company or its
authorized agent during the lifetime and good health of the person upon whose life the insurance is applied
for. The company reserves the absolute right of disapproval of such application.

The other form contains the statement that the company shall incur no liability under the
application until it has been received, approved at the house office of the company, and a policy issued
thereon. Hence, it’s clear that no liability has incurred by the company.

90
P er f ec ti o n of C o nt ra c t: Pa y m e nt o f P r e mi um

GULF RESORTS, INC., v. PCIC


G.R NO. 156167
May 16, 2005

Puno, J.

Facts:

Petitioner is the owner of a resort situated in Agoo, La Union which was originally insured
American Home Assurance Company. In the first four insurance policies issued by respondent, the risk of
loss from earthquake shock was extended only to plaintiffs two swimming pools, thus, earthquake shock.

In consideration of the payment by the insured to the company of the sum included additional
premium the Company agrees, notwithstanding what is stated in the printed conditions of this policy due
to the contrary, that this insurance covers loss or damage to shock to any of the property insured by this
Policy occasioned by or through or in consequence of earthquake. On July 16, 1990, an earthquake struck
Central Luzon and Northern Luzon and plaintiffs properties covered by Policy issued by defendant,
including the two swimming pools in its Agoo Playa Resort were damaged. After the earthquake,
petitioner advised respondent that it would be making a claim under its Insurance Policy for damages on
its properties. On August 11, 1990, petitioner filed its formal demand for settlement of the damage to all
its properties. On August 23, 1990, respondent denied petitioners claim on the ground that its insurance
policy only afforded earthquake shock coverage to the two swimming pools of the resort.

Petitioner filed a complaint. Petitioner contends, among others that pursuant to this rider, no
qualifications were placed on the scope of the earthquake shock coverage. Thus, the policy extended
earthquake shock coverage to all of the insured properties.

Issue:

Whether or not the policy extended to all the properties of the insured.

Ruling:

An insurance premium is the consideration paid an insurer for undertaking to indemnify the
insured against a specified peril. In fire, casualty, and marine insurance, the premium payable becomes a
debt as soon as the risk attaches. In the subject policy, no premium payments were made with regard to
earthquake shock coverage, except on the two swimming pools.
There is no mention of any premium payable for the other resort properties with regard to earthquake
shock. There is no ambiguity in the terms of the contract and its riders. Petitioner cannot rely on the
general rule that insurance contracts are contracts of adhesion which should be liberally construed in
favor of the insured and strictly against the insurer company which usually prepares it. Consequently, any
ambiguity therein is resolved against the insurer, or construed liberally in favor of the insured.

91
P er f ec ti o n of C o nt ra c t: Pa y m e nt o f P r e mi um

UCPB GENERAL INSURANCE CO., INC. v. MASAGANA TELEMART, INC


G.R No. 137172
April 4, 2001

Davide, Jr., C.J.

Facts:

Respondent obtained from Petitioner five insurance policies on its properties. All five policies
reflect on their face the effectivity term: "from 4:00 P.M. of 22 May 1991 to 4:00 P.M. of May 22, 1992." On
June 13, 1992, respondent 's properties were razed by fire. On July 13, 1992, respondent tendered, and
petitioner accepted, five Checks in the total amount of P225,753.45 as renewal premium payments for
which was issued by petitioner. On July 14, 1992, Masagana made its formal demand for indemnification
for the burned insured properties.

On the same day, petitioner returned the five checks stating in its letter (that it was rejecting
Masagana's claim on the following grounds: a) Said policies expired last May 22, 1992 and were not
renewed for another term; b) The properties covered by the said policies were burned in a fire that took
place last June 13, 1992, or before tender of premium payment." Both the Court of Appeals and the trial
court found that sufficient proof exists that Respondent, which had procured insurance coverage from
Petitioner for a number of years, had been granted a 60 to 90-day credit term for the renewal of the
policies. Such a practice had existed up to the time the claims were filed.

Further, the both courts found that for years, Petitioner had been issuing fire policies to the
Respondent, and these policies were annually renewed; there was no valid notice of non-renewal of the
policies in question, as there is no proof at all that the notice sent by ordinary mail was received by
Respondent, and the copy thereof allegedly sent to Zuellig was ever transmitted to Respondent.

Issue:

Are the fire insurance policies issued by petitioner to the respondent covering the period from
May 22, 1991 to May 22, 1992 had been extended or renewed by an implied credit arrangement though
actual payment of premium?

Ruling:

Yes. Section 77 of the Insurance Code of 1978 provides that: An insurer is entitled to payment of
the premium as soon as the thing insured is exposed to the peril insured against. Notwithstanding any
agreement to the contrary, no policy or contract of insurance issued by an insurance company is valid and
binding unless and until the premium thereof has been paid, except in the case of a life or an industrial life
policy whenever the grace period provision applies.

In turn, this Section has its source in Section 72 of Act No. 2427 otherwise known as the Insurance
Act as amended by R.A. No. 3540, approved on 21 June 1963, which read: SEC. 72. An insurer is entitled to
payment of premium as soon as the thing insured is exposed to the peril insured against, unless there is
clear agreement to grant the insured credit extension of the premium due.
No policy issued by an insurance company is valid and binding unless and until the premium
thereof has been paid. It can be seen at once that Section 77 does not restate the portion of Section 72
expressly permitting an agreement to extend the period to pay the premium. But there are exceptions to
Section 77:

1. in case of a life or industrial life policy whenever the grace period provision applies.
2. is that covered by Section 78 of the Insurance Code, which provides: SEC. 78. Any
acknowledgment in a policy or contract of insurance of the receipt of premium is conclusive

92
P er f ec ti o n of C o nt ra c t: Pa y m e nt o f P r e mi um

evidence of its payment, so far as to make the policy binding, notwithstanding any stipulation
therein that it shall not be binding until premium is actually paid.
3. Section 77 may not apply if the parties have agreed to the payment in installments of the premium
and partial payment has been made at the time of loss. Section 77 merely precludes the parties
from stipulating that the policy is valid even if premiums are not paid, but does not expressly
prohibit an agreement granting credit extension, and such an agreement is not contrary to morals,
good customs, public order or public policy.
4. that the insurer may grant credit extension for the payment of the premium. This simply means
that if the insurer has granted the insured a credit term for the payment of the premium and loss
occurs before the expiration of the term, recovery on the policy should be allowed even though
the premium is paid after the loss but within the credit term.

Finally, it would be unjust and inequitable if recovery on the policy would not be permitted against
Petitioner, which had consistently granted a 60- to 90-day credit term for the payment of premiums
despite its full awareness of Section 77.

93
P er f ec ti o n of C o nt ra c t: Pr e m iu m Pa y m e nt in L i f e In su ra nc e

PEDRO ARCE v. THE CAPITAL INSURANCE & SURETY CO., INC.


G.R. No. L-28501
September 30, 1982

Abad Santos, J.

Facts:

The INSURED was the owner of a residential house in Tondo, Manila, which had been insured with
the COMPANY since 1961. On November 27, 1965, the COMPANY sent to the INSURED a Renewal
Certificate to cover the period December 5, 1965 to December 5, 1966. The COMPANY also requested
payment of the corresponding premium in the amount of P 38.10.

Anticipating that the premium could not be paid on time, the INSURED, thru his wife, promised to
pay it on January 4, 1966. The COMPANY accepted the promise but the premium was not paid on January
4, 1966. On January 8, 1966, the house of the INSURED was totally destroyed by fire.

On January 10, 1966, INSURED's wife presented a claim for indemnity to the COMPANY. She was
told that no indemnity was due because the premium on the policy was not paid. Nonetheless the
COMPANY tendered a check for P300.00 as financial aid which was received by the INSURED's daughter,
Evelina R. Arce. The voucher for the check which Evelina signed stated that it was "in full settlement (ex
gratia) of the fire loss." Thereafter the INSURED and his wife went to the office of the COMPANY to have
his signature on the check Identified preparatory to encashment. At that time the COMPANY reiterated
that the check was given "not as an obligation, but as a concession" because the renewal premium had not
been paid, The INSURED cashed the check but then sued the COMPANY on the policy.

The court a quo held that since the COMPANY could have demanded payment of the premium,
mutuality of obligation requires that it should also be liable on its policy. The court a quo also held that the
INSURED was not bound by the signature of Evelina on the check voucher because he did not authorize
her to sign the waiver.

Issue:

Is the insurer liable to pay the insured?

Ruling:

No. It is obvious from both the Insurance Act, as amended, and the stipulation of the parties that
time is of the essence in respect of the payment of the insurance premium so that if it is not paid the
contract does not take effect unless there is still another stipulation to the contrary. In the instant case, the
INSURED was given a grace period to pay the premium but the period having expired with no payment
made, he cannot insist that the COMPANY is nonetheless obligated to him.

The Court commiserates with the INSURED. We are well aware that many insurance companies
have fallen into the condemnable practice of collecting premiums promptly but resort to all kinds of
excuses to deny or delay payment of just claims. Unhappily the instant case is one where the insurer has
the law on its side.

94
P er f ec ti o n of C o nt ra c t: Pr e m iu m Pa y m e nt in L i f e In su ra nc e

ACME SHOE, RUBBER & PLASTIC CORPORATION AND CHUA PAC v. CA


G.R. No. 103576
August 22, 1996

Vitug, J.

Facts:

Chua Pac executed on June 1978, for and in behalf of the company, a chattel mortgage in favor of
private respondent Producers Bank of the Philippines. The mortgage stood by way of security for
petitioner's corporate loan of three million pesos.

In due time, the loan of P3,000,000.00 was paid by petitioner corporation. Subsequently, in 1981,
it obtained from respondent bank additional financial accommodations totalling P2,700,000.00. These
borrowings were on due date also fully paid. On January 1984, the bank yet again extended to petitioner
corporation a loan of one million pesos (P1,000,000.00) covered by four promissory notes for
P250,000.00 each. Due to financial constraints, the loan was not settled at maturity. Respondent bank
thereupon applied for an extra judicial foreclosure of the chattel mortgage, herein before cited, with the
Sheriff of Caloocan City, prompting Petitioner Corporation to forthwith file an action for injunction, with
damages and a prayer for a writ of preliminary injunction, before the Regional Trial Court of Caloocan City.
Ultimately, the court dismissed the complaint and ordered the foreclosure of the chattel mortgage. It held
Petitioner Corporation bound by the stipulations of the chattel mortgage.

Issue:

Is Petitioner bound by the stipulations in the chattel mortgage it entered into with Respondent
Bank.

Ruling:

Yes. Contracts of security are either personal or real. In contracts of personal security, such as a
guaranty or a suretyship, the faithful performance of the obligation by the principal debt or is secured by
the personal commitment of another. In contracts of real security, such as a pledge, a mortgage or an
antichresis, that fulfillment is secured by an encumbrance of property — in pledge, the placing of movable
property in the possession of the creditor; in chattel mortgage, by the execution of the corresponding deed
substantially in the form prescribed by law; in real estate mortgage, by the execution of a public instrument
encumbering the real property covered thereby; and in antichresis, by a written instrument granting to
the creditor the right to receive the fruits of an immovable property with the obligation to apply such fruits
to the payment of interest, if owing, and thereafter to the principal of his credit — upon the essential
condition that if the obligation becomes due and the debtor defaults, then the property encumbered can
be alienated for the payment of the obligation, but that should the obligation be duly paid, then the contract
is automatically extinguished proceeding from the accessory character of the agreement. As the law so
puts it, once the obligation is complied with, then the contract of security becomes, ipso facto, null and void.

Refusal on the part of the borrower to execute the agreement so as to cover the after-incurred
obligation can constitute an act of default on the part of the borrower of the financing agreement whereon
the promise is written but, of course, the remedy of foreclosure can only cover the debts extant at the time
of constitution and during the life of the chattel mortgage sought to be foreclosed.

95
P er f ec ti o n of C o nt ra c t: Pr e m iu m Pa y m e nt in L i f e In su ra nc e

CAPITAL INSURANCE v. PLASTIC ERA CO., INC.


G.R. No. L-22375
July 18, 1975

Martin, J.

Facts:

Capital Insurance & Surety Co., Inc. delivered to the respondent Plastic Era Manufacturing Co., Inc.,
its open Fire Policy wherein the former undertook to insure the latter's building, equipment, raw
materials, products and accessories located at Sheridan Street, Mandaluyong, Rizal. The policy expressly
provides that if the property insured would be destroyed or damaged by fire after the payment of the
premiums, at anytime between the 15th day of December 1960 and one o'clock in the afternoon of the
15th day of December 1961, the insurance company shall make good all such loss or damage in an amount
not exceeding P100, 000.00. When the policy was delivered, Plastic Era failed to pay the corresponding
insurance premium.

On January 1961, in partial payment of the insurance premium, Plastic Era delivered to Capital
Insurance, a check for the amount of P1,000.00, payable to the order of the latter and drawn against the
Bank of America. However, Capital Insurance tried to deposit the check and the same was dishonored by
the bank for lack of funds.

Two days after the insurance premium became due, at about 4:00 to 5:00 o'clock in the morning,
the property insured by Plastic Era was destroyed by fire. In due time, the latter notified Capital Insurance
of the loss of the insured property by fire and accordingly filed its claim for indemnity thru the Manila
Adjustment Company. The loss and/or damage suffered by Plastic Era was estimated by the Manila
Adjustment Company to be P283,875. However, according to the records the same property has been
insured by Plastic Era with the Philamgen Insurance Company for P200,000.00.

In less than a month Plastic Era demanded from Capital Insurance the payment of the sum of
P100,000.00 as indemnity for the loss of the insured property but the latter refused for the reason that,
among others, Plastic Era failed to pay the insurance premium.

Issue:

Is there a contract of insurance duly perfected between petitioner and respondent?

Ruling:

Capital Insurance accepted the promise of Plastic Era to pay the insurance premium within thirty
days from the effective date of policy. By so doing, it has implicitly agreed to modify the tenor of the
insurance policy and in effect, waived the provision therein that it would only pay for the loss or damage
in case the same occurs after the payment of the premium. Considering that the insurance policy is silent
as to the mode of payment, Capital Insurance is deemed to have accepted the promissory note in payment
of the premium. This rendered the policy immediately operative on the date it was delivered.

When the damage or loss of the insured property occurred, the insurance policy was in full force
and effect. The fact that the check issued by Plastic Era in partial payment of the promissory note was later
on dishonoured did not in any way operate as a forfeiture of its rights under the policy, there being no
express stipulation therein to that effect.

Therefore, Plastic Era has complied with its obligation to pay the insurance premium and
therefore Capital Insurance is obliged to make good its undertaking to Plastic Era.

96
P er f ec ti o n of C o nt ra c t: Pr e m iu m Pa y m e nt in L i f e In su ra nc e

PHILIPPINE PHOENIX SURETY & INSURANCE COMPANY v. WOODWORKS, INC.


G.R. no. L-25317
August 6, 1979

Melencio-Jerrera, J.

Facts:

Upon WOODWORKS’s application, PHIL. PHOENIX issued in its favor a fire insurance policy
whereby PHIL. PHOENIX insured WOODWORKS’ building, machinery and equipment for a term of one
year from against loss by fire. The premium and other charges amounted to P10,593.36. It is undisputed
that WOODWORKS did not pay the premium stipulated in the Policy when it was issued nor at any time
thereafter. Before the expiration of the one-year term, PHIL. PHOENIX notified WOODWORKS of the
cancellation of the Policy allegedly upon request of WOODWORKS. The latter has denied having made such
a request. PHIL. PHOENIX credited WOODWORKS with the amount of P3,110.25 for the unexpired period
of 94 days, and claimed the balance of P7,483.11 representing , earned premium. Thereafter, PHIL.
PHOENIX demanded in writing for the payment of said amount.

WOODWORKS disclaimed any liability contending, in essence, that it need not pay premium
“because the Insurer did not stand liable for any indemnity during the period the premiums were not
paid.”For this reason, PHIL. PHOENIX commenced action in the CFI of Manila. Judgment was rendered in
PHIL. PHOENIX’s favor. From this adverse Decision, WOODWORKS appealed to the Court of Appeals
which certified the case to SC on a question of law.

Issue:

May the insurer collect the earned premiums?

Ruling:

NO. The Courts findings are buttressed by Section 77 of the Insurance Code (Presidential Decree
No. 612, promulgated on December 18, 1974), which now provides that “no contract of insurance issued
by an insurance company is valid and binding unless and until the premium thereof has been paid,
notwithstanding any agreement to the contrary.” Since the premium had not been paid, the policy must be
deemed to have lapsed.

The non-payment of premiums does not merely suspend but put, an end to an insurance contract,
since the time of the payment is peculiarly of the essence of the contract.

In fact, if the peril insured against had occurred, PHIL. PHOENIX, as insurer, would have had a valid
defense against recovery under the Policy it had issued. Explicit in the Policy itself is PHIL. PHOENIX’s
agreement to indemnify WOODWORKS for loss by fire only “after payment of premium,” Compliance by
the insured with the terms of the contract is a condition precedent to the right of recovery.

The burden is on an insured to keep a policy in force by the payment of premiums, rather than on
the insurer to exert every effort to prevent the insured from allowing a policy to elapse through a failure
to make premium payments. The continuance of the insurer’s obligation is conditional upon the payment
of premiums, so that no recovery can be had upon a lapsed policy, the contractual relation between the
parties having ceased.

Moreover, “an insurer cannot treat a contract as valid for the purpose of collecting premiums and
invalid for the purpose of indemnity.”

97
P er f ec ti o n of C o nt ra c t: Pr e m iu m Pa y m e nt in L i f e In su ra nc e

VELASCO v. APOSTOL
G.R. No. L-44588
May 9, 1989

J. Regalado

Facts:

Plaintiffs Velasco and Acosta were riding in their Mercury car near Speaker Perez Street, Quezon
City, toward the direction of Manila. An N/S taxicab driven by defendant Santos, crossed the center island
towards their direction, and finally collided with their car. The taxicab tried to return to its original lane,
but was unable to climb the island, and instead, backtracked, hitting again plaintiffs' car in the left near
portion, causing the latter's back portion to turn toward the center hitting a jeepney on its right.

Maharlika Insurance Co., Inc. was impleaded as a defendant with an allegation that the N/S taxicab
involved was insured against third party liability for P20,000.00 with private respondent at the time of the
accident.

The company claimed that there was no cause of action against it because at the time of the
accident, the alleged insurance policy was not in force due to non-payment of the premium. They alleged
that even if the taxicab had been insured, the complaint would still be premature since the policy provides
that the insurer would be liable only when the insured becomes legally liable.

The trial court rendered judgment in favor of the plaintiff affirming Santos’ negligence as the
proximate cause. Defendants were made to pay P17,061.95 for the repair of their car and 37,000 for other
damages. Maharlika Insurance Co. was exonerated on the ground that the policy was not in force for failure
of the defendants to pay the initial premium.
Petitioners appealed the case to the Supreme Court with the averment that only questions of law are
involved.

Issue:

Whether defendant Maharlika Insurance Co. Inc. is liable under the insurance policy on account
of the negligence of defendant Dominador Santos.

Ruling:

No. Petition dismissed

Petitioners averred that the respondent had agreed to grant the then prospective insured a credit
extension for the premium due.

The accident arose when the old insurance law, Act No. 2427 was in effect. The accident occurred
on November 27, 1973 while the complaint was filed on July 20, 1974, both before effectivity of
Presidential Decree No. 612. The former insurance law, which applies to the case under consideration,
provided that:

An insurer is entitled to the payment of premium as soon as the thing insured is exposed to the
peril insured against, unless there is clear agreement to grant the insured credit extension of the
premium due. No policy issued by an insurance company is valid and binding unless and until the
premium thereof has been paid.

98
P er f ec ti o n of C o nt ra c t: Pr e m iu m Pa y m e nt in L i f e In su ra nc e

The insurance policy in question would be valid and binding even without the non-payment of the
premium if there was a clear agreement to grant to the insured credit extension.

Petitioners claim that “a condition requiring pre-payment of the premium is waived by a parol
agreement to that effect, acceptance of the premium after delivery of the policy, the unconditional delivery
of the policy, the giving of credit for the premiums, ... or any other circumstances showing that pre-payment
was not intended to be insisted upon.”

The accident for which respondent insurance company is sought to be held liable occurred on
November 27, 1973 while the initial premium was paid only on December 11, 1973.

Petitioners still maintained that the policy is nevertheless binding because there was an implied
agreement to grant a credit extension so as to make the policy effective.

This was not tenable, because the delivery of the policy was made on March 28, 1974. Also, the
premium was had been paid, in fact, more than three months before such delivery.

The payment was accepted by the insurer without any knowledge that the risk insured had
occurred since such fact was concealed by the insured and was not revealed to the insurer. The delivery of
the policy was far from being unconditional. Had there really been a credit extension, the insured would
not have had any apprehension or hesitation to inform the respondent insurance company at the time of
or before the payment of the premium that an accident for which the insurer may be held liable had
already happened.
Petitioners failed to point out "any other circumstances showing that prepayment of premium were not
intended to be insisted upon." They have thus failed to discharge the burden of proving their allegation of
the existence of the purported credit extension agreement.

Section 77 of the Insurance Code of 1978 has deleted the clause "unless there is clear agreement
to grant the insured credit extension of the premium due" which was then involved in this controversy.

There is bad faith on the part of defendants due to a reprehensible disregard of the principle that
insurance contracts are “uberrimae fidae and demand the most abundant good faith.”

99
P er f ec ti o n of C o nt ra c t: Pr e m iu m Pa y m e nt in L i f e In su ra nc e

VALENZUELA v. CA
G.R. No. 83122
October 19, 1990

J. Gutierrez Jr.

Facts:

Petitioner Valenzuela, a General Agent respondent Philamgen, was authorized to solicit and sell
all kinds of non-life insurance. He had a 32.5% commission rate. From 1973 to 1975, Valenzuela solicited
marine insurance from Delta Motors, Inc. in the amount of P4.4 Million from which he was entitled to a
commission of 32%. However, Valenzuela did not receive his full commission which amounted to P1.6
Million from the P4.4 Million. Premium payments amounting to P1,946,886.00 were paid directly to
Philamgen. Valenzuela’s commission amounted to P632,737.00.

Philamgen wanted to cut Valenzuela’s commission to 50% of the amount. He declined.When


Philamgen offered again, Valenzuela firmly reiterated his objection.

Philamgen took drastic action against Valenzuela. They: reversed the commission due him,
threatened the cancellation of policies issued by his agency, and started to leak out news that Valenzuela
has a substantial debt with Philamgen. His agency contract was terminated.

The petitioners sought relief by filing the complaint against the private respondents. The trial
court found that the principal cause of the termination as agent was his refusal to share his Delta
commission.

The court considered these acts as harassment and ordered the company to pay for the resulting
damage in the value of the commission. They also ordered the company to pay 350,000 in moral damages.

The company appealed. The CA ordered Valenzuela to pay the entire amount of the commission.
Hence, this appeal by Valenzuela.

Issues:

1. WON the agency contract is coupled with interest on the part of agent Valenzuela.
2. Whether or not Philamgen can be held liable for damages due to the termination of the General
Agency Agreement it entered into with the petitioners.
3. WON Valenzuela should pay the premiums he collected.

Ruling:

1. In any event the principal's power to revoke an agency at will is so pervasive, that the Supreme
Court has consistently held that termination may be effected even if the principal acts in bad faith, subject
only to the principal's liability for damages.

The Supreme Court accorded great weight on the trial court’s factual findings and found the cause
of the conflict to be Valenzuela’s refusal to share the commission. Philamgen told the petitioners of its
desire to share the Delta Commission with them. It stated that should Delta back out from the agreement,
the petitioners would be charged interests through a reduced commission after full payment by Delta.

Philamgen proposed reducing the petitioners' commissions by 50% thus giving them an agent's
commission of 16.25%. The company insisted on the reduction scheme. The company pressured the
agents to share the income with the threat to terminate the agency. The petitioners were also told that the
Delta commissions would not be credited to their account. This continued until the agency was terminated.

10 0
P er f ec ti o n of C o nt ra c t: Pr e m iu m Pa y m e nt in L i f e In su ra nc e

Records also show that the agency is one "coupled with an interest," and, therefore, should not be
freely revocable at the unilateral will of the company. The records sustain the finding that the private
respondent started to covet a share of the insurance business that Valenzuela had built up, developed and
nurtured. The company appropriated the entire insurance business of Valenzuela. Worse, despite the
termination of the agency, Philamgen continued to hold Valenzuela jointly and severally liable with the
insured for unpaid premiums.

Under these circumstances, it is clear that Valenzuela had an interest in the continuation of the
agency when it was unceremoniously terminated not only because of the commissions he procured, but
also Philamgen’s stipulation liability against him for unpaid premiums. The respondents cannot state that
the agency relationship between Valenzuela and Philamgen is not coupled with interest.

There is an exception to the principle that an agency is revocable at will and that is when the
agency has been given not only for the interest of the principal but also for the mutual interest of the
principal and the agent. The principal may not defeat the agent's right to indemnification by a termination
of the contract of agency. Also, if a principal violates a contractual or quasi-contractual duty which he owes
his agent, the agent may as a rule bring an appropriate action for the breach of that duty.

2. Hence, if a principal acts in bad faith and with abuse of right in terminating the agency, then he
is liable in damages. The Civil Code says that "every person must in the exercise of his rights and in the
performance of his duties act with justice, give every one his due, and observe honesty and good faith: (Art.
19, Civil Code), and every person who, contrary to law, wilfully or negligently causes damages to another,
shall indemnify the latter for the same (Art. 20, Civil Code).

3. As to the issue of whether or not the petitioners are liable to Philamgen for the unpaid and
uncollected premiums which the appellate court ordered Valenzuela to pay, the respondent court erred in
holding Valenzuela liable.

Under Section 77 of the Insurance Code, the remedy for the non-payment of premiums is to put
an end to and render the insurance policy not binding. Philippine Phoenix- non-payment of premium does
not merely suspend but puts an end to an insurance contract since the time of the payment is peculiarly of
the essence of the contract.

Section 776 of the insurance Code says that no contract of insurance by an insurance company is
valid and binding unless and until the premium has been paid, notwithstanding any agreement to the
contrary. Since the premiums have not been paid, the policies issued have lapsed. The insurance coverage
did not go into effect or did not continue and the obligation of Philamgen as insurer ceased. Philam can’t
demand from or sue Valenzuela for the unpaid premiums.

The court held that the CA’s giving credence to an audit that showed Valenzuela owing Philamgen
P1,528,698.40 was unwarranted. Valenzuela had no unpaid account with Philamgen. But facts show that
the beginning balance of Valenzuela's account with Philamgen amounted to P744,159.80. 4 statements of
account were sent to the agent.

It was only after the filing of the complaint that a radically different statement of accounts surfaced
in court. Certainly, Philamgen's own statements made by its own accountants over a long period of time
and covering examinations made on four different occasions must prevail over unconfirmed and
unaudited statements made to support a position made in the course of defending against a lawsuit.

The records of Philamgen itself are the best refutation against figures made as an afterthought in
the course of litigation. Moreover, Valenzuela asked for a meeting where the figures would be reconciled.
Philamgen refused to meet with him and, instead, terminated the agency agreement.

10 1
P er f ec ti o n of C o nt ra c t: Pr e m iu m Pa y m e nt in L i f e In su ra nc e

After off-setting the amount, Valenzuela had overpaid Philamgen the amount of P530,040.37 as
of November 30, 1978. Philamgen cannot later be heard to complain that it committed a mistake in its
computation. The alleged error may be given credence if committed only once. But as earlier stated, the
reconciliation of accounts was arrived at four (4) times on different occasions where Philamgen was duly
represented by its account executives. On the basis of these admissions and representations, Philamgen
cannot later on assume a different posture and claim that it was mistaken in its representation with respect
to the correct beginning balance as of July 1977 amounting to P744,159.80. The audit report
commissioned by Philamgen is unreliable since its results are admittedly based on an unconfirmed and
unaudited beginning balance of P1,758,185.43.

Philamgen has been appropriating for itself all these years the gross billings and income that it
took away from the petitioners. A principal can be held liable for damages in cases of unjust termination
of agency. This Court ruled that where no time for the continuance of the contract is fixed by its terms,
either party is at liberty to terminate it at will, subject only to the ordinary requirements of good faith. The
right of the principal to terminate his authority is absolute and unrestricted, except only that he may not
do so in bad faith.

The circumstances of the case, however, require that the contractual relationship between the
parties shall be terminated upon the satisfaction of the judgment. No more claims arising from or as a
result of the agency shall be entertained by the courts after that date.

10 2
P er f ec ti o n of C o nt ra c t: Pr e m iu m Pa y m e nt in L i f e In su ra nc e

MAKATI TUSCANY v. CA
G.R. No. 95546
November 6, 1992

Bellosillo, J.

Facts:

Sometime in early 1982, private respondent American Home Assurance Co. (AHAC), represented
by American International Underwriters (Phils.), Inc., issued in favor of petitioner Makati Tuscany
Condominium Corporation (TUSCANY) Insurance Policy No. AH-CPP-9210452 on the latter's building and
premises, for a period beginning 1 March 1982 and ending 1 March 1983, with a total premium of
P466,103.05. The premium was paid on installments on 12 March 1982, 20 May 1982, 21 June 1982 and
16 November 1982, all of which were accepted by private respondent.

Successive renewals of the policies were made in the same manner. On 1984, the policy was again
renewed and petitioner made two installment payments, both accepted by private respondent, the first
on 6 February 1984 for P52,000.00 and the second, on 6 June 1984 for P100,000.00. Thereafter, petitioner
refused to pay the balance of the premium.

Private respondent filed an action to recover the unpaid balance of P314,103.05 for Insurance
Policy. Petitioner explained that it discontinued the payment of premiums because the policy did not
contain a credit clause in its favor. Petitioner further claimed that the policy was never binding and valid,
and no risk attached to the policy. It then pleaded a counterclaim for P152,000.00 for the premiums
already paid for 1984-85, and in its answer with amended counterclaim, sought the refund of P924,206.10
representing the premium payments for 1982-85.

Issue:

Whether payment by installment of the premiums due on an insurance policy invalidates the
contract of insurance, in view of Sec. 77 of P.D. 612, otherwise known as the Insurance Code, as amended.

Ruling:

No, the contract remains valid even if the premiums were paid on installments. Certainly, basic
principles of equity and fairness would not allow the insurer to continue collecting and accepting the
premiums, although paid on installments, and later deny liability on the lame excuse that the premiums
were not prepared in full. At the very least, both parties should be deemed in estoppel to question the
arrangement they have voluntarily accepted. Moreover, as correctly observed by the appellate court,
where the risk is entire and the contract is indivisible, the insured is not entitled to a refund of the
premiums paid if the insurer was exposed to the risk insured for any period, however brief or momentary.
The obligation to pay premiums when due is ordinarily an indivisible obligation to pay the entire premium.

10 3
P er f ec ti o n of C o nt ra c t: Pr e m iu m Pa y m e nt in L i f e In su ra nc e

TIBAY v. COURT OF APPEALS


GR No. 119655
24 May 1996

Bellosillo, J.

Facts:

In January 22 1987, the Petitioner Violeta Tibay (and Nicolas Roralso) obtained a fire insurance
policy for their 2-storey from the Private Respondent Fortune Life Insurance Co. The said policy covers the
period from January 23, 1987 until January 23, 1988 or one year for P600, 000 and at the agreed premium
of P2, 983.50. On January 23 or the next day, petitioner made a partial payment of the premium with P600.

Unfortunately, on March 8 1987, the said building was burned to the ground. It was only two days
after the fire that Petitioner Violeta advanced the full payment of the policy premium which was accepted
by the insurer. On this same day, petitioner likewise filed the claim that was then referred to the insurer's
adjuster. Investigation of the cause of fire commenced and the petitioner submitted the required proof of
loss.

Despite that, the private respondent Fortune refused to pay the insurance claim saying it as not
liable due to the non-payment by petitioner of the full amount of the premium as stated in the policy

The petitioner then brought the matter to the Insurance Commission but nothing good came out.
Hence this case filed.

The trial court ruled in favor of the petitioner. Upon appeal, the Court of Appeals reversed the
lower court's decision and held that Fortune is not liable but ordered it to return the premium paid with
interest to the petitioner.

Issue:

Whether or not the partial payment of the premium rendered the insurance policy ineffective?

Ruling:

YES. Insurance is a contract whereby one undertakes for a consideration to indemnify another
against loss, damage or liability arising from an unknown or contingent event. The consideration is the
premium, which must be paid at the time, way and manner as stated in the policy, and if not so paid as in
this case, the policy is therefore forfeited by its own terms. In this case, the policy taken out by the petitioner
provides for payment of premium in full. Since the petitioner only made partial payment with the
remaining balance paid only after the fire or peril insured against has occurred, the insurance contract
therefore did not take effect barring the insured from claiming or collecting from the loss of her building.

Under Section 77 of the Insurance Code (Philippine), it provides therein that "An insurer is entitled
to payment of the premium as soon as the thing insured is exposed to the peril insured against.
Notwithstanding any agreement to the contrary, no policy or contract of insurance issued by an insurance
company is valid and binding unless and until the premium thereof has been paid, except in the case of a
life or an industrial life policy whenever the grace period provision applies." Herein case, the controversy
is on the payment of the premium. It cannot be disputed that premium is the elixir vitae of the insurance
business because the insurer is required by law to maintain a reserve fund to meet its contingent
obligations to the public. Due to this, it is imperative that the premium is paid fully and promptly. To allow
the possibility of paying the premium even after the peril has ensued will surely undermine the foundation
of the insurance business.

10 4
P er f ec ti o n of C o nt ra c t: Pr e m iu m Pa y m e nt in L i f e In su ra nc e

10 5
P er f ec ti o n of C o nt ra c t: Pr e m iu m Pa y m e nt in L i f e In su ra nc e

SOUTH SEA SURETY & INSURANCE CO. v. CA


G.R. No. 102253,
June 2, 1995

J. Vitug

Facts:

Valenzuela Hardwood entered into an agreement with the defendant Seven Brothers whereby
the latter undertook to load the former's 940 lauan logs for shipment to Manila. South Sea insured the logs
for P2,000,000.00 in its marine policy. Valenzuela then gave the check in payment of the premium on the
insurance policy to Mr. Victorio Chua. Seven Brothers’ ship sank resulting in the loss of the logs.

A check for P5,625.00 to cover payment of the premium tendered to the insurer but was not
accepted. Instead, the South Sea Surety and Insurance Co., Inc. cancelled the insurance policy it issued as
of the date of inception for non-payment of the premium due in accordance with Section 77 of the
Insurance Code. Valenzuela demanded from South Sea the payment of the proceeds of the policy but the
latter denied liability under the policy. Plaintiff likewise filed a formal claim with defendant Seven Brothers
Shipping Corporation for the value of the lost logs but the latter denied the claim.

Valenzuela filed a complaint a complaint for the recovery of the value of lost logs and freight
charges from Seven Brothers Shipping Corporation or from South Sea Surety and Insurance Company, the
insurer. The trial court rendered judgment in favor of plaintiff Valenzuela. The Court of Appeals affirmed
the judgment only against the insurance corporation and absolved the shipping entity from liability. The
court held that there was a stipulation in the charter party exempted the ship owner from liability in case
of loss.

In the SC petition, petitioner argues that it should have been freed from any liability to Hardwood.
It faults the appellate court (a) for having disregarded Section 77 of the insurance Code and (b) for holding
Victorio Chua to have been an authorized representative of the insurer.

Issue:

Whether or not Mr. Chua acted as an agent of the surety company or of the insured when he
received the check for insurance premiums.

Ruling:

Agent of the surety. Petition denied.

To determine if there was a valid contract of insurance, it must be determined if the premium was
validly paid to the company or its agents at the time of the loss.

The appellate and trial courts have found that Chua acted as an agent. South Sea insisted that Chua
has been an agent for less than ten years of the Columbia Insurance Brokers, a different company.
Appellant argued that Mr. Chua, having received the premiums, acted as an agent under Section 301 of the
Insurance Code which provides:
Sec. 301 Any person who for any compensation, commission or other thing of value, acts,
or aids in soliciting, negotiating or procuring the making of any insurance contract or in
placing risk or taking out insurance, on behalf of an insured other than himself, shall be
an insurance broker within the intent of this Code, and shall thereby become liable to all
the duties requirements, liabilities and penalties to which an insurance broker is subject.

Valenzuela claimed that the second paragraph of Section 306 of the Insurance Code provided:

10 6
P er f ec ti o n of C o nt ra c t: Pr e m iu m Pa y m e nt in L i f e In su ra nc e

Sec. 306 Any insurance company which delivers to an insurance agent or insurance
broker a policy or contract of insurance shall be deemed to have authorized such agent
or broker to receive on its behalf payment of any premium which is due on such policy of
contract of insurance at the time of its issuance or delivery or which becomes due
thereon.

Mr. Chua testified that the marine cargo insurance policy logs was by South Sea to be given to the
wood company. When South Sea delivered to Mr. Chua the marine cargo insurance policy for Valenzuela’s
logs, he is deemed to have been authorized by former to receive the premium which is due on its behalf.

When the logs were lost, the insured had already paid the premium to an agent of the South Sea
Surety and Insurance Co., Inc., which is consequently liable to pay the insurance proceeds under the policy
it issued to the insured.

10 7
P er f ec ti o n of C o nt ra c t: Pr e m iu m Pa y m e nt in L i f e In su ra nc e

SERVICEWIDE SPECIALISTS INCORPORATED v. CA


G.R. No. 110597
May 8, 1996

Romero, J.

Facts:

Spouses Ricardo and Elisa Trinidad purchased one unit Isuzu Gemini car from Autoworld Sales
Corporation. To secure payment the Trinidads executed on the same date a promissory note and a deed
of chattel mortgage on the subject car in favor of Autoworld Sales Corporation. Autoworld assigned its
interests on the promissory note and chattel mortgage to Filinvest Credit Corporation (Filinvest). Private
respondents delivered seventeen (17) checks to Filinvest Credit Corporation in full payment of the car.
Filinvest assigned all its rights and interests on the promissory note and chattel mortgage in favor of
petitioner.

Private respondent Ricardo Trinidad received a demand letter from petitioner stating that an
assignment of credit had been made by Filinvest in its favor and that the Trinidads had not paid two
successive installments on the car which had matured on July 15 and August 15, 1985. No mention was
made in the letter that Filinvest had paid insurance premiums to Perla Compania de Seguros to insure the
car against loss and damage corresponding to two years. Private respondents were also never informed
by Filinvest that their installment payments on the car were converted to premium payments on the
insurance.

Petitioner demanded that either they pay the whole remaining balance of P6,977.67, including
accrued interest, or return possession of the car to petitioner. Petitioner filed an action for replevin and
damages. The MTC ruled in favor of the petitioner. However, RTC held that petitioner had no cause of
action against private respondents because the latter issued the checks with the understanding that they
were to be applied to the payment in full of the car and that the same were all duly encashed by petitioner.
The Court of Appeals affirmed the decision of the Regional Trial Court.

Petitioner contends that the matter about the notice is deemed waived by private respondents
because the car should be fully covered at all times. Petitioner claims that if, as stated in the Chattel
Mortgage, private respondents failed to renew the insurance, petitioner is entitled to renew the same for
the account of private respondents without any notice to them.

Issue:

Can the petitioner apply the installment payments made by private respondents for the payment
of the car to the payment of the insurance premiums without prior notice to private respondents?

Ruling:

No. While it is true that the Chattel Mortgage does not say that notice to the mortgagor of the
renewal of the insurance premium by the mortgagee is necessary, at the same time, there is no provision
that authorizes petitioner to apply the payments made to it for the payment of the chattel to the payment
of the said premiums. From the records of the case, it is clear that private respondents had fully paid for
the car. The Court ruled that before the petitioner may affect the renewal of insurance, two conditions must
be met: (1) Default by the mortgagor (private respondents) in effecting renewal of the insurance; (2)
failure to deliver the policy with endorsement to petitioner and (3) petitioner as mortgagee was not duty-
bound to renew the insurance in the event that private respondents failed to do so; it was merely optional
on its part. Clear is it that petitioner is not obligated to convert any of the installments made by private
respondents for the car to the payment for the renewal of the insurance. Should it decide to do so, it has to
send notice to private respondents who had already paid in full the principal indebtedness in question.

10 8
P er f ec ti o n of C o nt ra c t: Pr e m iu m Pa y m e nt in L i f e In su ra nc e

AMERICAN HOME ASSURANCE v. TANTUCO ENTERPRISES


G.R. No. 138941
October 8, 2001

Puno, J.

Facts:

Two oil mills were separately covered by fire insurance policies issued by petitioner American
Home Assurance Co., Philippine Branch. The first oil mill was insured for three million pesos for the period
March 1, 1991 to 1992. The new oil mill was insured for six million pesos for the same term. Official
receipts indicating payment for the full amount of the premium were issued by the petitioner's agent.

A fire that broke out in the early morning of September 30,1991 gutted and consumed the new oil
mill. Respondent immediately notified the petitioner of the incident. The latter then sent its appraisers
who inspected the burned premises and the properties destroyed. Thereafter petitioner rejected
respondent's claim for the insurance proceeds on the ground that no policy was issued by it covering the
burned oil mill. The respondent stated that the description of the insured establishment referred to
another building thus: "Our policy nos. 306-7432321-9 (Ps 6M) and 306-7432324-4 (Ps 3M) extend
insurance coverage to your oil mill under Building No. 5, whilst the affected oil mill was under Building No.
14.

Issue:

Did the Court of Appeals commit error in its legal interpretation of the Fire Extinguishing
Appliances Warranty' of the policy?

Ruling:

In construing the words used descriptive of a building insured, the greatest liberality is shown by
the courts in giving effect to the insurance. In view of the custom of insurance agents to examine buildings
before writing policies upon them, and since a mistake as to the identity and character of the building is
extremely unlikely, the courts are inclined to consider that the policy of insurance covers any building
which the parties manifestly intended to insure, however inaccurate the description may be.

Notwithstanding, therefore, the misdescription in the policy, it is beyond dispute, to our mind, that
what the parties manifestly intended to insure was the new oil mill. This is obvious from the categorical
statement embodied in the policy, extending its protection: "On machineries and equipment with
complete accessories usual to a coconut oil mill including stocks of copra, copra cake and copra mills whilst
contained in the new oil mill building, situate (sic) at UNNO. ALONG NATIONAL HIGH WAY, BO. IYAM,
LUCENA CITY UNBLOCKED.''

If the parties really intended to protect the first oil mill, then there is no need to specify it as new.

Indeed, it would be absurd to assume that respondent would protect its first oil mill for different
amounts and leave uncovered its second one. As mentioned earlier, the first oil mill is already covered
under Policy No. 306-7432324-4 issued by the petitioner. It is unthinkable for respondent to obtain the
other policy from the very same company. The latter ought to know that a second agreement over that
same realty results in its over insurance.

10 9
P er f ec ti o n of C o nt ra c t: Pr e m iu m Pa y m e nt in L i f e In su ra nc e

JOSE MARQUES AND MAXILITE TECHNOLOGIES, INC. v. FAR EAST BANK, ET. AL.
G.R. No. 171419
January 10, 2011

Carpio, J.

Facts:

Maxilite Technologies, Inc. (Maxilite) is a domestic corporation engaged in the importation and
trading of equipment for energy-efficiency systems. Jose N. Marques (Marques) is the President and
controlling stockholder of Maxilite.

Far East Bank and Trust Co. (FEBTC) is a local bank which handled the financing and related
requirements of Marques and Maxilite. Marques and Maxilite maintained accounts with FEBTC.
Accordingly, FEBTC financed Maxilite’s capital and operational requirements through loans secured with
properties of Marques under the latter’s name.

Sometime in August 1993, FEBIBI, upon the advice of FEBTC, facilitated the procurement and
processing from Makati Insurance Company of four separate and independent fire insurance policies over
the trust receipted merchandise.

On 19 August 1994, Insurance Policy No. 1024439, covering the period 24 June 1994 to 24 June
1995, was released to cover the trust receipted merchandise. The policy relevantly provides:
2. This policy including any renewal thereof and/or any endorsement thereon is not in
force until the premium has been fully paid to and duly receipted by the Company in the
manner provided herein.

Any supplementary agreement seeking to amend this condition prepared by agent, broker or
Company official, shall be deemed invalid and of no effect.

Finding that Maxilite failed to pay the insurance premium in the sum of ₱8,265.60 for Insurance
Policy No. 1024439 covering the period 24 June 1994 to 24 June 1995, FEBIBI sent written reminders to
FEBTC, dated 19 October 1994, 24 January 1995, and 6 March 1995, to debit Maxilite’s account.

On 24 and 26 October 1994, Maxilite fully settled its trust receipt account. On 9 March 1995, a fire
gutted the Aboitiz Sea Transport Building along M.J. Cuenco Avenue, Cebu City, where Maxilite’s office and
warehouse were located. As a result, Maxilite suffered losses amounting to at least ₱2.1 million, which
Maxilite claimed against the fire insurance policy with Makati Insurance Company. Makati Insurance
Company denied the fire loss claim on the ground of non-payment of premium. FEBTC and FEBIBI
disclaimed any responsibility for the denial of the claim.
Maxilite and Marques sued FEBTC, FEBIBI, and Makati Insurance Company. The Regional Trial Court of
Cebu City, Branch 58 ruled in favor of Maxilite and Marques.he Court of Appeals affirmed the trial court’s
decision, with modifications.

Issue:

Was there valid payment of premiums?

Ruling:

Yes. Both trial and appellate courts basically agree that FEBTC is estopped from claiming that the
insurance premium has been unpaid. That FEBTC induced Maxilite and Marques to believe that the
insurance premium has in fact been debited from Maxilite’s account is grounded on the the following facts:
(1) FEBTC represented and committed to handle Maxilite’s financing and capital requirements, including

11 0
P er f ec ti o n of C o nt ra c t: Pr e m iu m Pa y m e nt in L i f e In su ra nc e

the related transactions such as the insurance of the trust receipted merchandise; (2) prior to the subject
Insurance Policy No. 1024439, the premiums for the three separate fire insurance policies had been paid
through automatic debit arrangement; (3) FEBIBI sent FEBTC, not Maxilite nor Marques, written
reminders dated 19 October 1994, 24 January 1995, and 6 March 1995 to debit Maxilite’s account,
establishing FEBTC’s obligation to automatically debit Maxilite’s account for the premium amount; (4)
there was no written demand from FEBTC or Makati Insurance Company for Maxilite or Marques to pay
the insurance premium; (5) the subject insurance policy was released to Maxilite on 19 August 1994; and
(6) the subject insurance policy remained uncancelled despite the alleged non-payment of the premium,
making it appear that the insurance policy remained in force and binding.

11 1
P er f ec ti o n of C o nt ra c t: R e fu nd of Pr e m iu ms

GREAT PACIFIC LIFE INSURANCE CORPORATION v. COURT OF APPEALS


G.R. No. L-57308
April 23, 1990

Griño-Aquino, J.

Facts:

Private respondent Teodoro Cortez, upon the solicitation of Margarita Siega an underwriter for
the petitioner Great Pacific Insurance Corporation, applied for a 20-year endowment policy for P30,000.
His application, with the requisite medical examination, was accepted and approved by the company and
in due course, the policy was issued in his name. It was released for delivery on January 24, 1973, and was
actually delivered to him by the underwriter, Mrs. Siega on January 25, 1973. The effective date indicated
on the face of the policy in question was December 25, 1972. The annual premium was P1,416.60. Mrs.
Siega assured him that the first premium may be paid within the grace period of thirty (30) days from date
of delivery of the policy. The first premium of P1,416.60 was paid by him in three (3) installments.

In a letter dated June 1, 1973, defendant advised plaintiff that the policy was not in force. To make
it enforceable and operative, plaintiff was asked to remit the balance of P1,015.60 to complete his initial
annual premium due December 15, 1972, and to see Dr. Felipe V. Remollo for another full medical
examination at his own expense.

Cortez' reaction to the company's act was to immediately inform it that he was cancelling the
policy and he demanded the return of his premium plus damages. When the company ignored his demand,
Cortez filed with the RTC a complaint for damages. The RTC found in favor of plaintiff. On appeal, the CA
affirmed such decision in toto. Hence, this appeal.

Issue:

Is Cortez entitled to a refund of the premiums he paid?

Ruling:

YES. The Court found that since his policy was in fact inoperative or ineffectual from the beginning,
the company was never at risk, hence, it is not entitled to keep the premium and should reimburse the
same to Cortez.

Sections 79, 81 and 82 of P.D. 612 of the Insurance Code of 1978 provides when the insured is
entitled to the return of premium paid
SECTION 79. A person insured is entitled to a return of premium, as follows:
(a) To the whole premium, if no part of his interest in the thing insured be exposed to any of
the perils insured against.
(b) Where the insure is made for a definite period of time and the insured surrenders his
policy, to such portion of the premium as corresponds with the unexpired time, at a pro
rata rate, unless a short period rate has been agreed upon and appears on the face of the
policy, after deducting from the whole premium any claim for loss or damage under the
policy which has previously accrued: Provided, That no holder of a life insurance policy
may avail himself of the privileges of this paragraph without sufficient causes as
otherwise provided by law.
SECTION 81. A person insured is entitled to a return of the premium when the contract
is voidable on account of the fraud or misrepresentation of the insurer or of his agent or
on account of facts the existence of which the insured was ignorant without his fault; or
when, by any default of the insured other than actual fraud, the insurer never incurred
any liability under the policy.

11 2
P er f ec ti o n of C o nt ra c t: R e fu nd of Pr e m iu ms

SECTION 82. In case of an over-insurance by several insurers, the insured is entitled to a


ratable return of the premium, proportioned to the amount by which the aggregate sum
insured in all the policies exceeds the insurable value of the thing at risk.

There was mistake on the part of the insurer when the petitioner advised private respondent on
June 1, 1973, four months after he had paid the first premium, that his policy had never been in force, and
that he must pay another premium and undergo another medical examination to make the policy effective,
the petitioner committed a serious breach of the contract of insurance.

Petitioner should have informed Cortez of the deadline for paying the first premium before or at
least upon delivery of the policy to him, so he could have complied with what was needful and would not
have been misled into believing that his life and his family were protected by the policy, when actually they
were not. And, if the premium paid by Cortez was unacceptable for being late, it was the company's duty
to return it. By accepting his premiums without giving him the corresponding protection, the company
acted in bad faith.

11 3
P er f ec ti o n of C o nt ra c t: Del iv e ry of t h e P ol icy

VDA DE SINDAYEN v. INSULAR LIFE


G.R. No. 41702
September 4, 1935

Butte, J

Facts:

Arturo Sindayen was employed as a linotype operator. Arturo made a written application on
December 26, 1932, to the defendant Insular through its agent, Cristobal Mendoza, for a policy of insurance
on his life to be delivered to his aunt Felicidad Estrada. Arturo paid P15 as first premium and left P26.06
to his aunt to complete the annual payment. On January 11, 1933, The Company accepted the risk and
issued policy and mailed the same to its agent for delivery. On January 15, it was discovered that Arturo
was suffering from acute nephritis and uremia and later on died on January 19, 1933. The agent learned
of the death of Arturo and asked Felicidad to return the policy. Felicidad then returned the policy.

Thereupon this action was brought to enforce payment of the policy. The dispute centres around
the provision on the application of the insured which stated that the said policy shall not take effect until
the first premium has been paid and the policy has been delivered to and accepted by the insured.

Issue:

Was there delivery to the insured making the policy effective?

Ruling:

Yes, the policy is effective. The Court held that delivery to the insured in person may not necessary
for delivery can be made by mail or to a duly constituted agent. The Court based its findings on a number
of cases ultimately it held that the delivery of the policy by the agent to the insured consummates the
contract even though the agent knew that the insured was not in good health at the time, the theory being
that his knowledge is the company's knowledge and his delivery of the policy is the company's delivery;
that when the delivery is made notwithstanding this knowledge of the defect, the company is deemed to
have waived the defect.

Further the Court held that Mendoza was authorized by the company to make the delivery of the
policy when he received the payment of the first premium and he was satisfied that the insured was in
good health. Mendoza was duly licensed by the Insurance Commissioner to act as the agent of the
defendant insurance company. As such Mendoza’s delivery was regarded as the delivery of the policy by
the company to the insured.

On a note, the Court stressed that the business of insurance is of the public interest, for the public
is profoundly and generally interested in life insurance, as well as in the interest of the insurance
companies themselves by giving certainly and security to their policies and as such the Court held that the
delivery of the policy to the insured by an agent of the company who is authorized to make delivery or
without delivery is the final act which binds the company and the insured in the absence of fraud or other
legal ground for rescission. As such, the Court held that respondent should pay the proceeds.

11 4
P er f ec ti o n of C o nt ra c t: Del iv e ry of t h e P ol icy

PEREZ v. COURT OF APPEALS


G.R. No. 112329
January 28, 2000

Ynares-Santiago, J.

Facts:

Primitivo B. Perez had been insured with the BF Lifeman Insurance Corporation since 1980 for
P20, 000.00. He was then convinced to avail of additional insurance coverage of P50, 000.00 which Perez
accomplished. Lalog forwarded the application of Perez, to the office of BF Lifeman Insurance Corporation
in Quezon which office was supposed to forward the papers to the Manila office.

On November 25, 1987, Perez died in an accident. At the time of his death, his application papers
for the additional insurance of P50, 000.00 were still with the Quezon office. It was only on November 27,
1987 that said papers were received in Manila. BF Lifeman Insurance Corporation approved the
application without knowledge of the death on December 2, 1987.
Petitioner Virginia Perez went to Manila to claim the benefits under the insurance policies of the deceased.
She was paid P40,000.00 under the first insurance policy but the insurance company refused to pay the
claim under the additional policy coverage of P50,000.00 it reasoned that such has not been perfected.

On September 21, 1990, private respondent BF Lifeman Insurance Corporation filed a complaint
against Virginia A. Perez seeking the rescission and declaration of nullity of the insurance contract in
question. Petitioner Virginia A. Perez, on the other hand, averred all the elements of a valid contract are
present.

On October 25, 1991, the trial court rendered a decision ordering the insurance company to pay
the insurance proceeds. The Court of Appeals, however, reversed the decision of the trial court saying that
the insurance contract for P50, 000.00 could not have been perfected since at the time that the policy was
issued, Primitivo was already dead.

Issue:

Was there a perfected contract of insurance?

Ruling:

There was no Contract. Consent must be manifested by the meeting of the offer and the
acceptance upon the thing and the cause which are to constitute the contract. The offer must be certain
and the acceptance absolute. When Primitivo died on November 25, 1987, his application papers for
additional insurance coverage were still with the branch office of Respondent Corporation. Consequently,
there was absolutely no way the acceptance of the application could have been communicated to the
applicant for the applicant was already dead. A contract of insurance, like other contracts, must be
assented to by both parties either in person or by their agents. So long as an application for insurance has
not been either accepted or rejected, it is merely an offer or proposal to make a contract.

Further the Court held that the assent of private respondent BF Lifeman Insurance Corporation
therefore was not given when it merely received the application form and all the requisite supporting
papers of the applicant. Its assent is only given when it issues a corresponding policy to the applicant. It is
only when the applicant pays the premium and receives and accepts the policy while he is in good health
that the contract of insurance is deemed to have been perfected.

11 5
Cov e r N o t es o r T e m po r a ry o r Pr ov isi o na l I nsu ra nc e

GREAT PACIFIC LIFE v. CA


G.R. L – 31845
April 30, 1979

De Castro, J.

Facts:

On March 14, 1957, private respondent Ngo Hing filed an application with the Great Pacific Life
Assurance Company for a twenty year endowment policy in the amount of P50,000 on the life of Helen Go,
his one year old daughter. He supplied the essential data which petitioner Mondragon, the Branch
Manager, wrote on the form. He then paid the annual premium of P1,077.75. Upon payment of the
insurance premium, the binding deposit receipt was issued to Ngo Hing. Mondragon handwrote at the
bottom of the back page of the application from his strong recommendation for the approval of the
insurance application. Then Mondragon received a letter from Pacific Life disapproving the Insurance
application. The letter stated that the said life insurance application for 20 year endowment plan is not
available for minors below seven years old, but Pacific Life can consider the same under the Juvenile Triple
Action Plan, and advised that if the offer is acceptable, the Juvenile NonMedical Declaration be sent to the
company. The non-acceptance of the insurance plan by Pacific Life was allegedly not communicated by
petitioner Mondragon to private respondent Ngo Hing. Instead, on May 6, 1957, Mondragon wrote back
Pacific Life again strongly recommending the approval of the 20 year endowment insurance plan to
children, pointing out that since the customers were asking for such coverage. On May 28, 1957, Helen Go
died of influenza. Ngo Hing sought the payment of the proceeds but failed. He filed the action for the
recovery before the CFI of Cebu but ruled adversely.

Issues:

1. Whether or not the binding deposit receipt constituted a temporary contract of life insurance
2. Whether or not Ngo Hing concealed the state of health and physical condition of Helen Go, which
rendered the policy void

Ruling:

1. No. The court held that the binding deposit receipt is intended to be merely a provisional
insurance contract subject to compliance of the following conditions: a) that the company shall be satisfied
that the applicant was insurable on standard rates; b) that if the company does not accept the application
and offers to issue a policy for a different plan, the insurance contract shall not be binding until the
applicant accepts the policy offered; otherwise, the deposit shall be refunded; and c) that if the company
disapproves the application, the insurance applied for shall not be in force at any time, and the premium
paid shall be returned to the applicant. The binding deposit receipt is merely an acknowledgment, on
behalf of the company, that the latter’s branch office had received from the applicant the insurance
premium and had accepted the application subject for processing by the insurance company. There was
still approval or rejection of the same on the basis of whether or not the applicant is “insurable on standard
rates”. Since Pacific Life disapproved the insurance application of respondent Ngo Hing, the binding
deposit receipt had never become in force at any time. The binding deposit receipt is conditional and does
not insure outright. The deposit paid by private respondent shall have to be refunded by Pacific Life.

2. Yes. Ngo Hing had deliberately concealed the state of health of his daughter Helen Go. When he
supplied data, he was fully aware that his one year old daughter is typically a mongoloid child. He withheld
the fact material to the risk to be assumed by the insurance company. The contract of insurance is one of
perfect good faith “uberrimei fidei” meaning good faith, absolute and perfect candor or openness and
honesty; the absence of any concealment or demotion, however slight.” Whether intentional or
unintentional the concealment entitles the insurer to rescind the contract of insurance.

11 6
Ty p es o f P ol ic i es

DEVELOPMENT INSURANCE v. IAC, et al.


G.R. No. 71360
July 16, 1986

Cruz, J.

Facts:

A fire occurred in the building of the Philippine Union Realty Development Corporation. It sued
for recovery of damages from Development Insurance Corporation on the basis of an insurance contract
between them. Development Insurance failed to answer on time and was declared in default by the trial
court. A judgment of default was subsequently rendered, which allowed Philippine Union Realty full
recovery of its claimed damages. The IAC affirmed the decision of the trial court in toto. Development
Insurance claims that the insurance covered only the building and not the elevators.

Issue:

Did the insurance also cover the elevators? If so, what is the amount of the indemnity due to
Philippine Union Realty?

Ruling:

Yes. The circumstance that the building insured is seven stories high and so had to be provided
with elevators – a legal requirement known to Development Insurance as an insurance company – makes
its contention all the more ridiculous. The policy, by virtue of which Development Insurance insured the
Philippine Union Realty's building against fire, is for P2.5M. This must be considered, by agreement of the
insurer and the insured, the actual value of the property insured on the day the fire occurred. The policy is
an open one. As such, the actual loss will represent the total indemnity due the insured from the insurer
except only that the total indemnity shall not exceed the face value of the policy. Applying the open policy
clause, the SC held that Philippine Union Realty is entitled to the payment of indemnity under the said
contract in the total amount of P508K.

11 7
Pa r t ie s ( I n su r e r, I ns ur e d, B e n ef ic ia r i es)

VILLANUEVA v. ORO
G.R. No. L-2227
August 31, 1948

Paras, J.

Facts:

West Coast Life Insurance Company issued 2 policies of insurance on the life of Esperanza J.
Villanueva: 2,000 php - maturing on April 1, 1943, if living, on the 1st day of April 1943 - to insured. Upon
death during the continuance of this policy - to the beneficiary Bartolome Villanueva, father of the insured,
with right on the part of the insured to change the beneficiary. On 1940, Bartolome Villanueva died,
Mariano J. Villanueva duly substituted as beneficiary, a brother of the insured, 3,000 php - maturing on
March 31, 1943. Esperanza J. Villanueva survived the insurance period, for she died only on October 15,
1944, without, however, collecting the insurance proceeds. CFI held that the estate of the insured
Esperanza is entitled to the insurance proceeds.

Issue:

Is the estate of the insured Esperanza entitled to the insurance proceeds since she outlived the
insurance policy?

Ruling:

Yes. The appealed order is, therefore, hereby affirmed. To sustain the beneficiary's claim would be
altogether eliminate from the policies the condition that the insurer "agrees to pay . . . to the insured
hereunder, if living Upon the insured's death, within the period, the beneficiary will take, as against the
personal representative or the assignee of the insured. Upon the other hand, if the insured survives the
endowment period, the benefits are payable to him or to his assignee, notwithstanding a beneficiary is
designated in the policy.

11 8
Pa r t ie s ( I n su r e r, I ns ur e d, B e n ef ic ia r i es)

FILIPINAS COMPAÑIA DE SEGUROS v. CHRISTERN, HUENEFELD and CO., INC.


G.R. No. L-2294
May 25, 1951

Paras, C.J.

Facts:

On Oct. 1, 1941, Domestic Corp Christern, after payment of the premium, obtained from Filipinas,
Fire Policy No. 29333 for P100T covering merchandise contained in a building located in Binondo, Manila.
On Feb. 27, 1942, during the Jap occupation, the building and the insured merchandise were
burned. Christern submitted to Filipinas its claim.

The salvage goods were sold at public auction and, after deducting their value, the total loss
suffered by Christern was fixed at P92,650. Filipinas refused to pay the claim on the ground that the policy
in favor of the Christern had ceased to be in force on the date the United States declared war against
Germany, the Filipinas Corporation (though organized under and by virtue of the laws of the Philippines)
being controlled by the German subjects and the Filipinas being a company under American jurisdiction
when said policy was issued on October 1, 1941.

Issue:

Can Filipinas deny the claim of Christern on the ground that it is a public enemy?

Ruling:

The Philippine Insurance Law (Act No. 2427, as amended,) in section 8, provides that "anyone
except a public enemy may be insured." It stands to reason that an insurance policy ceases to be allowable
as soon as an insured becomes a public enemy.
Where an insurance policy ceases to be effective by reason of war, which has made the insured an enemy,
the premiums paid for the period covered by the policy from the date of war is declared, should be
returned.

Christern having become an enemy corporation on December 10, 1941, the insurance policy
issued in its favor on October 1, 1941, by Filipinas had ceased to be valid and enforcible, and since the
insured goods were burned after December 10, 1941, and during the war, Christern was not entitled to
any indemnity under said policy from the Filipinas. However, elementary rules of justice (in the absence
of specific provision in the Insurance Law) require that the premium paid by Christern for the period
covered by its policy from December 11, 1941, should be returned by Filipinas.

11 9
Pa r t ie s ( I n su r e r, I ns ur e d, B e n ef ic ia r i es)

DELFIN NARIO and ALEJANDRA SANTOS-NARIO v. THE PHILIPPINE AMERICAN LIFE


INSURANCE COMPANY
G.R. No. L-22796
June 26, 1967

Reyes, J.B.L., J.

Facts:

Mrs. Alejandra Santos-Nario was issued on June 12, 1959 by the Philippine American Life
Insurance Co., a life insurance policy under a 20-year endowment plan, with a face value of P5,000.00. She
designated her husband, Delfin Nario, and their unemancipated minor son, Ernesto Nario, as her
irrevocable beneficiaries. Then, she applied for a loan on the policy with Philamlife which she is entitled to
as policy holder, after the policy has been in force for 3 years. The application bore the written signature
and consent of Delfin Nario as one of the irrevocable beneficiaries of the policy; and the other, as the father-
guardian of said minor son. Philamlife denied the loan application contending that written consent of the
minor son must not only be given by his father as legal guardian but it must also be authorized by the court
in a competent guardianship proceeding. Mrs. Nario signified her decision to surrender her policy and
demand its cash value which then amounted to P 520. The Insurance Company also denied the surrender
of the policy on the same ground as that given in disapproving the loan application.

Issue:

Is the Philamlife justified in refusing to grant the loan application and the surrender of the policy?

Ruling:

NO. The vested interest or right of the beneficiaries in the policy should be measured on its full
face value and not on its cash surrender value, for in case of death of the insured, said beneficiaries are paid
on the basis of its face value and in case the insured should discontinue paying premiums, the beneficiaries
may continue paying it and are entitled to automatic extended term or paid-up insurance options, etc. and
that said vested right under the policy cannot be divisible at any given time. The proposed transactions in
question (policy loan and surrender of policy) constitute acts of disposition or alienation of property rights
and not merely of management or administration because they involve the incurring or termination of
contractual obligations.

In this case, the full face value of the policy is P5,000.00 and the minor's vested interest therein, as
one of the two (2) irrevocable beneficiaries, consists of one-half (½) of said amount or P2,500.00.

12 0
Pa r t ie s ( I n su r e r, I ns ur e d, B e n ef ic ia r i es)

RE: CLAIMS FOR BENEFITS OF THE HEIRS OF THE LATE MARIO v. CHANLIONGCO
A.M. No. 190
October 18, 1977

Makasiar, J.

Facts:

The matter refers to the claims for retirement benefits by the heirs of the late Atty. Chanliongco of
the Supreme Court, who was more than 63 years of age, with more than 38 years of service in the
government. His widow, one legitimate child and 2 illegitimate children were left as heirs of Atty.
Chanliongco. He died intestate and stated in his application for membership with the GSIS the beneficiary,
of his retirement benefits, should he die before retirement.

Issue:

How should the retirement benefits and the monetary value of terminal leave of Atty. Chanliongco
be settled?

Ruling:

Where a government employee failed to name the beneficiary of his retirement benefits, the
proceeds thereof shall accrue to his estate. In this case, the record shows that the late Atty. Chanliongco
died ab intestato and that he filed or over to state in his application for membership with the GSIS the
beneficiary or benefits of his retirement benefits, should he die before retirement. Hence, the retirement
benefits shall accrue to his estate and will be distributed among his Legal heirs in with the benefits on
intestate, as in the caw of a fife if no benefit is named in the policy.

12 1
Pa r t ie s ( I n su r e r, I ns ur e d, B e n ef ic ia r i es)

THE PHILIPPINE AMERICAN INSURANCE COMPANY v. HONORABLE GREGORIO G. PINEDA


and RODOLFO C. DIMAYUGA
G.R. No. L-54216
July 19, 1989

Paras, J.

Facts:

On January 15, 1968, Dimayuga procured an ordinary life insurance policy from Philamlife and
designated his wife and children as irrevocable beneficiaries of said policy. Dimayuga filed a civil case at
CFI Rizal to amend the designation of the beneficiaries in his life policy from irrevocable to revocable. The
lower court granted the petition of Pineda.

Issue:

Can the insured change the designation of the irrevocable beneficiaries without their consent?

RULING:

The applicable law in the instant case is the Insurance Act, otherwise known as Act No. 2427 as
amended, the policy having been procured in 1968. Under the said law, the beneficiary designated in a life
insurance contract cannot be changed without the consent of the beneficiary because he has a vested
interest in the policy.

The Beneficiary Designation Indorsement in the policy which forms part of Policy Number
0794461 in the name of Rodolfo Cailles Dimayuga states that the designation of the beneficiaries is
irrevocable.

Inevitably therefore, based on the provision of the contract, not to mention the law then
applicable, it is only with the consent of all the beneficiaries that any change or amendment in the policy
concerning the irrevocable beneficiaries may be legally and validly effected. Both the law and the policy do
not provide for any other exception, thus, abrogating the contention of the private respondent that said
designation can be amended if the Court finds a just, reasonable ground to do so.

Similarly, the alleged acquiescence of the six (6) children beneficiaries of the policy (the
beneficiary-wife predeceased the insured) cannot be considered an effective ratification to the change of
the beneficiaries from irrevocable to revocable. Indubitable is the fact that all the six (6) children named
as beneficiaries were minors at the time, for which reason, they could not validly give their consent. Neither
could they act through their father insured since their interests are quite divergent from one another.

Of equal importance is the well-settled rule that the contract between the parties is the law
binding on both of them and for so many times, this court has consistently issued pronouncements
upholding the validity and effectivity of contracts. Where there is nothing in the contract which is contrary
to law, good morals, good customs, public policy or public order the validity of the contract must be
sustained. Likewise, contracts which are the private laws of the contracting parties should be fulfilled
according to the literal sense of their stipulations, if their terms are clear and leave no room for doubt as to
the intention of the contracting parties, for contracts are obligatory, no matter in what form they may be,
whenever the essential requisites for their validity are present

12 2
Ot h er Ca s e s i n th e Pol i c y of I ns ura nc e , Pa rt i e s a nd t h ei r R i gh ts

PILAR C. DE LIM v. SUN LIFE ASSURANCE COMPANY OF CANADA


G.R. No. L-15774
November 29, 1920

Malcolm, J

Facts:

On July 6, 1917, Luis Lim y Garcia of Zamboanga applied for a policy of life insurance with Sunlife
in the amount of P5 000.00. He designated his wife Pilar Lim as the beneficiary. The first premium of P433
was paid by Lim and company issued a “provisional policy”. Such policy contained the following
provisions “xx the abovementioned life is to be assured in accordance with the terms and conditions
contained or inserted by the Company in the policy which may be granted by it in this particular case for
4 months only from the date of the application, provided that the company shall confirm this agreement
by issuing a policy on said application xxx. Should the company not issue such a policy, then this agreement
shall be null and void ab initio and the Company shall be held not to have been on the risk at all, but in such
case, the amount herein shall be returned.”

Lim died on Aug. 23, 1917 after the issuance of the provisional policy but before the approval of
the application by the home office of the insurance company. The instant action is brought by the
beneficiary to recover from Sun Life the sum of P5000.00.

Issue:

Can Pilar Lim, as beneficiary of Luis Lim, collect P5000.00 from Sun Life Assurance Company of
Canada under the policy of life insurance?

Ruling:

In this case, the contract of insurance was not consummated by the parties and that, consequently,
the widow of the deceased cannot recover the amount of the insurance from the insurance company.

A contract of insurance, like other contracts, must be assented to by both parties either in person
or by their agents. So long as an application for insurance has not been either accepted or rejected, it is
merely an offer or proposal to make a contract. The contract, to be binding from the date of the application,
must have been a completed contract, one that leaves nothing to be done, nothing to be completed, nothing
to be passed upon, or determined, before it shall take effect. There can be no contract of insurance unless
the minds of the parties have met in agreement.

Where an agreement is made between the applicant and the agent whether by signing an
application containing such condition, or otherwise, that no liability shall attach until the principal
approves the risk and a receipt is given buy the agent, such acceptance is merely conditional, and it
subordinated to the act of the company in approving or rejecting; so in life insurance a "binding slip" or
"binding receipt" does not insure of itself.

Where the acceptance by the agent is within the scope of his authority a receipt containing a
contract for insurance for a specific time which is not absolute but conditional, upon acceptance or
rejection by the principal, covers the specified period unless the risk is declined within that period.

12 3
Ot h er Ca s e s i n th e Pol i c y of I ns ura nc e , Pa rt i e s a nd t h ei r R i gh ts

RAFAEL ENRIQUEZ v. SUN LIFE ASSURANCE COMPANY OF CANADA


G.R. No. L-15895;
November 29, 1920

Malcolm, J.

Facts:

On September 24, 1917, Joaquin Herrer made application to the Sun Life Assurance Company of
Canada through its office in Manila for a life annuity. Two days later, he paid P6,000 to the manager of the
company's Manila office and was given a receipt according to the provisional receipt, 3 things had to be
accomplished by the insurance company before there was a contract: (a) There had to be a medical
examination of the applicant; (b) there had to be approval of the application by the head office of the
company; and (c) this approval had in some way to be communicated by the company to the applicant. On
November 26, 1917, the head office at Montreal, Canada gave notice of acceptance by cable to Manila but
this was not mailed.

On December 4, 1917, policy was issued at Montreal. Attorney Torres wrote to the Manila office
of the company stating that Herrer desired to withdraw his application

The local office replied to Mr. Torres, stating that the policy had been issued, and called attention
to the notification of November 26, 1917. On December 20, 1917, Mr. Herrer died. On the morning of
December 21, 1917, the notice received by Mr. Torres. Rafael Enriquez, as administrator of the estate of
the late Joaquin Herrer, filed to recover from Sun Life Assurance Company of Canada through its office in
Manila for a life annuity.

Issue:

Can Rafael Enriquez recover the value under the life policy from Sun Life Assurance Company of
Canada?

Ruling:

The contract for a life annuity was not perfected because it had not been proved satisfactorily that
the acceptance of the application ever came to the knowledge of the applicant.
An acceptance made by letter shall not bind the person making the offer except from the time it came to
his knowledge. An acceptance of an offer of insurance not actually or constructively communicated to the
proposer does not make a contract. Only the mailing of acceptance, it has been said, completes the contract
of insurance, as the locus poenitentiae is ended when the acceptance has passed beyond the control of the
party.

When a letter or other mail matter is addressed and mailed with postage prepaid there is a
rebuttable presumption of fact that it was received by the addressee as soon as it could have been
transmitted to him in the ordinary course of the mails. But if any one of these elemental facts fails to appear,
it is fatal to the presumption.

12 4
Ot h er Ca s e s i n th e Pol i c y of I ns ura nc e , Pa rt i e s a nd t h ei r R i gh ts

CONCORDIA GO v. ANGELA REDFERN and THE INTERNATIONAL ASSURANCE CO., LTD


GR 47705
25 April 1941

Horrilleno, J.

Facts:

In October 1937, Edward K. Redfern obtained an insurance policy against accidents from the
International Assurance Co, Ltd. On 31 August 1938, Redfern died from an accident. The mother of the
deceased, presenting the necessary evidence of the death of Redfern, sought to claim the proceeds of the
insurance policy from the insurance company. The company, however, denied such claim, on the ground
that the insurance policy was amended on 22 November 1937 to include another beneficiary, Concordia
Go. Hence, an action was filed to determine who has the right to collect the insurance proceeds of the
deceased Redfern. The mother claimed that the addition of the co-beneficiary is illegal. Go, on her part,
alleged the contrary.

Issue:

Is the addition of Go’s name as co-beneficiary allowed for her share in the insurance proceeds?

Ruling:

When designated in a policy, the beneficiary acquires a right of which he cannot be deprived of
without his consent, unless the right has been reserved specifically to the insured to modify the policy.
Thus, unless the insured has reserved specifically the right to change or to modify the policy, with respect
to the beneficiary, said policy constitutes an acquired right of the beneficiary, which cannot be modified
except with the consent of the latter. In this case, it is admitted that Redfern did not reserve expressly his
right to change or modify the policy. Change implies the idea of an alteration. The addition of Go's name as
one of the beneficiaries of the policy constitutes change as all addition is an alteration. The addition of Go's
name changed the policy inasmuch as there are two beneficiaries instead of one, and thus in effect the
original beneficiary cannot receive the full amount of the policy.

12 5
Ot h er Ca s e s i n th e Pol i c y of I ns ura nc e , Pa rt i e s a nd t h ei r R i gh ts

PAULO ANG and SALLY C. ANG v. FULTON FIRE INSURANCE CO., ET AL


G.R. No. L-15862
July 31, 1961

Labrador, J.

Facts:

On September 9, 1953, P&S Department Store owned by Sally Ang was insured with Fulton Fire
Insurance Co. over stocks of general merchandise, consisting principally of dried goods. It contained a
stipulation stating “if the claim is made and rejected but no action is commenced within 12 months after
such rejection, all benefits under the policy would be forfeited.” On December 17, 1954, a fire consumed
the store and Ang filed claims. However, on April 1956, Fulton denied claims. On January 1955, Paulo Ang
and 10 others were charged for arson in the Justice of Peace Court of Ilocos Norte. The case was remanded
to CFI and the latter acquitted Paulo Ang of the crime of arson.

On May 1956, Ang filed a case against Fulton’s agent. On September 1957, this case was dismissed
without prejudice. CFI ruled in favor of Ang, holding that the 12-month prescription period (from insurer’s
denial of claim) was suspended by the case against the agent.

Issue:

Is the filing of the previous suit against the agent suspended the running of the prescriptive
period?

Ruling:

NO. The condition contained in the insurance policy that claims must be presented within one
year after rejection is not merely a procedural requirement. The condition is an important matter, essential
to a prompt settlement of claims against insurance companies, as it demands that insurance suits be
brought by the insured while the evidence as to the origin and cause of destruction have not yet
disappeared. It is in the nature of a condition precedent to the liability of the insurer, or in other terms, a
resolutory cause, the purpose of which is to terminate all liabilities in case the action is not filed by the
insured within the period stipulated.

The contractual limitations in an insurance policy prevails over the statutory limitation, as well as
over the exceptions to the statutory limitations that the contract necessarily supersedes the statute (of
limitations) and the limitation is in all phases governed by the former. The rights of the parties flow from
the contract of insurance; hence they are not bound by the statute of limitations or by exemptions thereto.
Their contract is the law between the parties, and their agreement that an action on a claim denied by the
insurer must be brought within one year (1) from the denial, governs, not the rules on the prescription of
actions. If there is no condition in the policy that an action should be filed by the insured against the agent
for his claim, the filing of such action has no legal effect and serves no other purpose except that of notifying
the agent of the claim. There is no law giving any effect to such action upon the principal, and courts cannot
by interpretation extend the clear scope of the agreement beyond what is agreed upon by the parties.

12 6
Ot h er Ca s e s i n th e Pol i c y of I ns ura nc e , Pa rt i e s a nd t h ei r R i gh ts

MELECIO COQUIA, MARIA ESPANUEVA and MANILA YELLOW TAXICAB CO., INC v
FIELDMEN'S INSURANCE CO. INC.
G.R. No. L-23276
November 29, 1968

Concepcion, C.J

Facts:

On Dec. 1, 1961, Fieldmen’s Insurance co. issued in favor of the Manila Yellow Taxicab a common
carrier insurance policy with a stipulation that the company shall indemnify the insured of the sums which
the latter will be held liable for with respect to “death or bodily injury to any fair-paying passenger
including the driver and conductor”. The policy also stated that in “the event of the death of the driver, the
Company shall indemnify his personal representatives and at the Company’s option may make indemnity
payable directly to the claimants or heirs of the claimants.”

During the policy’s lifetime, a taxicab of the insured driven by Coquia met an accident and Coquia
died. When the company refused to pay the heirs of Coquia, the heirs instituted this complaint. The
company contends that plaintiffs have no cause of action since the Coquias have no contractual
relationship with the company.

Issue:

Are the heirs of Coquia have the right to collect under the policy?

Ruling:

NO. In the case at bar, the insurance policy contains stipulations pursuant to which the insurance
company "will indemnify any authorized Driver who is driving the Motor Vehicle" of the Insured and, in
the event of death of said driver, the Company shall, likewise, "indemnify his personal representatives." In
fact, the Company "may, at its option, make indemnity payable directly to the claimants or heirs of
claimants ... it being the true intention of this Policy to protect ... the liabilities of the Insured towards the
passengers of the Motor Vehicle and the Public" — in other words, third parties.

Thus, the policy under consideration is typical of contracts pour autrui, this character being made
more manifest by the fact that the deceased driver paid fifty percent (50%) of the corresponding
premiums, which were deducted from his weekly commissions. Under these conditions, it is clear that the
Coquias — who, admittedly, are the sole heirs of the deceased — have a direct cause of action against the
Company, and, since they could have maintained this action by themselves, without the assistance of the
Insured, it goes without saying that they could and did properly join the latter in filing the complaint herein.

The section 17 of the policy states: “If any difference or dispute shall arise with respect to the
amount of the Company's liability under this Policy, the same shall be referred to the decision of a single
arbitrator to be agreed upon by both parties or failing such agreement of a single arbitrator, to the decision
of two arbitrators, one to be appointed in writing by each of the parties within one calendar month after
having been required in writing so to do by either of the parties and in case of disagreement between the
arbitrators, to the decision of an umpire who shall have been appointed in writing by the arbitrators before
entering on the reference and the costs of and incident to the reference shall be dealt with in the Award.
And it is hereby expressly stipulated and declared that it shall be a condition precedent to any right of
action or suit upon this Policy that the award by such arbitrator, arbitrators or umpire of the amount of
the Company's liability hereunder if disputed shall be first obtained.”

The record shows, however, that none of the parties to the contract invoked this section, or made
any reference to arbitration, during the negotiations preceding the institution of the present case. In fact,

12 7
Ot h er Ca s e s i n th e Pol i c y of I ns ura nc e , Pa rt i e s a nd t h ei r R i gh ts

counsel for both parties stipulated, in the trial court, that none of them had, at any time during said
negotiations, even suggested the settlement of the issue between them by arbitration, as provided in said
section. Their aforementioned acts or omissions had the effect of a waiver of their respective right to
demand arbitration.

12 8
Ot h er Ca s e s i n th e Pol i c y of I ns ura nc e , Pa rt i e s a nd t h ei r R i gh ts

PACIFIC TIMBER EXPORT CORPORATION v. THE HONORABLE COURT OF APPEALS and


WORKMEN'S INSURANCE COMPANY, INC.
G.R. No. L-38613
February 25, 1982

De Castro, J

Facts:

On March 13, 1963, Pacific Timber Export Corp. secured temporary insurance from the
Workmen’s Insurance Co. for its exportation of logs to Japan. Workmen’s Insurance Co. issued on said
date Cover Note 1010 insuring said cargo. The regular marine policies were issued by the company in
favor of Pacific Timber Export Corp. on April 2, 1963. The two (2) marine policies bore the number
53H01032 and 53H01033. After the issuance of the cover note but before the issuance of the 2 policies,
some of the logs intended to be exported were lost due to a typhoon. Pacific Timber Export Corp. filed its
claim with the company, but the latter refused, contending that said loss may not be considered as covered
under the cover note because such became null and void by virtue of the issuance of the marine policies.

Issue:

1. Is the policy considered as null and void due to non-payment of premiums?


2. Is the Workmen’s Insurance Co. released from liability under the cover note due to
unreasonable delay in giving notice of loss?

Ruling:

The fact that no separate premium was paid on the Cover Note before the loss insured against
occurred, does not militate against the validity of Pacific's contention, for no such premium could have
been paid, since by the nature of the Cover Note, it did not contain, as all Cover Notes do not contain
particulars of the shipment that would serve as basis for the computation of the premiums. As a logical
consequence, no separate premiums are intended or required to be paid on a Cover Note. This is a fact
admitted by an official of Workmen’s company, Juan Jose Camacho, in charge of issuing cover notes of the
company.

At any rate, it is not disputed that Pacific paid in full all the premiums as called for by the statement
issued by Workmen after the issuance of the two regular marine insurance policies, thereby leaving no
account unpaid by Pacific due on the insurance coverage, which must be deemed to include the Cover Note.
If the Note is to be treated as a separate policy instead of integrating it to the regular policies subsequently
issued, the purpose and function of the Cover Note would be set at naught or rendered meaningless, for it
is in a real sense a contract, not a mere application for insurance which is a mere offer.

The non-payment of premium on the Cover Note is no cause for the Pacific to lose what is due it
as if there had been payment of premium, for non-payment by it was not chargeable against its fault. Had
all the logs been lost during the loading operations, but after the issuance of the Cover Note, liability on the
note would have already arisen even before payment of premium. This is how the cover note as a "binder"
should legally operate otherwise, it would serve no practical purpose in the realm of commerce, and is
supported by the doctrine that where a policy is delivered without requiring payment of the premium, the
presumption is that a credit was intended and policy is valid.

The defense of delay as raised by Workmen in resisting the claim cannot be sustained. The law
requires this ground of delay to be promptly and specifically asserted when a claim on the insurance
agreement is made. The undisputed facts show that instead of invoking the ground of delay in objecting to
Pacific's claim of recovery on the cover note, it took steps clearly indicative that this particular ground for
objection to the claim was never in its mind. The nature of this specific ground for resisting a claim places

12 9
Ot h er Ca s e s i n th e Pol i c y of I ns ura nc e , Pa rt i e s a nd t h ei r R i gh ts

the insurer on duty to inquire when the loss took place, so that it could determine whether delay would be
a valid ground upon which to object to a claim against it.

In the proceedings that took place later in the Office of the Insurance Commissioner, Workmen
should then have raised this ground of delay to avoid liability. It did not do so. It must be because it did not
find any delay, as this Court fails to find a real and substantial sign thereof. But even on the assumption that
there was delay, this Court is satisfied and convinced that as expressly provided by law, waiver can
successfully be raised against Workmen.

13 0
Ot h er Ca s e s i n th e Pol i c y of I ns ura nc e , Pa rt i e s a nd t h ei r R i gh ts

DEVELOPMENT INSURANCE v. INTERMEDIATE APPELLATE COURT


G.R. No. 71360
July 16, 1986

J. Cruz

Facts:

A fire occurred in the building of Philippine Union. It sued for recovery of damages from the
Development Insurance Corp. on the basis of an insurance contract between them. The Development
Insurance Corp. failed to answer on time despite the numerous extensions it asked for. It was declared in
default by the trial court. A judgment of default was subsequently rendered on the strength of the evidence
given by the respondent, which was allowed damages. The Development Insurance Corp moved to lift the
order of default. Its motion was denied. It went to the appellate court, which affirmed the decision of the
trial court.

Issue:

Was Philippine Union required to jointly indemnify the insured building?

Ruling:

No. In this case, the building was insured at P2,500,000.00 by agreement of the insurer and
the insured.

The agreement is known as an open policy and is subject to the express condition that: “In the
event of loss, whether total or partial, it is understood that the amount of the loss shall be subject to
appraisal and the liability of the company, if established, shall be limited to the actual loss, subject to the
applicable terms, conditions, warranties and clauses of this Policy, and in no case shall exceed the amount
of the policy.” Under an open policy of insurance value of the damaged portion shall be paid in full by
insurer, in the absence of evidence of greater value of entire building over the amount of insurance bought
and where the damage was worth less than the latter.

Section 60 of the Insurance Code defines an open policy is one in which the value of the
thing insured is not agreed upon but is left to be ascertained in case of loss. This means that the actual loss,
as determined, will represent the total indemnity due the insured from the insurer except only that the
total indemnity shall not exceed the face value of the policy. Applying the open policy clause as expressly
agreed upon, the private respondent is entitled to indemnity in the total amount of P508,867.00. The
refusal of its vice-president to receive the private respondent's complaint was the first indication of the
petitioner's intention to prolong this case and postpone the discharge of its obligation to the private
respondent under this agreement. They still evaded payment for 5 years.

13 1
Ot h er Ca s e s i n th e Pol i c y of I ns ura nc e , Pa rt i e s a nd t h ei r R i gh ts

FINMAN GENERAL ASSURANCE CORP. v. WILLIAM INOCENCIO, ET AL.


G.R. No. 90273-75
November 15, 1989

Feliciano, J.

Facts:

Pan Pacific Overseas Recruiting Services, Inc. is a private, fee-charging, recruitment and
employment agency. William Inocencio, Perfecto Palero, Jr., Edwin Cardones and one Edwin Hernandez
filed with the POEA complaints against Pan Pacific for violation of Articles 32 and 34 (a) of the Labor Code,
and for refund of placement fees paid to Pan Pacific. They alleged that Pan Pacific charged and collected
such fees from them but did not secure employment for them.

The POEA Administrator motu proprio impleaded Finman as party respondent in its capacity as
surety for Pan Pacific. Finman denied liability and plead that the POEA had no jurisdiction over surety
bonds, that jurisdiction being vested in the Insurance Commission or the regular courts, that it (Finman)
had not violated Articles 32 and 34 (a) of the Labor Code and complainants' claims had accrued during the
suspension of Pan Pacific, and complainants had no cause of action against Finman.

Issue:

Can the complainants Innocencio et al. collect from Finman General Assurance Corp as surety?

Ruling:

YES. While Finman has refrained from attaching a copy of the bond it had issued to its Petition for
Certiorari, there can be no question that the conditions of the Finman surety bond Pan Pacific had posted
with the POEA include the italicized portions of Section 4, Rule 11, Book I quoted above. It is settled
doctrine that the conditions of a bond specified and required in the provisions of the statute or regulation
providing for the submission of the bond, are incorporated or built into all bonds tendered under that
statute or regulation, even though not there set out in printer's ink.

Clearly, Finman is a party-in-interest in, certainly a proper party to, the proceedings Innocencio et
al had initiated against Pan Pacific the principal obligor. Since Pan Pacific had thoughtfully refrained from
notifying the POEA of its new address and from responding to the complaints, Finman may well be
regarded as an indispensable party to the proceedings before the POEA. Whether Finman was an
indepensable or merely a proper party to the proceedings, the Court believes and so holds that the POEA
could properly implead it as respondent either upon the request of Innocencio et al or, as it
happened, motu propio. Such is the situation under the Revised Rules of Court and the application thereof,
directly or by analogy, by the POEA can certainly not be regarded as arbitrary, oppressive or capricious.
The POEA and DOLE have the power to compel surety to make good on its solidary undertaking in the
same proceedings where the liability of the principal obligor is determined.

13 2
Ot h er Ca s e s i n th e Pol i c y of I ns ura nc e , Pa rt i e s a nd t h ei r R i gh ts

FORTUNE INSURANCE AND SURETY CO., INC. v. COURT OF APPEALS and PRODUCERS
BANK OF THE PHILIPPINES
G.R. No. 115278
May 23, 1995

Davide, Jr., J.

Facts:

Producers Bank of the Philippines was insured by Fortune Insurance. Producers Bank filed
against Fortune Insurance a complaint for recovery of the sum of P725,000.00 under the policy issued by
Fortune. The sum was allegedly lost during a robbery of Producer’s armored vehicle while it was in transit
to transfer the money from Pasay City Branch to its head office in Makati City.

The armored vehicle was robbed by its driver Magalong and security guard Atiga tasked to man
the same. Both of them are not Producers Bank’s employees but were merely assigned by and affiliated
with PRC Management Systems and Unicorn Security Services.

Fortune Insurance refused to pay the amount as the loss, according to it, is excluded from the
coverage of the insurance policy. Producers Bank opposed the contention of Fortune Insurance and
contends that Atiga and Magalong are not its officer, employee, trustee, or authorized representative at the
time of the robbery. When Producers commissioned a guard and a driver to transfer its funds from one
branch to another, they effectively and necessarily became its authorized representatives in the care and
custody of the money. Assuming that they could not be considered authorized representatives, they were
employees of Producers.

Issue:

Is the Fortune Insurance and Surety Co. Inc. liable under the Money, Security, and Payroll Robbery
policy it issued to Producers Bank of the Philippines?
Is the recovery precluded under the general exceptions clause of the policy?

Ruling:

Except with respect to compulsory motor vehicle liability insurance, the Insurance Code contains
no other provisions applicable to casualty insurance or to robbery insurance in particular. These contracts
are, therefore, governed by the general provisions applicable to all types of insurance. Outside of these, the
rights and obligations of the parties must be determined by the terms of their contract, taking into
consideration its purpose and always in accordance with the general principles of insurance law.

It has been aptly observed that in burglary, robbery, and theft insurance, "the opportunity to
defraud the insurer — the moral hazard — is so great that insurers have found it necessary to fill up their
policies with countless restrictions, many designed to reduce this hazard. Seldom does the insurer assume
the risk of all losses due to the hazards insured against." Persons frequently excluded under such
provisions are those in the insured's service and employment. The purpose of the exception is to guard
against liability should the theft be committed by one having unrestricted access to the property. In such
cases, the terms specifying the excluded classes are to be given their meaning as understood in common
speech. The terms "service" and "employment" are generally associated with the idea of selection, control,
and compensation.

A contract of insurance is a contract of adhesion, thus any ambiguity therein should be resolved
against the insurer, or it should be construed liberally in favor of the insured and strictly against the
insurer. Limitations of liability should be regarded with extreme jealousy and must be construed in such a
way, as to preclude the insurer from non-compliance with its obligation. It goes without saying then that if

13 3
Ot h er Ca s e s i n th e Pol i c y of I ns ura nc e , Pa rt i e s a nd t h ei r R i gh ts

the terms of the contract are clear and unambiguous, there is no room for construction and such terms
cannot be enlarged or diminished by judicial construction.

An insurance contract is a contract of indemnity upon the terms and conditions specified
therein. It is settled that the terms of the policy constitute the measure of the insurer's liability. In the
absence of statutory prohibition to the contrary, insurance companies have the same rights as individuals
to limit their liability and to impose whatever conditions they deem best upon their obligations not
inconsistent with public policy.

13 4
Ot h er Ca s e s i n th e Pol i c y of I ns ura nc e , Pa rt i e s a nd t h ei r R i gh ts

COUNTRY BANKERS INSURANCE CORPORATION v. LIANGA BAY AND COMMUNITY MULTI-


PURPOSE COOPERATIVE, INC.
G.R. No. 136914
January 25, 2002

De Leon, Jr., J.

Facts:

Llanga Bay and Community Multi-Purpose Cooperative, and Country Bankers Insurance
Corporation entered into a contract of fire insurance to protect Lianga Bay’s stocks-in-trade against fire
loss, damage or liability for P200,000. Lianga Bay’s building was gutted in a fire resulting in the total loss
of the stocks-in-trade, pieces of furniture and fixtures, equipment and records. Lianga Bay filed its claim
under the fire insurance policy.

Country Banker’s Insurance Corp. denied the insurance claim on the ground that the building was
set on fire by two (2) NPA rebels who wanted to obtain canned goods, rice and medicines as provisions for
their comrades in the forest, and that such loss was an excepted risk under paragraph No. 6 of the policy
conditions of Fire Insurance Policy. Paragraph 6 provides that the insurance does not cover any loss or
damage through or in consequence of mutiny, riot, military or popular uprising, insurrection, rebellion,
revolution, military or usurped power. The RTC ruled in favor of Lianga Bay. The CA affirmed the decision
of the RTC.

Issue:

Are the stocks-in-trade burned by NPA rebels considered as an excepted risk under the fire
insurance policy?

Ruling:

Where a risk is excepted by the terms of a policy which insures against other perils or hazards,
loss from such a risk constitutes a defense which the insurer may urge, since it has not assumed that risk,
and from this it follows that an insurer seeking to defeat a claim because of an exception or limitation in
the policy has the burden of proving that the loss comes within the purview of the exception or limitation
set up. If a proof is made of a loss apparently within a contract of insurance, the burden is upon the insurer
to prove that the loss arose from a cause of loss which is excepted or for which it is not liable, or from a
cause which limits its liability. Stated elsewise, since the petitioner in this case is defending on the ground
of non-coverage and relying upon an exemption or exception clause in the fire insurance policy, it has the
burden of proving the facts upon which such excepted risk is based, by a preponderance of evidence. But
Country Bankers Insurance Corp. failed to do so. The Country Bankers Insurance Corp. relies only on the
sworn statements of Jose Lomocso and Ernesto Urbiztondo and spot report of Pfc. Arturo V. Juarbal. These
are inadmissible in evidence, for being hearsay, inasmuch as they did not take the witness stand and could
not therefore be cross-examined.

13 5
Ot h er Ca s e s i n th e Pol i c y of I ns ura nc e , Pa rt i e s a nd t h ei r R i gh ts

PARAMOUNT INSURANCE CORPORATION v. SPOUSES YVES and MARIA TERESA


REMONDEULAZ
G.R. No. 173773
November 28, 2012

Peralta, J.

Facts:

On May 26, 1994, Spouses Yves insured with Paramount Insurance Corp their Toyota Corolla
Sedan under a comprehensive motor vehicle insurance policy for one year. Spouses Yves’ car was
unlawfully taken by a certain Ricardo Sales who took possession of the subject vehicle to add
accessories and improvements, however, Sales failed to return the subject vehicle within the agreed three-
day period. The spouses notified Paramount to claim for the reimbursement of their lost vehicle but
Paramount refused to pay. A complaint for a sum of money against Paramount before the Regional Trial
Court of Makati City was filed but was denied as loss of the vehicle is not a peril covered by the policy. The
Court of Appeals reversed and set aside the Order issued by the trial court as well as the motion for
reconsideration.

Issue:

Is the Paramount Insurance Corp. liable to pay the loss of the vehicle of Spouses Yves?

Ruling:

Yes. Records would show that spouses Yves entrusted possession of their vehicle only to the
extent that Sales will introduce repairs and improvements thereon, and not to permanently deprive
them of possession thereof. Since, theft can also be committed through misappropriation, the fact that
Sales failed to return the subject vehicle to spouses Yves constitutes Qualified Theft. Hence, since spouses
Yves’ car is undeniably covered by a Comprehensive Motor Vehicle Insurance Policy that allows for
recovery in cases of theft, Paramount is liable under the policy for the loss of the vehicle under the "theft
clause." Sales act of depriving spouses Yves of their motor vehicle at, or soon after the transfer of physical
possession of the movable property, constitutes theft under the insurance policy, which is compensable.

13 6
Co nc ea l m e n t, M is r ep r e se n ta t io n a nd B r ea ch o f Wa r ra nt i es

FRANCISCA EGUARAS vs. THE GREAT EASTERN LIFE ASSURANCE COMPANY


G.R. No. L-10436
January 24, 1916

Torres, J

Facts:

In the present civil suit, it is not a question whether the acts performed by Eguaras and others
interested in the proceeds of the insurance were criminal, but whether in taking out the insurance on the
life of Dominador Albay there occurred in the operation deceit and fraud of a civil nature, in the form and
under the conditions defined by the Civil Code. On October 14, 1912, through the efforts of the defendant
company’s agent, Ponciano Remigio, Dominador Albay got the insurance company to insure his life for the
sum of P5,000 and that through the representations and statements made by said Dominador Albay in his
application and the favorable medical examination made by Dr. Jose A. Vidal, the company agreed to the
life insurance sought, and on November 6, 1912, issued the policy No. 5592, the value whereof was payable
to the insured’s mother-in-law, Francisca Eguaras.

One month after said insurance policy had been issued, that is, on December 6, 1912, the insured
Dominador Albay died in the municipality of Santa Cruz, Laguna, of intestinal occlusion, according to the
certificate of Dr. R. Kamatoy, after an illness of three days, with medical attendance. The defendant
company, according to the declaration of its own agent in these Islands, despite having received
satisfactory proofs of the death of the insured, refused to pay the amount of the insurance, alleging that it
had been secured through fraud and deceit and was therefore illegal and void. The court was therefore
asked to render judgment against the Great Eastern Life Assurance Company, Ltd., and its general agent,
West G. Smith, by sentencing them to pay to the plaintiff the sum of P5,000, the value of policy No. 5592,
plus the sum of P1,000 for damages inflicted upon them, in addition to the costs of the suit.

Issue:

Is life insurance issued through fraud and deceit, and in such case, by the defendant, The Great
Eastern Life Assurance Company, Ltd., no longer under obligation to pay the value thereof to the plaintiff?

Ruling:

Yes. Article 1269 of the Civil Code states: “There is deceit when by words or insidious
machinations on the part of one of the contracting parties the other is induced to execute a contract which
without them he would not have made.”

It is essential to the nature of the deceit, to which the foregoing article refers, that said deceit be
prior to or contemporaneous with the consent that is a necessary requisite for perfecting the contract, but
not that it may have occurred or happened thereafter. A contract is therefore deceitful, for the execution
whereof the consent of one of the parties has been secured by means of fraud, because he was persuaded
by words or insidious machinations, statements or false promises, and a defective consent wrung from
him, even though such do not constitute estafa or any other criminal act subject to the penal law.

The defendant company accepted the application for insurance made by Dominador Albay and
executed the contract comprised under articles 416 to 431 of the Code of Commerce, although for the
perfecting thereof the insured, Albay, as he was not in good health, by connivance with the insurance
company’s agent, presented Castor Garcia to the physician Vidal, who was commissioned by the company
to examine applicants for life insurance and in view of the favorable report of the said physician, who
reported and certified that the person examined by him under the name of Dominador Albay was in good
health and possessed the qualifications required by said insurance company for perfecting the contract, so
the company freely and willingly consented to the execution thereof, effectively induced thereto by the

13 7
Co nc ea l m e n t, M is r ep r e se n ta t io n a nd B r ea ch o f Wa r ra nt i es

result of the medical examination and of the favorable professional report issued in view of the appearance
of an individual who was in good health, but different from the invalid who was seeking to be insured and
who died one month and twenty-three days after the insurance had been granted.
The fraud which gave rise to the mistaken consent, given by the defendant company to the application
for insurance made by Albay and to the execution of the contract through deceit, is plain and
unquestionable. This fraud consisted in the substitution at the examination of Castor Garcia in place of
the insured Dominador Albay, and as the deceit practiced in the said contract is of a serious nature, the
same is ipso facto void and ineffective, in accordance with the provisions of article 1270 of the Civil Code.

For all the foregoing reasons the first part of the judgment appealed from, with reference to the
payment of P5,000 to the plaintiff, must be reversed and the defendant, The Great Eastern Life Assurance
Company, Ltd., absolved from the complaint, as we do absolve it; and we affirm the second part of said
judgment in so far as it absolves W. G. Smith and dismisses the petition for damages; without special
finding as to the costs in both instances.

13 8
Co nc ea l m e n t, M is r ep r e se n ta t io n a nd B r ea ch o f Wa r ra nt i es

MRS. HENRI HARDING v. COMMERCIAL UNION ASSURANCE COMPANY


G.R. No. L-12707
August 10, 1918

Fisher, J.

Facts:

In February 1916, Mrs. Harding applied for car insurance for a Studebaker she received as a gift
from her husband. She was assisted by Smith, Bell, and Co. which was the duly authorized representative
(insurance agent) of Commercial Union Assurance Company in the Philippines. The car’s value was
estimated with the help of an experienced mechanic (Mr. Server) of the Luneta Garage. The car was bought
by Mr. Harding for P2,800.00. The mechanic, considering some repairs done, estimated the value to be at
P3,000.00. This estimated value was the value disclosed by Mrs. Harding to Smith, Bell, and Co. She also
disclosed that the value was an estimate made by Luneta Garage (which also acts as an agent for Smith,
Bell, and Co).

In March 1916, a fire destroyed the Studebaker. Mrs. Harding filed an insurance claim but
Commercial Union denied it as it insisted that the representations and averments made as to the cost of
the car were false; and that said statement was a warranty. Commercial Union also stated that the car does
not belong to Mrs. Harding because such a gift [from her husband] is void under the Civil Code.

Issue:

Is Mrs. Harding entitled to the insurance claim?

Ruling:

Yes. Commercial Union is not the proper party to attack the validity of the gift made by Mr. Harding
to his wife.

The statement made by Mrs. Harding as to the cost of the car is not a warranty. The evidence does
not prove that the statement is false. In fact, the evidence shows that the cost of the car is more than the
price of the insurance. The car was bought for P2,800.00 and then thereafter, Luneta Garage made some
repairs and body paints which amounted to P900.00. Mr. Server attested that the car is as good as new at
the time the insurance was effected.

Commercial Union, upon the information given by Mrs. Harding, and after an inspection of the
automobile by its examiner, having agreed that it was worth P3,000, is bound by this valuation in the
absence of fraud on the part of the insured. All statements of value are, of necessity, to a large extent
matters of opinion, and it would be outrageous to hold that the validity of all valued policies must depend
upon the absolute correctness of such estimated value.

13 9
Co nc ea l m e n t, M is r ep r e se n ta t io n a nd B r ea ch o f Wa r ra nt i es

TAN CHAY HENG v. THE WEST COAST LIFE INSURANCE COMPANY,


G.R. No. L-27541
November 21, 1927

Avanceña, C.J.

Facts:

In 1926, Tan Chay Heng sued West Coast on the policy allegedly issued to his “uncle”, Tan Caeng
who died in 1925. He was the sole beneficiary thereof. West Coast refused on the ground that the policy
was obtained by Tan Caeng with the help of agents Go Chuilian, Francisco Sanchez and Dr. Locsin of West
Coast. West Coast said that it was made to appear that Tan Caeng was single, a merchant, health and not a
drug user, when in fact he was married, a laborer, suffering from tuberculosis and addicted to drugs.

West Coast now denies liability based on these misrepresentations. Tan Chay contends that West
Coast may not rescind the contract because an action for performance has already been filed. Trial court
found for Tan Chay holding that an insurer cannot avoid a policy which has been procured by fraud unless
he brings an action to rescind it before he is sued thereon.

Issue:

Is West Coast’s action for rescission barred by the collection suit filed by Tan Chay?

Ruling:

No. Precisely, the defense of West Coast was that through fraud in its execution, the policy is void
ab initio, and therefore, no valid contract was ever made. Its action then cannot be fore rescission because
an action to rescind is founded upon and presupposes the existence of the contract. Hence, West Coast’s
defense is not barred by Sec. 47.

In the instant case, it will be noted that even in its prayer, the defendant does not seek to have the
alleged insurance contract rescinded. It denies that it ever made any contract of insurance on the life of
Tan Caeng, or that any such a contract ever existed, and that is the question which it seeks to have litigated
by its special defense. In the very nature of things, if the defendant never made or entered into the contract
in question, there is no contract to rescind, and, hence, section 47 upon which the lower court based its
decision in sustaining the demurrer does not apply.

As stated, an action to rescind a contract is founded upon and presupposes the existence of the
contract which is sought to be rescinded. If all of the material matters set forth and alleged in the
defendant's special plea are true, there was no valid contract of insurance, for the simple reason that the
minds of the parties never met and never agreed upon the terms and conditions of the contract. We are
clearly of the opinion that, if such matters are known to exist by a preponderance of the evidence, they
would constitute a valid defense to plaintiff's cause of action. Upon the question as to whether or not they
are or are not true, we do not at this time have or express any opinion, but we are clear that section 47
does not apply to the allegations made in the answer, and that the trial court erred in sustaining the
demurrer.

14 0
Co nc ea l m e n t, M is r ep r e se n ta t io n a nd B r ea ch o f Wa r ra nt i es

ARGENTE v. WEST COAST


G.R. No. L-24899
March 19, 1928

Malcolm, J.

Facts:

In February 1925, Bernardo Argente and his wife applied for a joint life insurance under West
Coast Life Insurance Company (West Coast). The couple was examined by the insurance company doctor
(Doctor Sta. Ana). The couple disclosed to the doctor that they never had any serious medical histories;
that they were never confined; that Vicenta De Ocampo (wife of Argente) was not an alcoholic. Doctor Sta.
Ana then recommended the approval of the application. In May 1925, the couple were issued with the
insurance policy. In November 1925, Vicenta died. West Coast Life denied the subsequent insurance claim
filed by Argente as it averred that the application made in June was attended by fraud because the couple
failed to disclose the fact that each of them were actually confined prior to their application; that Vicenta
in particular was diagnosed for alcoholism and ultimately for psycho-neurosis; that in sum, their statement
as to their health and previous illnesses within the last 5-7 years prior to their application were untrue.

Argente conceded to the allegations of West Coast however he stated that those facts were actually
disclosed to Dr. Sta. Ana however Dr. Sta. Ana connived with the insurance agent hence he failed to record
them in the medical reports. Further, Argente averred that if West Coast did have the right to rescind the
insurance, it should have done so prior to the filing of a suit involving the insurance claim.

Issue:

Is Argente entitled to the insurance claim?

Ruling:

No. In an action on a life insurance policy where the evidence conclusively shows that the answers
to questions concerning diseases were untrue, the truth or falsity of the answers become the determining
factor. If the policy was procured by fraudulent representations, the contract of insurance apparently set
forth therein was never legally existent. It can fairly be assumed that had the true facts been disclosed by
the assured, the insurance would never have been granted. The allegations of Argente do not have a leg to
stand on, Dr. Sta. Ana has no motive whatsoever and such alleged illicit act will only destroy his reputation
as a physician.

As to the allegation of Argente regarding the failure of West Coast to rescind the insurance prior to the
filing of this case, there are two answers:

1. The failure of West Coast to rescind the contract cannot prejudice any defense to the suit which
concealment may furnish.
2. Prior to the filing of this case, West Coast sent a notice to Argente advising him that the policy is
being canceled due to the concealment and that his premium is being refunded – this operates as
a rescission to the contract of insurance.

14 1
Co nc ea l m e n t, M is r ep r e se n ta t io n a nd B r ea ch o f Wa r ra nt i es

MUSÑGI v. WEST COAST LIFE ASSURANCE CO.


G.R. No. L-41794
August 30, 1935
Imperial. J.

Facts:

Arsenio Garcia was insured by West Coast twice in 1931. In both policies, he was asked to answer
the question: “what physician or practitioners have you consulted or been treated by, and for what illness or
ailment?

In both policies, he answered in the negative. It turned out that from 1929 to 1939, he went to see
several physicians for a number of ailments. So when he died in 1942, the company refused to pay the
proceeds of the insurance.

Issue:

Did the answer given by Arsenio in the policies justifies the company’s refusal to pay?

Ruling:

Yes. One ground for the rescission of a contract of insurance under the Insurance Act is a
"concealment", which in section 25 is defined as "A neglect to communicate that which a party knows and
ought to communicate". Appellant argues that the alleged concealment was immaterial and insufficient to
avoid the policy. We cannot agree. In an action on a life insurance policy where the evidence conclusively
shows that the answers to questions concerning diseases were untrue, the truth or falsity of the answers
become the determining factor. If the policy was procured by fraudulent representations, the contract of
insurance apparently set forth therein was never legally existent. It can fairly be assumed that had the true
facts been disclosed by the assured, the insurance would never have been granted.

Aresenio knew that he was suffering from a number of ailments, yet, he concealed this. Such
concealment and his false statements constituted fraud, because the insurance company by reasons of
such statement accepted the risk which it would otherwise have rejected.

14 2
Co nc ea l m e n t, M is r ep r e se n ta t io n a nd B r ea ch o f Wa r ra nt i es

INSULAR LIFE ASSURANCE CO., LTD. v. SERAFIN FELICIANO


G.R. No. L-47593
December 29, 1943

Ozaeta, J.

Facts:

From the court’s decision rendered in the case of Insular Life Assurance v. Feliciano (1941), Insular
Life filed a motion for reconsideration. Insular avers that Feliciano is not entitled to the claim because the
insurance policy is void ab initio; that he connived with the insurance agent and the medical examiner; and
that at best, Feliciano is only entitled to refund or the reimbursement of what he has paid in premium.

Issue:

Is Insular Life correct?


Ruling:

Yes. This time, the Supreme Court held that Insular Life’s contention is correct. When Evaristo
Feliciano, the applicant for insurance, signed the application in blank and authorized the soliciting agent
and/or medical examiner of Insular to write the answers for him, he made them his own agents for that
purpose, and he was responsible for their acts in that connection. If they falsified the answers for him, he
could not evade the responsibility for the falsification. He was not supposed to sign the application in blank.
He knew that the answers to the questions therein contained would be “the basis of the policy,” and for
that very reason he was required with his signature to vouch for truth thereof.

14 3
Co nc ea l m e n t, M is r ep r e se n ta t io n a nd B r ea ch o f Wa r ra nt i es

YU PANG CHENG alias YU PANG CHING v. CA


G.R. No. L-12465
May 29, 1959

Bautista Angelo, J.

Facts:

Yu Pang Eng submitted his application for insurance to the defendant. He answered “no” to
questions on his medical history as well as to the question of whether he consulted any physician
regarding said diseases. Upon payment of the first premium, the company issued to him an insurance
policy. On December 1950, he went to St. Luke’s for medical treatment but he died two months later due
to infiltrating medullary carcinoma, Grade 4, advanced cardiac and of lesser curvature, stomach
metastases spleen.

His brother and beneficiary, Yu Pang Cheng, demanded the payment of the policy proceeds, but
his demand was refused. The insurance company’s defense was that the insured was guilty of
misrepresentation and concealment of material facts, hence, the effect is the avoiding of the policy.

Issue:

Is the insured guilty of concealment of material facts?

Ruling:

Yes. A neglect to communicate that which a party knows and ought to communicate is called
concealment. Whenever intentional the concealment entitles the insurer to rescind the contract of
insurance.

The insurance law requires the insured to communicate to the insurer all facts within his
knowledge which are material to the contract and which the other party has not the means of ascertaining
(Section 27), and the materiality is to be determined not by the event but solely by the probable and
reasonable influence of the facts upon the party to whom the communication is due (Section 30 of Act
2427.)

14 4
Co nc ea l m e n t, M is r ep r e se n ta t io n a nd B r ea ch o f Wa r ra nt i es

ESTEFANIA SATURNINO v. THE PHILIPPINE AMERICAN LIFE INSURANCE COMPANY


G.R. No. L-16163
February 28, 1963

Makalintal, J.

Facts:

In September 1957, Estefania Saturnino was operated for cancer in which her right breast was
removed. She was advised by her surgeon that she’s not totally cured because her cancer was malignant.
In November 1957, she applied for an insurance policy under Philamlife (Philippine American Life
Insurance Company). She did not disclose the fact that she was operated nor did she disclose any medical
histories. Philamlife, upon seeing the clean bill of health from Estefania waived its right to have Estefania
undergo a medical checkup. In September 1958, Estefania died of pneumonia secondary to influenza. Her
heirs now seek to enforce the insurance claim.

Issue:

Is Saturnino entitled to the insurance claim?

Ruling:

No. The concealment of the fact of the operation is fraudulent. Even if, as argued by the heirs,
Estefania never knew she was operated for cancer, there is still fraud in the concealment no matter what
the ailment she was operated for. Note also that in order to avoid a policy, it is not necessary that actual
fraud be established otherwise insurance companies will be at the mercy of any one seeking insurance.

In this jurisdiction a concealment, whether intentional or unintentional, entitles the insurer to


rescind the contract of insurance, concealment being defined as “negligence to communicate that which a
party knows and ought to communicate.”

Also, the fact that Philamlife waived its right to have Estefania undergo a medical examination is
not negligence. Because of Estefania’s concealment, Philamlife considered medical checkup to be no longer
necessary. Had Philamlife been informed of her operation, she would have been made to undergo medical
checkup to determine her insurability.

14 5
Co nc ea l m e n t, M is r ep r e se n ta t io n a nd B r ea ch o f Wa r ra nt i es

GREAT PACIFIC v. CA
G.R. No. L-31845
April 30, 1979

De Castro, J.

Facts:

Ngo Hing filed an application with the Great Pacific for a twenty-year endowment policy in on the
life of his one-year old daughter Helen. He supplied the essential data which petitioner Mondragon, the
Branch Manager, wrote on the form.

Upon the payment of the insurance premium, the binding deposit receipt was issued Ngo Hing.
Likewise, petitioner Mondragon handwrote at the bottom of the back page of the application form his
strong recommendation for the approval of the insurance application. Then Mondragon received a letter
from Pacific Life disapproving the insurance application. The letter stated that the said life insurance
application for 20-year endowment plan is not available for minors below seven years old, but Pacific Life
can consider the same under the Juvenile Triple Action Plan, and advised that if the offer is acceptable, the
Juvenile Non-Medical Declaration be sent to the company.

The non-acceptance of the insurance plan by Pacific Life was allegedly not communicated by
petitioner Mondragon to private respondent Ngo Hing. Helen Go died of influenza. Ngo Hing sought the
payment of the proceeds of the insurance, but it was denied.

Issues:

1. Did the binding deposit receipt constitute a temporary contract of the life insurance in question?
2. Did Ngo Hing conceal the state of health and physical condition of Helen Go, which rendered void
the policy?

Ruling:
1. Yes. The receipt was intended to be merely a provisional insurance contract. Its perfection was
subject to compliance of the following conditions: (1) that the company shall be satisfied that the applicant
was insurable on standard rates; (2) that if the company does not accept the application and offers to issue
a policy for a different plan, the insurance contract shall not be binding until the applicant accepts the policy
offered; otherwise, the deposit shall be refunded; and (3) that if the company disapproves the application,
the insurance applied for shall not be in force at any time, and the premium paid shall be returned to the
applicant.

The receipt is merely an acknowledgment that the latter's branch office had received from the
applicant the insurance premium and had accepted the application subject for processing by the insurance
company. There was still approval or rejection the same on the basis of whether or not the applicant is
"insurable on standard rates." Since Pacific Life disapproved the insurance application of respondent Ngo
Hing, the binding deposit receipt in question had never become in force at any time. The binding deposit
receipt is conditional and does not insure outright. This was held in Lim v. Sun.

The deposit paid by private respondent shall have to be refunded by Pacific Life.

2. No. Ngo Hing had deliberately concealed the state of health of his daughter Helen Go. When he
supplied data, he was fully aware that his one-year old daughter is typically a mongoloid child. He withheld
the fact material to the risk insured.

14 6
Co nc ea l m e n t, M is r ep r e se n ta t io n a nd B r ea ch o f Wa r ra nt i es

“The contract of insurance is one of perfect good faith uberrima fides meaning good faith, absolute
and perfect candor or openness and honesty; the absence of any concealment or demotion, however
slight.” The concealment entitles the insurer to rescind the contract of insurance.

14 7
Co nc ea l m e n t, M is r ep r e se n ta t io n a nd B r ea ch o f Wa r ra nt i es

REGINA EDILLON v. MANILA BANKER LIFE ASSURANCE CORPORATION


G.R. No. L-34200
September 30, 1982

Vasquez, J.

Facts:

In April 1969, Carmen Lapuz filled out an application form for insurance under Manila Banker Life
Assurance Corporation. She stated that her date of birth was July 11, 1904. Upon payment of the Php 20.00
premium, she was issued the insurance policy in April 1969. In May 1969, Carmen Lapuz died in a
vehicular accident. Regina Edillon, who was named a beneficiary in the insurance policy sought to collect
the insurance proceeds but Manila Banker denied the claim. Apparently, it is a rule of the insurance
company that they were not to issue insurance policies to “persons who are under the age of sixteen (16)
years of age or over the age of sixty (60) years …” Note, that Lapuz was already 65 years old when she was
applying for the insurance policy.

Issue:

Is Edillon entitled to the insurance claim as a beneficiary?

Ruling:

Yes. Carmen Lapuz did not conceal her true age. Despite this, the insurance company still received
premium from Lapuz and issued the corresponding insurance policy to her. When the accident happened,
the insurance policy has been in force for 45 days already and such time was already sufficient for Manila
Banker to notice the fact that Lapuz is already over 60 years old and thereby cancel the insurance policy.
If Manila Banker failed to act, it is either because it was willing to waive such disqualification; or, through
the negligence or incompetence of its employees for which it has only itself to blame, it simply overlooked
such fact. Under the circumstances, Manila Banker is already deemed in estoppel.

14 8
Co nc ea l m e n t, M is r ep r e se n ta t io n a nd B r ea ch o f Wa r ra nt i es

NG GAN ZEE v. ASIAN CRUSADER LIFE ASSURANCE CORPORATION


G.R. No. L-30685
May 30, 1983

Escolin, J.

Facts:

In May 1962, Kwong Nam applied for a 20-year endowment policy with Asian Crusader Life
Assurance Corporation. Asian Crusader asked the following question:

Has any life insurance company ever refused your application for insurance or for
reinstatement of a lapsed policy or offered you a policy different from that applied for? If, so,
name company and date.

Kwong Nam answered “No” to the above question. Kwong Nam was also examined by Asian
Crusader’s medical examiner to whom he disclosed that he was once operated and a tumor was removed
from his stomach and such was “associated with ulcer of the stomach.”

Kwong Nam’s application was approved. In May 1963, he died. His widow, Ng Gan Zee, filed an
insurance claim but Asian Crusader refused her claim as it insisted that Kwong Nam concealed material
facts from them when he was applying for the insurance; that he misrepresented the fact that he was
actually denied application by Insular Life when he was renewing his application with them; that Kwong
Nam was actually operated for peptic ulcer.

Issue:

Can Ng Gan Zee collect from the insurance claim?

Ruling:

Yes. Asian Crusader was not able to prove that Kwong Nam’s statement that Insular Life did not
deny his insurance renewal with them is untrue. In fact, evidence showed that in April 1962, Insular Life
approved Kwong Nam’s request of reinstatement only with the condition that Kwong Nam’s plan will be
lowered from P50,000.00 to P20,000.00 considering his medical history.

Kwong Nam did not conceal anything from Asian Crusader. His statement that his operation, in
which a tumor the size of a hen’s egg was removed from his stomach, was only “associated with ulcer of
the stomach” and not peptic ulcer can be considered as an expression made in good faith of his belief as to
the nature of his ailment and operation. Indeed, such statement must be presumed to have been made by
him without knowledge of its incorrectness and without any deliberate intent on his part to mislead Asian
Crusader.

While it may be conceded that, from the viewpoint of a medical expert, the information
communicated was imperfect, the same was nevertheless sufficient to have induced Asian Crusader to
make further inquiries about the ailment and operation of Kwong Nam. It has been held that where, upon
the face of the application, a question appears to be not answered at all or to be imperfectly answered, and
the insurers issue a policy without any further inquiry, they waive the imperfection of the answer and
render the omission to answer more fully immaterial.

14 9
Co nc ea l m e n t, M is r ep r e se n ta t io n a nd B r ea ch o f Wa r ra nt i es

PACIFIC BANKING CORP. v. CA


G.R. No. L-41014
November 28, 1988

Paras, J.

Facts:

An open fire insurance policy, was issued to Paramount Shirt Manufacturing by Oriental
Assurance Corporation to indemnify by fire to the factory’s stocks, materials and supplies. The insured was
a debtor of Pacific Banking in the amount of (P800,000.00) and the goods described in the policy were held
in trust by the insured for Pacific Banking under trust receipts. The policy was endorsed to Pacific Banking
as mortgagee/ trustor of the properties insured, with the knowledge and consent of private respondent to
the effect that "loss if any under this policy is payable to the Pacific Banking Corporation".

A fire broke out on the premises destroying the goods contained in the building. The bank sent a
letter of demand to Oriental for indemnity. The company wasn’t ready to give since it was awaiting the
adjuster’s report. The company then made an excuse that the insured had not filed any claim with it, nor
submitted proof of loss which is a clear violation of Policy Condition No.11, as a result, determination of
the liability of private respondent could not be made.

Pacific Banking filed in the trial court an action for a sum of money for P61,000.00 against Oriental
Assurance. The defense of fraud, in the form of non-declaration of co-insurances which was not pleaded in
the answer, was also not pleaded in the Motion to Dismiss.

Issues:

Does unrevealed co-insurances violate conditions No. 3 of the policy?

Ruling:

Yes. Policy Condition No. 3 explicitly provides:

“3. The Insured shall give notice to the Company of any insurance already effected, or
which may subsequently be effected, covering any of the property hereby insured, and
unless such notice be given and the particulars of such insurance or insurances be stated
in or endorsed on this Policy by or on behalf of the Company before the occurrence of any
loss or damage, all benefit under this policy shall be forfeited.”

The insured failed to reveal before the loss three other insurances. Had the insurer known that
there were many co-insurances, it could have hesitated or plainly desisted from entering into such
contract. Hence, the insured was guilty of clear fraud. Concrete evidence of fraud or false declaration by
the insured was furnished by the petitioner itself when the facts alleged in the policy under clauses "Co-
Insurances Declared" and "Other Insurance Clause" are materially different from the actual number of co-
insurances taken over the subject property.

As the insurance policy against fire expressly required that notice should be given by the insured
of other insurance upon the same property, the total absence of such notice nullifies the policy.

Petitioner points out that Condition No. 3 in the policy in relation to the "other insurance clause"
supposedly to have been violated, cannot certainly defeat the right of the petitioner to recover the
insurance as mortgagee/assignee. Hence, they claimed that the purpose for which the endorsement or
assignment was made was to protect the mortgagee/assignee against any untoward act or omission of the

15 0
Co nc ea l m e n t, M is r ep r e se n ta t io n a nd B r ea ch o f Wa r ra nt i es

insured. It would be absurd to hold that petitioner is barred from recovering the insurance on account of
the alleged violation committed by the insured.

It is obvious that petitioner has missed all together the import of subject mortgage clause which
specifically provides:
“Loss, if any, under this policy, shall be payable to the PACIFIC BANKING CORPORATION
Manila mortgagee/trustor as its interest may appear, it being hereby understood and
agreed that this insurance as to the interest of the mortgagee/trustor only herein, shall
not be invalidated by any act or neglect—except fraud or misrepresentation, or arson—
of the mortgagor or owner/trustee of the property insured; provided, that in case the
mortgagor or owner/ trustee neglects or refuses to pay any premium, the mortgagee/
trustor shall, on demand pay the same.”

The paragraph clearly states the exceptions to the general rule that insurance as to the interest of
the mortgagee, cannot be invalidated; namely: fraud, or misrepresentation or arson. Concealment of the
aforecited co-insurances can easily be fraud, or in the very least, misrepresentation. Undoubtedly, it is but
fair and just that where the insured who is primarily entitled to receive the proceeds of the policy has by
its fraud and/or misrepresentation, forfeited said right.

15 1
Co nc ea l m e n t, M is r ep r e se n ta t io n a nd B r ea ch o f Wa r ra nt i es

TAN v. CA and THE PHILAM LIFE INSURANCE COMPANY


G.R. No. 48049
June 29, 1989

Gutierrez Jr., J.

Facts:

Tan Lee Siong, father of the petitioners, applied for life insurance with Philamlife. It was approved.
Tan Lee Siong died of hepatoma. Petitioners then filed a claim for the proceeds. The company denied
petitioners' claim and rescinded the policy by reason of the alleged misrepresentation and concealment of
material facts. The premiums paid on the policy were refunded. The petitioners filed a complaint in the
Insurance Commission. The latter dismissed the complaint.

Issue:

Did Philam have the right to rescind the contract of insurance as rescission must allegedly be done
during the lifetime of the insured within two years and prior to the commencement of action?

Ruling:

No. The Insurance Code states in Section 48:


“Whenever a right to rescind a contract of insurance is given to the insurer by any
provision of this chapter, such right must be exercised previous to the commencement of
an action on the contract.

After a policy of life insurance made payable on the death of the insured shall have been
in force during the lifetime of the insured for a period of two years from the date of its
issue or of its last reinstatement, the insurer cannot prove that the policy is void ab initio
or is rescindable by reason of the fraudulent concealment or misrepresentation of the
insured or his agent.”

The so-called "incontestability clause" in the second paragraph prevents the insurer from raising
the defenses of false representations insofar as health and previous diseases are concerned if the insurance
has been in force for at least two years during the insured's lifetime.

The policy was in force for a period of only one year and five months. Considering that the insured
died before the two-year period had lapsed, respondent company is not, therefore, barred from proving
that the policy is void ab initio by reason of the insured's fraudulent concealment or misrepresentation.

The "incontestability clause" added by the second paragraph of Section 48 is in force for two years.
After this, the defenses of concealment or misrepresentation no longer lie.

The petitioners argue that no evidence was presented to show that the medical terms were
explained in a layman's language to the insured. They also argue that no evidence was presented by
respondent company to show that the questions appearing in Part II of the application for insurance were
asked, explained to and understood by the deceased so as to prove concealment on his part. This couldn’t
be accepted because the insured signed the form. He affirmed the correctness of all the entries.

The company records show that the deceased was examined by Dr. Victoriano Lim and was found
to be diabetic and hypertensive. He was also found to have suffered from hepatoma. Because of the
concealment made by the deceased, the company was thus misled into accepting the risk and approving
his application as medically fit.

15 2
Co nc ea l m e n t, M is r ep r e se n ta t io n a nd B r ea ch o f Wa r ra nt i es

THELMA VDA. DE CANILANG v. CA


G.R. No. 92492
June 17, 1993

Feliciano, J.

Facts:

Jaime Canilang applied for a “non-medical” insurance policy with respondent Great Pacific Life
Assurance Company naming his wife, Thelma Canilang as his beneficiary. But he did not disclose the fact
that he was diagnosed as suffering from sinus tachycardia and that he has consulted a doctor twice. Jaime
was issued an ordinary life insurance policy with the face value of P19,700.00. Jaime died of “congestive
heart failure”, “anemia”, and “chronic anemia”. Petitioner widow and beneficiary of the insured, filed a
claim with Great Pacific which the insurer denied upon the ground that the insured had concealed material
information from it. Hence, Thelma filed a complaint against Great Pacific with the Insurance Commission
for recovery of the insurance proceeds.

Issue:

Is the non-disclosure of certain facts about the insured’s previous health conditions material to
warrant the denial of the claims of Thelma Canilang?

Ruling:

Yes. The Supreme Court agreed with the Court of Appeals that the information which Jaime
Canilang failed to disclose was material to the ability of Great Pacific to estimate the probable risk he
presented as a subject of life insurance. Had Canilang disclosed his visits to his doctor, the diagnosis made
and medicines prescribed by such doctor, in the insurance application, it may be reasonably assumed that
Great Pacific would have made further inquiries and would have probably refused to issue a non-medical
insurance policy or, at the very least, required a higher premium for the same coverage. The materiality of
the information withheld by Great Pacific did not depend upon the state of mind of Jaime Canilang. A man’s
state of mind or subjective belief is not capable of proof in our judicial process, except through proof of
external acts or failure to act from which inferences as to his subjective belief may be reasonably drawn.
Neither does materiality depend upon the actual or physical events which ensure. Materiality relates
rather to the “probable and reasonable influence of the facts” upon the party to whom the communication
should have been made, in assessing the risk involved in making or omitting to make further inquiries and
in accepting the application for insurance; that “probable and reasonable influence of the facts” concealed
must, of course, be determined objectively, by the judge ultimately.

15 3
Co nc ea l m e n t, M is r ep r e se n ta t io n a nd B r ea ch o f Wa r ra nt i es

GREAT PACIFIC LIFE ASSURANCE CORPORATION v. CA


G.R. No. 113899
October 13, 1999

Quisimbing, J.

Facts:

Great Pacific Life Assurance Corporation (Grepalife) executed a contract of group life insurance
with DBP wherein Grepalife agreed to insure the lives of eligible housing loan mortgagors of DBP.

One such loan mortgagor is Dr. Wilfredo Leuterio. Dr. Leuterio answered questions concerning
his test, attesting among others that he does not have any heart conditions and that he is in good health to
the best of his knowledge. However, after about a year, Dr. Leuterio died due to “massive cerebral
hemorrhage.” When DBP submitted a death claim to Grepalife, the latter denied the claim, alleging that Dr.
Leuterio did not disclose he had been suffering from hypertension, which caused his death. Allegedly, such
non-disclosure constituted concealment that justified the denial of the claim.

Hence, the widow of the late Dr. Leuterio filed a complaint against Grepalife for “Specific
Performance with Damages.” Both the trial court and the Court of Appeals found in favor of the widow and
ordered Grepalife to pay DBP.

Issue:

Is Grepalife liable to DBP as beneficiary in a group life insurance contract from a complaint filed
by the widow of the decedent/mortgagor?

Ruling:

Yes. The rationale of a group of insurance policy of mortgagors, otherwise known as the “mortgage
redemption insurance,” is a device for the protection of both the mortgagee and the mortgagor. On the part
of the mortgagee, it has to enter into such form of contract so that in the event of the unexpected demise
of the mortgagor during the subsistence of the mortgage contract, the proceeds from such insurance will
be applied to the payment of the mortgage debt, thereby relieving the heirs of the mortgagor from paying
the obligation. In a similar vein, ample protection is given to the mortgagor under such a concept so that in
the event of death, the mortgage obligation will be extinguished by the application of the insurance
proceeds to the mortgage indebtedness. In this type of policy insurance, the mortgagee is simply an
appointee of the insurance fund. Such loss-payable clause does not make the mortgagee a party to the
contract.

The insured, being the person with whom the contract was made, is primarily the proper person
to bring suit thereon. Subject to some exceptions, insured may thus sue, although the policy is taken wholly
or in part for the benefit of another person, such as a mortgagee.

And since a policy of insurance upon life or health may pass by transfer, will or succession to any
person, whether he has an insurable interest or not, and such person may recover it whatever the insured
might have recovered, the widow of the decedent Dr. Leuterio may file the suit against the insurer,
Grepalife.

15 4
Co nc ea l m e n t, M is r ep r e se n ta t io n a nd B r ea ch o f Wa r ra nt i es

MA. LOURDES S. FLORENDO v. PHILAM PLANS, INC.


G.R. No. 186983
February 22, 2012

Abad, J.

Facts:

Manuel Florendo filed an application for comprehensive pension plan with respondent Philam
Plans. Manuel signed the application and left to Perla the task of supplying the information needed in the
application. Respondent Abcede, Perla’s daughter, signed the application as sales counselor. Philam Plans
issued Pension Plan Agreement to Manuel, with petitioner Ma. Lourdes S. Florendo, his wife, as beneficiary.
In time, Manuel paid his quarterly premiums. Eleven months later, Manuel died of blood poisoning.
Subsequently, Lourdes filed a claim with Philam Plans for the payment of the benefits under her husband’s
plan but Philam Plans declined her claim prompting her to file the present action against the pension plan
company before the courts.

Issue:

Can Ma. Lourdes claim benefits as the beneficiary of her husband under the insurance plan despite
consideration that her husband Manuel concealed the true condition of his health?

Ruling:

No. The comprehensive pension plan that Philam Plans issued contains a one-year
incontestability period. It states:
“After this Agreement has remained in force for one (1) year, we can no longer contest f
or health reasons any claim for insurance under this Agreement, except for the reason
thatinstallment has not been paid (lapsed), or that you are not insurable at the time you
bought thispension program by reason of age. If this Agreement lapses but is reinstated
afterwards, theone (1) year contestability period shall start again on the date of approval
of your request for reinstatement.”

The above incontestability clause precludes the insurer from disowning liability under the policy
it issued on the ground of concealment or misrepresentation regarding the health of the insured after a
year of its issuance. Since Manuel died on the eleventh month following the issuance of his plan, the one
year incontestability period has not yet set in. Consequently, Philam Plans was not barred from
questioning Lourdes’ entitlement to the benefits of her husband’s pension plan.

15 5
Co nc ea l m e n t, M is r ep r e se n ta t io n a nd B r ea ch o f Wa r ra nt i es

MANILA BANKERS LIFE INSURANCE CORPORATION v. ABAN


G.R. No. 175666
July 29, 2013

Del Castillo, J.

Facts:

On July 3, 1993, Delia Sotero (Sotero) took out a life insurance policy from Manila Bankers Life
Insurance Corporation (Bankers Life), designating respondent Cresencia P. Aban (Aban), her niece, as her
beneficiary. Petitioner issued the policy in Sotero’s favor on August 30, 1993, after the requisite medical
examination and payment of the insurance premium. On April 10, 1996, when the insurance policy had
been in force for more than two years and seven months, Sotero died.

Respondent filed a claim for the insurance proceeds. Petitioner conducted an investigation into
the claim, and came out with the following findings: 1. Sotero did not personally apply for insurance
coverage, as she was illiterate; 2. Sotero was sickly since 1990; 3. Sotero did not have the financial
capability to pay the insurance premiums on Insurance Policy No. 747411; 4. Sotero did not sign the July
3, 1993 application for insurance; and 5. Respondent was the one who filed the insurance application, and
x x x designated herself as the beneficiary.

For the above reasons, petitioner denied respondent’s claim on April 16, 1997 and refunded the
premiums paid on the policy.

Issue:

Is Manila Bankers barred from denying the insurance claims based on fraud or concealment?

Ruling:

Yes. The “incontestability clause” is a provision in law that after a policy of life insurance made
payable on the death of the insured shall have been in force during the lifetime of the insured for a period
of two (2) years from the date of its issue or of its last reinstatement, the insurer cannot prove that the
policy is void ab initio or is rescindible by reason of fraudulent concealment or misrepresentation of the
insured or his agent.

The purpose of the law is to give protection to the insured or his beneficiary by limiting the
rescinding of the contract of insurance on the ground of fraudulent concealment or misrepresentation to
a period of only two (2) years from the issuance of the policy or its last reinstatement.

The insurer is deemed to have the necessary facilities to discover such fraudulent concealment or
misrepresentation within a period of two (2) years. It is not fair for the insurer to collect the premiums as
long as the insured is still alive, only to raise the issue of fraudulent concealment or misrepresentation
when the insured dies in order to defeat the right of the beneficiary to recover under the policy.

Section 48 serves a noble purpose, as it regulates the actions of both the insurer and the insured.
Under the provision, an insurer is given two years – from the effectivity of a life insurance contract and
while the insured is alive – to discover or prove that the policy is void ab initio or is rescindible by reason
of the fraudulent concealment or misrepresentation of the insured or his agent. After the two-year period
lapses, or when the insured dies within the period, the insurer must make good on the policy, even though
the policy was obtained by fraud, concealment, or misrepresentation. This is not to say that insurance
fraud must be rewarded, but that insurers who recklessly and indiscriminately solicit and obtain business
must be penalized, for such recklessness and lack of discrimination ultimately work to the detriment of
bona fide takers of insurance and the public in general.

15 6
Co nc ea l m e n t, M is r ep r e se n ta t io n a nd B r ea ch o f Wa r ra nt i es

SUN LIFE v. SIBYA


G.R. No. 211212
June 08, 2016

Reyes, J.

Facts:

Atty. Sibya applied for life insurance with Sun Life. In his application, he indicated that he had
sought advice for kidney problems. On February 5, 2001, the application was approved. The policy
indicated the respondents as beneficiaries and entitles them to a death benefit of P1,000,000.00 should
Atty. Jesus Jr. dies on or before February 5, 2021, or a sum of money if Atty. Jesus Jr. is still living on the
endowment date. On May 11, 2001, Atty. Sibya died as a result of a gunshot wound. As such, his
beneficiaries claimed from Sun Life, who denied the claim of the beneficiaries stating that Atty. Sibya did
not fully disclose his medical history in the application. Sun Life then filed a Complaint for Rescission.

Issue:

Was there concealment or misrepresentation?

Ruling:

Sun Life issued Atty. Jesus Jr.'s policy on February 5, 2001. Thus, it has two years from its issuance,
to investigate and verify whether the policy was obtained by fraud, concealment, or misrepresentation.
Upon the death of Atty. Jesus Jr., however, on May 11, 2001, or a mere three months from the issuance of
the policy, Sun Life loses its right to rescind the policy. As discussed in Manila Bankers, the death of the
insured within the two-year period will render the right of the insurer to rescind the policy nugatory. As
such, the incontestability period will now set in.

Assuming, however, for the sake of argument, that the incontestability period has not yet set in,
the Court agrees, nonetheless, with the CA when it held that Sun Life failed to show that Atty. Jesus Jr.
committed concealment and misrepresentation.

As correctly observed by the CA, Atty. Jesus Jr. admitted in his application his medical treatment
for kidney ailment. Moreover, he executed an authorization in favor of Sun Life to conduct investigation in
reference with his medical history.

The intent to defraud on the part of the insured must be ascertained to merit rescission of the
insurance contract. Concealment as a defense for the insurer to avoid liability is an affirmative defense and
the duty to establish such defense by satisfactory and convincing evidence rests upon the provider or
insurer. In the present case, Sun Life failed to clearly and satisfactorily establish its allegations, and is
therefore liable to pay the proceeds of the insurance.

15 7
Co nc ea l m e n t, M is r ep r e se n ta t io n a nd B r ea ch o f Wa r ra nt i es

MANULIFE PHILIPPINES, INC., v. HERMENEGILDA YBANEZ


G.R. No. 204736
November 28, 2016
Del Castillo, J.

Facts:

It is alleged in the Complaint that Insurance Policy Nos. 6066517-18 and 6300532-69 which
Manulife issued on October 25, 2002 and on July 25, 2003, respectively, both in favor of Dr. Ybañez, were
void due to concealment or misrepresentation of material facts in the latter's applications for life
insurance. Hermenegilda, wife of the said insured, was revocably designated as beneficiary in the subject
insurance policies. On November 17, 2003, when one of the subject insurance policies had been in force
for only one year and three months, while the other for only four months, the insured died. On December
10, 2003, Hermenegilda, now widow to the said insured, filed a claim with respect to the subject insurance
policies. The Death Certificate dated November 17, 2003 stated that the insured had "Hepatocellular CA.,
Crd Stage 4, secondary to Uric Acid Nephropathy; SAM Nephropathy recurrent malignant pleural effusion;
NASCVC”. Manulife conducted an investigation into the circumstances leading to the said insured's death,
in view of the aforementioned entries in the said insured's Death Certificate. Manulife thereafter concluded
that the insured misrepresented or concealed material facts at the time the subject insurance policies were
applied for; and that for this reason Manulife accordingly denied Hermenegilda's death claims and
refunded the premiums that the insured paid on the subject insurance policies.

Issue:

Did CA committed any reversible error in affirming the RTC Decision dismissing Manulife's
Complaint for rescission of insurance contracts for failure to prove concealment on the part of the insured?

Ruling:

No. Manulife failed to prove concealment on the part of the insured. The RTC correctly held that
the CDH's medical records that might have established the insured's purported misrepresentation/s or
concealment/s was inadmissible for being hearsay, given the fact that Manulife failed to present the
physician or any responsible official of the CDH who could confirm or attest to the due execution and
authenticity of the alleged medical records. Manulife had utterly failed to prove by convincing evidence
that it had been beguiled, inveigled, or cajoled into selling the insurance to the insured who purportedly
with malice and deceit passed himself off as thoroughly sound and healthy, and thus a fit and proper
applicant for life insurance. Manulife's sole witness gave no evidence at all relative to the particulars of the
purported concealment or misrepresentation allegedly perpetrated by the insured. In fact, Victoriano
merely perfunctorily identified the documentary exhibits adduced by Manulife; she never testified in
regard to the circumstances attending the execution of these documentary exhibits much less in regard to
its contents. Of course, the mere mechanical act of identifying these documentary exhibits, without the
testimonies of the actual participating parties thereto, adds up to nothing. These documentary exhibits did
not automatically validate or explain themselves. "The fraudulent intent on the part of the insured must be
established to entitle the insurer to rescind the contract. Misrepresentation as a defense of the insurer to
avoid liability is an affirmative defense and the duty to establish such defense by satisfactory and
convincing evidence rests upon the insurer."For failure of Manulife to prove intent to defraud on the part
of the insured, it cannot validly sue for rescission of insurance contracts.

15 8
Wa r ra nty

BACHRACH v. BRITISH AMERICAN


G.R. No. L-5715
December 20, 1910

Johnson, J.

Facts:

In July 1908, E.M. Bachrach commenced an action against British American Assurance Company
to recover P9, 841.50, the amount due, deducting the salvage, upon a fire insurance policy. British
American alleged certain facts under which it claimed that it was released from all obligations. First, that
Bachrach maintained a paint and varnish shop in the building where the goods which were insured were
stored. Second, that Bachrach transferred his interest in the property covered by the policy and that the
sanction of British American had not been obtained by Bachrach as required by the policy. Third, that
Bachrach, immediately preceding the outbreak of the alleged fire, willfully placed a gasoline can in the
upper story of the building, thereby greatly increasing the risk of fire. Fourth, that Bachrach made no proof
of the loss within the time required by the policy, nor did he file a statement of the goods alleged to have
been in the building at the time of the fire, nor of the goods saved, nor the loss suffered. The lower court
found that British American was liable.

Issue:

Is British American liable to Bachrach?

Ruling:

Yes. First, there was no provision in the policy prohibiting the keeping of paints and varnishes
upon the premises where the insured property was stored. If the company intended to rely upon a
condition of that character, it ought to have been plainly expressed in the policy. Second, there is also no
provision in the policy prohibiting Bachrach from placing a mortgage upon the property insured. But,
admitting that such a provision was intended, the interest in property insured does not pass by the mere
execution of a chattel mortgage and that, while a chattel mortgage is a conditional sale, there is no
alienation within the meaning of the insurance law until the mortgagee acquires a right to take possession
by default under the terms of the mortgage. No such right is claimed to have accrued in this case, and the
alienation clause is therefore inapplicable. Third, while the evidence shows some very peculiar and
suspicious circumstances concerning the burning of the goods covered by the policy, the SC did not find
that there is a preponderance of evidence showing that Bachrach did actually set fire or cause fire to be set
to the goods in question. Fourth, there was no requirement in the policy that a notice of loss be given.

15 9
Wa r ra nty

YOUNG v. MIDLAND
G.R. No. L-9370,
March 31, 1915

Johnson, J.

Facts:

Young, who ran a candy and fruit store in Escolta, Manila, entered into a contract of insurance with
defendant. One of the conditions stipulated in said contract of insurance is found in Warranty B prohibiting
the plaintiff from storing or keeping for sale hazardous goods or to carry out hazardous trade or process
in the building subject of the insurance and which Young used as residence and storehouse. Young,
however, stored fireworks in said building when the authorities prohibited their use in the celebration of
the Chinese New Year. When the building was destroyed by fire, said fireworks were found in the part of
the building not destroyed by fire. The trial court held that Young should be able to collect on the insurance
policy.

Issue:

Did the placing of the fireworks in the insured building violate the terms of the contract of
insurance, especially warranty B?

Ruling:

Yes. The compliance of the insured with the terms of the contract is a condition precedent to the
right of recovery. If the insured has violated or failed to perform the conditions of the contract, and such a
violation or want of performance has not been waived by the insurer, then the insured cannot recover. The
appellant argues that in view of the fact that the storing of the fireworks in the building did not contribute
in any way to the damage occasioned by the fire, he should be permitted to recover. That argument,
however, is beside the question, if the storing was a violation of the terms of the contract. The plaintiff paid
a premium based upon the risk at the time the policy was issued. It cannot be denied that the placing of the
firecrackers in the building insured increased the risk. The plaintiff had not paid a premium based upon
the increased risk, neither had the defendant issued a policy upon the theory of a different risk. The
defendant had neither been paid nor had issued a policy to cover the increased risk. An increase of risk
which is substantial and which is continued for a considerable period of time, is a direct and certain injury
to the insurer, and changes the basis upon which the contract of insurance rests.

16 0
Wa r ra nty

AMERICAN HOME v. TANTUCO


G.R. No. 138941
October 8, 2001

Puno, J.

Facts:

Tantuco Enterprises, Inc. is engaged in the coconut oil milling and refining industry, and owns two
oil mills which were separately covered by fire insurance policies issued by petitioner American Home
Assurance Co. When the newer one of the oil mills was destroyed by fire, respondent immediately notified
the petitioner. Petitioner rejected respondent’s claim for the insurance proceeds on the ground that no
policy was issued by it covering the burned oil mill. It stated that the policies extended insurance coverage
to the older one of the oil mills and not the one affected by the fire. The respondent thus instituted a
complaint for specific performance and damages with the Regional Trial Court which rendered a Decision
finding the petitioner liable on the insurance policy. The Court of Appeals affirmed the trial court’s decision
in toto.

Issue:

Did the Court of Appeals err in its legal interpretation of 'Fire Extinguishing Appliances Warranty'
of the Policy?

Ruling:

No. Respondent was able to comply with the warranty. Within the vicinity of the new oil mill can
be found the following devices: numerous portable fire extinguishers, two fire hoses, fire hydrant, and an
emergency fire engine. All of these equipments were in efficient working order when the fire occurred. Not
only are warranties strictly construed against the insurer, but they should, likewise, by themselves be
reasonably interpreted. That reasonableness is to be ascertained in light of the factual conditions
prevailing in each case. Here, we find that there is no more need for an internal hydrant considering that
inside the burned building were: (1) numerous portable fire extinguishers, (2) an emergency fire engine,
and (3) a fire hose which has a connection to one of the external hydrants.

16 1
Wa r ra nty

PRUDENTIAL v. TRANS ASIA


G.R. No. 151890
June 20, 2006

Chico-Nazario, J.

Facts:
Respondent Trans-Asia Shipping Lines insured its vessel M/V Asia Korea with petitioner
Prudential Guarantee and Assurance Inc. for loss/damage of the hull and machinery arising from perils,
inter alia, of fire and explosion. While the policy was in force, a fire broke out while M/V Asia Korea was
undergoing repairs at the port of Cebu. Trans-Asia filed its notice of claim for damage but this was
subsequently denied by Prudential. TransAsia filed a Complaint for Sum of Money against Prudential with
the Regional Trial Court of Cebu City which rendered Judgment finding for Prudential. According to the
trial court, Trans-Asia failed to prove compliance of the terms of the warranty, thereby entitling Prudential,
the injured party, to rescind the contract. The Court of Appeals reversed the judgment of the RTC.

Issue:

Does Prudential have any liability to Trans-Asia arising from the subject insurance contract?

Ruling:

Yes. Section 74 of the Insurance Code provides that, "the violation of a material warranty, or other
material provision of a policy on the part of either party thereto, entitles the other to rescind." It is generally
accepted that "a warranty is a statement or promise set forth in the policy, or by reference incorporated
therein, the untruth or non-fulfillment of which in any respect, and without reference to whether the
insurer was in fact prejudiced by such untruth or non-fulfillment, renders the policy voidable by the
insurer". However, it is similarly indubitable that for the breach of a warranty to avoid a policy, the same
must be duly shown by the party alleging the same. An allegation that is unfounded cannot be sustained.
Consequently, Prudential, not having shown that Trans-Asia breached the warranty condition, CLASSED
AND CLASS MAINTAINED, it remains that Trans-Asia must be allowed to recover its rightful claims on the
policy. However, assuming arguendo that Trans-Asia violated the policy condition on WARRANTED
VESSEL CLASSED AND CLASS MAINTAINED, Prudential made a valid waiver of the same. Prudential, in
renewing Trans-Asia’s insurance policy for two consecutive years after the loss covered by the subject
policy was considered to have waived Trans-Asia’s breach of the subject warranty, if any. Breach of a
warranty or of a condition renders the contract defeasible at the option of the insurer; but if he so elects,
he may waive his privilege and power to rescind by the mere expression of an intention so to do. In that
event his liability under the policy continues as before. There can be no clearer intention of the waiver of
the alleged breach than the renewal of the policy of insurance granted by Prudential to Trans-Asia.

16 2
Wa r ra nty

MALAYAN v. PHIL NAILS AND WIRES CORP


G.R. No. 138084
April 10, 2002

Quisumbing, J.

Facts:

Respondent Philippine Nails and Wires Corporation insured against all risks its shipment of 10,
053.400 metric tons of steel billets valued at P67, 156,300 with petitioner Malayan Insurance Company
Inc. The shipment delivered was short by 377.168 metric tons. For this shortage, respondent claimed
insurance for P2, 698,637.04, representing the value of undelivered steel billets, plus customs duties, taxes
and other charges paid by respondent. Petitioner refused to pay. On July 28, 1993, respondent filed a
complaint against petitioner for sum of money with the RTC of Pasig representing said lost and/or
undelivered cargo. On September 8, 1993, respondent filed a motion to admit an amended complaint
which the trial court granted. It sent petitioner summons and a copy of the complaint on October 13, 1993
and also gave petitioner until October 31, 1993 to file its answer. On November 4, 1993, respondent moved
to declare petitioner in default. The trial court granted and allowed the presentation of evidence ex parte
before the branch clerk of court. Respondent presented its lone witness, Jeanne King. On December 10,
1993, the trial court rendered a judgment by default. Respondent moved to execute judgment pending
appeal. The trial court granted the motion. Meanwhile, petitioner filed its notice of appeal which was given
due course. Pursuant to the notice of appeal, the entire records of the case were elevated to the Court of
Appeals, where petitioner argued that the trial court erred in rendering judgment by default
notwithstanding that issues were joined by petitioners filing of an answer; in awarding damages to
respondent based on unauthenticated documentary evidence and hearsay; and in admitting documentary
evidence which is irregular in nature and not in accordance with the Rules of Court. The Court of Appeals
concurred with the trial court.

Issue:

Is there sufficient evidence to prove petitioner's liability and therefore warrant a rescission of the
insurance contract?

Ruling:

No. At the outset, we must stress that respondents cause of action is founded on breach of
insurance contract covering cargo consisting of imported steel billets. To hold petitioner liable, respondent
has to prove, first, its importation of 10,053.400 metric tons of steel billets valued at P67, 156,300.00, and
second, the actual steel billets delivered to and received by the importer, namely the respondent. Witness
Jeanne King, who was assigned to handle respondents’ importations, including their insurance coverage,
has personal knowledge of the volume of steel billets being imported, and therefore competent to testify
thereon.

Her testimony is not hearsay, as this doctrine is defined in Section 36, Rule 130 of the Rules of
Court. However, she is not qualified to testify on the shortage in the delivery of the imported steel billets.
She did not have personal knowledge of the actual steel billets received. Even though she prepared the
summary of the received steel billets, she based the summary only on the receipts prepared by other
persons. Her testimony on steel billets received was hearsay. It has no probative value even if not objected
to at the trial.

Under Section 20, Rule 132, Rules of Court, before a private document is admitted in evidence, it
must be authenticated either by the person who executed it, the person before whom its execution was
acknowledged, any person who was present and saw it executed, or who after its execution, saw it and

16 3
Wa r ra nty

recognized the signatures, or the person to whom the parties to the instruments had previously confessed
execution thereof. In this case, respondent admits that King was none of the aforementioned persons. She
merely made the summary of the weight of steel billets based on the unauthenticated bill of lading and the
SGS report. Thus, the summary of steel billets actually received had no proven real basis, and Kings’
testimony on this point could not be taken at face value. In sum, the Court finds no sufficient competent
evidence to prove petitioners liability.

16 4
Wa iv e r o f th e R i gh ts t o R esc i n d

MALAYAN v. PAP
G.R. No. 200784
August 7, 2013

Mendoza, J.

Facts:

On May 13, 1996, Malayan Insurance Company issued Fire Insurance Policy No. F00227-000073
to PAP Co., Ltd. for the latter’s machineries and equipment located at Sanyo Precision Phils. Building. The
insurance, which was for P15,000,000.00 and effective for a period of 1 year, was procured by PAP Co. for
RCBC, the mortgagee of the insured machineries and equipment. After the passage of almost a year but
prior to the expiration of the insurance coverage, PAP Co. renewed the policy on an "as is" basis. Pursuant
thereto, a renewal policy, Fire Insurance Policy No. F-00227-000079, was issued. On October 12, 1997 and
during the subsistence of the renewal policy, the insured machineries and equipment were totally lost by
fire. Hence, PAP Co. filed a fire insurance claim with Malayan in the amount insured. Malayan denied the
claim upon the ground that, at the time of the loss, the insured machineries and equipment were
transferred by PAP Co. to a location different from that indicated in the policy. Specifically, that the insured
machineries were transferred in September 1996 from the Sanyo Building to the Pace Pacific. Contesting
the denial, PAP Co. argued that Malayan cannot avoid liability as it was informed of the transfer by RCBC,
the party duty-bound to relay such information. However, Malayan reiterated its denial of PAP Co.’s claim.
Distraught, PAP Co. filed the complaint against Malayan. RTC ordered Malayan to pay PAP an indemnity
for the loss under the fire insurance policy. CA affirmed RTC’s decision.

Issue:

Is Malayan liable for the loss of the insured properties under the fire insurance policy?

Ruling:

No. The policy forbade the removal of the insured properties unless sanctioned by Malayan.
Evidently, by the clear and express condition in the renewal policy, the removal of the insured property to
any building or place required the consent of Malayan. Any transfer effected by the insured, without the
insurer’s consent, would free the latter from any liability. The respondent failed to notify, and to obtain the
consent of, Malayan regarding the removal. The records are bereft of any convincing and concrete
evidence that Malayan was notified of the transfer of the insured properties from the Sanyo factory to the
Pace factory. Considering that the original policy was renewed on an "as is basis," it follows that the
renewal policy carried with it the same stipulations and limitations. There being an unconsented removal,
the transfer was at PAP’s own risk. Consequently, it must suffer the consequences of the fire. It can also be
said that with the transfer of the location of the subject properties, without notice and without Malayan’s
consent, after the renewal of the policy, PAP clearly committed concealment, misrepresentation and a
breach of a material warranty.

16 5
Wa iv e r o f th e R i gh ts t o R esc i n d

AMERICAN HOME v. CHUA


G.R. No. 130421
June 28, 1999

Davide, Jr., J.

Facts:

Respondent Chua obtained from petitioner a fire insurance covering the stock-intrade of his
business, Moonlight Enterprises. Respondent issued PCIB Check in the amount of P2, 983.50 to petitioner's
agent, James Uy, as payment for the renewal of the policy. The check was drawn against a Manila bank and
deposited in petitioner's bank account in Cagayan de Oro City. The corresponding official receipt was
issued on 10 April. Subsequently, a new insurance policy was issued. Moonlight Enterprises was
completely razed by fire. Respondent filed an insurance claim with petitioner. Petitioner refused to honor
the claim notwithstanding several demands by respondent, thus, the latter filed an action against
petitioner before the trial court. Petitioner claimed there was no existing insurance contract when the fire
occurred since respondent did not pay the premium. It also alleged that even assuming there was a
contract, respondent violated several conditions of the policy, particularly: (1) his submission of
fraudulent income tax return and financial statements; (2) his failure to establish the actual loss, which
petitioner assessed at P70, 000; and (3) his failure to notify to petitioner of any insurance already effected
to cover the insured goods. These violations, petitioner insisted, justified the denial of the claim. The trial
Court ruled in favor of trial court ruled in favor of respondent. The Court of Appeals found that
respondent's claim was substantially proved and petitioner's unjustified refusal to pay the claim entitled
respondent to the award of damages.

Issue:

1. Was there was a valid payment of premium, considering that respondent's check was
cashed after the occurrence of the fire?
2. Did respondent violated the policy by his submission of fraudulent documents and non-
disclosure of the other existing insurance contracts?

Ruling:

(1) Yes. Section 306 of the Insurance Code provides that any insurance company which delivers a
policy or contract of insurance to an insurance agent or insurance broker shall be deemed to have
authorized such agent or broker to receive on its behalf payment of any premium which is due on such
policy or contract of insurance at the time of its issuance or delivery or which becomes due thereon. In the
instant case, the best evidence of such authority is the fact that petitioner accepted the check and issued
the official receipt for the payment. It is, as well, bound by its agent's acknowledgment of receipt of
payment.

(2) No. There is a showing that respondent paid its income tax during the said years. Ordinarily,
where the insurance policy specifies as a condition the disclosure of existing coinsurers, non-disclosure
thereof is a violation that entitles the insurer to avoid the policy. This condition is common in fire insurance
policies and is known as the "other insurance clause." The purpose for the inclusion of this clause is to
prevent an increase in the moral hazard. To constitute a violation the other existing insurance contracts
must be upon the same subject matter and with the same interest and risk. It is true that respondent
acquired several co-insurers and he failed to disclose this information to petitioner. Nonetheless,
petitioner is estopped from must invoking this argument.it cannot be said that petitioner was deceived by
respondent by the latter's non-disclosure of the other insurance contracts when petitioner actually had
prior knowledge thereof. Petitioner's loss adjuster had known all along of the other existing insurance
contracts, yet, he did not use that as basis for his recommendation of denial. The loss adjuster, being an
employee of petitioner, is deemed a representative of the latter whose awareness of the other insurance

16 6
Wa iv e r o f th e R i gh ts t o R esc i n d

contracts binds petitioner. We, therefore, hold that there was no violation of the "other insurance" clause
by respondent

16 7
Wa iv e r o f th e R i gh ts t o R esc i n d

QUA CHEE GAN v. LAW UNION


G.R. No. L-4611
December 17, 1955

Reyes, J. B. L., J.

Facts:

Qua Chee Gan was a merchant who owned four warehouses for storage of copra and hemp in
Tabaco, Albay. These warehouses and their contents had been insured with Law Union and Rock
Insurance Co. Ltd. since 1937. Fire of undetermined origin and which lasted almost a week completely
destroyed Bodegas 1, 2 and 4 and the merchandise stored therein. When plaintiff submitted the
corresponding fire claims, the Insurance Company resisted payment, claiming violation of warranties and
conditions, filing of fraudulent claims, and that the fire had been deliberately caused by the insured or by
other persons in connivance with him.

Issue:

Should the policies be avoided for breach of warranty?

Ruling:

No. Appellant is barred by estoppel to claim violation of the fire hydrants warranty, for the reason
that knowing fully all that the number of hydrants demanded therein never existed from the very
beginning, the appellant nevertheless issued the policies in question subject to such warranty, and
received the corresponding premiums. According to American Jurisprudence, “where the insurer, at the
time of the issuance of a policy of insurance, has knowledge of existing facts which, if insisted on, would
invalidate the contract from its very inception, such knowledge constitutes a waiver of conditions in the
contract inconsistent with the facts, and the insurer is stopped thereafter from asserting the breach of such
conditions. The law is charitable enough to assume, in the absence of any showing to the contrary, that an
insurance company intends to execute a valid contract in return for the premium received; and when the
policy contains a condition which renders it voidable at its inception, and this result is known to the
insurer, it will be presumed to have intended to waive the conditions and to execute a binding contract,
rather than to have deceived the insured into thinking he is insured when in fact he is not, and to have
taken his money without consideration.” As to the alleged violation of the insured of the hemp warranty
provisions against the storage of gasoline, gasoline was not specifically mentioned among the prohibited
articles listed in the so-called hemp warranty. The cause relied upon by the insurer speaks of “oils.” As held
by the Supreme Court in Moore v. Aetna Life Insurance Co., “An insurer should not be allowed, by the use
of obscure phrases and exceptions, to defeat the very purpose for which the policy was procured.”
Moreover, in Bachrach v. British American Assurance Co., the Court Ruling: “If the company intended to
rely upon a condition of that character, it ought to have been plainly expressed in the policy.”

16 8
Sp ec if i e d R isk v. Al l R i sk Pol ic y

CHOA TIEK SENG v. CA


G.R. No. 84507
March 15, 1990

Gancayco, J.

Facts:

On November 4, 1976, petitioner imported lactose crystals from Holland. It involved 15 metric
tons packed in 600 6-ply paper bags with polythelene inner bags. The goods were loaded at the port at
Rotterdam. The goods were insured by respondent Filipino Merchants against all risks under the terms of
the insurance cargo policy. Upon arrival, the goods were discharged in the custody of the arrastre operator.
The survey shows that the bad order bags suffered spillage and loss later valued at Php 33, 117. 63.

Petitioner filed for a claim against respondent, who in turn rejected the claim stating that
assuming that spillage took place while the goods were in transit, petitioner and his agent failed to avert
or minimize the loss by failing to recover spillage from the sea van, thus violating the terms of the insurance
policy sued upon; and that assuming that the spillage did not occur while the cargo was in transit, the said
400 bags were loaded in bad order, and that in any case, the van did not carry any evidence of spillage.

Issue:

Is respondent liable under the “all risk” policy?

Ruling:

Yes. In Gloren Inc. v. Filipinas Cia de Seguros, it was held that an all risk insurance policy insures
against all causes of conceivable loss or damage, except as otherwise excluded in the policy or due to fraud
or intentional misconduct on the part of the insured. It covers all losses during the voyage whether arising
from a marine peril or not, including pilferage losses during the war.

The terms of the policy are so clear and require no interpretation. The insurance policy covers all
loss or damage to the cargo except those caused by delay or inherent vice or nature of the cargo insured.
It is the duty of the respondent insurance company to establish that said loss or damage falls within the
exceptions provided for by law, otherwise it is liable therefor.

An "all risks" provision of a marine policy creates a special type of insurance which extends
coverage to risks not usually contemplated and avoids putting upon the insured the burden of establishing
that the loss was due to peril falling within the policy's coverage. The insurer can avoid coverage upon
demonstrating that a specific provision expressly excludes the loss from coverage. In this case, the damage
caused to the cargo has not been attributed to any of the exceptions provided for nor is there any
pretension to this effect. Thus, the liability of respondent insurance company is clear.

16 9
R isk a n d Co v er a g e – Ex c e pt io ns a nd E xcl u si on s

COUNTRY BANKERS v. LIANGA BAY


G.R. No. 136914
25 January 2002

De Leon, Jr., J.

Facts:

The petitioner Country Bankers Insurance Corporation is a domestic corporation principally


engaged in the insurance business wherein it undertakes, for a consideration, to indemnify another against
loss, damage or liability from an unknown or contingent event including fire while the respondent Lianga
Bay and Community Multi-Purpose Cooperative is a duly registered cooperative judicially declared
insolvent and represented by the elected assignee, Cornelio Jamero. The petitioner and the respondent
entered into a contract of fire insurance. The petitioner insured the respondents stocks-in-trade against
fire loss, damage or liability during the period starting from June 20, 1989 at 4:00 p.m. to June 20, 1990 at
4:00 p.m., for the sum of P200, 000.00. On July 1, 1989, at or about 12:40 a.m., the respondents building
located at Barangay Diatagon, Lianga, Surigao del Sur was gutted by fire and reduced to ashes, resulting in
the total loss of the respondents stocks-in-trade, pieces of furniture and fixtures, equipment and records.
Due to the loss, the respondent filed an insurance claim with the petitioner under its Fire Insurance Policy.
The petitioner, however, denied the insurance claim on the ground that, based on the submitted
documents, the building was set on fire by two NPA rebels who wanted to obtain canned goods, rice and
medicines as provisions for their comrades in the forest, and that such loss was an excepted risk under
paragraph No. 6 of the policy conditions of Fire Insurance Policy. Finding the denial of its claim
unacceptable, the respondent then instituted in the trial court the complaint for recovery of loss, damage
or liability against petitioner. The Regional Trial Court held that the Country Bankers has to be irreversibly
adjudged liable, as it should be, to Insolvent Cooperative, represented in this action by its Assignee,
Cornelio Jamero, and thus, ordering said defendant-Country Bankers to fully pay the insurance claim for
the loss the insured-plaintiff Insolvent Cooperative sustained as a result of the fire under its Fire Insurance
Policy in its full face value of P200,000.00 with interest of 12% per annum from date of filing of the
complaint until the same is fully paid The appellate court affirmed the challenged decision of the trial court
in its entirety.

Issue:

Is petitioner Country Bankers liable for 12% interest per annum on the face value of the policy
from the filing of the complaint until fully paid?

Ruling:

No. Concerning the application of the proper interest rates, the following guidelines were set in
Eastern Shipping Lines, Inc. v. Court of Appeals wherein:

When an obligation, regardless of its source, is breached, the contravenor can be held liable for
damages.

With regard particularly to an award of interest in the concept of actual and compensatory
damages, the rate of interest, as well as the accrual thereof, is imposed, as follows: 1. When the obligation
is breached, and it consists in the payment of a sum of money, the interest due should be that which may
have been stipulated in writing. Furthermore, the interest due shall itself earn legal interest from the time
it is judicially demanded. In the absence of stipulation, the rate of interest shall be 12% per annum to be
computed from default. 2. When an obligation, not constituting a loan or forbearance of money, is
breached, an interest on the amount of damages awarded may be imposed at the discretion of the court at

17 0
R isk a n d Co v er a g e – Ex c e pt io ns a nd E xcl u si on s

the rate of 6% per annum. No interest, however, shall be adjudged on unliquidated claims or damages
except when or until the demand can be established with reasonable certainty.

17 1
R isk a n d Co v er a g e – Ex c e pt io ns a nd E xcl u si on s

UNITED MERCHANTS v. COUNTRY BANKERS


G.R. No. 198588
11 July 2012

Carpio, J.

Facts:

Petitioner leased a warehouse where it assembled and stored its products. Petitioner’s General
Manager insured the stocks in trade of Christmas lights with defendant. An endorsement and fire invoice
were executed to form part of the Insurance policy. The goods were insured against additional perils. On
July 3, 1996, fire gutted the warehouse. CRM to investigate and evaluate UMC’s loss by reason of the fire.
CBIC’s reinsurer, Central Surety, likewise requested the National Bureau of Investigation (NBI) to conduct
a parallel investigation. On 6 July 1996, UMC, through CRM, submitted to CBIC its Sworn Statement of
Formal Claim, with proofs of its loss. UMC demanded for at least fifty percent (50%) payment of its claim
from CBIC. UMC received CBIC’s letter rejecting UMC’s claim due to breach of Condition No. 15 of the
Insurance Policy. CBIC alleged that UMC’s claim was fraudulent because UMC’s Statement of Inventory
showed that it had no stocks in trade as of 31 December 1995, and that UMC’s suspicious purchases for
the year 1996 did not even amount to ₱25,000,000.00. UMC’s GIS and Financial Reports further revealed
that it had insufficient capital, which meant UMC could not afford the alleged ₱50,000,000.00 worth of
stocks in trade.

Issue:

Is petitioner entitled to claim?

Ruling:

No. The invoices cannot be taken as genuine. The invoices reveal that the stocks in trade
purchased for 1996 amounts to ₱20,000,000.00 which were purchased in one month. Thus, UMC needs
to prove purchases amounting to ₱30,000,000.00 worth of stocks in trade for 1995 and prior years.
However, in the Statement of Inventory it submitted to the BIR, which is considered an entry in official
records, UMC stated that it had no stocks in trade as of 31 December 1995. It has long been settled that a
false and material statement made with an intent to deceive or defraud voids an insurance policy. The
most liberal human judgment cannot attribute such difference to mere innocent error in estimating or
counting but to a deliberate intent to demand from insurance companies’ payment for indemnity of goods
not existing at the time of the fire. This constitutes the so-called "fraudulent claim" which, by express
agreement between the insurers and the insured, is a ground for the exemption of insurers from civil
liability.

17 2
R isk a n d Co v er a g e – Ex c e pt io ns a nd E xcl u si on s

DBP v. RADIO MINDANAO


G.R. No. 147039
January 27, 2006

Austria- Martinez, J.

Facts:

In the evening of July 27, 1988, the radio station of Radio Mindanao Network located at the SSS
Building in Bacolod City was burned down causing damage in the amount of over one million pesos.
Respondent sought to recover under two insurance policies but the claims were denied on the basis that
the case of the loss was an excepted risk under condition no. 6 (c) and (d), to wit:
6. This insurance does not cover any loss or damage occasioned by or through or in
consequence, directly or indirectly, of any of the following consequences, namely:
(c) War, invasion, act of foreign enemies, hostilities, or warlike operations (whether war
be declared or not), civic war.
(d) Mutiny, riot, military or popular uprising, insurrection, rebellion, revolution, military
or usurped power.

The insurers maintained that based on witnesses and evidence gathered at the site, the fire was
caused by the members of the Communist Party of the Philippines/New People’s Army. Hence the refusal
to honor their obligations.

The trial court and the CA found in favor of the respondent. In its findings, both courts mentioned
the fact that there was no credible evidence presented that the CCP/NPA did in fact cause the fire that
gutted the radio station in Bacolod.

Issue:

Are the insurance companies liable to pay Radio Mindanao Network under the insurance policies?

Ruling:

Yes. The Court will not disturb the factual findings of the appellant and trial courts absent
compelling reason. Under this mode of review, the jurisdiction of the court is limited to reviewing only
errors of law.

In cases of insurance disputes with regard to excepted risks, it is the insurance companies which
have the burden to prove that the loss comes within the purview of the exception or limitation set up. It is
sufficient for the insured to prove the fact of damage or loss. Once the insured makes out a prima facie case
in its favor, the duty or burden of evidence shifts to the insurer to controvert said prima facie case.

17 3
Pr i nci pl es i n Ca u sa t io n

FGU INSURANCE v. CA
G.R. No. 137775
March 31, 2005

Chico- Nazario, J.

Facts:

On April 21, 1987, a car owned by private respondent FILCAR Transport Inc., rented to and driven
by Dahl-Jensen, a Danish tourist, swerved into the right and hit the car owned by Lydia Soriano and driven
by Benjamin Jacildone. Dahl-Jensen did not possess a Philippine driver’s license. Petitioner, as the insurer
of Soriano’s car, paid the latter P25, 382.20 and, by way of subrogation, sued FILCAR, Dahl-Jensen, and
Fortune Insurance Corporation, FILCAR’s insurer, for quasi-delict. The trial court dismissed the petition
for failure to substantiate the claim for subrogation. The Court of Appeals affirmed the decision, but on the
ground that only Dahl-Jensen’s negligence was proven, not that of FILCAR. Hence, this instant petition.

Issue:

Is FGU liable under the insurance contract considering the circumstances surrounding the loss of
the cargoes?

Ruling:

No. One of the purposes for taking out insurance is to protect the insured against the
consequences of his own negligence and that of his agents. Thus, it is a basic rule in insurance that the
carelessness and negligence of the insured or his agents constitute no defense on the part of the insurer.
This rule however presupposes that the loss has occurred due to causes which could not have been
prevented by the insured, despite the exercise of due diligence. The question now is whether there is a
certain degree of negligence on the part of the insured or his agents that will deprive him the right to
recover under the insurance contract. We say there is. However, to what extent such negligence must go
in order to exonerate the insurer from liability must be evaluated in light of the circumstances surrounding
each case. When evidence show that the insured’s negligence or recklessness is so gross as to be sufficient
to constitute a willful act, the insurer must be exonerated. According to the Court, while mistake and
negligence of the master or crew are incident to navigation and constitute a part of the perils that the
insurer is obliged to incur, such negligence or recklessness must not be of such gross character as to
amount to misconduct or wrongful acts; otherwise, such negligence shall release the insurer from liability
under the insurance contract.

17 4
R isk a n d Co v er a g e – L if e I nsu ra nc e

GREAT PACIFIC v. CA
G.R. No. 113899
October 13, 1999

Quisumbing, J.

Facts:

There was an existing group life insurance executed between Great Pacific Life Assurance and the
Development Bank of the Philippines. Grepalife agreed to insure the lives of eligible housing loan
mortgagors of DBP. In November 1983, Wilfredo Leuterio, mortgagor of DBP applied to be a member of
the group life insurance. He filled out a form where he indicated he never consulted any physician
regarding any illness and that he is in good health. He was eventually included in the group life insurance
and he was covered for the amount of his indebtedness.

In August 1984, Wilfredo died. DBP submitted a death claim but it was denied by Grepalife as it
insisted that Wilfredo actually concealed that he was suffering from hypertension at the time of his
insurance application. Grepalife relied on the statement made by the doctor who issued Wilfredo’s death
certificate wherein it was stated that Wilfredo’s immediate cause of death was massive cerebral
hemorrhage secondary to hypertension or hypertension as a “possible cause of death”.

Since Grepalife refused to pay the insurance claim filed by DBP, Medarda Leuterio sued Grepalife.
Grepalife assailed the suit and insisted that Medarda is not a proper party in interest. The lower court ruled
in favor of Medarda and the court ordered Grepalife to pay the amount of the insurance to DBP. The Court
of Appeals affirmed this decision in 1993. Grepalife appealed to the Supreme Court. In 1995, pending
resolution of the case in the SC, DBP foreclosed the property of Medarda.

Issue:

Whether or not Grepalife is liable to pay the insurance claim.

Ruling:

Yes. Grepalife is liable to pay the insurance claim. Medarda is a proper party in interest (note that
it was Wilfredo who has been paying the premium, as the insured, he is the real party in interest and this
status was transferred to his widow). The group life insurance or “mortgage redemption insurance”
provides that DBP as the mortgagee is merely an assignee of Wilfredo; and that in the event of Wilfredo’s
death before his indebtedness to DBP is paid, proceeds from the insurance shall first be applied to the sum
of the balance insured. But this does not cease Wilfredo to be a party to the insurance contract.

Grepalife failed to prove that Wilfredo concealed that he was suffering from hypertension at the
time of his application. The doctor’s finding as to the cause of death is not conclusive because no autopsy
was conducted. The doctor who issued the death certificate has no knowledge of Wilfredo’s hospital
confinement [if there are any]. The fraudulent intent on the part of the insured must be established to
entitle the insurer to rescind the contract. Misrepresentation as a defense of the insurer to avoid liability is
an affirmative defense and the duty to establish such defense by satisfactory and convincing evidence rests
upon the insurer. Grepalife must however pay the claim to Medarda considering that DBP already
foreclosed the property.

17 5
R isk a n d Co v er a g es

FINMAN GENERAL ASSURANCE CORPORATION v. COURT OF APPEALS


G.R. No. 100970
September 2, 1992

Nocon, J.

Facts:

October 22, 1986, deceased, Carlie Surposa was insured with petitioner Finman General
Assurance Corporation under Finman General Teachers Protection Plan Master Policy No. 2005 and
Individual Policy No. 08924 with his parents, spouses Julia and Carlos Surposa, and brothers Christopher,
Charles, Chester and Clifton, all surnamed, Surposa, as beneficiaries.

Carlie Surposa, died on October 18, 1988 as a result of a stab wound inflicted by one of the three
(3) unidentified men without provocation and warning on the part of the former as he and his cousin,
Winston Surposa, were waiting for a ride on their way home along Rizal-Locsin Streets, Bacolod City after
attending the celebration of the "Maskarra Annual Festival."

Private respondent and the other beneficiaries of said insurance policy filed a written notice of
claim with the petitioner insurance company which denied said claim contending that murder and assault
are not within the scope of the coverage of the insurance policy. Given this, Julia filed a suit before the
Insurance Commission against Finman. Finman alleges that death resulting from murder and/or assault
is impliedly excluded in said insurance policy considering that the cause of death of the insured was not
accidental but rather a deliberate and intentional act of the assailant in killing the former as indicated by
the location of the lone stab wound on the insured. Therefore, said death was committed with deliberate
intent which, by the very nature of a personal accident insurance policy, cannot be indemnified.

Issues:

1. Is Finman still liable notwithstanding of its allegation that the death of Carlie is not through
accident?
2. Is the insurance contract clear on what instances are excluded under the policy?

Ruling:

The generally accepted rule is that, death or injury does not result from accident or accidental
means within the terms of an accident-policy if it is the natural result of the insured's voluntary act,
unaccompanied by anything unforeseen except the death or injury. There is no accident when a deliberate
act is performed unless some additional, unexpected, independent, and unforeseen happening occurs
which produces or brings about the result of injury or death. In other words, where the death or injury is
not the natural or probable result of the insured's voluntary act, or if something unforeseen occurs in the
doing of the act which produces the injury, the resulting death is within the protection of the policies
insuring against death or injury from accident.

In the case at bar, it cannot be pretended that Carlie Surposa died in the course of an assault or
murder as a result of his voluntary act considering the very nature of these crimes. In the first place, the
insured and his companion were on their way home from attending a festival. They were confronted by
unidentified persons. The record is barren of any circumstance showing how the stab wound was inflicted.
Nor can it be pretended that the malefactor aimed at the insured precisely because the killer wanted to
take his life. In any event, while the act may not exempt the unknown perpetrator from criminal liability,
the fact remains that the happening was a pure accident on the part of the victim.

Furthermore, the personal accident insurance policy involved herein specifically enumerated
only ten (10) circumstances wherein no liability attaches to petitioner insurance company for any injury,

17 6
R isk a n d Co v er a g es

disability or loss suffered by the insured as a result of any of the stimulated causes. The principle of
" expresso unius exclusio alterius" — the mention of one thing implies the exclusion of another thing — is
therefore applicable in the instant case since murder and assault, not having been expressly included in
the enumeration of the circumstances that would negate liability in said insurance policy cannot be
considered by implication to discharge the petitioner insurance company from liability for, any injury,
disability or loss suffered by the insured. Thus, the failure of the petitioner insurance company to include
death resulting from murder or assault among the prohibited risks leads inevitably to the conclusion that
it did not intend to limit or exempt itself from liability for such death.

Moreover, it is well settled that contracts of insurance are to be construed liberally in favor of the
insured and strictly against the insurer. Thus ambiguity in the words of an insurance contract should be
interpreted in favor of its beneficiary.

17 7
R isk a n d Co v er a g es

VIRGINIA CALANOC v. COURT OF APPEALS


G.R. No. L-8151
December 16, 1955

Bautista Angelo, J.

Facts:

Melencio Basilio was a watchman of the Manila Auto Supply located at the corner of Avenida Rizal
and Zurbaran. He secured a life insurance policy from the Philippine American Life Insurance Company in
the amount of P2,000 to which was attached a supplementary contract covering death by accident. On
January 25, 1951, he died of a gunshot wound on the occasion of a robbery committed in the house of Atty.
Ojeda at the corner of Oroquieta and Zurbaan streets. Virginia Calanoc, the widow, was paid the sum of
P2,000, face value of the policy, but when she demanded the payment of the additional sum of P2,000
representing the value of the supplemental policy, the company refused alleging, as main defense, that the
deceased died because he was murdered by a person who took part in the commission of the robbery and
while making an arrest as an officer of the law which contingencies were expressly excluded in the contract
and have the effect of exempting the company from liability.

Issue:

Whether or not the death of the insured was a risk excluded by the supplementary contract which
exempts the company from liability.

Ruling:

The circumstance that he was a mere watchman and had no duty to heed the call of Atty. Ojeda
should not be taken as a capricious desire on his part to expose his life to danger considering the fact that
the place he was in duty-bound to guard was only a block away. He cannot therefore be blamed solely for
doing what he believed was in keeping with his duty as a watchman and as a citizen. And he cannot be
considered as making an arrest as an officer of the law, as contended, simply because he went with the
traffic policeman, for certainly he did not go there for that purpose nor was he asked to do so by the
policeman.

While as a general rule "the parties may limit the coverage of the policy to certain particular
accidents and risks or causes of loss, and may expressly except other risks or causes of loss therefrom",
however, it is to be desired that the terms and phraseology of the exception clause be clearly expressed so
as to be within the easy grasp and understanding of the insured, for if the terms are doubtful or obscure
the same must of necessity be interpreted or resolved against the one who has caused the obscurity. And
so it has bene generally held that the "terms in an insurance policy, which are ambiguous, equivacal, or
uncertain . . . are to be construed strictly and most strongly against the insurer, and liberally in favor of the
insured so as to effect the dominant purpose of indemnity or payment to the insured, especially where a
forfeiture is involved", and the reason for this rule is that he "insured usually has no voice in the selection
or arrangement of the words employed and that the language of the contract is selected with great care
and deliberation by experts and legal advisers employed by, and acting exclusively in the interest of, the
insurance company."
Insurance is, in its nature, complex and difficult for the layman to understand. Policies are
prepared by experts who know and can anticipate the bearings and possible complications of every
contingency. So long as insurance companies insist upon the use of ambiguous, intricate and technical
provisions, which conceal rather than frankly disclose, their own intentions, the courts must, in fairness to
those who purchase insurance, construe every ambiguity in favor of the insured. An insurer should not be
allowed, by the use of obscure phrases and exceptions, to defeat the very purpose for which the policy was
procured.

17 8
R isk a n d Co v er a g es

BIAGTAN v. THE INSULAR LIFE ASSURANCE COMPANY, LTD


G.R. No. L-25579
March 29, 1972

Makalintal, J.

Facts:
Juan S. Biagtan was insured with defendant Insular Life Assurance Company under Policy No.
398075 for the sum of P5,000.00 and, under a supplementary contract denominated "Accidental Death
Benefit Clause, for an additional sum of P5,000.00 if "the death of the Insured resulted directly from bodily
injury effected solely through external and violent means sustained in an accident and independently of
all other causes." The clause, however, expressly provided that it would not apply where death resulted
from an injury "intentionally inflicted by another party."
On the night of May 20, 1964 or the first hours of May 21, 1964, while the said life policy and
supplementary contract were in full force and effect, the house of insured Juan S. Biagtan was robbed by a
band of robbers who were charged in and convicted by the Court of First Instance of Pangasinan for
robbery with homicide; that in committing the robbery, the robbers, on reaching the staircase landing on
the second floor, rushed towards the door of the second floor room, where they suddenly met a person
near the door of oneof the rooms who turned out to be the insured Juan S. Biagtan who received thrusts
from their sharp-pointed instruments, causing wounds on the body of said Juan S. Biagtan resulting in his
death at about 7 a.m. on the same day, May 21, 1964.
Plaintiffs, as beneficiaries of the insured, filed a claim under the policy. The insurance company
paid the basic amount of P5,000.00 but refused to pay the additional sum of P5,000.00 under the accidental
death benefit clause, on the ground that the insured's death resulted from injuries intentionally inflicted
by third parties and therefore was not covered. Plaintiffs filed suit to recover, and after due hearing the
court a quo rendered judgment in their favor. Hence the present appeal by the insurer.
Issue:
Whether under the facts are stipulated and found by the trial court the wounds received by the
insured at the hands of the robbers were inflicted intentionally.
Ruling:
The court, in ruling negatively on the issue, stated that since the parties presented no evidence
and submitted the case upon stipulation, there was no "proof that the act of receiving thrust from the
sharp-pointed instrument of the robbers was intended to inflict injuries upon the person of the insured or
any other person or merely to scare away any person so as to ward off any resistance or obstacle that
might be offered in the pursuit of their main objective which was robbery."
The trial court committed a plain error in drawing the conclusion it did from the admitted facts. It
should be noted that the exception in the accidental benefit clause invoked by the appellant does not speak
of the purpose — whether homicidal or not — of a third party in causing the injuries, but only of the fact
that such injuries have been "intentionally" inflicted — this obviously to distinguish them from injuries
which, although received at the hands of a third party, are purely accidental. This construction is the basic
idea expressed in the coverage of the clause itself, namely, that "the death of the insured resulted directly
from bodily injury effected solely through external and violent means sustained in an accident ... and
independently of all other causes." A gun which discharges while being cleaned and kills a bystander; a
hunter who shoots at his prey and hits a person instead; an athlete in a competitive game involving
physical effort who collides with an opponent and fatally injures him as a result: these are instances where
the infliction of the injury is unintentional and therefore would be within the coverage of an accidental
death benefit clause such as that in question in this case. But where a gang of robbers enter a house and
coming face to face with the owner, even if unexpectedly, stab him repeatedly, it is contrary to all reason
and logic to say that his injuries are not intentionally inflicted, regardless of whether they prove fatal or
not. As it was, in the present case they did prove fatal, and the robbers have been accused and convicted of
the crime of robbery with homicide.
For while a single shot fired from a distance, and by a person who was not even seen aiming at the
victim, could indeed have been fired without intent to kill or injure, nine wounds inflicted with bladed

17 9
R isk a n d Co v er a g es

weapons at close range cannot conceivably be considered as innocent insofar as such intent is concerned.
The manner of execution of the crime permits no other conclusion.
Court decisions in the American jurisdiction, where similar provisions in accidental death benefit
clauses in insurance policies have been construed, may shed light on the issue before Us. Thus, it has been
held that "intentional" as used in an accident policy excepting intentional injuries inflicted by the insured
or any other person, etc., implies the exercise of the reasoning faculties, consciousness and volition. Where
a provision of the policy excludes intentional injury, it is the intention of the person inflicting the injury that
is controlling. If the injuries suffered by the insured clearly resulted from the intentional act of a third
person the insurer is relieved from liability as stipulated.

18 0
R isk a n d Co v er a g es

MANUFACTURERS LIFE INSURANCE CO. v. MEER


G.R. No. L-2910
June 29, 1951

Bengzon, J.

Facts:

The plaintiff, the Manufacturer Life Insurance Company in a corporation duly organized in Canada
with head office at Toronto. It is duly registered and licensed to engage in life insurance business in the
Philippines, and maintains a branch office in Manila. It was engaged in such business in the Philippines for
more than five years before and including the year 1941. But due to the exigencies of the war it closed the
branch office at Manila during 1942 up to September 1945.

In the course of its operations before the war, plaintiff issued a number of life insurance policies
in the Philippines containing stipulations referred to as non-forfeiture clauses.

From January 1, 1942 to December 31, 1946 for failure of the insured under the above policies to
pay the corresponding premiums for one or more years, the plaintiff's head office of Toronto, applied the
provision of the automatic premium loan clauses; and the net amount of premiums so advanced or loaned
totalled P1,069,254.98. On this sum the defendant Collector of Internal Revenue assessed P17,917.12 —
which plaintiff paid supra protest —. The assessment was made pursuant to section 255 of the National
Internal Revenue Code as amended.

It is the plaintiff's contention that when it made premium loans or premium advances, as above
stated, by virtue of the non-forfeiture clauses, it did not collect premiums within the meaning of the above
sections of the law, and therefore it is not amendable to the tax therein provided.

Issue:

1. Whether or not, in the application of the automatic premium loan clause of plaintiff-appellant's
policies, there is "payment in money, notes, credit, or any substitutes for money"
2. Whether the making of premium advances, granting for the sake of argument that it amounted
to collection of premiums, were done in Toronto, Canada, or in the Philippines.

Ruling:

Yes. The operation of the automatic loan provision contributed no additional cash to the funds of
the insurer. Yet it must be admitted that the insurer agreed to consider the premium paid on the strength
of the automatic loan. The premium was therefore paid by means of a "note" or "credit" or "other
substitute for money" and the taxis due because section 255 above quoted levies taxes according to the
total premiums collected by the insurer "whether such premiums are paid in money, notes, credits or any
substitutes for money.

No. The thesis overlooks the actual fact that the loans are made to policyholders in the Philippines,
who in turn pay therewith the premium to the insurer thru the Manila branch. Approval of appellants
position will enable foreign insurers to evade the tax by contriving to require that premium payments shall
be made at their head offices. What is important, the law does not contemplate premiums collected in the
Philippines. It is enough that the insurer is doing insurance business in the Philippines, irrespective of the
place of its organization or establishment.

18 1
R isk a n d Co v er a g es

JAMES MCGUIRE v. THE MANUFACTURERS LIFE INSURANCE CO.


G.R. No. L-3581
September 21, 1950

Ozaeta, J.

Facts:

On August 18, 1932, the defendant issued an insurance policy on the life of Jaime McGuire for the
sum of $5,000, and an additional sum of $5,000 as double indemnity accident benefit, payable to the
plaintiff as beneficiary. The insured paid the premiums on said policy up to and including that due on July
19, 1940. On June 22, 1940, the insured secured from the defendant a loan of $760 on said insurance policy.
The insured failed to pay the loan with the interest thereon on January 1, 1941, when it became due, or on
any other date thereafter. He likewise failed to pay the premiums which fell due on July 19, 1941, as well
as those payable thereafter.

Upon those facts the trial court rendered judgment in favor of the plaintiff, adjudging the
defendant to pay to him the sum of P20,000, minus the premiums due and unpaid up to the date of the
death of the insured, with legal interest thereon from the date of the filing of the complaint, and the costs.

According to the complaint, plaintiff's theory is that, although the policy lapsed on March 1, 1942,
the insured had the privilege of reinstating it so as to keep it in force up to the time of his death upon a
written application within three years from the date of lapse and upon production of evidence of
insurability satisfactory to the company and the payment of all overdue premiums and any other
indebtedness to the company, but that the insured was unable to exercise that privilege because of the
war. Adopting another theory, the trial court held that it was unnecessary for the plaintiff to invoke the
reinstatement clause of the policy because it had not lapsed inasmuch as the failure to pay the premiums
was due to the war.

Issue:

Whether, defendant is liable under the policy on the ground that this was validly reinstated by the
insured.

Ruling:

Plaintiff's theory is untenable. Even if the insured had applied for reinstatement within three years
after the policy had lapsed, his right thereto was not absolute under the terms of the policy but
discretionary on the part of the insurance company, which had the right to deny the reinstatement if it was
not satisfied as to the insurability of the insured and if the latter did not pay all overdue premiums and all
other indebtedness to the company. After the death of the insured the insurance company could not be
compelled to entertain an application for reinstatement of the policy because the conditions precedent to
reinstatement could no longer be determined and satisfied.

In some cases we rejected the New York rule which holds that war between states in which the
parties reside suspends the contract of life insurance and that, upon tender of all premiums due by the
insured or his representative after the war was terminated, the contract revives and becomes fully
operative; and adopted the United States rule which declares that the contract is not merely suspended,
but is abrogated by reason of nonpayment of premiums, since the time of the payments is peculiarly of the
essence of the contract. Speaking through Mr. Justice Bengzon, this court, after a review of various
pertinent cases

Thus the fundamental character of the undertaking to pay premiums and high importance of the
defense of nonpayment thereof, was specifically recognized.

18 2
R isk a n d Co v er a g es

RUFINO D. ANDRES v. CROWN LIFE INSURANCE COMPANY


G.R. No. L-l0874
January 28, 1958

Reyes, J.B.L., J.

Facts:

Plaintiff and Severa G. Andres filed an application for insurance No. 536,423.On February 13,
1950, defendant isssued Crown Life Policy No. 536,423 for the sum of P5,000, in the name of Rufino D.
Andres, plaintiff, and Severa G. Andres.The premiums are to be paid as called for in the policy, semi-
annually, and the amount of P165.15 for the first semester beginning November 25, 1949 to May 25, 1950
was paid on November 25, 1949, and the premium likewise in the sum of P165.15 for the second semester
beginning May 25, 1950 to November 25, 1950, was paid on June 24, 1950 and the premium for the third
semester beginning November 25, 1950 to May 25, 1951 was not paid. Crown Life, through its branch
secretary, wrote to Mr. and Mrs. Andres advising them that their insurance policy lapsed on Dec. 26, 1950
and the amount of P165.15 was overdue, giving them 60 days from the date of lapse to file an application
for reinstatement. Crown Life later sent another letter telling the spouses Andres that their insurance
policy was no longer in force. Plaintiff and his wife executed a Statement of Health and application for
reinstatement of the aforesaid policy. Plaintiff wrote a letter to the defendant, enclosed with a money order
for P100. Upon acceptance, defendant advised Rufino that its main office had approved the application and
that the reinstatement of the lapsed policy was subject to the payment of the remaining premium balance
of P65.15. Severa Andres died of dystocia, contracted pelvis. Plaintiff then sent a letter enclosed with a
money order in the amount of P65, for the remaining balance due. Defendant sent a letter with official
receipt of the P165.15 paid by Rufino as well as a Certificate of Reinstatement. plaintiff presented his Death
Claim as survivor-beneficiary of the deceased Severa G. Andres which has been received in the office of the
defendant on June 11, 1951. On Jun 7, 1951, Rufino Andres presented his death claim as survivor-
beneficiary of the deceased Severa G. Andres, who died May 3, 1951.

On April 20, 1952, Rufino D. Andres filed a complaint in the Court of First Instance of Ilocos Norte
against the Crown Life Insurance Company for the recovery of the amount of P5,000, as the face value of a
joint 20-year endowment insurance policy issued in favor of the plaintiff Rufino D. Andres and his wife
Severa G. Andres on the 13th of February, 1950, by said insurance company. Payment having been denied
by the insurance company on April 20, 1952, this case was instituted.

Issue:

Was there a perfected contract of reinstatement after the policy lapsed due to non-payment of
premiums?

Ruling:

As stated by the lower court, the conditions set forth in the policy for reinstatement are the
following: (a) application shall be made within three years from the date of lapse; (b) there should be a
production of evidence of the good health of the insured: (c) if the rate of premium depends upon the age
of the Beneficiary, there should likewise be a production of evidence of his or her good health; (d) there
should be presented such other evidence of insurability at the date of application for reinstatement; (e)
there should be no change which has taken place in such good health and insurability subsequent to the
date of such application and before the policy is reinstated; and (f) all overdue premiums and other
indebtedness in respect of the policy, together with interest at six per cent, compounded annually, should
first be paid.

The plaintiff-appellant did not comply with the last condition; for he only paid P100 (on account
of the overdue semi-annual premium of P165.15) on February 20, 1951, before his wife's death

18 3
R isk a n d Co v er a g es

(Stipulation, par. 7); and, despite the Company's reminders on April 14 and 27, he remitted the balance of
P65 on May 5, 1951 (received by the Company's agency on May 11), two days after his wife died. On the
face of such facts, the Company had the right to treat the contract as lapsed and refuse payment of the
policy.

Clearly the Company did not consider the partial payment as sufficient consideration for the
reinstatement. Appellant's failure to remit the balance before the death of his wife operated to deprive him
of any right to waive the policy and recover the face value thereof.

18 4
R isk a n d Co v er a g es

SIMON DE LA CRUZ v. THE CAPITAL INSURANCE AND SURETY CO., INC.


G.R. No. L-21574
June 30, 1966

Barrera, J.

Facts:

Eduardo de la Cruz, employed as a mucker in the Itogon-Suyoc Mines, Inc. in Baguio, was the
holder of an accident insurance policy (No. ITO-BFE-170) underwritten by the Capital Insurance & Surety
Co., Inc., for the period beginning November 13, 1956 to November 12, 1957. On January 1, 1957, in
connection with the celebration of the New Year, the Itogon-Suyoc Mines, Inc. sponsored a boxing contest
for general entertainment wherein the insured Eduardo de la Cruz, a non-professional boxer participated.
In the course of his bout with another person, likewise a non-professional, of the same height, weight, and
size, Eduardo slipped and was hit by his opponent on the left part of the back of the head, causing Eduardo
to fall, with his head hitting the rope of the ring. He was brought to the Baguio General Hospital the
following day. The cause of death was reported as hemorrhage, intracranial, left.

Simon de la Cruz, the father of the insured and who was named beneficiary under the policy,
thereupon filed a claim with the insurance company for payment of the indemnity under the insurance
policy which was denied. Defendant insurer set up the defense that the death of the insured, caused by his
participation in a boxing contest, was not accidental and, therefore, not covered by insurance. After due
hearing the court rendered the decision in favor of the plaintiff which is the subject of the present appeal

Issue:

Whether, the beneficiary under said insurance policy can successfully claim indemnity from the
insurance company.

Ruling:

In the present case, while the participation of the insured in the boxing contest is voluntary, the
injury was sustained when he slid, giving occasion to the infliction by his opponent of the blow that threw
him to the ropes of the ring. Without this unfortunate incident, that is, the unintentional slipping of the
deceased, perhaps he could not have received that blow in the head and would not have died. The fact that
boxing is attended with some risks of external injuries does not make any injuries received in the course
of the game not accidental. In boxing as in other equally physically rigorous sports, such as basketball or
baseball, death is not ordinarily anticipated to result. If, therefore, it ever does, the injury or death can only
be accidental or produced by some unforeseen happening or event as what occurred in this case.
Furthermore, the policy involved herein specifically excluded from its coverage —
(e) Death or disablement consequent upon the Insured engaging in football, hunting,
pigsticking, steeplechasing, polo-playing, racing of any kind, mountaineering, or
motorcycling.

Death or disablement resulting from engagement in boxing contests was not declared outside of
the protection of the insurance contract. Failure of the defendant insurance company to include death
resulting from a boxing match or other sports among the prohibitive risks leads inevitably to the
conclusion that it did not intend to limit or exempt itself from liability for such death.

18 5
R isk a n d Co v er a g es

THE INSULAR LIFE ASSURANCE COMPANY, LTD v. EBRADO


G.R. No. L-44059
October 28, 1977

Martin, J.

Facts:

On September 1, 1968, Buenaventura Cristor Ebrado was issued by The Life Assurance Co., Ltd.,
Policy No. 009929 on a whole-life for P5,882.00 with a, rider for Accidental Death for the same amount
Buenaventura C. Ebrado designated T. Ebrado as the revocable beneficiary in his policy. He to her as his
wife. On October 21, 1969, Buenaventura C. Ebrado died as a result of an t when he was hit by a failing
branch of a tree. As the policy was in force, The Insular Life Assurance Co., Ltd. liable to pay the coverage
in the total amount of P11,745.73, representing the face value of the policy in the amount of P5,882.00 plus
the additional benefits for accidental death also in the amount of P5,882.00 and the refund of P18.00 paid
for the premium due November, 1969, minus the unpaid premiums and interest thereon due for January
and February, 1969, in the sum of P36.27.

Carponia T. Ebrado filed with the insurer a claim for the proceeds of the Policy as the designated
beneficiary therein, although she admits that she and the insured Buenaventura C. Ebrado were merely
living as husband and wife without the benefit of marriage. Pascuala Vda. de Ebrado also filed her claim as
the widow of the deceased insured. She asserts that she is the one entitled to the insurance proceeds, not
the common-law wife, Carponia T. Ebrado. In doubt as to whom the insurance proceeds shall be paid, the
insurer, The Insular Life Assurance Co., Ltd. commenced an action for Interpleader before the Court of First
Instance of Rizal on April 29, 1970.

Issue:

Can a common-law wife named as beneficiary in the life insurance policy of a legally married man
claim the proceeds thereof in case of death of the latter?

Ruling:

Common-law spouses are, definitely, barred from receiving donations from each other. Article
739 of the new Civil Code provides that the following donations shall be void: 1. Those made between
persons who were guilty of adultery or concubinage at the time of donation; 2. Those made between persons
found guilty of the same criminal offense, in consideration thereof; 3. Those made to a public officer or his
wife, descendants or ascendants by reason of his office.

In essence, a life insurance policy is no different from a civil donation insofar as the beneficiary is
concerned. Both are founded upon the same consideration: liberality. A beneficiary is like a donee, because
from the premiums of the policy which the insured pays out of liberality, the beneficiary will receive the
proceeds or profits of said insurance. As a consequence, the proscription in Article 739 of the new Civil
Code should equally operate in life insurance contracts. The mandate of Article 2012 cannot be laid aside:
any person who cannot receive a donation cannot be named as beneficiary in the life insurance policy of
the person who cannot make the donation. Under American law, a policy of life insurance is considered as
a testament and in construing it, the courts will, so far as possible treat it as a will and determine the effect
of a clause designating the beneficiary by rules under which wins are interpreted.

In the caw before Us, the requisite proof of common-law relationship between the insured and
the beneficiary has been conveniently supplied by the stipulations between the parties in the pre-trial
conference of the case. It case agreed upon and stipulated therein that the deceased insured Buenaventura
C. Ebrado was married to Pascuala Ebrado with whom she has six legitimate children; that during his
lifetime, the deceased insured was living with his common-law wife, Carponia Ebrado, with whom he has

18 6
R isk a n d Co v er a g es

two children. These stipulations are nothing less than judicial admissions which, as a consequence, no
longer require proof and cannot be contradicted. A fortiori, on the basis of these admissions, a judgment
may be validly rendered without going through the rigors of a trial for the sole purpose of proving the illicit
liaison between the insured and the beneficiary. In fact, in that pretrial, the parties even agreed "that a
decision be rendered based on this agreement and stipulation of facts as to who among the two claimants
is entitled to the policy."

18 7
R isk a n d Co v er a g es

SUN INSURANCE OFFICE LTD. v. COURT OF APPEALS


G.R. No. 92383
July 17, 1992

Cruz, J.

Facts:

The petitioner issued Personal Accident Policy No. 05687 to Felix Lim, Jr. with a face value of
P200,000.00. Two months later, he was dead with a bullet wound in his head. As beneficiary, his wife
Nerissa Lim sought payment on the policy but her claim was rejected. The petitioner agreed that there was
no suicide. It argued, however that there was no accident either.

Pilar Nalagon, Lim's secretary, was the only eyewitness to his death. It happened on October 6,
1982, at about 10 o'clock in the evening, after his mother's birthday party. According to Nalagon, Lim was
in a happy mood (but not drunk) and was playing with his handgun, from which he had previously
removed the magazine. As she watched television, he stood in front of her and pointed the gun at her. She
pushed it aside and said it might he loaded. He assured her it was not and then pointed it to his temple. The
next moment there was an explosion and Lim slumped to the floor. He was dead before he fell.

Issue:

Whether or not Lim died of an accident.

Ruling:

An accident is an event which happens without any human agency or, if happening through
human agency, an event which, under the circumstances, is unusual to and not expected by the person to
whom it happens. It has also been defined as an injury which happens by reason of some violence or
casualty to the injured without his design, consent, or voluntary co-operation.

In light of these definitions, the Court is convinced that the incident that resulted in Lim's death
was indeed an accident. The petitioner, invoking the case of De la Cruz v. Capital Insurance, says that "there
is no accident when a deliberate act is performed unless some additional, unexpected, independent and
unforeseen happening occurs which produces or brings about their injury or death." There was such a
happening. This was the firing of the gun, which was the additional unexpected and independent and
unforeseen occurrence that led to the insured person's death.

Lim was unquestionably negligent and that negligence cost him his own life. But it should not
prevent his widow from recovering from the insurance policy he obtained precisely against accident.
There is nothing in the policy that relieves the insurer of the responsibility to pay the indemnity agreed
upon if the insured is shown to have contributed to his own accident. Indeed, most accidents are caused
by negligence. There are only four exceptions expressly made in the contract to relieve the insurer from
liability, and none of these exceptions is applicable in the case at bar.

It bears noting that insurance contracts are as a rule supposed to be interpreted liberally in favor
of the assured. There is no reason to deviate from this rule, especially in view of the circumstances of this
case as above analyzed.

18 8
R isk a n d Co v er a g es

THE INSULAR LIFE ASSURANCE COMPANY, LTD. v. KHU


G.R. No. 195176
April 18, 2016

Del Castillo, J.

Facts:

Felipe N. Khu, Sr. applied for a life insurance policy with Insular Life under the latter’s Diamond
Jubilee Insurance Plan. Felipe accomplished the required medical questionnaire wherein he did not
declare any illness or adverse medical condition. Insular Life thereafter issued him Policy Number
A000015683 with a face value of P1 million. This took effect on June 22, 1997. Felipe’s policy lapsed due
to non-payment of the premium. Felipe applied for the reinstatement of his policy and paid P25,020.00 as
premium. Except for the change in his occupation of being self-employed to being the Municipal Mayor, all
the other information submitted by Felipe in his application for reinstatement was virtually identical to
those mentioned in his original policy.

Insular Life advised Felipe that his application for reinstatement may only be considered if he
agreed to certain conditions such as payment of additional premium and the cancellation of the riders
pertaining to premium waiver and accidental death benefits. Felipe agreed to these conditions and on
December 27, 1999 paid the agreed additional premium of P3,054.50.

Insular Life issued Endorsement No. PNA000015683. In consequence thereof, the premium rates
on this policy are adjusted to P28,000.00 annually, P14,843.00 semi-annually and P7,557.00 quarterly.
Felipe paid the annual premium. Felipe died. His Certificate of Death enumerated the following as causes
of death: Immediate cause: a. End stage renal failure, Hepatic failure; Antecedent cause: b. Congestive heart
failure, Diffuse myocardial ischemia.; Underlying cause: c. Diabetes Neuropathy, Alcoholism, and
Pneumonia.

Paz Y. Khu, Felipe Y. Khu, Jr. and Frederick Y. Khu filed with Insular Life a claim for benefit under
the reinstated policy. This claim was denied. Instead, Insular Life advised Felipe’s beneficiaries that it had
decided to rescind the reinstated policy on the grounds of concealment and misrepresentation by Felipe.
Hence, respondents instituted a complaint for specific performance with damages. Respondents prayed
that the reinstated life insurance policy be declared valid, enforceable and binding on Insular Life; and that
the latter be ordered to pay unto Felipe’s beneficiaries the proceeds of this policy, among others.

Issue:

Whether Felipe’s reinstated life insurance policy is already incontestable at the time of his death.

Ruling:

Sec. 48. Whenever a right to rescind a contract of insurance is given to the insurer by any provision
of this chapter, such right must be exercised previous to the commencement of an action on the contract.

After a policy of life insurance made payable on the death of the insured shall have been in force
during the lifetime of the insured for a period of two years from the date of its issue or of its last
reinstatement, the insurer cannot prove that the policy is void ab initio or is rescindible by reason of the
fraudulent concealment or misrepresentation of the insured or his agent.‘The insurer is deemed to have
the necessary facilities to discover such fraudulent concealment or misrepresentation within a period of
two (2) years. It is not fair for the insurer to collect the premiums as long as the insured is still alive, only
to raise the issue of fraudulent concealment or misrepresentation when the insured dies in order to defeat
the right of the beneficiary to recover under the policy.

18 9
R isk a n d Co v er a g es

At least two (2) years from the issuance of the policy or its last reinstatement, the beneficiary is
given the stability to recover under the policy when the insured dies. The provision also makes clear when
the two-year period should commence in case the policy should lapse and is reinstated, that is, from the
date of the last reinstatement’.

In this case, the parties differ as to when the reinstatement was actually approved. Insular Life
claims that it approved the reinstatement only on December 27, 1999. On the other hand, respondents
contend that it was on June 22, 1999 that the reinstatement took effect.The resolution of this issue hinges
on the following documents: 1) Letter of Acceptance; and 2) the Endorsement.

Based on the foregoing, we find that the CA did not commit any error in holding that the subject
insurance policy be considered as reinstated on June 22, 1999. This finding must be upheld not only
because it accords with the evidence, but also because this is favorable to the insured who was not
responsible for causing the ambiguity or obscurity in the insurance contract, The Court discerns a genuine
ambiguity or obscurity in the language of the two documents.

Indemnity and liability insurance policies are construed in accordance with the general rule of
resolving any ambiguity therein in favor of the insured, where the contract or policy is prepared by the
insurer. A contract of insurance, being a contract of adhesion, par excellence, any ambiguity therein should
be resolved against the insurer; in other words, it should be construed liberally in favor of the insured and
strictly against the insurer. Limitations of liability should be regarded with extreme jealousy and must be
construed in such a way as to preclude the insurer from noncompliance with its obligations.

19 0
F ir e I ns u ra nc e

MALAYAN INSURANCE CO. v. PAP CO., LTD.


G.R. No. 200784
August 7, 2013

Mendoza, J.

Facts:

Malayan Insurance Company issued Fire Insurance Policy No. F-00227-000073 to PAP Co., Ltd.
for the latter’s machineries and equipment located at Sanyo Precision Phils. Bldg., Phase III, Lot 4, Block
15, PEZA, Rosario, Cavite. The insurance, which was for Fifteen Million Pesos and effective for a period of
one (1) year, was procured by PAP Co. for Rizal Commercial Banking Corporation (RCBC), the mortgagee
of the insured machineries and equipment.

After the passage of almost a year but prior to the expiration of the insurance coverage, PAP Co.
renewed the policy on an "as is" basis. Pursuant thereto, a renewal policy, Fire Insurance Policy No. F-
00227-000079, was issued by Malayan to PAP Co.

On October 12, 1997 and during the subsistence of the renewal policy, the insured machineries
and equipment were totally lost by fire. Hence, PAP Co. filed a fire insurance claim with Malayan in the
amount insured.

In a letter, dated December 15, 1997, Malayan denied the claim upon the ground that, at the time
of the loss, the insured machineries and equipment were transferred by PAP Co. to a location different
from that indicated in the policy. Specifically, that the insured machineries were transferred from the
Sanyo Building to the Pace Pacific Bldg., Lot 14, Block 14, Phase III, PEZA, Rosario, Cavite (Pace Pacific).
Contesting the denial, PAP Co. argued that Malayan cannot avoid liability as it was informed of the transfer
by RCBC, the party duty-bound to relay such information. However, Malayan reiterated its denial of PAP
Co.’s claim. Distraught, PAP Co. filed the complaint below against Malayan.

Issue:

Is Malayan liable for the loss of the insured properties under the fire insurance policy?

Ruling:

The Court agrees with the position of Malayan that it cannot be held liable for the loss of the
insured properties under the fire insurance policy. The policy forbade the removal of the insured
properties unless sanctioned by Malayan.

Any transfer effected by the insured, without the insurer’s consent, would free the latter from any
liability. The respondent failed to notify, and to obtain the consent of, Malayan regarding the removal. The
records are bereft of any convincing and concrete evidence that Malayan was notified of the transfer of the
insured properties from the Sanyo factory to the Pace factory.

Also, the Court agrees with Malayan that the transfer to the Pace Factory exposed the properties
to a hazardous environment and negatively affected the fire rating stated in the renewal policy. The
increase in tariff rate from 0.449% to 0.657% put the subject properties at a greater risk of loss. Such
increase in risk would necessarily entail an increase in the premium payment on the fire policy.

Considering that the original policy was renewed on an "as is basis," it follows that the renewal
policy carried with it the same stipulations and limitations. The terms and conditions in the renewal policy
provided, among others, that the location of the risk insured against is at the Sanyo factory in PEZA. The
subject insured properties, however, were totally burned at the Pace Factory. Although it was also located

19 1
F ir e I ns u ra nc e

in PEZA, Pace Factory was not the location stipulated in the renewal policy. There being an unconsented
removal, the transfer was at PAP’s own risk. Consequently, it must suffer the consequences of the fire.
Thus, the Court agrees with the report of Cunningham Toplis Philippines, Inc., an international loss
adjuster which investigated the fire incident at the Pace Factory, which opined that "[g]iven that the
location of risk covered under the policy is not the location affected, the policy will, therefore, not respond
to this loss/claim."

It can also be said that with the transfer of the location of the subject properties, without notice
and without Malayan’s consent, after the renewal of the policy, PAP clearly committed concealment,
misrepresentation and a breach of a material warranty. Section 26 of the Insurance Code

Moreover, under Section 168 of the Insurance Code, the insurer is entitled to rescind the
insurance contract in case of an alteration in the use or condition of the thing insured. Section 168
of the Insurance CodeIn the case at bench, all these circumstances are present. It was clearly
established that the renewal policy stipulated that the insured properties were located at the
Sanyo factory; that PAP removed the properties without the consent of Malayan; and that the
alteration of the location increased the risk of loss.

19 2
F ir e I ns u ra nc e

TAN CHUCO v. YORKSHIRE FIRE AND LIFE INSURANCE


G.R. No. L-5069
October 15, 1909

Carson, J.

Facts:

Judgment in this action was rendered in the Court of First Instance of Manila against the
plaintiff upon his claim under an "open" fire insurance policy for compensation to the full extent
of the policy, for the alleged loss by fire of a certain stock of goods insured by the defendant
company; and against the defendant company on its counterclaim for losses resulting from the
plaintiff’s alleged intentional and fraudulent setting on fire of the building wherein the insured
goods were kept, thereby as it is alleged causing the destruction by fire of several stocks of goods
belonging to third parties, and insured by the defendant company.

Tan Chuco insured his property with Yorkshire Fire and Life Insurance. Yorkshire was
liable to indemnify Tan for the amount of losses unless it appears that Tan's right of recovery was
defeated by some failure on his part to live up to the terms of his contract. All of Tan's property
which was within the building where the insured goods were to be kept was destroyed by fire.
Yorkshire refused Tan's claim on the "open" fire insurance policy, alleging that Tan caused the
building to be set on fire. The CFI held the evidence did not sustain Yorkshire's allegation but was
of opinion that not only had Tan failed to establish the value of the goods but that he had failed to
live up to the terms of his contract as set out in the policy, thereby voiding the policy and defeating
his claim to indemnification.

Issue:

Is Tan entitled to recover to the full extent of the policy?

Ruling:

No. Tan failed to prove the value of the insured goods and that for the fraudulent purpose
of recovering the full amount of the policy, he submitted fabricated written evidence and false
testimony in support of his claim that the insured goods actually destroyed by fire were worth
more than the total amount of the insurance thereon.

We think that the action of the trial court in rejecting the proof offered by plaintiff as to
the amount of the loss must be sustained, and the contract of fire insurance being a contract of
indemnity, and the plaintiff only entitled therefore to recover the amount of the actual loss
sustained by him, there being no express valuation in the policy, judgment was properly entered
against him for lack of satisfactory proof of the amount of his loss.

19 3
F ir e I ns u ra nc e

BACHRACH v. BRITISH AMERICAN INSURANCE


G.R. No. L-5715
December 20, 1910

Johnson, J.

Facts:

The plaintiff commenced an action against the defendant to recover the sum of P9,841.50, the
amount due, deducting the salvage, upon the following fire insurance policy issued by the defendant to the
plaintiff the defendant also alleged certain facts under which it claimed that it was released from all
obligations whatever under said policy. These special facts are as follows: First. That the plaintiff
maintained a paint and varnish shop in the said building where the goods which were insured were stored;
Second. That the plaintiff transferred his interest in and to the property covered by the policy to H. W.
Peabody & Co. to secure certain indebtedness due and owing to said company, and also that the plaintiff
had transferred his interest in certain of the goods covered by the said policy to one Macke, to secure
certain obligations assumed by the said Macke for and on behalf of the insured. That the sanction of the
said defendant had not been obtained by the plaintiff, as required by the said policy; Third. That the
plaintiff, on the 18th of April, 1908, and immediately preceding the outbreak of the alleged fire, wilfully
placed a gasoline can containing 10 gallons of gasoline in the upper story of said building in close proximity
to a portion of said goods, wares, and merchandise, which can was so placed by the plaintiff as to permit
the gasoline to run on the floor of said second story, and after so placing said gasoline, he, the plaintiff,
placed in close proximity to said escaping gasoline a lighted lamp containing alcohol, thereby greatly
increasing the risk of fire; Fourth. That the plaintiff made no proof of the loss within the time required by
condition five of said policy, nor did the insured file a statement with he municipal or any other judge or
court of the goods alleged to have been in said building at the time of the alleged fire, nor of the goods
saved, nor the loss suffered.

The plaintiff, after denying nearly all of the facts set out in the special answer of the defendant,
alleged: First. That he had been acquitted in a criminal action against him, after a trial duly and regularly
had, upon a charge of arson, based upon the same alleged facts set out in the answer of the defendant;
Second. That her had made no proof of the loss set up in his complaint for the reason that immediately
after he had, on the 20th of April, 1908, given the defendant due notice in writing of said loss, the defendant,
on the 21st of April, 1908, and thereafter on other occasions, had waived all right to require proof of said
loss by denying all liability under the policy and by declaring said policy to be null and void; Third. That the
plaintiff, on the 18th of April, 1908, and immediately preceding the outbreak of the alleged fire, wilfully
placed a gasoline can containing 10 gallons of gasoline in the upper story of said building in close proximity
to a portion of said goods, wares, and merchandise, which can was so placed by the plaintiff as to permit
the gasoline to run on the floor of said second story, and after so placing said gasoline, he, the plaintiff,
placed in close proximity to said escaping gasoline a lighted lamp containing alcohol, thereby greatly
increasing the risk of fire; Fourth. That the plaintiff made no proof of the loss within the time required by
condition five of said policy, nor did the insured file a statement with he municipal or any other judge or
court of the goods alleged to have been in said building at the time of the alleged fire, nor of the goods
saved, nor the loss suffered.

Issue:

Is British American liable to Bachrach?

Ruling:

After hearing the evidence adduced during the trial of the cause, the lower court found
that the defendant was liable to the plaintiff and rendered a judgment against the defendant for
the sum of P9,841.50, with interest for a period of one year at 6 per cent, making a total of

19 4
F ir e I ns u ra nc e

P10,431.99, with costs. It may be added that there was no provision in the policy prohibiting the
keeping of paints and varnishes upon the premises where the insured property was stored. If the
company intended to rely upon a condition of that character, it ought to have been plainly
expressed in the policy.

With reference to this assignment of error, upon reading the policy of insurance issued by
the defendant to the plaintiff, it will be noted that there is no provision in said policy prohibiting
the plaintiff from placing a mortgage upon the property insured, but, admitting that such a
provision was intended, we think the lower court has completely answered this contention of the
defendant. He said, in passing upon this question as it was presented No such right is claimed to
have accrued in the case at bar, and the alienation clause is therefore inapplicable.

While the evidence shows some very peculiar and suspicious circumstances concerning the
burning of the goods covered by the said policy, yet, nevertheless, in view of the findings of the
lower court and in view of the apparent conflict in the testimony, we cannot find that there is a
preponderance of evidence showing that the plaintiff did actually set fire or cause fire to be set to
the goods in question.

19 5
F ir e I ns u ra nc e

DEVELOPMENT INSURANCE v. INTERMEDIATE APPELLATE COURT, et al.


G.R. No. 71360
July 16, 1986

Cruz, J.

Facts:

A fire occurred in the building of the private respondent and it sued for recovery of
damages from the petitioner on the basis of an insurance contract between them. The petitioner
allegedly failed to answer on time and was declared in default by the trial court. A judgment of
default was subsequently rendered on the strength of the evidence submitted ex parte by the
private respondent, which was allowed full recovery of its claimed damages. On learning of this
decision, the petitioner moved to lift the order of default, invoking excusable neglect, and to
vacate the judgment by default. Its motion was denied. It then went to the respondent court,
which affirmed the decision of the trial court in toto. Petitioner's claim that the insurance covered
only the building and not the elevators

Issue:

Did the insurance also cover the elevators? If so, what is the amount of the indemnity due
to Philippine Union Realty?

Ruling:

The petitioner's claim that the insurance covered only the building and not the elevators is absurd,
to say the least. The circumstance that the building insured is seven stories high and so had to be provided
with elevators-a legal requirement known to the petitioner as an insurance company-makes its contention
all the more ridiculous.

No less preposterous is the petitioner's claim that the elevators were insured after the occurrence
of the fire, a case of shutting the barn door after the horse had escaped, so to speak.4 This pretense merits
scant attention. Equally undeserving of serious consideration is its submission that the elevators were not
damaged by the fire, against the report of The arson investigators of the INP and, indeed, its own expressed
admission in its answer6 where it affirmed that the fire "damaged or destroyed a portion of the 7th floor
of the insured building and more particularly a Hitachi elevator control panel."

However, there is no evidence on record that the building was worth P5,800,000.00 at the
time of the loss; only the petitioner says so and it does not back up its self-serving estimate with
any independent corroboration. On the contrary, the building was insured at P2,500,000.00, and
this must be considered, by agreement of the insurer and the insured, the actual value of the
property insured on the day the fire occurred. This valuation becomes even more believable if it
is remembered that at the time the building was burned it was still under construction and not
yet completed.

As defined in the aforestated provision, which is now Section 60 of the Insurance Code, "an open
policy is one in which the value of the thing insured is not agreed upon but is left to be ascertained in case
of loss. " This means that the actual loss, as determined, will represent the total indemnity due the insured
from the insurer except only that the total indemnity shall not exceed the face value of the policy.

The actual loss has been ascertained in this case and, to repeat, this Court will respect such factual
determination in the absence of proof that it was arrived at arbitrarily. There is no such showing. Hence,
applying the open policy clause as expressly agreed upon by the parties in their contract, we hold that the

19 6
F ir e I ns u ra nc e

private respondent is entitled to the payment of indemnity under the said contract in the total amount of
P508,867.00.

19 7
F ir e I ns u ra nc e

NEW LIFE ENTERPRISES, et al. v. CA, et al.


G.R. No. 94071
March 31, 1992

Regalado, J.

Facts:

Julian Sy and Jose Sy Bang have formed a business partnership under the business name of New
Life Enterprises, the partnership engaged in the sale of construction materials at its place of business, a
two storey building. Julian Sy insured the stocks in trade of New Life Enterprises with Western Guaranty
Corporation, Reliance Surety and Insurance. Co., Inc., and Equitable Insurance Corporation.
Western Guaranty Corporation issued Fire Insurance Policy No. 37201 in the amount of
P350,000.00. This policy was renewed on May, 13, 1982. On July 30,1981, Reliance Surety and Insurance
Co., Inc. issued Fire Insurance Policy No. 69135 in the amount of P300,000.00 (Renewed under Renewal
Certificate No. 41997) An additional insurance was issued by the same company on November 12, 1981
under Fire Insurance Policy No. 71547 in the amount of P700,000.00. On February 8, 1982, Equitable
Insurance Corporation issued Fire Insurance Policy No. 39328 in the amount of P200,000.00.
When the building occupied by the New Life Enterprises was gutted by fire, the stocks in the trade
inside said building were insured against fire in the total amount of P1,550,000.00. The cause of fire was
electrical in nature.
After the fire, Julian Sy went to the agent of Reliance Insurance whom he asked to accompany him
to the office of the company so that he can file his claim. He averred that in support of his claim, he
submitted the fire clearance, the insurance policies and inventory of stocks. The three insurance
companies denied Julian's claim for payment.
In relation to the case against Reliance Surety and Insurance Company, a certain Atty. Serafin D.
Dator, acting in behalf of the plaintiff, sent a letter dated February 13, 1983 to Executive Vice-President
Mary Dee Co asking that he be informed as to the specific policy conditions allegedly violated by the
plaintiff. She replied stating that Sy violated Condition No. 3 of said insurance policies, otherwise known as
the "Other Insurance Clause," is uniformly contained in all the aforestated insurance contracts of herein
petitioners, as follows:
Because of the denial of their claims for payment by the three (3) insurance companies, petitioner
filed separate civil actions against the former before the Regional Trial Court of Lucena City, which cases
were consolidated for trial, and thereafter the court below rendered its decision on December 19, l986
with the following disposition:
Issue:
Whether the petitioner violated the insurance policies
Ruling:
New Life and Julian admitted that the insurance policies issued by the companies did not state or
endorse the other insurance coverage obtained or subsequently effected on the same stocks-in-trade. The
policy issued by Western did not declare Reliance and Equitable as co-insurers on the same stocks, while
Reliance's policies covering the same stocks did not likewise declare Western and Equitable as co-insurers.
Equitable's policy stated "nil" in the space thereon requiring indication of any co-insurance although there
were three policies subsisting on the same stocks-in-trade at the time of the loss. The insured is specifically
required to disclose to the insurer any other insurance and its particulars which he may have effected on
the same subject matter. The knowledge of such insurance by the insurer's agents, even assuming the
acquisition thereof by the former, is not the "notice" that would estopped the insurers from denying the
claim. The obvious purpose of the disclosure requirement is to prevent over-insurance and thus avert the
perpetration of fraud. The public, as well as the insurer, is interested in preventing the situation in which
a fire would be profitable to the insured.
While it is a cardinal principle of insurance law that a policy or contract of insurance is to be
construed liberally in favor of the insured and strictly against the insurer company, yet contracts of
insurance, like other contracts, are to be construed according to the sense and meaning of the terms which
the parties themselves have used. If such terms are clear and unambiguous, they must be taken and

19 8
F ir e I ns u ra nc e

understood in their plain, ordinary and popular sense. Moreover, obligations arising from contracts have
the force of law between the contracting parties and should be complied with in good faith.
As the insurance policy against fire expressly required that notice should be given by the insured
of other insurance upon the same property, the total absence of such notice nullifies the policy.

19 9
F ir e I ns u ra nc e

PHILIPPINE HOME ASSURANCE CORPORATION v. COURT OF APPEALS


G.R. No 106999
June 20, 1996

Kapunan, J.

Facts:

Eastern Shipping Lines, Inc. loaded on board SS Eastern Explorer in Kobe, Japan, for
carriage to Manila and Cebu, freight pre-paid and in good order and condition, viz: (a) two (2)
boxes internal combustion engine parts, consigned to William Lines, Inc.; (b) ten (10) metric tons
(334 bags) ammonium chloride, consigned to Orca's Company; (c) two hundred (200) bags Glue
300, consigned to Pan Oriental Match Company; and (d) garments, consigned to Ding Velayo.
While the vessel was off Okinawa, Japan, a small flame was detected on the acetylene cylinder
located in the accommodation area near the engine room on the main deck level. As the crew was
trying to extinguish the fire, the acetylene cylinder suddenly exploded sending a flash of flame
throughout the accommodation area, thus causing death and severe injuries to the crew and
instantly setting fire to the whole superstructure of the vessel. The incident forced the master and
the crew to abandon the ship.

Thereafter, SS Eastern Explorer was found to be a constructive total loss and its voyage was
declared abandoned. After the fire was extinguished, the cargoes which were saved were loaded
to another vessel for delivery to their original ports of destination. ESLI charged the consignees
several amounts corresponding to additional freight and salvage charges

The charges were all paid by Philippine Home Assurance Corporation (PHAC) under protest
for and in behalf of the consignees.

PHAC, as subrogee of the consignees, thereafter filed a complaint before the Regional Trial
Court against ESLI to recover the sum paid under protest on the ground that the same were
actually damages directly brought about by the fault, negligence, illegal act and/or breach of
contract of ESLI. In its answer, ESLI contended that it exercised the diligence required by law in
the handling, custody and carriage of the shipment; that the fire was caused by an unforeseen
event; that the additional freight charges are due and demandable pursuant to the Bill of
Lading; and that salvage charges are properly collectible under Act No. 2616, known as the
Salvage Law.

Issue:

1. Whether or not the fire on the vessel which was caused by the explosion of an acetylene
cylinder loaded on the same was the fault or negligence of the defendant.
2. Whether or not the court erroneously adopted the conclusion of the Trial Court's
conclusion that the expenses or averages incurred in saving the cargo constitute general average.

Ruling:

In our jurisprudence, fire may not be considered a natural disaster or calamity since it
almost always arises from some act of man or by human means. It cannot be an act of God unless
caused by lightning or a natural disaster or casualty not attributable to human agency.

In the case at bar, it is not disputed that a small flame was detected on the acetylene cylinder
and that by reason thereof, the same exploded despite efforts to extinguish the fire. Neither is
there any doubt that the acetylene cylinder, obviously fully loaded, was stored in the
accommodation area near the engine room and not in a storage area considerably far, and in a

20 0
F ir e I ns u ra nc e

safe distance, from the engine room. Moreover, there was no showing, and none was alleged by
the parties, that the fire was caused by a natural disaster or calamity not attributable to human
agency. On the contrary, there is strong evidence indicating that the acetylene cylinder caught fire
because of the fault and negligence of respondent ESLI, its captain and its crew.

First, the acetylene cylinder which was fully loaded should not have been stored in the
accommodation area near the engine room where the heat generated therefrom could cause the
acetylene cylinder to explode by reason of spontaneous combustion. Respondent ESLI should
have easily foreseen that the acetylene cylinder, containing highly inflammable material, was in a
real danger of exploding because it was stored in close proximity to the engine room.

Second, respondent ESLI should have known that by storing the acetylene cylinder in the
accommodation area supposed to be reserved for passengers, it unnecessarily exposed its
passengers to grave danger and injury. Curious passengers, ignorant of the danger the tank might
have on humans and property, could have handled the same or could have lighted and smoke
cigarettes while repairing in the accommodation area.

Third, the fact that the acetylene cylinder was checked, tested and examined and
subsequently certified as having complied with the safety measures and standards by qualified
experts before it was loaded in the vessel only shows to a great extent that negligence was present
in the handling of the acetylene cylinder after it was loaded and while it was on board the
ship. Indeed, had the respondent and its agents not been negligent in storing the acetylene
cylinder near the engine room, then that same would not have leaked and exploded during the
voyage.

Verily, there is no merit in the finding of the trial court to which respondent court
erroneously agreed that the fire was not fault or negligence of respondent but a natural disaster
or calamity. The records are simply wanting in this regard.

On the issue of whether or not respondent court committed an error in concluding that the
expenses incurred in saving the cargo are considered general average, we rule in the
affirmative. As a rule, general or gross averages include all damages and expenses which are
deliberately caused in order to save the vessel, its cargo, or both at the same time, from a real and
known risk. While the instant case may technically fall within the purview of the said provision,
the formalities prescribed under Article 813 and 814 of the Code of Commerce in order to incur
the expenses and cause the damage corresponding to gross average were not complied
with. Consequently, respondent ESLI's claim for contribution from the consignees of the cargo at
the time of the occurrence of the average turns to naught.

20 1
Ca sua l t y In su ra nc e

PAN MALAYAN INSURANCE CORPORATION v. COURT OF APPEALS


G.R. No. 81026
April 3, 1990

Cortes, J.
Facts:

PANMALAY insured a Mitsubishi Colt Lancer car and registered in the name of Canlubang Automotive
Resources Corporation. On May 26, 1985, due to the "carelessness, recklessness, and imprudence" of the
unknown driver of a pick-up, the insured car was hit and suffered damages in the amount of P42,052.00.
PANMALAY defrayed the cost of repair of the insured car and, therefore, was subrogated to the rights of
CANLUBANG against the driver of the pick-up and his employer, Erlinda Fabie. Despite repeated demands,
defendants, failed and refused to pay the claim of PANMALAY.

Private respondents filed a Motion for Bill of Particulars and a supplemental motion thereto. In
compliance therewith, PANMALAY clarified that the damage caused to the insured car was settled under the
"own damage", coverage of the insurance policy, and that the driver of the insured car was, at the time of the
accident, an authorized driver duly licensed to drive the vehicle. PANMALAY also submitted a copy of the
insurance policy and the Release of Claim and Subrogation Receipt executed by CANLUBANG in favor of
PANMALAY.

Issue:

Whether or not the insurer PANMALAY may institute an action to recover the amount it had paid.

Ruling:

Yes. It is a basic rule in the interpretation of contracts that the terms of a contract are to be construed
according to the sense and meaning of the terms which the parties thereto have used. PANMALAY could not
be deemed subrogated to the rights of its assured under Article 2207 of the Civil Code, PANMALAY would still
have a cause of action against private respondents.

It must be reiterated that , the insurer PANMALAY as subrogee merely prays that it be allowed to
institute an action to recover from third parties who allegedly caused damage to the insured vehicle, the
amount which it had paid its assured under the insurance policy. Having shown from the discussion that
PANMALAY has a cause of action against third parties whose negligence may have caused damage to
CANLUBANG's car, the Court holds that there is no legal obstacle to the filing by PANMALAY of a complaint for
damages against private respondents as the third parties allegedly responsible for the damage. It is now for
the trial court to determine if in fact the damage caused to the insured vehicle was due to the "carelessness,
recklessness and imprudence" of the driver of private respondent Erlinda Fabie.

20 2
Ca sua l t y In su ra nc e

SIMON DE LA CRUZ v. THE CAPITAL INSURANCE and SURETY CO., INC.,


G.R. No. L-21574
June 30, 1966

Barrera, J.

Facts:

Eduardo de la Cruz, employed as a mucker in the Itogon-Suyoc Mines, Inc. in Baguio, was the holder
of an accident insurance policy gainst death or disability caused by accidental means. January 1, 1957: For the
celebration of the New Year, the Itogon-Suyoc Mines, Inc. sponsored a boxing contest for general
entertainment wherein Eduardo, a non-professional boxer participated. In the course of his bout with another
non-professional boxer, Eduardo slipped and was hit by his opponent on the left part of the back of the head,
causing Eduardo to fall, with his head hitting the rope of the ring. He was brought to the Baguio General
Hospital the following day. He died due to hemorrhage, intracranial.

Simon de la Cruz, the father of the insured and who was named beneficiary under the policy,
thereupon filed a claim with the insurance company but Capital Insurance and Surety co., inc denied stating
that the death caused by his participation in a boxing contest was not accidental.

Issue:

Whether or not the cause of death was an accident.

RULING:

The fact that boxing is attended with some risks of external injuries does not make any injuries
received in the course of the game not accidental. In boxing as in other equally physically rigorous sports, such
as basketball or baseball, death is not ordinarily anticipated to result. If, therefore, it ever does, the injury or
death can only be accidental or produced by some unforeseen happening or event as what occurred in this
case.

Furthermore, the policy involved herein specifically excluded from its coverage —
(e) Death or disablement consequent upon the Insured engaging in football, hunting, pigsticking,
steeplechasing, polo-playing, racing of any kind, mountaineering, or motorcycling.

Death or disablement resulting from engagement in boxing contests was not declared outside of the
protection of the insurance contract.

20 3
Ca sua l t y In su ra nc e

SUN INSURANCE OFFICE, LTD., v. THE HON. COURT OF APPEALS


G.R. No. 92383
July 17, 1992

Cruz, J.
Facts:

The petitioner issued Personal Accident Policy No. 05687 to Felix Lim, Jr. with a face value of
P200,000.00. Two months later, he was dead with a bullet wound in his head. As beneficiary, his wife Nerissa
Lim sought payment on the policy but her claim was rejected. The petitioner agreed that there was no suicide.
It argued, however that there was no accident either.

Pilar Nalagon was the only eyewitness to his death. It happened on October 6, 1982, after his mother's
birthday party. According to Nalagon, Lim was in a happy mood and was playing with his handgun, from
which he had previously removed the magazine. As she watched television, he stood in front of her and pointed
the gun at her. She pushed it aside and said it might he loaded. He assured her it was not and then pointed it to
his temple. The next moment there was an explosion and Lim slumped to the floor. He was dead before he fell.

Issue:

Whether the incident was an accident.

Ruling:

Yes.

Lim was unquestionably negligent and that negligence cost him his own life. But it should not prevent
his widow from recovering from the insurance policy he obtained precisely against accident. There is nothing
in the policy that relieves the insurer of the responsibility to pay the indemnity agreed upon if the insured is
shown to have contributed to his own accident. Indeed, most accidents are caused by negligence. There are
only four exceptions expressly made in the contract to relieve the insurer from liability, and none of these
exceptions is applicable in the case at bar.

20 4
Ca sua l t y In su ra nc e

FORTUNE INSURANCE AND SURETY CO., INC., v. COURT OF APPEALS


G.R. No. 115278
May 23, 1995

Davide, Jr., J.

Facts:

Sun Insurance issued Personal Accident Policy to Felix Lim with a face value of P200,000.00. Lim
accidentally killed himself with his gun after removing the magazine, showing off, pointing the gun at his
secretary, and pointing the gun at his temple.

The widow, the beneficiary, sued the petitioner and won 200,000 as indemnity with additional
amounts for other damages and attorney’s fees. This was sustained in the Court of Appeals then sent to the
Supreme Court by the insurance company.

Issue:

Is Lim’s widow eligible to receive the benefits?

Ruling:

Yes. Under the insurance contract, the company wasn’t liable for bodily injury caused by attempted
suicide or by one needlessly exposing himself to danger except to save another’s life.

Lim wasn’t thought to needlessly expose himself to danger due to the witness testimony that he took
steps to ensure that the gun wasn’t loaded. He even assured his secretary that the gun was loaded.

There is nothing in the policy that relieves the insurer of the responsibility to pay the indemnity agreed
upon if the insured is shown to have contributed to his own accident.

20 5
Ca sua l t y In su ra nc e

ALPHA INSURANCE AND SURETY CO. v. ARSENIA SONIA CASTOR


G.R. No. 198174
September 2, 2013

Peralta, J.
FACTS:

On February 21, 2007, respondent entered into a contract of insurance, Motor Car Policy No.
MAND/CV-00186, with petitioner, involving her motor vehicle, a Toyota Revo DLX DSL. . The contract of
insurance obligates the petitioner to pay the respondent the amount of Six Hundred Thirty Thousand Pesos
(₱630,000.00) in case of loss or damage to said vehicle during the period covered, which is from February 26,
2007 to February 26, 2008.

On April 16, 2007, respondent instructed her driver Lanuza to bring the above described vehicle to a
nearby auto-shop for a tune-up. However, Lanuza no longer returned the motor vehicle to respondent and
despite diligent efforts to locate the same, said efforts proved futile.

In a letter dated July 5, 2007, petitioner denied the insurance claim of respondent
1.) The Company shall not be liable for:
xxxx
(4) Any malicious damage caused by the Insured, any member of his family or by "A
PERSON IN THE INSURED’S SERVICE."

Issue:

Whether the loss of respondent’s vehicle is excluded under the insurance policy.

Ruling:

No, the loss is not excluded under the insurance policy. Ruling in favor of respondent, the RTC
scrupulously elaborated that theft perpetrated by the driver of the insured is not an exception to the coverage
from the insurance policy, since Section III thereof did not qualify as to who would commit the theft. Theft
perpetrated by a driver of the insured is not an exception to the coverage from the insurance policy subject of
this case. This is evident from the very provision of Section III – "Loss or Damage." The insurance company,
subject to the limits of liability, is obligated to indemnify the insured against theft. Said provision does not
qualify as to who would commit the theft. Thus, even if the same is committed by the driver of the insured,
there being no categorical declaration of exception, the same must be covered.

Indemnity and liability insurance policies are construed in accordance with the general rule of
resolving any ambiguity therein in favor of the insured, where the contract or policy is prepared by the insurer.
A contract of insurance, being a contract of adhesion, par excellence, any ambiguity therein should be resolved
against the insurer; in other words, it should be construed liberally in favor of the insured and strictly against
the insurer.

20 6
Ca sua l t y In su ra nc e

PHIL-NIPPON KYOEI, CORP. v. ROSALIA T. GUDELOSAO


G.R. No. 181375
July 13, 2016

Jardeleza, J.

Facts:

Petitioner, a domestic shipping corporation, purchased a "Ro-Ro" passenger/cargo. For the vessel's
one-month conduction voyage from Japan to the Philippines, petitioner, as local principal, and Top Ever
Marine Management Maritime Co., Ltd. (TMCL), as foreign principal, hired Gudelosao, Tancontian, and six
other crewmembers. They were hired through the local manning agency of TMCL, Top Ever Marine
Management Philippine Corporation (TEMMPC). Petitioner secured a Marine Insurance Policy from SSSICI
over the vessel for P10,800,000.00 against loss, damage, and third party liability or expense, arising from the
occurrence of the perils of the sea for the voyage of the vessel from Onomichi, Japan to Batangas, Philippines.
This Marine Insurance Policy included Personal Accident Policies for the eight crewmembers for
P3,240,000.00 each in case of accidental death or injury.

On February 24, 2003, while still within Japanese waters, the vessel sank due to extreme bad weather
condition. Only Chief Engineer Nilo Macasling survived the incident while the rest of the crewmembers,
including Gudelosao and Tancontian, perished.

Issue:

Is the liability of petitioner extinguished only upon SSSICI's payment of insurance proceeds.

RULING:

No. Since, that while the Personal Accident Policies are casualty insurance, they do not answer for
petitioner's liabilities arising from the sinking of the vessel. It is an indemnity insurance procured by petitioner
for the benefit of the seafarers. As a result, petitioner is not directly liable to pay under the policies because it
is merely the policyholder of the Personal Accident Policies.

Petitioner's claim that the limited liability rule and its corresponding exception apply here is irrelevant
because petitioner was not found liable under tort or quasi-delict. Moreover, the insurance proceeds
contemplated under the exception in the case of a lost vessel are the insurance over the vessel and pending
freightage for the particular voyage. It is not the insurance in favor of the seafarers, the proceeds of which are
intended for their beneficiaries. Thus, if ever petitioner is liable for the value of the insurance proceeds under
tort or quasi-delict, it would be from the Marine Insurance Policy over the vessel and not from the Personal
Accident Policies over the seafarers.

20 7
Su r et ysh ip

MELECIO ARRANZ v. MANILA FIDELITY AND SURETY CO., INC.,


G.R. No. L-9674
April 29, 1957

Labrador, J.

Facts:

Manila Fidelity executed and delivered to Manila Ylang Ylang Distillery a surety bond, by virtue of
which defendant as surety understood to pay jointly and severally with plaintiff as principal, the sum of
P90,000. To secure the surety against loss arising from the surety bond, plaintiff executed a second mortgaged
over the properties which were transferred by the Manila Ylang Ylang Distillery to Arranz. As defendant surety
had no money with which to respond for the obligation, plaintiff made an arrangement with the Philippine
National Bank, whereby he would mortgage the same properties to the latter in order to raise the amount
needed to pay the amount of the loan. As the plaintiff feared that the credit accommodation he sought from
the Philippine National Bank could not be secured without release by the surety of its second mortgage, Arranz
paid the sums required by PNB except the P25,000, and thereupon the second mortgage executed in favor of
surety, defendant, was cancelled. A complaint to recover the premiums was filed. The trial court ruled that the
payment of the sum of P14,200 demanded in plaintiff's complaint was paid as a price for the release of the
properties held on second mortgage by the defendant, or that the same was the consideration for said release
in order to save his properties, and therefore dismissed the complaint.

Issue:

Is plaintiff obligated to pay the premium on the bond because of failure of his surety to pay the
indebtedness secured by it?

Ruling:

Yes. There is no allegation in the complaint or in any other paper in the case that the surety promised
the principal that it will pay the loan or obligation contracted by the principal (plaintiff herein) for the latter's
account. In the contract of suretyship the creditor was given the right to sue the principal, or the latter and the
surety at the same time. This does not imply, however, that the surety covenanted or agreed with the principal
that it will pay the loan for the benefit of the principal. Such a promise is not implied by law either.

Plaintiff, therefore, cannot claim that there has been a breach on the part of the surety of any obligation
it has made or undertaken under the suretyship contract. And the failure or refusal of the surety to pay the
debt for the principal's account did not have the effect of relieving the principal of his obligation to pay the
premium on the bond furnished. The premium is the consideration for furnishing the bond or the guaranty.
While the liability of the surety to the obligee subsists the premium is collectible from the principal. Under the
terms of the contract of suretyship the surety's obligation is that the principal pay the loan and the interest
thereon, and that the surety shall be relieved of his obligation when the loan or obligation secured is paid. As
the loan and interest remained unpaid the surety continued to be bound to the creditor-obligee, and as a
corollary its right to collect the premium on the bond also continued. Plaintiff-appellant, therefore, cannot
excuse himself from the payment of the premium on the bond upon the failure or refusal of the surety to pay
the loan and the interest. Even if, therefore, the payment of the premium were against his will, still plaintiff-
appellant has no cause of action for the return thereof, because the surety was entitled thereto.

20 8
Su r et ysh ip

REPARATIONS COM. v. UNIVERSAL DEEP SEA FISHING


A.M. No. 21901-96
June 27, 1978

Concepcion Jr., J

Facts:

Universal Deep-Sea Fishing was awarded six trawl boats by the Reparations Commission. The M/S
UNIFISH 1 and 2, were delivered to Universal on November 20, 1958, and the contract of Conditional Purchase
and Sale of Reparations Goods, executed by and between the parties on February 12, 1960, provided among
others, that the first installment representing 10% of the amount P53,642.84 shall be paid within 24 months
from the date of complete delivery thereof, the balance shall be paid in the manner stipulated. To guarantee
faithful compliance, a performance bond was executed in favor of RC. A Corresponding indemnity agreement
was executed to indemnify the surety company for any damage, loss charges, etc., which it may sustain or incur
as a consequence of having become a surety upon the performance bond. Bonds were likewise issued over the
other trawl boats. But when the RC sued Universal and its surety to recover various amounts of money due
under the contracts, they claimed that the amounts were not yet due and demandable. Universal alleged that
there was an obscurity in the terms of the contracts in question which was caused by the plaintiff as to the
amounts and due dates of the first installments which should have been first fixed before the creditor could
demand its payment from the debtors specifically referring to the schedule of payments which allegedly
indicated two due dates for the payment of the first installment.

Issue:

Are the first instalments under judgment already due and demandable when complaint was filed?

Ruling:

Yes. The terms of the contracts for the purchase and sale of the reparations vessels, however, are very
clear and leave no doubt as to the intent of the contracting parties. Thus, in the contract concerning the M/S
UNIFISH 1 and M/S UNIFISH 2, the parties expressly agreed that the first installment representing 10% of the
purchase price or P53, 642.84 shall be paid within 24 months from the date of complete delivery of the vessel
or on May 8, 1961, and the balance to be paid in ten equal yearly instalments. The amount of P56, 597.20 due
on May 8, 1962, which is also claimed to be a "first installment," is but the first of the 10 equal yearly
instalments of balance of judgment, purchase price.

20 9
Su r et ysh ip

CAPITAL INSURANCE v. RONQUILLO TRADING


G.R. No. L-36488
July 25, 1983

Gutierrez, Jr., J

Facts:

Capital Insurance executed and issued a surety bond in behalf of Ronquillo and in favor of S.S
Eurygenes, its master, and or its agents, Delgado Shipping. The bond was a guarantee for any additional freight
which may be determined to be due on a cargo of 258 surplus army vehicles consigned from Pusan, Korea to
the Ronquillo Trading on board S.S Eurygenes and booked on said vessel by the Philippine Merchants
Steamship. In consideration thereof, appelles executed an indemnity agreement whereby they jointly and
severally promised to pay the appellant the sum of P1,827 in advance as premium and documentary stamps
for each period of twelve months while the surety bond was in effect.

About 5 days before the expiration of the liability on the bond, PD Marchessini and Delgado Shipping
filed a civil case before the CFI against PMSI, Bautista, doing business under the name of Ronquillo Trading,
and Capital Insurance for the sum of the loss they allegedly suffered as a direct consequence of failure of
defendants to load the stipulated quantity of US surplus army vehicles. Upon the expiration of the bond, the
appellant made a formal demand for the payment of there newal premiums and cost of documentary stamps
for another year in the amount of P1,827.00. They refused to pay under the bond. Hence, a complaint was filed
to recover the sum of P1,827.00.

Issue:

Did the trial court err in holding that once surety's liability under the bond has accrued, defendants
are under no obligation to pay the premiums and costs of documentary stamps for the succeeding period that
it is in effect?

Ruling:

Yes. The surety bond stipulated that the "liability of surety on this bond will expire on May 5, 1963
and said bond will be cancelled 15 days after its expiration, unless surety is notified of any existing obligations
thereunder." Under this stipulation the bond expired on the stated date and the phrase "unless surety is
notified of any existing obligations thereunder" refers to obligations incurred during the term of the bond.
Furthermore, under the Indemnity Agreement, the appellees "agree to pay the COMPANY the sum of
P1,800.00, in advance as premium thereof for every twelve months or fraction thereof, while this bond or any
renewal or substitution thereof is in effect." Obviously, the duration of the bond is for "every twelve months or
fraction thereof, while this bond or any renewal or substitution is in effect." Since the appellees opted not to
renew the contract they cannot be obliged to pay the premiums. More specifically, where a contract of surety
is terminated under its terms, the liability of the principal for premiums after such termination ceases
notwithstanding the pendency of a lawsuit to enforce a liability that accrued during its stipulated lifetime.

21 0
Su r et ysh ip

ZARAGOZA v. FIDELINO
G.R. No. L-29723
July 14, 1988

Narvasa, J.
Facts:

A case for replevin was file by Zaragoza against Fidelino and/or John Doe. He alleged that he sold the
car to Fidelino but the latter failed to pay the price in the manner stipulated in their agreement. The car was
taken from Fidelino’s possession by the sheriff but was promptly returned to her on orders of the court when
a surety bond for the car’s release was posted in her behalf by Mabini Insurance & Fidelity. Zaragoza moved
for the amendment of the decision so as to include Mabini as party solidarily liable with defendant.

Issue:

Is Mabini liable although it had paid a counter-bond to release the property subject of replevin?

Ruling:

Yes. In Dee v. Masloff, a surety on a counter-bond given to release property from receivership, also
sought to avoid liability by asserting that it was not a party to the case, had never been made a party, and had
not been notified of the trial. The Court overruled the contention, and upheld the propriety of the amendment
of the judgment which ordered the appellant surety company to pay — to the extent of its bond and jointly
and severally with defendant — the judgment obligation. The Court ruled that since such "amended judgment
had been rendered after the appellant surety company as party jointly and severally liable with the defendant
for the damages already awarded to the appellees, to which the appellant surety company filed its
"Opposition" and "Rejoinder" to the "Reply to Opposition filed by the appellees, without putting in issue the
reasonableness of the amount awarded for damages but confining itself to the defense in avoidance of liability
on its bond that it was not a party to the case and never made a party therein and was not notified of the trial
of the case, and that the appellees were guilty of laches, the requirement of hearing was fully satisfied or
complied with; appellant surety company never prayed for an opportunity to present evidence in its behalf."

21 1
Su r et ysh ip

EASTERN ASSURANCE & SURETY CORPORATION v. INTERMEDIATE APPELLATE COURT


G.R. No. L-69450
November 22, 1988

Feliciano, J.

Facts:

On 8 January 1976, the Region 7 (Cebu) Office of respondent Department of Agrarian Reform put up
for public bidding a job or project consisting of the repair of seven units of Willys Mitsubishi/Eisenhower jeeps.
Among the bidders was Motor City, an automotive repair, company, which latter on emerged as the winning
bidder. The winning bid was accompanied by a Proposal Bond 2— required by the DAR of all bidders — in the
amount of P33,275.00 and issued by petitioner Eastern Assurance and Surety Corporation, as surety, on behalf
of Motor City, its principal.

On 31 January 1976, a Contract for Repair of Jeeps was entered into between DAR as owner and Motor
City as contractor. It turned out that only six out of the seven jeeps were repaired fully and delivered to DAR.
The seventh unit, continued to remain undelivered, despite the grant of several extensions in favor of and the
issuance. On March 13, 1978 of a final letter to Motor City, demanding that the latter complete the repair and
effect delivery of the seventh vehicle.

On 12 July 1978, DAR commenced a suit for specific performance and damages against Motor City.
Included there as a co-defendant was petitioner Eastern which, "had posted the performance bond
undertaking to answer and guarantee the true and faithful compliance and performance of the Contract for
Repair of Jeeps." The Trial Court ruled that Eastern Assurance and Motor City are liable. The IAC upheld the
decision of the lower court.

Issue:

Is Eastern Assurance liable as a surety for the contractual breach committed by Motor City?

Ruling:

YES. A proposal or bid bond has for its purpose to assure the owner of the project of the good faith of
the bidder and that the bidder will enter into a contract with the project owner should his proposal be
accepted. A performance bond is, upon the other hand designed to afford the project owner security that the
bidder, now the contractor, will faithfully comply with the requirements of the contract awarded to the
contractor and make good damages sustained by the project owner in case of the contractor's failure to so
perform.

21 2
Su r et ysh ip

STRONGHOLD INSURANCE v. COURT APPEALS


G.R. No. 88050
January 30, 1992

Cruz, J.

Facts:

Acting on behalf of its foreign principal, Qatar National Fishing Co., Pan Asian Logistics and Trading, a
domestic recruiting and placement agency, hired Adriano Urtesuela as captain of the vessel M/V Oryx for the
stipulated period of twelve months. The required surety bond, in the amount of P50,000.00, was submitted by
Pan Asian and Stronghold Insurance Co., Inc., to answer for the liabilities of the employer. Urtesuela assumed
his duties on April 18, 1982, but three months later his services were terminated and he was repatriated to
Manila. HeBthereupon filed a complaint against Pan Asian and his former employer with the Philippine
Overseas Employment Administration for breach of contract and damages.

Issue:

Is Stronghold Insurance, as a surety, liable to the payment of the prayed value?

Ruling:

YES. The surety bond required of recruitment agencies is intended for the protection of our citizens
who are engaged for overseas employment by foreign companies. The purpose is to ensure that if the rights of
these overseas workers are violated by their employers, recourse would still be available to them against the
local companies that recruited them for the foreign principal. The foreign principal is outside the jurisdiction
of our courts and would probably have no properties in this country against which an adverse judgment can
be enforced. This difficulty is corrected by the bond, which can be proceeded against to satisfy that judgment.

Given this purpose, and guided by the benign policy of social justice, the Court rejected the
technicalities raised by the petitioner against its established legal and even moral liability to the private
respondent. These technicalities do not impair the rudiments of due process or the requirements of the law
and must be rejected in deference to the constitutional imperative of justice for the worker.

21 3
Su r et ysh ip

NATIONAL POWER CORP. v. COURT APPEALS


G.R. No. L-43706
November 14, 1986

Paras, J.

Facts:

NPC entered into a contract with FFEI for the erection of the Angat Balintawak 115- KW-3-Phase
transmission lines for the Angat Hydroelectric Project. FEEI agreed to complete the work within 120 days from
signing the contract. On the other hand Philamgen issued a surety bond for the faithful compliance of the
undertaking by FEEI. FEEI started construction on December 26, 1962 but on May 30, 1963, both FEEI and
Philamgen wrote NPC requesting the assistance of the latter to complete the project due to unavailability of
the equipment of FEEI. The work was abandoned on June 26, 1963, leaving the construction unfinished.

On July 19, 1963, in a joint letter, Philamgen and FEEI informed NPC that FEEI was giving up the
construction due to financial difficulties. On the same date, NPC wrote Philamgen informing it of the
withdrawal of FEEI from the work and formally holding both FEEI and Philamgen liable for the cost of the
work to be completed as of July 20, 1962 plus damages. The work was completed by NPC.

On January 30, 1967 NPC notified Philamgen that FEEI had an outstanding obligation in the amount
of P75,019.85, exclusive of interest and damages, and demanded the remittance of the amount of the surety
bond the answer for the cost of completion of the work. In reply, Philamgen requested for a detailed statement
of account, but after receipt of the same, Philamgen did not pay as demanded but contended instead that its
liability under the bond has expired on September 20, 1964 and claimed that no notice of any obligation of the
surety was made within 30 days after its expiration.

Issue:

Is the surety liable under the bond?

Ruling:

Yes. The surety bond must be read in its entirety and together with the contract between NPC and the
contractors. The provisions must be construed together to arrive at their true meaning. Certain stipulations
cannot be segregated and then made to control.

Furthermore, it is well settled that contracts of insurance are to be construed liberally in favor of the
insured and strictly against the insurer. Thus ambiguity in the words of an insurance contract should be
interpreted in favor of its beneficiary. It cannot be denied that the breach of contract in this case, that is, the
abandonment of the unfinished work of the transmission line of the petitioner by the contractor Far Eastern
Electric, Inc. was within the effective date of the contract and the surety bond. Such abandonment gave rise to
the continuing liability of the bond as provided for in the contract which is deemed incorporated in the surety
bond executed for its completion. To rule therefore that private respondent was not properly notified would
be gross error.

21 4
Su r et ysh ip

PHILIPPINE PRYCE ASSURANCE v. COURT OF APPEALS


G.R. No. 107062
February 21, 1994

Nocon, J.

Facts:

Petitioner, Interworld Assurance Corporation, was the butt of the complaint for collection of sum of
money, filed on May 13, 1988 by respondent, Gegroco, Inc. before the Makati Regional Trial Court, Branch 138.
The complaint alleged that petitioner issued two surety bonds in behalf of its principal Sagum General
Merchandise for Php. 500,000.00 and Php. 1,000,000.00, respectively.

In its Answer, dated July 29, 1988, but filed only on August 4, 1988, petitioner admitted having
executed the said bonds, but denied liability because allegedly 1) the checks which were to pay for the
premiums bounced and were dishonored hence there is no contract to speak of between petitioner and its
supposed principal; and 2) that the bonds were merely to guarantee payment of its principal's obligation, thus,
excussion is necessary.

Petitioner or its officer or its counsel, despite being duly notified did not appear in Court. Thus, the
petitioner was declared in default. The Trial Court rendered a decision in favour of respond which was later
upheld by the appellate court.

Issue:

Is Philippine Pryce Assurance Corporation authorized to issue a surety bond?

Ruling:

Petitioner hinges its defense on two arguments, namely: a) that the checks issued by its principal
which were supposed to pay for the premiums, bounced, hence there is no contract of surety to speak of; and
2) that as early as 1986 and covering the time of the Surety Bond, Interworld Assurance Company was not yet
authorized by the insurance Commission to issue such bonds.

Petitioner hinges its defense on two arguments, namely: a) that the checks issued by its principal
which were supposed to pay for the premiums, bounced, hence there is no contract of surety to speak of; and
2) that as early as 1986 and covering the time of the Surety Bond, Interworld Assurance Company (now Phil.
Pryce) was not yet authorized by the insurance Commission to issue such bonds. Secondly, the testimony of
Mr. Leonardo T. Guzman, witness for the respondent, reveals that what were issued to respondent were
surety bonds.

21 5
Su r et ysh ip

PRUDENTIAL GUARANTEE AND ASSURANCE v. EQUINOX LAND CORP.


G.R. Nos. 152505-06
September 13, 2007

Sandoval-Gutierrez, J.

Facts:

Equinox Land Corporation, decided to construct five additional floors to its existing building, the
Eastgate Centre, located at 169 EDSA, Mandaluyong City. It then sent invitations to bid to various building
contractors. Four building contractors, including J’Marc Construction & Development Corporation, responded.
Finding the bid of J’Marc to be the most advantageous, Equinox offered the construction project to it. On
February 22, 1997, J’Marc accepted the offer. Two days later, Equinox formally awarded to J’Marc the contract
to build the extension for a consideration of P37,000,000.00.

On February 24, 1997, J’Marc submitted to Equinox two (2) bonds, namely: (1) a surety bond issued
by Prudential Guarantee and Assurance, Inc. (Prudential), herein petitioner, in the amount of P9,250,000.00
to guarantee the unliquidated portion of the advance payment payable to J’Marc; and (2) a performance bond
likewise issued by Prudential in the amount of P7,400,000.00 to guarantee J’Marc’s faithful performance of its
obligations under the construction agreement. On March 17, 1997, Equinox and J’Marc signed the contract and
related documents. Under the terms of the contract, J’Marc would supply all the labor, materials, tools,
equipment, and supervision required to complete the project.

In accordance with the terms of the contract, Equinox paid J’Marc a downpayment of P9,250,000.00
equivalent to 25% of the contract price. J’Marc did not adhere to the terms of the contract. It failed to finish the
required work, only 19% of the total project was finished.

Issue:

Is Prudential free from liability claiming that only J’Marc is solidarily liable?

Ruling:

NO. It is not disputed that Prudential entered into a suretyship contract with J’Marc. Section 175 of the
Insurance Code defines a suretyship as "a contract or agreement whereby a party, called the suretyship,
guarantees the performance by another party, called the principal or obligor, of an obligation or undertaking
in favor of a third party, called the obligee. It includes official recognizances, stipulations, bonds, or
undertakings issued under Act 5368, as amended." Corollarily, Article 2047 of the Civil Code provides that
suretyship arises upon the solidary binding of a person deemed the surety with the principal debtor for the
purpose of fulfilling an obligation.

21 6
Su r et ysh ip

AFP GENERAL INSURANCE v. MOLINA


G.R. No. 151133
June 30, 2008

Quisumbing, J.

Facts:

Radon Security appealed the Labor Arbiter’s decision to public respondent NLRC and posted a
supersedeas bond, issued by herein petitioner AFPGIC as surety. On April 6, 1998, the NLRC affirmed with
modification the decision of the Labor Arbiter. The NLRC found the herein private respondents constructively
dismissed and ordered Radon Security to pay them their separation pay, in lieu of reinstatement with
backwages, as well as their monetary benefits limited to three years, plus attorney’s fees equivalent to 10% of
the entire amount, with Radon Security and Ever Emporium, Inc. adjudged jointly and severally liable.

On February 5, 1999, the Labor Arbiter issued a Writ of Execution incorporating the computation of
the NLRC Research and Information Unit. That same date, the Labor Arbiter dismissed the Motion for
Recomputation filed by Radon Security. By virtue of the writ of execution, the NLRC Sheriff issued a Notice of
Garnishment against the supersedeas bond.

Issue:

Is the subject appeal bond already been cancelled for non-payment of premiums and thus it could not
be subject of execution or garnishment?

Ruling:

The instant case pertains to a surety bond; thus, the applicable provision of the Insurance Code is
Section 177, which specifically governs suretyship. It provides that a surety bond, once accepted by the obligee
becomes valid and enforceable, irrespective of whether or not the premium has been paid by the obligor. The
private respondents, the obligees here, accepted the bond posted by Radon Security and issued by the
petitioner. Hence, the bond is both valid and enforceable.

When petitioner surety company cancelled the surety bond because Radon Security failed to pay the
premiums, it gave due notice to the latter but not to the NLRC. By its failure to give notice to the NLRC, AFPGIC
failed to acknowledge that the NLRC had jurisdiction not only over the appealed case, but also over the appeal
bond. This oversight amounts to disrespect and contempt for a quasi-judicial agency tasked by law with
resolving labor disputes. Until the surety is formally discharged, it remains subject to the jurisdiction of the
NLRC.

21 7
Su r et ysh ip

INTRA-STRATA ASSURANCE CORPORATION v. REPUBLIC OF THE PHILIPPINES


G.R. No. 156571
July 9, 2008

Brion, J.

Facts:

Grand Textile is a local manufacturing corporation. In 1974, it imported from different countries
various articles such as dyestuffs, spare parts for textile machinery, polyester filament yarn, textile auxiliary
chemicals, trans open type reciprocating compressor, and trevira filament. Subsequent to the importation,
these articles were transferred to Customs Bonded Warehouse No. 462. As computed by the Bureau of
Customs, the customs duties, internal revenue taxes, and other charges due on the importations amounted to
₱2,363,147.00. To secure the payment of these obligations pursuant to Section 1904 of the Tariff and Customs
Code, Intra-Strata and PhilHome each issued general warehousing bonds in favor of the Bureau of Customs.
These bonds, the terms of which are fully quoted below, commonly provide that the goods shall be withdrawn
from the bonded warehouse "on payment of the legal customs duties, internal revenue, and other charges to
which they shall then be subject."

Without payment of the taxes, customs duties, and charges due and for purposes of domestic
consumption, Grand Textile withdrew the imported goods from storage. The Bureau of Customs demanded
payment of the amounts due from Grand Textile as importer, and from Intra-Strata and PhilHome as sureties.
All three failed to pay. The government responded on January 14, 1983 by filing a collection suit against the
parties with the RTC of Manila.

Issue:

What is the nature of the surety’s obligation?

Ruling:

By its very nature under the terms of the laws regulating suretyship, the liability of the surety is joint
and several but limited to the amount of the bond, and its terms are determined strictly by the terms of the
contract of suretyship in relation to the principal contract between the obligor and the obligee. The definition
and characteristics of a suretyship bring into focus the fact that a surety agreement is an accessory contract
that introduces a third party element in the fulfillment of the principal obligation that an obligor owes an
obligee. In short, there are effectively two contracts involved when a surety agreement comes into play – a
principal contract and an accessory contract of suretyship.

Under the accessory contract, the surety becomes directly, primarily, and equally bound with the
principal as the original promissor although he possesses no direct or personal interest over the latter’s
obligations and does not receive any benefit therefrom.

21 8
Su r et ysh ip

FIRST LEPANTO-TAISHO INSURANCE CORPORATION v. CHEVRON PHILIPPINES, INC.


G.R. No. 177839
January 18, 2012

Villarama, Jr., J.

Facts:

Respondent Chevron Philippines, Inc., formerly Caltex Philippines, Inc., sued petitioner First
Lepanto-Taisho Insurance Corporation (now known as FLT Prime Insurance Corporation) for the
payment of unpaid oil and petroleum purchases made by its distributor Fumitechniks Corporation
(Fumitechniks). Fumitechniks, represented by Ma. Lourdes Apostol, had applied for and was issued
Surety Bond FLTICG (16) No. 01012 by petitioner for the amount of ₱15,700,000.00. As stated in the
attached rider, the bond was in compliance with the requirement for the grant of a credit line with
the respondent "to guarantee payment/remittance of the cost of fuel products withdrawn within the
stipulated time in accordance with the terms and conditions of the agreement." The surety bond was
executed on October 15, 2001 and will expire on October 15, 2002. Fumitechniks defaulted on its
obligation. The check dated December 14, 2001 it issued to respondent in the amount of
₱11,461,773.10, when presented for payment, was dishonored for reason of "Account Closed."

On April 9, 2002, respondent formally demanded from petitioner the payment of its claim
under the surety bond. However, petitioner reiterated its position that without the basic contract
subject of the bond, it cannot act on respondent’s claim; petitioner also contested the amount of
Fumitechniks’ supposed obligation. After trial, the RTC rendered judgment dismissing the complaint
as well as petitioner’s counterclaim. Said court found that the terms and conditions of the oral credit
line agreement between respondent and Fumitechniks have not been relayed to petitioner and
neither were the same conveyed even during trial. Since the surety bond is a mere accessory contract,
the RTC concluded that the bond cannot stand in the absence of the written agreement secured
thereby. In holding that petitioner cannot be held liable under the bond it issued to Fumitechniks, the
RTC noted the practice of petitioner, as testified on by its witnesses, to attach a copy of the written
agreement (principal contract) whenever it issues a surety bond, or to be submitted later if not yet in
the possession of the assured, and in case of failure to submit the said written agreement, the surety
contract will not be binding despite payment of the premium.
The CA ruled in favor of respondent. According to the appellate court, petitioner cannot insist on the
submission of a written agreement to be attached to the surety bond considering that respondent
was not aware of such requirement and unwritten company policy. It also declared that petitioner is
estopped from assailing the oral credit line agreement, having consented to the same upon
presentation by Fumitechniks of the surety bond it issued. Considering that such oral contract
between Fumitechniks and respondent has been partially executed, the CA ruled that the provisions
of the Statute of Frauds do not apply.

Issue:

Whether a surety is liable to the creditor in the absence of a written contract with the
principal.

Ruling:

Section 175 of the Insurance Code defines a suretyship as a contract or agreement whereby a
party, called the surety, guarantees the performance by another party, called the principal or obligor,

21 9
Su r et ysh ip

of an obligation or undertaking in favor of a third party, called the obligee. It includes official
recognizances, stipulations, bonds or undertakings issued under Act 536, as amended. Suretyship
arises upon the solidary binding of a person – deemed the surety – with the principal debtor, for the
purpose of fulfilling an obligation. Such undertaking makes a surety agreement an ancillary contract
as it presupposes the existence of a principal contract. Although the contract of a surety is in essence
secondary only to a valid principal obligation, the surety becomes liable for the debt or duty of
another although it possesses no direct or personal interest over the obligations nor does it receive
any benefit therefrom. And notwithstanding the fact that the surety contract is secondary to the
principal obligation, the surety assumes liability as a regular party to the undertaking.

The extent of a surety’s liability is determined by the language of the suretyship contract or
bond itself. It cannot be extended by implication, beyond the terms of the contract. A reading of Surety
Bond FLTICG (16) No. 01012 shows that it secures the payment of purchases on credit by
Fumitechniks in accordance with the terms and conditions of the "agreement" it entered into with
respondent. The word "agreement" has reference to the distributorship agreement, the principal
contract and by implication included the credit agreement mentioned in the rider. However, it turned
out that respondent has executed written agreements only with its direct customers but not
distributors like Fumitechniks and it also never relayed the terms and conditions of its
distributorship agreement to the petitioner after the delivery of the bond. This was clearly admitted
by respondent’s Marketing Coordinator, Alden Casas Fajardo.

A surety contract is merely collateral one; its basis is the principal contract or undertaking
which it secures. Necessarily, the stipulations in such principal agreement must at least be
communicated or made known to the surety particularly in this case where the bond expressly
guarantees the payment of respondent’s fuel products withdrawn by Fumitechniks in accordance
with the terms and conditions of their agreement. The bond specifically makes reference to a written
agreement. It is basic that if the terms of a contract are clear and leave no doubt upon the intention
of the contracting parties, the literal meaning of its stipulations shall control. Moreover, being an
onerous undertaking, a surety agreement is strictly construed against the creditor, and every doubt
is resolved in favor of the solidary debtor. Having accepted the bond, respondent as creditor must be
held bound by the recital in the surety bond that the terms and conditions of its distributorship
contract be reduced in writing or at the very least communicated in writing to the surety. Such
noncompliance by the creditor (respondent) impacts not on the validity or legality of the surety
contract but on the creditor’s right to demand performance.

It bears stressing that the contract of suretyship imports entire good faith and confidence
between the parties in regard to the whole transaction, although it has been said that the creditor
does not stand as a fiduciary in his relation to the surety. The creditor is generally held bound to a
faithful observance of the rights of the surety and to the performance of every duty necessary for the
protection of those rights. Moreover, in this jurisdiction, obligations arising from contracts have the
force of law between the parties and should be complied with in good faith. Respondent is charged
with notice of the specified form of the agreement or at least the disclosure of basic terms and
conditions of its distributorship and credit agreements with its client Fumitechniks after its
acceptance of the bond delivered by the latter. However, it never made any effort to relay those terms
and conditions of its contract with Fumitechniks upon the commencement of its transactions with
said client, which obligations are covered by the surety bond issued by petitioner. Contrary to
respondent’s assertion, there is no indication in the records that petitioner had actual knowledge of
its alleged business practice of not having written contracts with distributors; and even assuming
petitioner was aware of such practice, the bond issued to Fumitechniks and accepted by respondent
specifically referred to a "written agreement."

22 0
Com p r eh e ns iv e M ot or V ehi c l e L ia bil ity Ins u ra nc e

PERLA COMPANIA DE SEGUROS v. HONORABLE ANCHETA


G.R. No. L-49699
August 8, 1988

Cortes, J.

Facts:

On December 27, 1977, in a collision between the IH Scout in which private respondents were
riding and a Superlines bus along the national highway in Sta. Elena, Camarines Norte, private
respondents sustained physics injuries in varying degrees of gravity. Thus, they filed with the Court
of First Instance of Camarines Norte on February 23,1978 a complaint for damages against
Superlines, the bus driver and petitioner, the insurer of the bus [Rollo, pp. 27-39.] The bus was
insured with petitioner for the amount of P50,000.00 as and for passenger liability and P50,000.00
as and for third party liability. The vehicle in which private respondents were riding was insured with
Malayan Insurance Co.

Petitioner denied in its Answer its alleged liability under the "no fault indemnity" provision
[Rollo, p. 44] and likewise moved for the reconsideration of the order. Petitioner held the position
that under Sec. 378 of the Insurance Code, the insurer liable to pay the P5,000.00 is the insurer of the
vehicle in which private respondents were riding, not petitioner, as the provision states that "[i]n the
case of an occupant of a vehicle, claim shall lie against the insurer of the vehicle in which the occupant
is riding, mounting or dismounting from." Respondent judge, however, denied reconsideration. A
second motion for reconsideration was filed by petitioner. However, in an order dated January 3,
1979, respondent judge denied the second motion for reconsideration and ordered the issuance of a
writ of execution [Rollo, p. 69.] Hence, the instant petition praying principally for the annulment and
setting aside of respondent judge's orders dated March 1, 1978 and January 3, 1979.

Issue:

Whether or not petitioner is the insurer liable to indemnify private respondents under Sec.
378 of the Insurance Code.

Ruling:

From a reading of the provision, which is couched in straight-forward and unambiguous


language, the following rules on claims under the "no fault indemnity" provision, where proof of fault
or negligence is not necessary for payment of any claim for death or injury to a passenger or a third
party, are established:
1. A claim may be made against one motor vehicle only.
2. If the victim is an occupant of a vehicle, the claim shall lie against the insurer of the vehicle. in
which he is riding, mounting or dismounting from.
3. In any other case (i.e. if the victim is not an occupant of a vehicle), the claim shall lie against
the insurer of the directly offending vehicle.
4. In all cases, the right of the party paying the claim to recover against the owner of the vehicle
responsible for the accident shall be maintained.

The law is very clear — the claim shall lie against the insurer of the vehicle in which the
"occupant" ** is riding, and no other. The claimant is not free to choose from which insurer he will
claim the "no fault indemnity," as the law, by using the word "shall, makes it mandatory that the claim

22 1
Com p r eh e ns iv e M ot or V ehi c l e L ia bil ity Ins u ra nc e

be made against the insurer of the vehicle in which the occupant is riding, mounting or dismounting
from.

That said vehicle might not be the one that caused the accident is of no moment since the law
itself provides that the party paying the claim under Sec. 378 may recover against the owner of the
vehicle responsible for the accident. This is precisely the essence of "no fault indemnity" insurance
which was introduced to and made part of our laws in order to provide victims of vehicular accidents
or their heir’s immediate compensation, although in a limited amount, pending final determination
of who is responsible for the accident and liable for the victims ‘injuries or death. In turn, the "no fault
indemnity" provision is part and parcel of the Insurance Code provisions on compulsory motor
vehicle ability insurance [Sec. 373-389] and should be read together with the requirement for
compulsory passenger and/or third party liability insurance [Sec. 377] which was mandated in order
to ensure ready compensation for victims of vehicular accidents.

Irrespective of whether or not fault or negligence lies with the driver of the Superlines bus, as
private respondents were not occupants of the bus, they cannot claim the "no fault indemnity"
provided in Sec. 378 from petitioner. The claim should be made against the insurer of the vehicle they
were riding. This is very clear from the law. Undoubtedly, in ordering petitioner to pay private
respondents the 'no fault indemnity,' respondent judge gravely abused his discretion in a manner that
amounts to lack of jurisdiction. The issuance of the corrective writ of certiorari is therefore
warranted.

22 2
Com p r eh e ns iv e M ot or V ehi c l e L ia bil ity Ins u ra nc e

FAR EASTERN SURETY v. MISA


G.R. No. L-24377
October 26, 1968

Reyes, J.B.L., J

Facts:

The record discloses that on 3 September 1957 the two respondents, Socorro Dancel Vda. de
Misa and Araceli Pinto, hired a taxicab operated by respondent La Mallorca in Quezon City. While
proceeding south toward the Archbishop's Palace in Shaw Boulevard, the taxicab collided with a
gravel and sand truck, driven by one Faustino Nabor, who was proceeding in the opposite direction.
As a result, the two passengers of the La Mallorca taxicab were injured, and filed suit for damages
against the taxicab company in the Court of First Instance. The operator denied liability, but instituted
a third party complaint against herein appellant, Far Eastern Surety and Insurance Company, to
recoup from the latter, based on its Common Carrier's Accident Insurance No. CCA 106, any damages
that might be recovered by the plaintiff’s taxicab passengers. The insurer, likewise, denied
responsibility.

After trial, the Court of First Instance of Quezon City awarded to plaintiffs Vda. de Misa and
Pinto (now respondents) actual, moral, and exemplary damages and attorney's fees, payable by the
taxicab operator, La Mallorca; and sentenced the insurance company to pay to La Mallorca P10,000.00
on its third party liability insurance.

On appeal, the Court of Appeals, while holding that the collision was due to the fault of the
driver of the sand truck nevertheless held the taxicab operator liable in damages to the passengers of
its motor vehicle on the strength of its representation that the passengers were insured against
accidents, as shown by the sticker affixed to the taxicab; and, overruling the defense of the insurance
company that it was not answerable except for whatever amounts the insured might be legally liable
for in the event of accident caused by, or arising out of, the use of the motor vehicle, the appellate
court adjudged the said insurer answerable to La Mallorca in view of its third party liability insurance
contract.

Issue:

Is Far Eastern Surety not liable under the insurance contract because the assured La Mallorca
is not "legally liable" to the plaintiffs-appellees?

Ruling:

The Court agrees with the appellant that the decision of the Court of Appeals on this point is
not legally tenable, for the reason that the policy of insurance limited the recovery of the insured to
"all sums including claimant's" (passengers in this case) "cost and expenses which the Insured shall
become legally liable" in the "event of accident caused by or arising out of the use of the Motor
Vehicle;" and the appealed decision itself shows that the indemnity awarded to the passengers of the
La Mallorca taxicab was not because of the accident but was exclusively predicated on the
representation made by the taxicab company to its passengers that the latter were insured against
accidents.

22 3
Com p r eh e ns iv e M ot or V ehi c l e L ia bil ity Ins u ra nc e

While the decision correctly held that la Mallorca was in estoppel, and could not be heard to
deny that its passengers were insured, it does not necessarily follow that the estoppel, likewise,
applied to the appellant insurer. The Court of Appeals concurred in the finding of the trial court that
only the negligence of the driver of the sand and gravel truck was the causative factor of the mishap,
and made no pronouncement that the driver of the taxicab in any way contributed thereto; so that,
had it not been for its representation that its passengers were insured, the taxicab company would
not have been liable at all. As it does not appear that the insurance company authorized or consented
to, or even knew of, the representation made by the taxicab company to its passengers, it follows that
the source of the award of damages against the taxicab company was beyond, or outside of, the
contemplation of the parties to the contract of Accident Insurance No. CCA 106, and that the insurer
may not be held liable for such damages.

22 4
Com p r eh e ns iv e M ot or V ehi c l e L ia bil ity Ins u ra nc e

SHAFER v. JUDGE
G.R. No. 78848
November 14, 1988

Padilla, J.

Facts:

On January 2, 1985, petitioner Sherman Shafer obtained a private car policy over his Ford
Laser car. During the effectivity of the policy, information for reckless imprudence resulting in
damage to property and serious physical injuries was filed against petitioner. As a result thereof one
Jovencio Poblete, Sr. who was on board of the said Volkswagen car sustained physical injuries.

The owner of the damaged Volkswagen car filed a separate civil action against petitioner for
damages, while Jovencio Poblete, Sr., who was a passenger in the Volkswagen car when allegedly hit
and bumped by the car driven by petitioner, did not reserve his right to file a separate civil action for
damages. Upon motion, petitioner was granted leave by the former presiding judge of the trial court
to file a third party complaint against the herein private respondent, Makati Insurance Company, Inc.
Said insurance company, however, moved to vacate the order granting leave to petitioner to file a third
party complaint against it and/or to dismiss the same.

On 24 April 1987, the court a quo issued an order dismissing the third party complaint on the
ground that it was premature. The court further stated that the better procedure is for the accused
(petitioner) to wait for the outcome of the criminal aspect of the case to determine whether or not
the accused, also the third party plaintiff, has a cause of action against the third party defendant for
the enforcement of its third party liability (TPL) under the insurance contract. It is the contention of
herein petitioner that the dismissal of the third party complaint amounts to a denial or curtailment
of his right to defend himself in the civil aspect of the case.

Issue:

Can the accused in a criminal action for reckless imprudence, where the civil action is jointly
prosecuted, legally implead the insurance company as third party defendant under its private car
insurance policy?

Ruling:

The Supreme court ruled that a third party complaint is a device allowed by the rules of
procedure by which the defendant can bring into the original suit a party against whom he will have
a claim for indemnity or remuneration as a result of a liability established against him in the original
suit.

Compulsory Motor Vehicle Liability Insurance (third party liability or TPL) is primarily
intended to provide compensation for the death or bodily injuries suffered by innocent third parties
or passengers as a result of a negligent operation and use of motor vehicles. The victims and/or their
dependents are assured of immediate financial assistance, regardless of the financial capacity of
motor vehicle owners. Further, the liability of the insurance company under the Compulsory Motor
Vehicle Liability Insurance is for loss or damage. Where an insurance policy insures directly against
liability, the insurer's liability accrues immediately upon the occurrence of the injury or event upon

22 5
Com p r eh e ns iv e M ot or V ehi c l e L ia bil ity Ins u ra nc e

which the liability depends, and does not depend on the recovery of judgment by the injured party
against the insured.

The injured for whom the contract of insurance is intended can sue directly the insurer. The
general purpose of statutes enabling an injured person to proceed directly against the insurer is to
protect injured persons against the insolvency of the insured who causes such injury, and to give such
injured person a certain beneficial interest in the proceeds of the policy, and statutes are to be
liberally construed so that their intended purpose may be accomplished. It has even been held that
such a provision creates a contractual relation which inures to the benefit of any and every person
who may be negligently injured by the named insured as if such injured person were specifically
named in the policy. There is no need on the part of the insured to wait for the decision of the trial
court finding him guilty of reckless imprudence. The occurrence of the injury to the third party
immediately gave rise to the liability of the insurer under its policy.

22 6
Com p r eh e ns iv e M ot or V ehi c l e L ia bil ity Ins u ra nc e

PEZA v. ALIKPALA
G.R. No. L-29749
April 15, 1988

Narvasa, J.

Facts:

Two (2) children ran across the path of a vehicle as it was running along the national highway
at barrio Makiling Calamba, Laguna. They were killed. The vehicle, a Chevrolet "Carry-All", belonged
to a partnership known as Diman & Company, and was then being driven by its driver, Perfecto Amar.
It was insured with the Empire Insurance Co., Inc. under a so-called 'comprehensive coverage" policy,
loss by theft excluded. The policy was in force at the time of the accident.

Placida Peza, the managing partner of Diman & Co. filed a claim with the insurance company,
hereafter simply, Empire, for payment of compensation to the family of the two (2) children who died
as a result of the accident. Empire refused to pay on the ground that the driver had no authority to
operate the vehicle, a fact which expressly excepted it from liability under the policy. What Peza did
was to negotiate directly with the deceased children father for an out-of-court settlement. The father
agreed to accept P 6,200.00 in fun settlement of the liability of the vehicles owner and driver, and
Peza paid him this sum.

Peza thereafter sued Empire to recover this sum of P6,200.00 as actual damages, as well as
P20,000.00 as moral damages, P10,000.00 as exemplary damages, and P10,000.00 as attorney's fees.
She amended her complaint shortly thereafter to include Diman & Co. as alternative party plaintiff.

Empire's basic defense to the suit was anchored on the explicit requirement in the policy
limiting the operation of the insured vehicle to the "authorized driver" therein defined, namely, (a)
the insured, or (b) any person driving on the insured order or with his permission, provided that-
... that the person driving is permitted in accordance with the licensing or other laws
or regulations to drive the Motor vehicle or has been so permitted and is not
disqualified by order of the Court of Law of by reason of any enactment or regulation
in that behalf from driving such Motor Vehicle.-

It appearing, according to Empire, that at the time of the mishap, the driver Perfecto Amar
only had a temporary operator's permit (TVR) — already expired — his driver’s license having earlier
been confiscated by an agent of the Land Transportation Commission for an alleged violation of Land
Transportation and Traffic Rules, he was not permitted by law and was in truth disqualified to operate
any motor vehicle; and this operated to relieve it (Empire) from liability under its policy.

Issue:

Whether Empire Insurance Co. is liable?

Ruling:

NO. It would seem fairly obvious that whether the LTC agent was correct or not in his opinion
that driver Amar had violated some traffic regulation warranting confiscation of his license and
issuance of a TVR in lieu thereof, this would not alter the undisputed fact that Amar's license had
indeed been confiscated and a TVR issued to him, and the TVR had already expired at the time that

22 7
Com p r eh e ns iv e M ot or V ehi c l e L ia bil ity Ins u ra nc e

the vehicle being operated by him killed two children by accident. Neither would proof of the renewal
of Amar's license change the fact that it had really been earlier confiscated by the LTC agent. The
plaintiffs' proffered proof therefore had no logical connection with the facts thereby sought to be
refuted, the proof had no rational tendency to establish the improbability of the facts demonstrated
by Empire's evidence. The proofs were thus correctly by the respondent Judge as being irrelevant.

Even positing error in the Judge's analysis of the evidence attempted to be introduced and his
rejection thereof, it is clear that it was at most an error of judgment, not such an error as may be
branded a grave abuse of discretion, i.e., such capricious and whimsical exercise of judgment as is
equivalent to lack of jurisdiction, against which the writ of certiorari will lie. In any event, the
established principle is "that ruling of the trial court on procedural questions and on admissibility of
evidence during the course of the trial are interlocutory in nature and may not be the subject of
separate appeal or review on certiorari, but are to be assigned as errors and reviewed in the appeal
properly taken from the decision rendered by the trial court on the merits of the case.

In the meantime, respondent Judge Alikpala rendered judgment on the merits, since the case
was then already ripe for adjudication. The judgment ordered dismissal of the case for failure on the
part of the plaintiff to prove their cause of action against Empire. Notice of the judgment was served
on the parties in due course. The plaintiffs did not appeal. Instead, they filed a motion praying that
Judge Alikpala be declared guilty of contempt of court for having decided the case on the merits
despite the pendency in this Court of the certiorari action instituted by the plaintiffs.

22 8
Com p r eh e ns iv e M ot or V ehi c l e L ia bil ity Ins u ra nc e

WESTERN GUARANTY v. CA
G.R. No. 91666
July 20, 1990

Facts:

Afternoon of March 27, 1982, while crossing Airport Road on a pedestrian lane on her way to
work, respondent Priscilla E. Rodriguez was struck by a De Dios passenger bus owned by respondent
De Dios Transportation Co., Inc., then driven by one Walter Saga y Aspero. The bus driver disregarded
the stop signal given by a traffic policeman to allow pedestrians to cross the road. Priscilla was thrown
to the ground, hitting her forehead. She was treated at the Protacio Emergency Hospital and later on
hospitalized at the San Juan De Dios Hospital. Her face was permanently disfigured, causing her
serious anxiety and moral distress. Respondent bus company was insured with petitioner Western
Guaranty Corporation.

Respondent Priscilla Rodriguez filed a complaint for damages before the Regional Trial Court
of Makati against De Dios Transportation Co. and Walter A. Saga Respondent De Dios Transportation
Co., in turn, filed a third-party complaint against its insurance carrier, petitioner Western. The trial
court ruled in favor of Priscilla Rodriguez. On appeal, the Court of Appeals affirmed in toto the
decision of the trial court. Petitioner moved for the reconsideration of the appellate court's decision.

Issue:

Is petitioner liable to pay beyond the limits set forth in the Schedule of Indemnities and in
finding Western liable for loss of earnings, moral damages and attorney's fees?

Ruling:

The Supreme Court ruled that the Schedule of Indemnities does not purport to limit, or to
enumerate exhaustively, the species of bodily injury occurrence of which generate liability for
petitioner Western. A car accident may, for instance, result in injury to internal organs of a passenger
or third party, without any accompanying amputation or loss of an external member (e.g., a foot or an
arm or an eye). But such internal injuries are surely covered by Section I of the Master Policy, since
they certainly constitute bodily injuries. Moreover, the reading urged by Western of the Schedule of
Indemnities comes too close to working fraud upon both the insured and the third party beneficiary
of Section 1. For Western's reading would drastically and without warning limit the otherwise
unlimited and comprehensive scope of liability assumed by the insurer Western under Section 1:
"All sums necessary to discharge liability of the insured in respect of bodily injury to a third
party”.

This result- which is not essentially different from taking away with the left hand what had
been given with the right hand we must avoid as obviously repugnant to public policy.
Petitioner Western would have us construe the Schedule of Indemnities as comprising contractual
limitations of liability which, as already noted, is comprehensively defined in Section 1 — Liability to
the Public" — of the Master Policy. It is well settled, however, that contractual limitations of liability
found in insurance contracts should be regarded by courts with a jaundiced eye and extreme care and
should be so construed as to preclude the insurer from evading compliance with its just obligations.

22 9
Com p r eh e ns iv e M ot or V ehi c l e L ia bil ity Ins u ra nc e

Finally, an insurance contract is a contract of adhesion. The rule is well entrenched in our
jurisprudence that the terms of such contract are to be construed strictly against the party which
prepared the contract, which in this case happens to be petitioner Western.

23 0
Com p r eh e ns iv e M ot or V ehi c l e L ia bil ity Ins u ra nc e

PERLA COMPANIA DE SEGUROS v. CA & LIM


G.R. No. 96452
May 7, 1992

Nocon, J.

Facts:

These are two petitions for review on certiorari, one filed by Perla Compania de Seguros, Inc.
in and the other by FCP Credit Corporation, both seeking to annul and set aside the decision dated
July 30, 1990 of the Court of Appeals. That on December 24, 1981, private respondents spouses
Herminio and Evelyn Lim executed a promissory note in favor Supercars, Inc. in the sum of
P77,940.00, payable in monthly installments according to the schedule of payment indicated in said
note, and secured by a chattel mortgage over a brand new red Ford Laser 1300 5DR Hatchback 1981
model with motor and serial numbers indicated therein, which is registered under the name of
private respondent Herminio Lim and insured with the petitioner Perla Compania de Seguros, Inc.

On the same date, Supercars, Inc., with notice to private respondent spouses, assigned to
petitioner FCP Credit Corporation (FCP for brevity) its rights, title and interest on said promissory
note and chattel mortgage as shown by the Deed of Assignment. At around 2:30 P.M. of November 9,
1982, said vehicle was carnapped while parked at the back of Broadway Centrum along N. Domingo
Street, Quezon City. At around 2:30 P.M. of November 9, 1982, said vehicle was carnapped while
parked at the back of Broadway Centrum along N. Domingo Street, Quezon City. Private respondent
Evelyn Lim, who was driving said car before it was carnapped, immediately called up the Anti-
Carnapping Unit of the Philippine Constabulary to report said incident and thereafter.

On November 11, 1982, private respondent filed a claim for loss with the petitioner Perla but
said claim was denied on November 18, 1982 on the ground that Evelyn Lim, who was using the
vehicle before it was carnapped, was in possession of an expired driver's license at the time of the
loss of said vehicle which is in violation of the authorized driver clause of the insurance policy.

On November 17, 1982, private respondent’s requests from petitioner FCP for a suspension
of payment on the monthly amortization agreed upon due to the loss of the vehicle and, since the
carnapped vehicle insured with petitioner Perla, said insurance company should be made to pay the
remaining balance of the promissory note and the chattel mortgage contract. Perla, however, denied
private respondents' claim. Consequently, petitioner FCP demanded that private respondents pay the
whole balance of the promissory note or to return the vehicle but the latter refused.

Petitioner FCP filed a complaint against private respondents, who in turn filed an amended
third party complaint against petitioner Perla on December 8, 1983. After petitioners' separate
motions for reconsideration were denied by the Court of Appeals in its resolution of December 10,
1990, petitioners filed these separate petitions for review on certiorari.

Issue:

Did the loss of the collateral exempt the debtor from his admitted obligations under the
promissory note particularly the payment of interest, litigation expenses and attorney's fees?

Ruling:

23 1
Com p r eh e ns iv e M ot or V ehi c l e L ia bil ity Ins u ra nc e

The court ruled that the terms of the contract constitute the measure of the insurer's liability
and compliance therewith is a condition precedent to the insured's right of recovery from the insurer.

In the case at bar, the insurance policy clearly and categorically placed petitioner's liability
for all damages arising out of death or bodily injury sustained by one person as a result of any one
accident at P12,000.00. Said amount complied with the minimum fixed by the law then prevailing,
Section 377 of Presidential Decree No. 612 (which was retained by P.D. No. 1460, the Insurance Code
of 1978), which provided that the liability of land transportation vehicle operators for bodily injuries
sustained by a passenger arising out of the use of their vehicles shall not be less than P12,000.

In like manner, we rule as valid and binding upon private respondent the condition above-
quoted requiring her to secure the written permission of petitioner before effecting any payment in
settlement of any claim against her. There is nothing unreasonable, arbitrary or objectionable in this
stipulation as would warrant its nullification. The same was obviously designed to safeguard the
insurer's interest against collusion between the insured and the claimants.

23 2
Com p r eh e ns iv e M ot or V ehi c l e L ia bil ity Ins u ra nc e

VDA. DE MAGLANA v. CONSOLACION


G.R. No. 60506
August 6, 1992

Romero, J.

Facts:

Lope Maglana was an employee of the Bureau of Customs stationed in Davao city. On
December 20, 1978, early morning, Lope Maglana was on his way to his work station, driving a
motorcycle owned by the Bureau of Customs. He met an accident that resulted in his death. He died
on the spot. The PUJ jeep that bumped the deceased was driven by Pepito Into, operated and owned
by defendant Destrajo.

Consequently, the heirs of Lope Maglana, Sr., here petitioners, filed an action for damages and
attorney's fees against operator Patricio Destrajo and the Afisco Insurance Corporation (AFISCO for
brevity) before the then Court of First Instance of Davao, Branch II. The lower court rendered a
decision in favor of the heirs of the deceased.

Petitioners filed a motion for the reconsideration of the second paragraph of the dispositive
portion of the decision contending that AFISCO should not merely be held secondarily liable because
the Insurance Code provides that the insurer's liability is "direct and primary and/or jointly and
severally with the operator of the vehicle, although only up to the extent of the insurance coverage."

In its comment on the motion for reconsideration, AFISCO argued that since the Insurance
Code does not expressly provide for a solidary obligation, the presumption is that the obligation is
joint.

Issue:

Is the insurance company directly and solidarily liable with the negligent operator up to the
extent of its insurance coverage?

Ruling:

The Supreme Court ruled in the affirmative. The AFISCO can be held directly liable by
petitioners. Under a jurisprudential tenet the court ruled that “where an insurance policy insures
directly against liability, the insurer's liability accrues immediately upon the occurrence of the injury
or even upon which the liability depends, and does not depend on the recovery of judgment by the
injured party against the insured”. Further, the underlying reason behind the third party liability
(TPL) of the Compulsory Motor Vehicle Liability Insurance is "to protect injured persons against the
insolvency of the insured who causes such injury, and to give such injured person a certain beneficial
interest in the proceeds of the policy." Since petitioners had received from AFISCO the sum of P5,
000.00 under the no fault clause, AFISCO's liability is now limited to P15, 000.00. However, we cannot
agree that AFISCO is likewise solidarily liable with Destrajo. While it is true that where the insurance
contract provides for indemnity against liability to third persons, such third persons can directly sue
the insurer, however, the direct liability of the insurer under indemnity contracts against third party
liability does not mean that the insurer can be held solidarily liable with the insured and/or the other
parties found at fault. The liability of the insurer is based on contract; that of the insured is based on
tort.

23 3
Com p r eh e ns iv e M ot or V ehi c l e L ia bil ity Ins u ra nc e

The Court then proceeded to distinguish the extent of the liability and manner of enforcing
the same in ordinary contracts from that of insurance contracts. While in solidary obligations, the
creditor may enforce the entire obligation against one of the solidary debtors, in an insurance
contract, the insurer undertakes for a consideration to indemnify the insured against loss, damage or
liability arising from an unknown or contingent event. 11 Thus, petitioner therein, which, under the
insurance contract is liable only up to P20,000.00, cannot be made solidarily liable with the insured
for the entire obligation of P29,013.00 otherwise there would result "an evident breach of the concept
of solidary obligation."

23 4
Com p r eh e ns iv e M ot or V ehi c l e L ia bil ity Ins u ra nc e

FIRST QUEZON CITY INSURANCE v. CA


G.R. No. 98414
February 8, 1993

Griño-Aquino, J.

Facts:

On June 10, 1984, at about 3:00 p.m., Plaintiff Jose V. del Rosario proceeded to the loading and
unloading zone for public utility bus stop, which was located in front of the Manila International
Airport (MIA) to wait for a passenger bus bound for Quezon City. While at the bus stop, the plaintiff
saw a DMTC bus, per its signboard, was plying the Pasay to Quezon City (passing Espana) route. The
plaintiff now was the last one to board the bus. While the plaintiff was still on the bus' running board
with his hand on the bus door's handle bar, the slowly moving bus sped forward at a high speed, as a
result of which, the plaintiff lost his balance and fell from the bus. As plaintiff clung instinctively to
the handle bar, he was dragged by the bus along the asphalted road for about two (2) seconds. Plaintiff
screamed of pain and anguished even as the other passengers shouted and the bus' driver, Gil Agpalo,
an employee of defendant and third-party plaintiff DMTC, abruptly stopped the bus. Then, Gil
forthwith fled from the scene, leaving the bus and the injured plaintiff behind.

Thereafter, the plaintiff was brought to the Manila Sanitarium and Hospital where he was
given immediate medical treatment at the emergency ward. The doctors performed a major surgical
operation on plaintiff's right leg. This leg was extensively lacerated, its skin and tissues were exposed
and detached from the muscles.

Plaintiff was confined at the hospital for a total period of forty (40) days, from June 10, 1984
to August 26, 1984. After his release, he returned to the hospital from time to time for further
treatment and checkup. The injuries had left plaintiff with a huge, ugly scar running almost the entire
length of his right leg. Also, the plaintiff incurred lost earning by way of unearned salaries amounting
to P7,500.00 due to said physical injuries and the consequent hospital confinement.

Plaintiff filed on June 26, 1985 the aforesaid complaint against DMTC and its driver, Gil
Agpalo. Agpalo was later dropped as a party defendant because he could not be served with
summons. Upon filing its answer on August 20, 1985, defendant DMTC filed a third-party complaint
against First Quezon City Insurance Co. Inc. Sometime on September 17, 1985 this third-party
defendant filed its answer to the third-party complaint.

Issue:

Is the Insurance company liable and up to what extent of liability?

Ruling:

The insurance company clearly passed the maximum limit of the petitioner's liability for
damages arising from death or bodily injury at P12,000.00 per passenger and its maximum liability
per accident at P50,000.00. Since only one passenger was injured in the accident, the insurer's
liability for the damages suffered by said passenger is pegged to the amount of P12,000.00 only. The
bus company may not recover from the insurance company (herein petitioner) more than P
12,000.00 per passenger killed or injured, or fifty thousand (P50,000.00) pesos per accident even if
under the judgment of the court, the erring bus operator will have to pay more than P12,000.00 to

23 5
Com p r eh e ns iv e M ot or V ehi c l e L ia bil ity Ins u ra nc e

each injured passenger. The trial court's interpretation of the insurance contract was the correct
interpretation.

23 6
Com p r eh e ns iv e M ot or V ehi c l e L ia bil ity Ins u ra nc e

FIRST INTEGRATED BONDING AND INSURANCE v. HONORABLE HERNANDO


G.R. No. L-51221
31 July 1991

Medialdea, J.

Facts:

Silverio Blanco was the owner of a passenger jeepney which he insured against liabilities for
death and injuries to third persons with First Integrated Bonding and Insurance Company, Inc. for
P30,000. The said jeepney driven by Blanco himself bumped a five-year old child, Deogracias
Advincula, causing the latter's death. The boy’s parents filed a complaint for damages against Blanco
and First Insurance, which was granted by the lower court. First Insurance filed a petition for
certiorari contending that the victim’s parents have no cause of action against it because they are not
parties to the insurance contract and that they may only proceed against the driver based on the
provisions of the New Civil Code.

Issue:

Can an injured party for whom the insurance contract was intended sue directly the insurer?

Ruling:

Yes. Where the insurance contract provides for indemnity against liability to a third party,
such third party can directly sue the insurer. The liability of the insurer to such third person is based
on contract while the liability of the insured to the third party is based on tort. It cannot evade its
liability as insurer by hiding under the cloak of the insured. Its liability is primary and not dependent
on the recovery of judgment from the insured.

23 7
Com p r eh e ns iv e M ot or V ehi c l e L ia bil ity Ins u ra nc e

PARAMOUNT INSURANCE CORP v. SPOUSES ROMUALDEZ


G.R. No. 173773
28 November 2012

Peralta, J

Facts:

On May 26, 1994, respondents insured with petitioner their 1994 Toyota Corolla sedan under
a comprehensive motor vehicle insurance policy for one year. During the effectivity of said insurance,
respondents’ car was unlawfully taken. Respondents alleged that a certain Ricardo Sales (Sales) took
possession of the subject vehicle to add accessories and improvements thereon, however, Sales failed
to return the subject vehicle within the agreed three-day period. Then, respondents notified
petitioner to claim for the reimbursement of their lost vehicle. However, petitioner refused to pay.
Accordingly, respondents lodged a complaint for a sum of money against petitioner before the
Regional Trial Court of Makati City but dismissed the complaint filed by respondents. Not in
conformity with the trial court’s Order, respondents filed an appeal to the Court of Appeals and in its
decision the appellate court reversed and set aside the Order issued by the trial court. Petitioner,
thereafter, filed a motion for reconsideration against said Decision, but the same was denied by the
appellate court.

Issue:

Is the insurance company liable for the loss of the vehicle?

Ruling:

Yes, Paramount Insurance Corporation is liable under the insurance policy. When one takes
the motor vehicle of another without the latter’s consent even if the motor vehicle is later returned,
there is theft – there being intent to gain as the use of the thing unlawfully taken constitutes gain.
Also, the taking of a vehicle by another person without the permission or authority from the owner
thereof is sufficient to place it within the ambit of the word theft as contemplated in the policy, and is
therefore, compensable. Records would show that respondents entrusted possession of their vehicle
only to the extent that Sales will introduce repairs and improvements thereon, and not to
permanently deprive them of possession thereof. Since, Theft can also be committed through
misappropriation, the fact that Sales failed to return the subject vehicle to respondents constitutes
Qualified Theft. Hence, since respondents’ car is undeniably covered by a Comprehensive Motor
Vehicle Insurance Policy that allows for recovery in cases of theft, petitioner is liable under the policy
for the loss of respondents’ vehicle under the "theft clause."

23 8
R e in su ra nc e

EQUITABLE INS. AND CASUALTY CO. v. RURAL INSURANCE AND SURETY COMPANY, INC.
G.R. No. L-17436
January 31, 1962

Barrera, J.

Facts:

On May 26, 1959, plaintiff Equitable Insurance and Casualty Company, Inc. filed with the Court of First
Instance of Manila a complaint against defendant Rural Insurance and Surety Company, Inc. alleging,
as first cause of action, that on November 11, 1957, plaintiff and defendant entered into a reciprocal
facultative reinsurance agreement, wherein they agreed to cede to each other, by way of facultative
reinsurance on policies of insurance or reinsurance issued by their respective fire insurance
departments on risks situated in the Philippines, subject to the stipulations of the agreement.
Pursuant to said agreement, plaintiff on January 29, 1958, reinsured for P2,000.00 with defendant
the stock covered by fire insurance Policy No. 5880 issued by plaintiff in behalf of Messrs. Jaen
Bermers' Cooperative Marketing Association, Inc. On July 4, 1958, the stock insured and covered was
burned, and the share of the loss assumed by defendant was computed at P2,024.87 including
adjuster's fee, for which plaintiff sent to defendant for payment by the latter, a statement of account
dated March 12, 1959. Despite repeated demands by plaintiff, defendant refused and failed to pay the
sum of P2,024.87.

On the second cause of action, plaintiff on March 24, 1958 reinsured in the sum of P2,000.00
with defendant, stock covered by fire insurance, issued by plaintiff in behalf of Electric and Lamp
Supplies (Mr. Pedro Casipe). On October 13, 1958, said stock was burned and the share of loss
assumed by defendant as per reinsurance agreement with plaintiff was computed at P1,334.80
including adjuster's fee, for which plaintiff likewise sent a statement of account dated February 4,
1959, to defendant with the request that the same be paid. Notwithstanding repeated demands,
defendant refused and failed to pay plaintiffs. Because the defendant failed to pay its share of the
losses assumed by it, plaintiff has been compelled to institute the present action and to incur
attorney's fees and expenses of litigation amounting to P500.00. Plaintiff prayed for judgment
ordering the defendant to pay said sums of P2,024.80 and P1,334.80 with legal interest thereon from
the date of the filing of the complaint until fully paid, P500.00 as attorney's fees, and the costs of the
suit.

Issue:

Is the plaintiff entitled to the sums claimed by it?

Ruling:

No. It is true that the Reciprocal Facultative Reinsurance Agreement required that 'in the
event of any question arising as to the meaning of, or any way connected with or relating to this
Agreement, whether before or after its termination, the parties shall endeavor to arrive at a
satisfactory compromise by amicable settlement rather than by court action'; and that the dispute
should be referred to the decision of two arbitrators and umpire, as provided, therein. However, in
this particular case, there is absolutely no dispute between the two parties, because in the stipulation
of facts, the defendant has admitted that plaintiff has paid its liability to the insured as per its fire
insurance policies specified in the two causes of action of the complaint. In the second cause of action,
appellant claims that "the court a quo erred in failing to rule that in a facultative obligation the right

23 9
R e in su ra nc e

to choose an alternative remedy lies only with the debtor, who in this case is the herein defendant-
appellant", and in support thereof, cites Article 1206 of the new Civil Code. The Court found no
connection whatsoever between this article and the agreement subject of this action, except the word
"facultative" used in both. The term "facultative" is used in reinsurance contracts, and it is so used in
this particular case, merely to define the right of the reinsurer to accept or not to accept participation
in the risk insured. But once the share is accepted, as it was in the case at bar, the obligation is absolute
and the liability assumed thereunder can be discharged by one and only way — payment of the share
of the losses. There is neither alternative nor substitute prestation.

24 0
R e in su ra nc e

THE PHILIPPINE AMERICAN LIFE INSURANCE COMPANY v. THE AUDITOR GENERAL


G.R. No. L-19255
January 18, 1968

Sanchez, J

Facts:

On January 1, 1950, Philippine American Life Insurance Company [Philamlife], a domestic life
insurance corporation, and American International Reinsurance Company [Airco] of Pembroke,
Bermuda, a corporation organized under the laws of the Republic of Panama, entered into an
agreement whereby Philamlife agrees to reinsure with AIRCO the entire first excess of such life
insurance on the lives of persons as may be written by the Ceding Company under direct application
over and above its maximum limit of retention for life insurance, and AIRCO binds itself to accept such
reinsurances on the same terms and for an amount not exceeding its maximum limit for automatic
acceptance of life reinsurance. It is also stipulated that even though Philamlife "is already on a risk
for its maximum retention under policies previously issued, when new policies are applied for and
issued can cede automatically any amount, within the limits specified, on the same terms on which it
would be willing to accept the risk for its own account, if it did not already have its limit of retention."

Philamlife agrees to pay premiums for all reinsurances "on an annual premium basis." The
Central Bank of the Philippines collected the sum of P268, 747.48 as foreign exchange margin on
Philamlife remittances to Airco purportedly totalling $610,998.63 and made subsequent to July 16,
1959. Philamlife subsequently filed with the Central Bank a claim for the refund of the above sum of
P268, 747.48. The ground therefor was that the reinsurance premiums so remitted were paid
pursuant to the reinsurance treaty, and, therefore, were pre-existing obligations expressly exempt
from the margin fee.

On June 7, 1960, the Monetary Board, in line with the opinion of its Acting Legal Counsel
resolved that "reinsurance contracts entered into and approved by the Central Bank before July 17,
1959 are exempt from the payment of the 25% foreign exchange margin, even if remittances thereof
are made after July 17, 1959," because such remittances "are only made in the implementation of a
mother contract, a continuing contract, which is the reinsurance treaty."

The foregoing resolution notwithstanding, the Auditor of the Central Bank, on April 19, 1961,
refused to pass in audit Philamlife's claim for refund.

Issue:

Is the petitioner’s claim covered by the exemption?

Ruling:

The thrust of petitioner's argument is that the premia remitted were in pursuance of its
reinsurance treaty with Airco of January 1, 1950, a contract antedating the Margin Law, which took
effect only on July 16, 1959.For an exemption to come into play, there must be a reinsurance policy
or, as in the reinsurance treaty provided, a "reinsurance cession" 9 which may be automatic or
facultative. There should not be any misapprehension as to the distinction between a reinsurance
treaty, on the one hand, and a reinsurance policy or a reinsurance cession, on the other. A reinsurance
policy is thus a contract of indemnity one insurer makes with another to protect the first insurer from

24 1
R e in su ra nc e

a risk it has already assumed. In contradistinction a reinsurance treaty is merely an agreement


between two insurance companies whereby one agrees to cede and the other to accept reinsurance
business pursuant to provisions specified in the treaty. The practice of issuing policies by insurance
companies includes, among other things, the issuance of reinsurance policies on standard risks and
also on substandard risks under special arrangements. The lumping of the different agreements
under a contract has resulted in the term known to the insurance world as "treaties." Such a treaty is,
in fact, an agreement between insurance companies to cover the different situations described.
Reinsurance treaties and reinsurance policies are not synonymous. Treaties are contracts for
insurance; reinsurance policies or cessions. are contracts of insurance.

Philamlife's obligation to remit reinsurance premiums becomes fixed and definite upon the
execution of the reinsurance cession. Because, for every life insurance policy ceded to Airco,
Philamlife agrees to pay premium. It is only after a reinsurance cession is made that payment of
reinsurance premium may be exacted, as it is only after Philamlife seeks to remit that reinsurance
premium that the obligation to pay the margin fee arises. Upon the premise that the margin fee of
P268,747.48 was collected on remittances made on reinsurance effected on or after the Margin Law
took effect, refund thereof does not come within the coverage of the exemption circumscribed in
Section 3 of the said law.

24 2
R e in su ra nc e

IVOR ROBERT DAYTON GIBSON v. HON. PEDRO A. REVILLA


G.R. No. L-41432
July 30, 1979

Guerrero, J.

Facts:

Lepanto Consolidated Mining Company filed a complaint against Malayan Insurance


Company, Inc. The civil suit thus instituted by Lepanto against Malayan was founded on the fact that
Malayan issued a Marine Open Policy covering all shipments of copper, gold, and silver concentrates
in bulk from Poro, San Fernando, La Union to Tacoma, Washington or to other places in the United
States. Thereafter, Malayan obtained reinsurance abroad through Sedgwick, Collins & Co., Limited, a
London insurance brokerage. The Memorandum of Insurance issued by Sedgwick to Malayan listed
three groups of underwriters or reinsurers – Lloyds 62.808%, Companies (I.L.U.) 34.705%, Other
companies 2.487%. At the top of the list of underwriting members of Lloyds is Syndicate No. 448,
assuming 2.48% of the risk assumed by the reinsurer, which syndicate number petitioner Ivor Robert
Dayton Gibson claims to be himself. Petitioner then filed a motion to intervene as defendant, which
motion was denied by the lower court.

Issue:

Whether or not a separate premium is needed for cover notes.

Ruling:

No. The Cover Note was not without consideration for which the Court of Appeals held the
Cover Note as null and void, and denied recovery therefrom. The fact that no separate premium was
paid on the Cover Note before the loss insured against occurred, does not militate against the validity
of PTEC’s contention, for no such premium could have been paid, since by the nature of the Cover
Note, it did not contain, as all Cover Notes do not contain particulars of the shipment that would serve
as basis for the computation of the premiums. As a logical consequence, no separate premiums are
intended or required to be paid on a Cover Note. At any rate, it is not disputed that PTEC paid in full
all the premiums as called for by the statement issued by WIC after the issuance of the two regular
marine insurance policies, thereby leaving no account unpaid by PTEC due on the insurance coverage,
which must be deemed to include the Cover Note. If the Note is to be treated as a separate policy
instead of integrating it to the regular policies subsequently issued, the purpose and function of the
Cover Note would be set at naught or rendered meaningless, for it is in a real sense a contract, not a
mere application for insurance which is a mere offer.

24 3
R e in su ra nc e

PIONEER INSURANCE & SURETY CORPORATION v. COURT OF APPEALS


G.R. No. 84197
July 28, 1989

Gutierrez, Jr., J

Facts:

In 1965, Jacob S. Lim (petitioner in G.R. No. 84157) was engaged in the airline business as
owner-operator of Southern Air Lines (SAL) a single proprietorship.

On May 17, 1965, at Tokyo, Japan, Japan Domestic Airlines (JDA) and Lim entered into and
executed a sales contract for the sale and purchase of two (2) aircrafts and one (1) set of necessary
spare parts for the total agreed price of US $109,000.00 to be paid in installments. One DC-3 Aircraft
with Registry No. PIC-718, arrived in Manila on June 7,1965 while the other aircraft, arrived in Manila
on July 18,1965.

On May 22, 1965, Pioneer Insurance and Surety Corporation as surety executed and issued
its Surety Bond No. 6639in favor of JDA, in behalf of its principal, Lim, for the balance price of the
aircrafts and spare parts.

It appears that Border Machinery and Heavy Equipment Company, Inc. (Bormaheco),
Francisco and Modesto Cervantes (Cervanteses) and Constancio Maglana (respondents in both
petitions) contributed some funds used in the purchase of the above aircrafts and spare parts. The
funds were supposed to be their contributions to a new corporation proposed by Lim to expand his
airline business. They executed two (2) separate indemnity agreements in favor of Pioneer, one
signed by Maglana and the other jointly signed by Lim for SAL, Bormaheco and the Cervanteses. The
indemnity agreements stipulated that the indemnitors principally agree and bind themselves jointly
and severally to indemnify and hold and save harmless Pioneer from and against any/all damages,
losses, costs, damages, taxes, penalties, charges and expenses of whatever kind and nature which
Pioneer may incur in consequence of having become surety upon the bond/note and to pay,
reimburse and make good to Pioneer, its successors and assigns, all sums and amounts of money
which it or its representatives should or may pay or cause to be paid or become liable to pay on them
of whatever kind and nature.

On June 10, 1965, Lim doing business under the name and style of SAL executed in favor of
Pioneer as deed of chattel mortgage as security for the latter's suretyship in favor of the former. It
was stipulated therein that Lim transfer and convey to the surety the two aircrafts.

Lim defaulted on his subsequent installment payments prompting JDA to request payments
from the surety. Pioneer paid a total sum of P298,626.12.

Issue:

Do the respondents have interest in the reinsurance?

Ruling:

24 4
R e in su ra nc e

Article 2207 of the Civil Code provides that “If the plaintiffs property has been insured, and
he has received indemnity from the insurance company for the injury or loss arising out of the wrong
or breach of contract complained of, the insurance company shall be subrogated to the rights of the
insured against the wrongdoer or the person who has violated the contract. If the amount paid by the
insurance company does not fully cover the injury or loss, the aggrieved party shall be entitled to
recover the deficiency from the person causing the loss or injury.”

In the case of Phil. Air Lines, Inc. v. Heald Lumber Co., it was held that if a property is insured
and the owner receives the indemnity from the insurer, it is provided in said article that the insurer
is deemed subrogated to the rights of the insured against the wrongdoer and if the amount paid by
the insurer does not fully cover the loss, then the aggrieved party is the one entitled to recover the
deficiency. Evidently, under this legal provision, the real party in interest with regard to the portion
of the indemnity paid is the insurer and not the insured.

It is clear from the records that Pioneer sued in its own name and not as an attorneyin-fact of
the reinsurer. Accordingly, the appellate court did not commit a reversible error in dismissing the
petitioner's complaint as against the respondents for the reason that the petitioner was not the real
party in interest in the complaint and, therefore, has no cause of action against the respondents.

24 5
Ma r in e I ns u ra nc e – D e f in it io n a nd Sco p e

LA RAZON SOCIAL "GO TIAOCO Y HERMANOS" v. UNION INSURANCE SOCIETY OF CANTON


G.R. No. 13983
September 1, 1919

Facts:

A cargo of rice belonging to the Go Tiaoco Brothers, was transported in the early days of May
1915, on the steamship Hondagua from the port of Saigon to Cebu. On discharging the rice from one
of the compartments in the after hold, upon arrival at Cebu, it was discovered that 1,473 sacks had
been damaged by sea water.

The policy of insurance, covering the shipment, was signed upon a form long in use among
companies engaged in maritime insurance. It purports to insure the cargo from the following among
other risks: "Perils of the seas, men, of war, fire, enemies, pirates, rovers, thieves, jettisons, barratry
of the master and mariners, and of all other perils, losses, and misfortunes that have or shall come to
the hurt, detriment, or damage of the said goods and merchandise or any part thereof."

It was found out that the drain pipe which served as a discharge from the water closet passed
down through the compartment where the rice in question was stowed and thence out to sea through
the wall of the compartment, which was a part of the wall of the ship. The joint or elbow where the
pipe changed its direction was of cast iron; and in course of time it had become corroded and abraded
until a longitudinal opening had appeared in the pipe about one inch in length. This hole had been in
existence before the voyage was begun, and an attempt had been made to repair it by filling with
cement and bolting over it a strip of iron.

Issue:

Whether or not perils of the sea includes “entrance of water into the ship’s hold through a
defective pipe.”

Ruling:

It is determined that the words "all other perils, losses, and misfortunes" are to be interpreted
as covering risks which are of like kind (ejusdem generis) with the particular risks which are
enumerated in the preceding part of the same clause of the contract. According to the ordinary rules
of construction these words must be interpreted with reference to the words which immediately
precede them. They were no doubt inserted in order to prevent disputes founded on nice distinctions.
Their office is to cover in terms whatever may be within the spirit of the cases previously enumerated,
and so they have a greater or less effect as a narrower or broader view is taken of those cases.

24 6
Ma r in e I ns u ra nc e – G e n e ra l Av e ra g es

MAGSAYSAY, INC. v. ANASTACIO AGAN


G.R. No. L-6393
January 31, 1955

Reyes, A. J.

Facts:

In 1949, SS San Antonio, owned by AMInc, embarked on its voyage to Batanes via Aparri. It
was carrying various cargoes, one of which was owned by Agan. One fine weather day, it accidentally
ran aground the mouth of the Cagayan River due to the sudden shifting of the sands below. SS San
Antonio then needed the services of Luzon Stevedoring Co. to tow the ship and make it afloat so that
it can continue its journey. Later, AMInc required the cargo owners to pay the expenses incurred in
making the ship afloat (P841.40 each). The expenses, AMInc claims, fall under the General Averages
Rule under the Code of Commerce, which is to be shared by ship owner and cargo owners as well.

Issue:

Whether or not general averages exist in the case at bar.

Ruling:

No. General averages contemplate that the stranding of the vessel is intentionally done in
order to save the vessel itself from a certain and imminent danger. Here, the stranding was accidental
and it was made afloat for the purpose of saving the voyage and not the vessel. Note that this
happened on a fine weather day. Also, it cannot be said that the towing was made to save the cargos,
for the cargos were not in danger imminent danger.

24 7
Ma r in e I ns u ra nc e – G e n e ra l Av e ra g es

FRANCISCO JARQUE v. SMITH, BELL & CO., LTD., ET AL. and UNION FIRE INSURANCE CO.
G.R. No. L-32986
November 11, 1930

Ostrand, J

Facts:

The plaintiff was the owner of the motorboat Pandan and held a marine insurance policy for
the sum of P45,000 on the boat, the policy being issued by the National Union Fire Insurance
Company and according to the provisions of a "rider" attached to the policy, the insurance was against
the "absolute total loss of the vessel only." On October 31, 1928, the ship ran into very heavy sea off
the Islands of Ticlin, and it became necessary to jettison a portion of the cargo. As a result of the
jettison, the National Union Fire Insurance Company was assessed in the sum of P2,610.86 as its
contribution to the general average. The insurance company, insisting that its obligation did not
extend beyond the insurance of the "absolute total loss of the vessel only, and to pay proportionate
salvage of the declared value," refused to contribute to the settlement of the general average. The
present action was thereupon instituted, and after trial the court below rendered judgment in favor
of the plaintiff and ordered the defendant National Union Fire Insurance Company to pay the plaintiff
the sum of P2,610.86 as its part of the indemnity for the general average brought about by the jettison
of cargo.

Issues:

1. Whether or not the lower court erred in disregarding the typewritten clause endorsed upon
the policy, Exhibit A, expressly limiting insurer's liability thereunder of the total loss of the
wooden vessel Pandan and to proportionate salvage charges; and
2. Whether or not the lower court erred in concluding that defendant and appellant, National
Union Fire Insurance Company is liable to contribute to the general average resulting from
the jettison of a part of said vessel's cargo.
3.
Ruling:

I. The insurance contract, Exhibit A, is printed in the English common form of marine policies.
One of the clauses of the document originally read as follows:
Touching the Adventures and Perils which the said National Union Fire Insurance
Company is content to bear, and to take upon them in this Voyage; they are of the Seas,
Men-of-War, Fire, Pirates, Rovers, Thieves, Jettison, Letters of Mart and Countermart,
Surprisals, and Takings at Sea. Arrest, Restraint and Detainments, of all Kings Princes
and People of what Nation, Condition or Quality so ever; Barratry of the Master and
Marines, and of all other Perils, Losses and Misfortunes, that have or shall come to the
Hurt, Detriment, or Damage of the said Vessel or any part thereof; and in case of any
Loss or Misfortunes, it shall be lawful for the Assured, his or their Factors, Servants,
or assigns, to sue, labour and travel for, in and about the Defense. Safeguard, and
recovery of the said Vessel or any Charges whereof the said Company, will contribute,
according to the rate and quantity of the sum herein assured shall be of as much force
and Virtue as the surest Writing or Policy of Insurance made in LONDON.

Attached to the policy over and above the said clause is a "rider" containing typewritten
provisions, among which appears in capitalized type the following clause:

24 8
Ma r in e I ns u ra nc e – G e n e ra l Av e ra g es

AGAINST THE ABSOLUTE TOTAL LOSS OF THE VESSEL ONLY, AND TO PAY
PROPORTIONATE SALVAGE CHARGES OF TEH DECLARED VALUE.

At the bottom of the same rider following the type written provisions therein set forth are the
following words: "Attaching to and forming part of the National Union Fire Insurance Co., Hull Policy
No. 1055."

It is a well settled rule that in case repugnance exists between written and printed portions
of a policy, the written portion prevails, and there can be no question that as far as any inconsistency
exists, the above-mentioned typed "rider" prevails over the printed clause it covers. Section 291 of
the Code of Civil Procedure provides that "when an instrument consists partly of written words and
partly of a printed form and the two are inconsistent, the former controls the latter." (See also Joyce
on Insurance, 2d ed., sec. 224, page 600; Arnould on Marine Insurance, 9th ed., sec. 73; Marine
Equipment Corporation v. Automobile Insurance Co., 24 Fed. (2d), 600; and Marine Insurance
Company v. McLahanan, 290 Fed., 685, 688.)

II. In the absence of positive legislation to the contrary, the liability of the defendant insurance
company on its policy would, perhaps, be limited to "absolute loss of the vessel only, and to pay
proportionate salvage of the declared value." But the policy was executed in this jurisdiction and
"warranted to trade within the waters of the Philippine Archipelago only." Here the liability for
contribution in general average is not based on the express terms of the policy, but rest upon the
theory that from the relation of the parties and for their benefit, a quasi-contract is implied by law.
Article 859 of the Code of Commerce is still in force and reads as follows:
ART. 859. The underwriters of the vessel, of the freight, and of the cargo shall be
obliged to pay for the indemnity of the gross average in so far as is required of each
one of these objects respectively.

The articles are mandatory in its terms, and the insurers, whether for the vessel or for the
freight or for the cargo, are bound to contribute to the indemnity of the general average. And there is
nothing unfair in that provisions; it simply places the insurer on the same footing as other persons
who have an interest in the vessel, or the cargo therein at the time of the occurrence of the general
average and who are compelled to contribute (art. 812, Code of Commerce).

In the present case it is not disputed that the ship was in grave peril and that the jettison of
part of the cargo was necessary. If the cargo was in peril to the extent of call for general average, the
ship must also have been in great danger, possibly sufficient to cause its absolute loss. The jettison
was therefore as much to the benefit of the underwriter as to the owner of the cargo. The latter was
compelled to contribute to the indemnity; why should not the insurer be required to do likewise? If
no jettison had taken place and if the ship by reason thereof had foundered, the underwriter's loss
would have been many times as large as the contribution now demanded.

24 9
Ma r in e I ns u ra nc e – G e n e ra l Av e ra g es

COMPAGNIE DE COMMERCE ET DE NAVIGATION D'EXTREME ORIENT v. THE HAMBURG


AMERIKA PACKETFACHT ACTIEN GESELLSCHAFT
G.R. No. L-10986
March 31, 1917

Carson, J.

Facts:

The defendant chartered and hired unto the plaintiff the steamship or vessel called the Sambia for the
purpose of carrying a full cargo of rice, rice bran and cargo meal from the port of Saigon to the port of Dunkirk
and Hamburg, via Suez Canal, upon the terms and conditions set forth and contained in the written charter
party made and executed between the said parties on said date. There were rumors of impending war
between Germany and France and other nations of Europe. The master of the steamship Sambia received an
order from the owner of said steamship to proceed at once to a neutral port for refuge, the port of Saigon being
a French port. Plaintiff asked for compulsory detention of its vessel to prevent its property from leaving Saigon.
However, the Governor of Saigon refused to issue an order because he had not been officially notified of the
declaration of the war. The steamship Sambia came directly from Saigon to Manila, because it was issued a bill
of health for Manila, issued by the United States consul at Saigon. The steamship stayed continuously in Manila
and where it contends it will be compelled to stay until the war ceases. No attempt was made on the part of
the defendants to transfer and deliver the cargo to the destinations as stipulated in the charter party. Behn,
Meyer & Company, offered to purchase the cargo from plaintiff, but the latter never received the cable
messages so they never answered. When a survey was done on the ship, it was found that the cargo was
infested with beetles, so Behn, Meyer and Company asked for court authority to sell the cargo and the balance
to be dumped at sea. The proceeds of the sale was deposited to the court, waiting for orders as to what to do
with it. Behn, Meyer & Company still waiting for orders as to what to do. Now, the plaintiff wants all the
proceeds of the sale to be given to them as damages for the defendants’ failure to deliver the cargo to the
destinations Dunkirk and Hamburg, while defendants contended that they have lien on the proceeds of the
sale.

The trial court ruled in favour of the plaintiffs. Counsel for the defendant-appellant made the following
assignments of error on appeal:
1. The court had no jurisdiction
2. Fear of capture was not force majeure
3. Defendant is liable for non-delivery of cargo, and the value of the awards of damges

Issue:

Is the claim of the defendant for general average can be sustained

Ruling:

The claim of the shipowner for general average cannot be sustained under the provisions of the York-
Antwerp Rules of 1890, by reference to which, it was expressly stipulated in the charter party, all such
questions should be settled, Rules X and XI, which treat of "Expenses at Port of Refuge, etc.," and "Wages and
Maintenance of Crew in Port of Refuge, etc.," provide for general average "When a ship shall have entered a
port or place of refuge, or shall have returned to her port or place of loading, in consequence of accident,
sacrifice, or other extraordinary circumstances which render that necessary for the common safety . . .;" and
an examination of the entire body of these rules discloses that general average is never allowed thereunder
unless the loss or damage sought to be made good as general average has been incurred for the "common

25 0
Ma r in e I ns u ra nc e – G e n e ra l Av e ra g es

safety." It is very clear that in fleeing from the port of Saigon and taking refuge in Manila Bay the master of the
Sambia was not acting for the common safety of the vessel and her cargo. The French cargo was absolutely
secure from danger of seizure or confiscation so long as it remained in the port of Saigon, and there can be no
question that the flight of the Sambia was a measure of precaution adopted solely and exclusively for the
preservation of the vessel from danger of seizure or capture.

If then, any doubt could properly arise as to the meaning and effect of the words "common safety" as
used in this body of rules, we would be justified in resolving it in accordance with settled principles of maritime
law; and an examination of the authorities discloses a substantial unanimity of opinion as to the general
doctrine which provides that claims for contribution in general average must be supported by proof that
sacrifices on account of which such claims are submitted were made to avert a common imminent peril, and
that extraordinary expenses for which reimbursement is sought, were incurred for the joint benefit of ship
and cargo.

The doctrine is discussed at length in numerous decisions of the Supreme Court of the United States,
a number of which are cited in the court below, but for our purposes it will be sufficient to insert here a few
extracts from two of the leading cases. In the cases of The Star of Hope v. Annano, the Court held that in order
to constitute a valid claim for general average contribution: First, there must be a common danger to which
the ship, cargo and crew were all exposed, and that danger must be imminent and apparently inevitable,
except by incurring a loss of a portion of the associated interests to save the remainder. Second, there must be
the voluntary sacrifice of a part for the benefit of the whole, as, for example, a voluntary jettison or casting away
of some portion of the associated interests for the purpose of avoiding the common peril, or a voluntary
transfer of the common peril from the whole to a particular portion of those interests. Third, the attempt so
made to avoid the common peril to which all those interests were exposed must be to some practical extent
successful, for if nothing is saved there cannot be any such contribution in any case.

25 1
Ma r in e I ns u ra nc e – G e n e ra l Av e ra g es

A. MAGSAYSAY INC. v. ANASTACIO AGAN


G.R. No. L-6393
January 31, 1955

Reyes, A. J

Facts:

In 1949, SS San Antonio, owned by AMInc, embarked on its voyage to Batanes via Aparri. It was
carrying various cargoes, one of which was owned by Agan. One fine weather day, it accidentally ran aground
the mouth of the Cagayan River due to the sudden shifting of the sands below. SS San Antonio then needed the
services of Luzon Stevedoring Co. to tow the ship and make it afloat so that it can continue its journey. Later,
AMInc required the cargo owners to pay the expenses incurred in making the ship afloat (P841.40 each). The
expenses, AMInc claims, fall under the General Averages Rule under the Code of Commerce, which is to be
shared by ship owner and cargo owners as well.

Issue:

Whether or not general averages exist in the case at bar.

Ruling:

No. Tolentino, in his commentaries on the Code of Commerce, gives the following requisites for
general average: First, there must be a common danger. This means, that both the ship and the cargo, after has
been loaded, are subject to the same danger, whether during the voyage, or in the port of loading or unloading;
that the danger arises from the accidents of the sea, dispositions of the authority, or faults of men, provided
that the circumstances producing the peril should be ascertained and imminent or may rationally be said to
be certain and imminent. This last requirement exclude measures undertaken against a distant peril. Second,
that for the common safety part of the vessel or of the cargo or both is sacrificed deliberately. Third, that from
the expenses or damages caused follows the successful saving of the vessel and cargo. Fourth, that the
expenses or damages should have been incurred or inflicted after taking proper legal steps and authority.

With respect to the first requisite, the evidence does not disclose that the expenses sought to be
recovered from defendant were incurred to save vessel and cargo from a common danger. The vessel ran
aground in fine weather inside the port at the mouth of a river, a place described as "very shallow". It would
thus appear that vessel and cargo were at the time in no imminent danger or a danger which might "rationally
be sought to be certain and imminent." It is, of course, conceivable that, if left indefinitely at the mercy of the
elements, they would run the risk of being destroyed. But as stated at the above quotation, "this last
requirement excludes measures undertaken against a distant peril." It is the deliverance from an immediate,
impending peril, by a common sacrifice, that constitutes the essence of general average. (The Columbian
Insurance Company of Alexandria v. Ashby & Stribling et al., 13 Peters 331; 10 L. Ed., 186). In the present case
there is no proof that the vessel had to be put afloat to save it from imminent danger. What does appear from
the testimony of plaintiff's manager is that the vessel had to be salvaged in order to enable it "to proceed to its
port of destination." But as was said in the case just cited it is the safety of the property, and not of the voyage,
which constitutes the true foundation of the general average. As to the second requisite, we need only repeat
that the expenses in question were not incurred for the common safety of vessel and cargo, since they, or at
least the cargo, were not in imminent peril. The cargo could, without need of expensive salvage operation, have
been unloaded by the owners if they had been required to do so.

25 2
Ma r in e I ns u ra nc e – G e n e ra l Av e ra g es

With respect to the third requisite, the salvage operation, it is true, was a success. But as the sacrifice
was for the benefit of the vessel — to enable it to proceed to destination — and not for the purpose of saving
the cargo, the cargo owners are not in law bound to contribute to the expenses.The final requisite has not been
proved, for it does not appear that the expenses here in question were incurred after following the procedure
laid down in article 813 et seq.

In conclusion we found that plaintiff not made out a case for general average, with the result that its
claim for contribution against the defendant cannot be granted.

25 3
Ma r in e I ns u ra nc e – Pa r tic ul a r A v era g es

PHILIPPINE HOME ASSURANCE CORP v. CA AND EASTERN SHIPPING LINES


G.R. No. 106999
June 20, 1996

Kapunan, J.

Facts:

Eastern Shipping Lines, Inc. (ESLI) loaded on board SS Eastern Explorer in Kobe, Japan, the following
shipment for carriage to Manila and Cebu, freight pre-paid and in good order and condition, viz: (a) two (2)
boxes internal combustion engine parts, consigned to William Lines, Inc. under Bill of Lading No. 042283; (b)
ten (10) metric tons (334 bags) ammonium chloride, consigned to Orca's Company; (c) two hundred (200)
bags Glue 300, consigned to Pan Oriental Match Company; and (d) garments, consigned to Ding Velayo . While
the vessel was off Okinawa, Japan, a small flame was detected on the acetylene cylinder located in the
accommodation area near the engine room on the main deck level. As the crew was trying to extinguish the
fire, the acetylene cylinder suddenly exploded sending a flash of flame throughout the accommodation area,
thus causing death and severe injuries to the crew and instantly setting fire to the whole superstructure of the
vessel. The incident forced the master and the crew to abandon the ship. Thereafter, SS Eastern Explorer was
found to be a constructive total loss and its voyage was declared abandoned. After the fire was extinguished,
the cargoes which were saved were loaded to another vessel for delivery to their original ports of destination.
ESLI charged port. After the fire was extinguished, the cargoes which were saved were loaded to another
vessel for delivery to their original ports of destination.

PHAC, as subrogee of the consignees, thereafter filed a complaint before the Regional Trial Court of
Manila, Branch 39, against ESLI to recover the sum paid under protest on the ground that the same were
actually damages directly brought about by the fault, negligence, illegal act and/or breach of contract of ESL.
In its answer, ESLI contended that it exercised the diligence required by law in the handling, custody and
carriage of the shipment; that the fire was caused by an unforeseen event; that the additional freight charges
are due and demandable pursuant to the Bill of Lading; and that salvage charges are properly collectible under
Act No. 2616, known as the Salvage Law.

The trial court dismissed PHAC's complaint and ruled in favor of ESLI ratiocinating thus:
The question to be resolved is whether or not the fire on the vessel which was caused by the explosion of an
acetylene cylinder loaded on the same was the fault or negligence of the defendant.

Issue:

Is Phil Assurance entitled to recover what it had paid?

Ruling:

Yes. The goods were not lost or damaged by the fire. The goods were all delivered to the consignees,
even if the transshipment took longer. What is at issue, therefore, is NOT whether or not the carrier is liable
for the loss, damage, or deterioration of the goods but WHO, among the carrier, consignee or insurer of the
goods, is liable for the additional charges incurred by the owner of the ship in the salvage operations and in
the transshipment of the goods via a different carrier.

In the case at bar, it is not disputed that a small flame was detected on the acetylene cylinder and that
by reason thereof, the same exploded despite efforts to extinguish the fire. Neither is there any doubt that the
acetylene cylinder, obviously fully loaded, was stored in the accommodation area near the engine room and

25 4
Ma r in e I ns u ra nc e – Pa r tic ul a r A v era g es

not in a storage area considerably far, and in a safe distance, from the engine room. Moreover, there was no
showing, and none was alleged by the parties, that the fire was caused by a natural disaster or calamity not
attributable to human agency. On the contrary, there is strong evidence indicating that the acetylene cylinder
caught fire because of the fault and negligence of respondent ESLI, its captain and its crew.

On the issue of whether or not respondent court committed an error in concluding that the expenses
incurred in saving the cargo are considered general average, we rule in the affirmative. As a rule, general or
gross averages include all damages and expenses which are deliberately caused in order to save the vessel, its
cargo, or both at the same time, from a real and known risk. While the instant case may technically fall within
the purview of the said provision, the formalities prescribed under Article 813 and 814 of the Code of
Commerce in order to incur the expenses and cause the damage corresponding to gross average were not
complied with. Consequently, respondent ESLI's claim for contribution from the consignees of the cargo at the
time of the occurrence of the average turns to naught.

Prescinding from the foregoing premises, it indubitably follows that the cargo consignees cannot be
made liable to respondent carrier for additional freight and salvage charges. Consequently, respondent carrier
must refund to herein petitioner the amount it paid under protest for additional freight and salvage charges in
behalf of the consignee.

25 5
P er il s o f th e S ea a n d P e ril s of t h e Sh i p

LA RAZON SOCIAL "GO TIAOCO Y HERMANOS," v. UNION INSURANCE SOCIETY OF CANTON,


LTD.
G.R. No. 13983
September 1, 1919

Street, J.

Facts:

The Go Tiaoco Brothers owned a shipment of rice from Saigon to Cebu. During the transit, it was
discovered that 1,473 sacks of rice was damaged by sea water. It was later found out that the damage was
caused b y a corroded pipe, the purpose of which was to drain from the water closet. The court found in effect
that the opening above described had resulted in course of time from ordinary wear and tear and not from the
straining of the ship in rough weather on that voyage. The trial court ruled in favor of Union Insurance. The
cause of the loss was due to the defect in one of the drain pipes thus, the loss is not covered.

Issue:

Was the reason for the loss of the cargo covered by the insurance?

Ruling:

No. What is covered is "peril of the sea" and not "peril of the ship".

Peril of the sea v. peril of the ship 1.1 It must be considered to be settled, furthermore, that a loss which,
in the ordinary course of events, results from the natural and inevitable action of the sea, from the ordinary
wear and tear of the ship, or from the negligent failure of the ship's owner to provide the vessel with proper
equipment to convey the cargo under ordinary conditions, is not a peril of the sea. Such a loss is rather due to
what has been aptly called the "peril of the ship." The insurer undertakes to insure against perils of the sea and
similar perils, not against perils of the ship. As was well said by Lord Herschell in Wilson, Sons & Co. v. Owners
of Cargo per the Xantho, there must, in order to make the insurer liable, be "some casualty, something which
could not be foreseen as one of the necessary incidents of the adventure. The purpose of the policy is to secure
an indemnity against accidents which may happen, not against events which must happen. The purpose of
the policy is to secure an indemnity against accidents which may happen, not against events which must
happen." Implied warranty as to seaworthiness of the ship. It is universally accepted that in every contract of
insurance upon anything which is the subject of marine insurance, a warranty is implied that the ship shall be
seaworthy at the time of the inception of the voyage. It is also well settled that a ship which is seaworthy for
the purpose of insurance upon the ship may yet be unseaworthy for the purpose of insurance upon the cargo.

In order to make the insurer liable, it must be "some casualty, something which could not be foreseen
as one of the necessary incidents of the adventure.

25 6
P er il s o f th e S ea a n d P e ril s of t h e Sh i p

ISABELA ROQUE and ONG CHIONG v. HON. INTERMEDIATE APPELATE COURT and PIONEER
INSURANCE AND SURETY CORPORATION
G.R. No. L-66935
November 11, 1985

Gutierrez, Jr., J.

Facts:

Isabela Roque (Roque of Isabela Roque Timber Enterprises) hired the Manila Bay Lighterage Corp.
(Manila Bay) to load and carry its logs from Palawan to North Harbor, Manila. The logs were insured with
Pioneer Insurance and Surety Corp. (Pioneer). The logs never reached Manila due to certain circumstances (as
alleged by Roque and found by the appellate court), such as the fact that the barge was not seaworthy that it
developed a leak, that one of the hatches were left open causing water to enter, and the absence of the
necessary cover of tarpaulin causing more water to enter the barge. When Roque demanded payment from
Pioneer, but the latter refused on the ground that its liability depended upon the “Total Loss by Total Loss of
Vessel Only.” The trial court ruled in favour of Roque in the civil complaint filed by the latter against Pioneer,
but the decision was reversed by the appellate court.

Issue:

Whether or not in cases of marine insurance, there is a warranty of seaworthiness by the cargo
owner; Whether or not the loss of the cargo was due to perils of the sea, not perils of the ship.

Ruling:

Yes, there is. The liability of the insurance company is governed by law. Section 113 of the Insurance
Code provides that “In every marine insurance upon a ship or freight, or freightage, or upon anything which is
the subject of marine insurance, a warranty is implied that the ship is seaworthy.” Hence, there can be no
mistaking the fact that the term "cargo" can be the subject of marine insurance and that once it is so made, the
implied warranty of seaworthiness immediately attaches to whoever is insuring the cargo whether he be the
shipowner or not. Moreover, the fact that the unseaworthiness of the ship was unknown to the insured is
immaterial in ordinary marine insurance and may not be used by him as a defense in order to recover on the
marine insurance policy. As to the second issue, by applying Sec. 113 of the Insurance Code, there is no doubt
that the term 'perils of the sea' extends only to losses caused by sea damage, or by the violence of the elements,
and does not embrace all losses happening at sea; it is said to include only such losses as are of extraordinary
nature, or arise from some overwhelming power, which cannot be guarded against by the ordinary exertion
of human skill and prudence. t is also the general rule that everything which happens thru the inherent vice of
the thing, or by the act of the owners, master or shipper, shall not be reputed a peril, if not otherwise borne in
the policy. It must be considered to be settled, furthermore, that a loss which, in the ordinary course of events,
results from the natural and inevitable action of the sea, from the ordinary wear and tear of the ship, or from
the negligent failure of the ship's owner to provide the vessel with proper equipment to convey the cargo
under ordinary conditions, is not a peril of the sea. Such a loss is rather due to what has been aptly called the
"peril of the ship." The insurer undertakes to insure against perils of the sea and similar perils, not against
perils of the ship. Neither barratry can be used as a ground by Roque. Barratry as defined in American
Insurance Law is "any willful misconduct on the part of master or crew in pursuance of some unlawful or
fraudulent purpose without the consent of the owners, and to the prejudice of the owner's interest." Barratry
necessarily requires a willful and intentional act in its commission. No honest error of judgment or mere
negligence, unless criminally gross, can be barratry. In the case at bar, there is no finding that the loss was

25 7
P er il s o f th e S ea a n d P e ril s of t h e Sh i p

occasioned by the willful or fraudulent acts of the vessel's crew. There was only simple negligence or lack of
skill.

25 8
P er il s o f th e S ea a n d P e ril s of t h e Sh i p

CATHAY INSURANCE CO. v. HON. COURT OF APPEALS and REMINGTON INDUSTRIAL SALES
CORPORATION
G.R. No. 76145
June 30, 1987

Paras, J.

Facts:

Originally, this was a complaint filed by private respondent corporation against petitioner (then
defendant) company seeking collection of the sum of P868,339.15 representing private respondent's losses
and damages incurred in a shipment of seamless steel pipes under an insurance contract in favor of the said
private respondent as the insured, consignee or importer of aforesaid merchandise while in transit from Japan
to the Philippines on board vessel SS "Eastern Mariner." The total value of the shipment was P2,894,463.83 at
the prevailing rate of P7.95 to a dollar in June and July 1984, when the shipment was made. The trial court
decided in favor of private respondent corporation.

Respondent contends, among others, that the alleged contractual limitations contained in insurance
policies are regarded with extreme caution by courts and are to be strictly construed against the insurer;
obscure phrases and exceptions should not be allowed to defeat the very purpose for which the policy was
procured. They also contend that The placing of notation "rusty" in the way bills is not only private
respondent's right but a natural and spontaneous reaction of whoever received the seamless steel pipes in a
rusty condition at private respondent's bodega.

Petitioner on the other hand contends that private respondent has admitted that the question
shipment in not covered by a " square provision of the contract," but private respondent claims implied
coverage from the phrase " perils of the sea" mentioned in the opening sentenced of the policy and that the
insistence of private respondent that rusting is a peril of the sea is erroneous.

Issue:

Whether the rusting of the steel pipes in the course of the voyage is considered as a peril of the sea.

Ruling:

There is no question that the rusting of steel pipes in the course of a voyage is a "peril of the sea" in
view of the toll on the cargo of wind, water, and salt conditions. At any rate if the insurer cannot be held
accountable therefor, the Court would fail to observe a cardinal rule in the interpretation of contracts, namely,
that any ambiguity therein should be construed against the maker/issuer/drafter thereof, namely, the insurer.
Besides the precise purpose of insuring cargo during a voyage would be rendered fruitless. Be it noted that
any attack of the 15-day clause in the policy was foreclosed right in the pre-trial conference.

25 9
Ins u ra bl e I n t e r est i n M a ri n e I ns ura nc e

PUROMINES, INC. v. COURT OF APPEAL and PHILIPP BROTHERS OCEANIC, INC


G.R. No. 91228
March 22, 1993

Nocon, J

Facts:

Pursuant to a contract of sale executed between Puromines Inc (petitioner and Philipp Brothers
Oceanic, Inc, (private respondent) as charterer of M/V Liliana Dimitrova, 3 Bills of Lading were executed
bound for the Iloilo and Manila of 15,000 metric tons of prilled urea. However, upon reaching the port of
Manila it was found out that the shipment (urea) were already contaminated with rust and dirt. This
prompted petitioner to file an action for breach of contract of carriage against Maritime Factors, Inc as ship
agent here in the Philippines for the owners of M/V Liliana in the complaint moreover private respondent
Philipp Brothers

Oceanic Inc., was impleaded as charterer of the said vessel. Private respondent, Philipp Brothers,
instead of filing its answer filed a motion to dismiss on the ground of no cause of action. Private respondent
also avers that Puromines Inc. should comply with the arbitration clause provided for in the sales contract.
Facts slow that the sales contract executed between Puromines Inc., and P!ilipp Brothers Oceanic, Inc.,
provides for an arbitration clause wherein it states that:
“Any disputes arising under this contract shall be settled by arbitration in London in
accordance with the Arbitration Act 1950 and any statutory amendment or modification
thereof. Each party is to appoint an Arbitrator, and should they be unable to agree, the
decision of an Umpire appointed by the them be final. The Arbitrators and Umpire are all to
be commercial men and resident in London. !his submission may be made a rule of the High
Court of Justice in England by either party.”

Petitioner moved to oppose the said motion to dismiss alleging that the arbitration clause is
not applicable on the case because the complaint did not arise from the violation of the terms and
conditions of the sales contract but rather for claims of cargo agreement. The trial court ruled in favor
of the petitioner. On appeal, the CA reversed the decision of the trial court and found that the
arbitration clause is applicable. Hence, this petition.

Issue:

Assuming that the cause of action arises from the contract of carriage, whether Philipp, as charterer,
would be liable for the loss or damage?

Ruling:

Assuming that the liability of Philipp is not based on the sales contract, but rather on the contract of
carriage, being the charterer of the vessel MV "Liliana Dimitrova," it is material to show what kind of charter
party Philipp had with owner of vessel to determine former's liability. American jurisprudence defines charter
party as a contract by which an entire ship or some principal part thereof is let by the owner to another person
for a specified time or use. Charter or charter parties are of two kinds. Charter of demise or bareboat and
contracts of affreightment.

In the present case, assuming that the charter party is a demise or bareboat charter, then Philipp
Brothers is liable to Puromines, Inc., subject to the terms and conditions of the sales contract. On the other

26 0
Ins u ra bl e I n t e r est i n M a ri n e I ns ura nc e

hand, if the contract between Philipp and the owner of the vessel MV "Liliana Dimitrova" was merely that of
affreightment, then it cannot be held liable for the damages caused by the breach of contract of carriage, the
evidence of which is the bill of lading.

26 1
Ins u ra bl e I n t e r est i n M a ri n e I ns ura nc e

COASTWISE LIGHTERAGE CORPORATION v. COURT OF APPEALS and the PHILIPPINE


GENERAL INSURANCE COMPANY
G.R. No. 114167
July 12, 1995

Francisco, R., J

Facts:

Pag-asa Sales, Inc. entered into a contract to transport molasses from the province of Negros to Manila
with Coastwise Lighterage Corporation using the latter's dumb barges. The barges were towed in tandem by
the tugboat MT Marica, which is likewise owned by Coastwise. Upon reaching Manila Bay, while approaching
Pier 18, one of the barges, "Coastwise 9", struck an unknown sunken object. As a consequence, the molasses
at the cargo tanks were contaminated and rendered unfit for the use it was intended. This prompted the
consignee, Pag-asa Sales, Inc. to reject the shipment of molasses as a total loss.

Thereafter, Pag-asa Sales, Inc. filed a formal claim with the insurer of its lost cargo, herein private
respondent, Philippine General Insurance Company and against the carrier, herein petitioner, Coastwise
Lighterage. Coastwise Lighterage denied the claim and it was PhilGen which paid the consignee, Pag-asa Sales,
Inc., the amount of P700,000.00, representing the value of the damaged cargo of molasses. In turn, PhilGen
then filed an action against Coastwise Lighterage before the Regional Trial Court of Manila, seeking to recover
the amount of P700,000.00 which it paid to Pag-asa Sales, Inc. for the latter's lost cargo. PhilGen now claims to
be subrogated to all the contractual rights and claims which the consignee may have against the carrier, which
is presumed to have violated the contract of carriage. The RTC awarded the amount prayed for by PhilGen. On
Coastwise Lighterage's appeal to the Court of Appeals, the award was affirmed.

Issue:

Whether or not petitioner Coastwise Lighterage was transformed into a private carrier, by virtue of
the contract of affreightment which it entered into with the consignee, Pag-asa Sales, Inc.

Ruling:

Petitioner contends that the RTC and the Court of Appeals erred in finding that it was a common
carrier. It stresses the fact that it contracted with Pag-asa Sales, Inc. to transport the shipment of molasses from
Negros Oriental to Manila and refers to this contract as a "charter agreement". It then proceeds to cite the case
of Home Insurance Company v. American Steamship Agencies, Inc.2 wherein this Court Ruling: ". . . a common
carrier undertaking to carry a special cargo or chartered to a special person only becomes a private carrier."

Petitioner's reliance on the aforementioned case is misplaced. In its entirety, the conclusions of the
court are as follows: “Accordingly, the charter party contract is one of affreightment over the whole vessel,
rather than a demise. As such, the liability of the shipowner for acts or negligence of its captain and crew, would
remain in the absence of stipulation.”

The distinction between the two kinds of charter is more clearly set out in the case of Puromines, Inc.
v. Court of Appeals, wherein we ruled:

Under the demise or bareboat charter of the vessel, the charterer will generally be regarded
as the owner for the voyage or service stipulated. To create a demise, the owner of a vessel
must completely and exclusively relinquish possession, command and navigation thereof to

26 2
Ins u ra bl e I n t e r est i n M a ri n e I ns ura nc e

the charterer, anything short of such a complete transfer is a contract of affreightment (time
or voyage charter party) or not a charter party at all. On the other hand a contract of
affreightment is one in which the owner of the vessel leases part or all of its space to haul
goods for others. It is a contract for special service to be rendered by the owner of the vessel
and under such contract the general owner retains the possession, command and navigation
of the ship, the charterer or freighter merely having use of the space in the vessel in return for
his payment of the charter hire…

Although a charter party may transform a common carrier into a private one, the same however is
not true in a contract of affreightment on account of the aforementioned distinctions between the two.
Petitioner admits that the contract it entered into with the consignee was one of affreightment. The Court
agrees. Pag-asa Sales, Inc. only leased three of petitioner's vessels, in order to carry cargo from one point to
another, but the possession, command and navigation of the vessels remained with petitioner Coastwise
Lighterage.

The law and jurisprudence on common carriers both hold that the mere proof of delivery of goods in
good order to a carrier and the subsequent arrival of the same goods at the place of destination in bad order
makes for a prima facie case against the carrier.

It follows then that the presumption of negligence that attaches to common carriers, once the goods
it transports are lost, destroyed or deteriorated, applies to the petitioner. This presumption, which is overcome
only by proof of the exercise of extraordinary diligence, remained unrebutted in this case.

26 3
Ins u ra bl e I n t e r est i n M a ri n e I ns ura nc e

LEA MER INDUSTRIES, INC., v. MALAYAN INSURANCE CO., INC.,


G.R. No. 161745
September 30, 2005

Panganiban, J.

Facts:

Ilian Silica Mining entered into a contract of carriage with the petitioner, Lea Mer Industries Inc. for
the shipment of 900 metric tons of silica sand worth P565,000. The cargo was consigned to Vulcan Industrial
and Mining Corporation and was to be shipped from Palawan to Manila. The silica sand was boarded to Judy
VII, the vessel leased by Lea Mer. However, during the course of its voyage, the vessel sank which led to the
loss of the cargo.

Consequently, the respondent, as the insurer, paid Vulcan the value of the lost cargo. Malayan
Insurance Co., Inc. then collected from the petitioner the amount it paid to Vulcan as reimbursement and as its
exercise on the right of subrogation. Lea Mer refused to pay which led Malayan to institute a complaint with
the RTC. The RTC dismissed the complaint stating that the loss was due to a fortuitous event, Typhoon Trining.
Petitioner did not know that a typhoon was coming and that it has been cleared by the Philippine Coast Guard
to travel from Palawan to Manila. The CA reversed the ruling of the trial court for the reason that said vessel
was not seaworthy when it sailed to Manila.

Issue:

Whether or not the petitioner is liable for the loss of the cargo.

Ruling:

Common carriers are persons, corporations, firms or associations engaged in the business of carrying
or transporting passengers or goods, or both — by land, water, or air — when this service is offered to the
public for compensation. Petitioner is clearly a common carrier, because it offers to the public its business of
transporting goods through its vessels. Thus, the Court corrects the trial court's finding that petitioner became
a private carrier when Vulcan chartered it. Charter parties are classified as contracts of demise (or bareboat)
and affreightment, which are distinguished as follows:
"Under the demise or bareboat charter of the vessel, the charterer will generally be considered as owner for
the voyage or service stipulated. The charterer mans the vessel with his own people and becomes, in effect,
the owner pro hac vice, subject to liability to others for damages caused by negligence. To create a demise, the
owner of a vessel must completely and exclusively relinquish possession, command and navigation thereof to
the charterer; anything short of such a complete transfer is a contract of affreightment (time or voyage charter
party) or not a charter party at all."

The distinction is significant, because a demise or bareboat charter indicates a business undertaking
that is private in character. Consequently, the rights and obligations of the parties to a contract of private
carriage are governed principally by their stipulations, not by the law on common carriers. The Contract in the
present case was one of affreightment, as shown by the fact that it was petitioner's crew that manned the
tugboat M/V Ayalit and controlled the barge Judy VII.

Common carriers are bound to observe extraordinary diligence in their vigilance over the goods and
the safety of the passengers they transport, as required by the nature of their business and for reasons of public

26 4
Ins u ra bl e I n t e r est i n M a ri n e I ns ura nc e

policy. Extraordinary diligence requires rendering service with the greatest skill and foresight to avoid damage
and destruction to the goods entrusted for carriage and delivery.

Jurisprudence defines the elements of a "fortuitous event" as follows: (a) the cause of the unforeseen
and unexpected occurrence, or the failure of the debtors to comply with their obligations, must have been
independent of human will; (b) the event that constituted the caso fortuito must have been impossible to
foresee or, if foreseeable, impossible to avoid; (c) the occurrence must have been such as to render it
impossible for the debtors to fulfill their obligation in a normal manner; and (d) the obligor must have been
free from any participation in the aggravation of the resulting injury to the creditor. To excuse the common
carrier fully of any liability, the fortuitous event must have been the proximate and only cause of the loss.
Moreover, it should have exercised due diligence to prevent or minimize the loss before, during and after the
occurrence of the fortuitous event. As required by the pertinent law, it was not enough for the common carrier
to show that there was an unforeseen or unexpected occurrence. It had to show that it was free from any fault
— a fact it miserably failed to prove.

26 5
Ins u ra bl e I n t e r est i n M a ri n e I ns ura nc e

LOADSTAR SHIPPING CO. v. COURT OF APPEALS


G.R. No. 157481
January 24, 2006

Quisumbing, J.

Facts:

On November 19, 1984, Loadstar received on board its vessel M/V Cherokee the following goods for
shipment:
1. 705 bales of lawanit hardwood
2. 27 boxes and crates of tilewood assemblies and others
3. 49 bundles of mouldings R & W (3) Apitong Bolidenized

The goods, amounting to P6,067,178, were insured by Manila Insurance Co. The vessel is insured by
Prudential Guarantee and Assurance, Inc. On November 20, 1984, on its way to Manila from Agusan, the vessel
sank off Limasawa Island. MIC paid the consignee P6,075,000 for the value of the goods lost, and filed a
complaint against Loadstar and PGAI, claiming subrogation into the rights of the consignee. When PGAI paid
Loadstar, it was dropped from the complaint. The trial court ruled against Loadstar, and this was affirmed by
the Court of Appeals.

Loadstar submits that the vessel was a private carrier because it was not issued a certificate of public
convenience, it did not have a regular trip or schedule nor a fixed route, and there was only "one shipper, one
consignee for a special cargo." In refutation, MIC argues that the issue as to the classification of the M/V
"Cherokee" was not timely raised below; hence, it is barred by estoppel. While it is true that the vessel had on
board only the cargo of wood products for delivery to one consignee, it was also carrying passengers as part
of its regular business. Moreover, the bills of lading in this case made no mention of any charter party but only
a statement that the vessel was a "general cargo carrier." Neither was there any "special arrangement"
between LOADSTAR and the shipper regarding the shipment of the cargo. The singular fact that the vessel was
carrying a particular type of cargo for one shipper is not sufficient to convert the vessel into a private carrier.

Loadstar argues that as a private carrier, it cannot be presumed to have been negligent, and the burden of
proving otherwise devolved upon MIC. It also maintains that the vessel was seaworthy, and that the loss was
due to force majeure. Loadstar goes on to argue that, being a private carrier, any agreement limiting its liability,
such as what transpired in this case, is valid. Since the cargo was being shipped at "owner’s risk," Loadstar was
not liable for any loss or damage to the same. Finally, Loadstar avers that MIC’s claim had already prescribed,
the case having been instituted beyond the period stated in the bills of lading for instituting the same — suits
based upon claims arising from shortage, damage, or non-delivery of shipment shall be instituted within sixty
days from the accrual of the right of action. MIC, on the other hand, claims that Loadstar was liable,
notwithstanding that the loss of the cargo was due to force majeure, because the same concurred with
Loadstar’s fault or negligence. Secondly, Loadstar did not raise the issue of prescription in the court below;
hence, the same must be deemed waived. Thirdly, the "limited liability" theory is not applicable in the case at
bar because Loadstar was at fault or negligent, and because it failed to maintain a seaworthy vessel.
Authorizing the voyage notwithstanding its knowledge of a typhoon is tantamount to negligence.

Issue:

Whether or not Loadstar exercised the degree of diligence required under the circumstances.

Ruling:

26 6
Ins u ra bl e I n t e r est i n M a ri n e I ns ura nc e

The doctrine of limited liability does not apply where there was negligence on the part of the vessel
owner or agent. Loadstar was at fault or negligent in not maintaining a seaworthy vessel and in having allowed
its vessel to sail despite knowledge of an approaching typhoon. In any event, it did not sink because of any
storm that may be deemed as force majeure, inasmuch as the wind condition in the area where it sank was
determined to be moderate. Since it was remiss in the performance of its duties, Loadstar cannot hide behind
the "limited liability" doctrine to escape responsibility for the loss of the vessel and its cargo.

Three kinds of stipulations have often been made in a bill of lading. The first is one exempting the
carrier from any and all liability for loss or damage occasioned by its own negligence. The second is one
providing for an unqualified limitation of such liability to an agreed valuation. And the third is one limiting the
liability of the carrier to an agreed valuation unless the shipper declares a higher value and pays a higher rate
of freight. According to an almost uniform weight of authority, the first and second kinds of stipulations are
invalid as being contrary to public policy, but the third is valid and enforceable. Since the stipulation in question
is null and void, it follows that when MIC paid the shipper, it was subrogated to all the rights which the latter
has against the common carrier, Loadstar.

26 7
Ins u ra bl e I n t e r est i n M a ri n e I ns ura nc e

PLANTERS PRODUCTS, INC. v. COURT OF APPEALS, SORIAMONT STEAMSHIP AGENCIES AND


KYOSEI KISEN KABUSHIKI KAISHA
G.R. No. 101503
September 15, 1993

Bellosillo, J

Facts:

Planters Product Inc. purchased from Mitsubishi international corporation metric tons of Urea
fertilizer, which the latter shipped aboard the cargo vessel M/V Sun Plum owned by private respondent Kyosei
Kisen Kabushiki Kaisha. Prior to its voyage, a time charter-party on the vessel respondent entered into
between Mitsubishi as shipper/charterer and KKKK as ship owner, in Tokyo, Japan.

Before loading the fertilizer aboard the vessel, (4) of her holds were presumably inspected by the
charterer’s representative and found fit to take a load of urea in bulk. After the Urea fertilizer was loaded in
bulk by stevedores hired by and under the supervision of the shipper, the steel hatches were closed with heavy
iron lids. Upon arrival of vessel at port, the petitioner unloaded the cargo pursuant to the terms and conditions
of the charter-party. The hatches remained open throughout the duration of the discharge.

Upon arrival at petitioner’s warehouse a survey conducted over the cargo revealed a shortage and the
most of the fertilizer was contaminated with dirt. As such, Planters filed an action for damages. The defendant
argued that the public policy governing common carriers do not apply to them because they have become
private carriers by reason of the provisions of the charter-party.

Issue:

Whether or not the charter-party contract between the ship owner and the charterer transforms a
common carrier into a private carrier?

Ruling:

A charter party may either be time charter wherein the vessel is leased to the charterer, wherein the
ship is leased to the charterer for a fixed period of time or voyage charter, wherein the ship is leased for a single
voyage. In both cases, the charter party provides for the hire of the vessel only, either for a determinate time
or for a single or consecutive voyage.

It is therefore imperative that such common carrier shall remain as such, notwithstanding the charter
of the whole or part of the vessel by one or more persons, provided the charter is limited to the ship only, as in
the case of a time-charter or voyage-charter. It is only when the charter includes both ship and its crew as in
bareboat or demise that it becomes a private carrier. Undoubtedly, a shipowner in a time or voyage charter
retains in possession and control of the ship, although her holds may be the property of the charterer.

26 8
Actua l To ta l L oss

PAN MALAYAN INSURANCE CORP. v. COURT OF APPEALS


G.R. No. 95070
September 5, 1991

Regalado, J.

Facts:

On December 10, 1985, PANMALAY filed a complaint for damages with the RTC of Makati against
private respondents Erlinda Fabie and her driver. PANMALAY averred the following: that it insured a
Mitsubishi Colt Lancer car with plate No. DDZ-431 and registered in the name of Canlubang Automotive
Resources Corporation [CANLUBANG]; that on May 26, 1985, due to the "carelessness, recklessness, and
imprudence" of the unknown driver of a pick-up with plate no. PCR-220, the insured car was hit and suffered
damages in the amount of P42,052.00; that PANMALAY defrayed the cost of repair of the insured car and,
therefore, was subrogated to the rights of CANLUBANG against the driver of the pick-up and his employer,
Erlinda Fabie; and that, despite repeated demands, defendants, failed and refused to pay the claim of
PANMALAY. private respondents filed a Motion to Dismiss alleging that PANMALAY had no cause of action
against them. They argued that payment under the "own damage" clause of the insurance policy precluded
subrogation under Article 2207 of the Civil Code, since indemnification thereunder was made on the
assumption that there was no wrongdoer or no third party at fault.

Issue:

May the insurer PANMALAY institute an action to recover the amount it had paid its assured in
settlement of an insurance claim against private respondents as the parties allegedly responsible for the
damage caused to the insured vehicle?

Ruling:

PANMALAY is correct. Article 2207 of the Civil Code is founded on the well-settled principle of
subrogation. If the insured property is destroyed or damaged through the fault or negligence of a party other
than the assured, then the insurer, upon payment to the assured, will be subrogated to the rights of the assured
to recover from the wrongdoer to the extent that the insurer has been obligated to pay. Payment by the insurer
to the assured operates as an equitable that the insurer has been obligated to pay. Payment by the insurer to
the assured operates as an equitable or negligence of a third party. CANLUBANG is apparently of the same
understanding. Based on a police report assignment to the former of all remedies that the latter may have
against the third party whose negligence or wrongful act caused the loss. The right of subrogation is not
dependent upon, nor does it grow out of, any privity of contract or upon written assignment of claim. It accrues
simply upon payment of the insurance claim by the insurer.

The exceptions are:


(1) if the assured by his own act releases the wrongdoer or third party liable for the loss or
damage, from liability, the insurer's right of subrogation is defeated
(2) where the insurer pays the assured the value of the lost goods without notifying the carrier
who has in good faith settled the assured's claim for loss, the settlement is binding on both
the assured and the insurer, and the latter cannot bring an action against the carrier on his
right of subrogation
(3) where the insurer pays the assured for a loss which is not a risk covered by the policy,
thereby effecting "voluntary payment", the former has no right of subrogation against the
third party liable for the loss

26 9
Actua l To ta l L oss

None of the exceptions are availing in the present case. Also, even if under the above circumstances
PANMALAY could not be deemed subrogated to the rights of its assured under Article 2207 of the Civil Code,
PANMALAY would still have a cause of action against private respondents. In the pertinent case of Sveriges
Angfartygs Assurans Forening v. Qua Chee Gan, supra., the Court ruled that the insurer who may have no rights
of subrogation due to "voluntary" payment may nevertheless recover from the third party responsible for the
damage to the insured property under Article 1236 of the Civil Code.

27 0
Actua l To ta l L oss

PHILIPPINE MANUFACTURING CO. v. UNION INSURANCE SOCIETY OF CANTON, LTD.


G.R. No. L-16473
November 22, 1921

Johns, J

Facts:

Phil. Manufacturing Co. duly organized under the laws of the Philippine Islands with its principal office and
place of business at Manila, and at the times alleged was the owner of the steel tank lighter named Philmaco.
Union Insurance Company organized under the laws of Hongkong and duly authorized to transact business
here. Respondent insured the petitioner's lighter for the sum of P16,000, and issued its policy for such
insurance. In consideration, petitioner paid the defendant P960 as a premium. During the life of the policy and
as a result of a typhoon, the lighter was sunk in the Manila Bay, petitioner notified the respondent and
demanded payment of the full amount of its policy, however, respondent refused, and denied its liability.
Petitioner commenced this action and alleged: 1)steel tank lighter Philmaco became a total loss by sinking in
the waters of the Bay of Manila while operating within the places noted in the said insurance policy, and 2)
such loss immediately became due and payable. Respondent answered that it is only liable for an absolute
total loss, and that there was not a total destruction of the lighter.

Issue:

Is the loss an absolute total loss within the terms and provisions of the policy?

Ruling:

An act revising the insurance laws and regulating insurance business in the Philippine Islands, No.
2427, was enacted by the Philippine Legislature December 12, 1914, and, under the heading of "Loss", contains
the following provisions: SEC. 120. A loss may be either total or partial. SEC. 121. Every loss which is not total
is partial. SEC. 122. A total loss may be either actual or constructive. SEC. 123. An actual total loss is caused by:
(a) A total destruction of the thing insured;
(b) The loss of the thing by sinking, or by being broken up;
(c) Damage to the thing which renders it valueless to the owner for the purpose for which he held it. . . .

At the time the lighter was sunk and in the bottom of the bay under the conditions then there existing, it
was of no value to the owner, and, if it was of no value to the owner, it would be a actual total loss. To render it
valueless to the owner, it is not necessary that there should be an actual or total loss or destruction of all the
different parts of the entire vessel. The question here is whether, under the conditions then and there existing,
and as the lighter laid in the bottom of the bay, was it of any value to the owner. If it was not of any value to the
owner, then there was an actual loss or a "total destruction of the thing insured" within the meaning of the
above sections of Act No. 2427 of the insurance code. The lighter was sunk about July 1, 1918. After several
futile attempts, it was finally raised on September 20, 1918.

It is fair to assume that in its then condition much further time would be required to make the necessary
repairs and install the new machinery before it could again be placed in commission.

During all that time the owner would be deprived of the use of its vessel or the interest on its investment.
When those questions are considered the testimony is conclusive that the cost of salvage, repair, and
reconstruction was more than the original cost of the vessel of its value at the time the policy was issued.

27 1
Actua l To ta l L oss

As found by the trial court “it is difficult to see how there could have been a more complete loss of the
vessel as that which actually occurred." Upon the facts that shown here, any other construction would nullify
the statute, and, as applied to the conditions existing in the Manila Bay, this kind of a policy would be worthless,
and there would not be any consideration for the premium. Moreover, where a policy which was issued at
Manila provides that it "shall be of as much force and effect as the surest writing or policy of insurance made
in London," and there is no allegation or proof of the Marine Law of Great Britain, the policy should be
construed under sections 120, 121, 122, and 123 of Act No. 2427 of the Philippine Legislature.

27 2
Co nst ru ct iv e T ota l L os s

ORIENTAL ASSURANCE CORPORATION v. COURT OF APPEALS AND PANAMA SAW MILL CO.,
INC.
G.R. No. 94052
August 9, 1991

Melencio-Herrera, J

Facts:

An action to recover on a marine insurance policy, issued by petitioner in favor of private respondent,
arising from the loss of a shipment of apitong logs from Palawan To Manila. Sometime in January1986, private
respondent Panama Sawmill Co., Inc.(Panama) bought, in Palawan, 1,208 pieces of apitong logs, with a total
volume of 2,000 cubic meters. It hired Transpacific Towage, Inc., to transport the logs by sea to Manila and
insured it against loss for P1 million with petitioner Oriental Assurance. There is a claim by Panama, however,
that the insurance coverage should have been for P 3million were it not for the fraudulent act of one Benito Sy
Yee Long to whom it had entrusted the amount of P6,000 for the payment of the premium for a P3 million
policy. The logs were loaded on 2 barges: (1) on barge PCT-7000,610 pieces of logs with a volume of 1,000
cubic meters and, (2) on Barge TPAC 1000,598 pieces of logs, also with a volume of 1,000 cubic meters. On
January 28, 1986, the two barges were towed by one tug-boat. During the voyage, rough seas and strong winds
caused damage to Barge TPAC-1000 resulting in the loss of 497 pieces of logs out of the 596 pieces loaded.
Panama demanded payment for the loss but Oriental Assurance refuse on the ground that its contracted
liability was for 7T-TA1 1-SS -810.79TC and CA rendered a decision ordering petitioner to pay Panama. Both
Courts shared the view that the insurance contract should be liberally construed in order to avoid a denial of
substantial justice and that the logs loaded in the two barges should be treated separately such that the loss
sustained by the shipment in one of them may be considered as constructive total loss and correspondingly
compensable.

Issue:

Whether or not oriental Assurance can be held liable under its marine insurance policy based on the
theory of a divisible contract of insurance and, consequently, a constructive total loss.

Ruling:

No liability attaches. The terms of the contract constitute the measure of the insurer liability and
compliance therewith is a condition precedent to the insured's right to recovery from the insurer (Perla
Compania de Seguros, Inc. v. Court of Appeals,185 SCRA 741). Whether a contract is entire or severable is a
question of intention to be determined by the language employed by the parties. The policy in question shows
that the subject matter insured was the entire shipment of 2,000 cubic meters of apitong logs. The fact that the
logs were loaded on two different barges did not make the contract several and divisible as to the items
insured. The logs on the two barges were not separately valued or separately insured. Only one premium was
paid for the entire shipment, making for only one cause or consideration. The insurance contract must,
therefore, be considered indivisible. More importantly, the insurer's liability was for "total loss only." A total
loss may be either actual or constructive (Sec. 129, Insurance Code). An actual total loss is caused by:

(a) A total destruction of the thing insured;


(b) The irretrievable loss of the thing by sinking, or by being broken up;
(c) Any damage to the thing which renders it valueless to the owner for the purpose for which he
held it; or

27 3
Co nst ru ct iv e T ota l L os s

(d) Any other event which effectively deprives the owner of the possession, at the port of destination,
of the thing insured. (Section 130, Insurance Code).

A constructive total loss is one which gives to a person insured a right to abandon, under Section 139 of
the Insurance Code.

Respondent Appellate Court treated the loss as a constructive total loss, and for the purpose of computing
the more than three-fourths value of the logs actually lost, considered the cargo in one barge as separate from
the logs in the other. Thus, it concluded that the loss of 497 pieces of logs from barge TPAC-1000,
mathematically speaking, is more than three-fourths (¾) of the 598 pieces of logs loaded in that barge and
may, therefore, be considered as constructive total loss.

In the absence of either actual or constructive total loss, there can be no recovery by the insured Panama
against the insurer, Oriental Assurance. By reason of the conclusions arrived at, Panama's asseverations in its
Comment need no longer be passed upon, besides the fact that no review, in proper form, has been sought by
it.

27 4
Pa r t ia l L oss

ABOITIZ SHIPPING CORPORATION v. COURT OF APPEALS


GR No. 84458
November 6, 1989

Regalado, J.

Facts:

Anacleto Viana boarded the vessel M/V Antonia owned by petitioner Aboitiz Shipping Corp. at port
San Jose, Occidental Mindoro, bund for Manila. The vessel arrived at Oier 4, North Harbor, Manila and was
taken over by Pioneer Stevedoring for the latter to unload the cargoes from the said vessel pursuant to their
Memorandum of Agreement. An hour after the passengers and the vessel the crane operator began its
unloading operation. While the crane was being operated, Viana who had already disembarked the vessel
remembered that some of his cargoes were still loaded there. He went back and while he was pointing to the
crew where his cargoes were, the crane hit him pinning him between the side of the vessel and the crane
resulting to his death. A complaint for damages was filed against petitioner for breach of carriage. Petitioner
contends that Viana ceased to be a passenger when he disembarked the vessel and that consequently his
presence there was no longer reasonable. CA affirmed the trial court’s order holding Aboitiz liable. Hence, the
petition.

Issue:

Is the petitioner still responsible as a carrier to Viana after the latter had already disembarked the
vessel?

Ruling:

Yes. The rule that the relation of the carrier and passenger continues until the passenger has been
landed at the port of destination and has left the vessel owner’s dock or premises. Once created, the
relationship will not ordinarily terminate until the passenger has, after reaching his destination, safely alighted
from the carrier’s premises. All persons who remain on the premises. All persons who remain on the premises
a reasonable time after leaving the conveyance are to be deemed passengers, and what is a reasonable delay
within this rule is to be determined from all the circumstances, and includes a reasonable time to see after his
baggage and prepare for his departure. The carrier-passenger relationship is not terminated merely by the
fact that the person has been carried to his destination if, for example, such person remains in the carrier’s
premises to claim his baggage.

The primary factor to be considered is the existence of a reasonable cause as will justify the presence
of the victim on or near the petitioner’s vessel. We believe there exists such a justifiable cause. The accident
occurred, the victim was in the act of unloading the cargoes, which he had every right to do, from petitioner’s
vessel. As earlier stated, a carrier is duty bound not only to bring its passengers safely to their destination but
also to afford them a reasonable time to claim their baggage.

Consequently, under the foregoing circumstances, the victim Anacleto Viana is still deemed a
passenger of said carrier at the time of his tragic death.

27 5
Cl a im S et tl em e n t A nd S ub r oga ti o n: Co n di ti o ns b ef o r e th e I nsu r e d ma y r ec ei v e o n th e
pol icy a f t e r th e l o ss

FILIPINO MERCHANTS INSURANCE CO., INC. v. COURT OF APPEALS and CHOA TIEK SENG
G.R. No. 85141
November 28, 1989

Regalado, J.

Facts:

In December 1976, plaintiff insured said shipment with defendant insurance company for the sum of
P267,653.59 for the goods described as 600 metric tons of fishmeal in new gunny bags of 90 kilos each from
Bangkok, Thailand to Manila against all risks under warehouse to warehouse terms. Actually, what was
imported was 59.940 metric tons not 600 tons at $395.42 a ton CNF Manila. The fishmeal in 666 new gunny
bags were unloaded from the ship on December 11, 1976 at Manila unto the arrastre contractor E. Razon, Inc.
and defendant's surveyor ascertained and certified that in such discharge 105 bags were in bad order
condition as jointly surveyed by the ship's agent and the arrastre contractor. The condition of the bad order
was reflected in the turn over survey report of Bad Order cargoes. The cargo was also surveyed by the arrastre
contractor before delivery of the cargo to the consignee and the condition of the cargo on such delivery was
reflected in E. Razon's Bad Order Certificate. Defendant's surveyor has conducted a final and detailed survey
of the cargo in the warehouse for which he prepared a survey report with the findings on the extent of shortage
or loss on the bad order bags totalling 227 bags amounting to 12,148 kilos. Based on said computation the
plaintiff made a formal claim against the defendant Filipino Merchants Insurance Company for P51,568.62.
The trial, court after trial on the merits, rendered judgment in favor of private respondent. On appeal, the
respondent court affirmed the decision of the lower court.

Issue:

Whether the private respondent had insurable interest in the subject cargo.

Ruling:

The court held that private respondent, as consignee of the goods in transit under an invoice
containing the terms under “C & F Manila,” has insurable interest in said goods. Section 13 of the Insurance
Code defines insurable interest in property as every interest in property, whether real or personal, or any
relation thereto, or liability in respect thereof, of such nature that a contemplated peril might directly damnify
the insured. In principle, anyone has an insurable interest in property who derives a benefit from its existence
or would suffer loss from its destruction whether he has or has not any title in, or lien upon or possession of
the property. private respondent, as vendee/consignee of the goods in transit has such existing interest therein
as may be the subject of a valid contract of insurance. His interest over the goods is based on the perfected
contract of sale. The perfected contract of sale between him and the shipper of the goods operates to vest in
him an equitable title even before delivery or before be performed the conditions of the sale. The contract of
shipment, whether under F.O.B., C.I.F., or C. & F. as in this case, is immaterial in the determination of whether
the vendee has an insurable interest or not in the goods in transit. The perfected contract of sale even without
delivery vests in the vendee an equitable title, an existing interest over the goods sufficient to be the subject of
insurance.

27 6
Cl a im S et tl em e n t A nd S ub r oga ti o n: Co n di ti o ns b ef o r e th e I nsu r e d ma y r ec ei v e o n th e
pol icy a f t e r th e l o ss

CALTEX (PHILIPPINES), INC. v. SULPICIO LINES, INC., et al.


G.R. No. 131166
September 30, 1999

Pardo, J.

Facts:

The motor tanker MT Vector left Bataan to Masbate and loaded with 8,800 barrels of Petroleum
products shipped by petitioner. During the voyage, MT Vector, owned and operated by Vector Shipping
Corporation, carried on board the products of the Caltex under a contract of charter. While the passenger ship
MV Dona Paz left the port of Tacloban headed to Manila, with 59 crew members and total passengers of 1,493.
MV Dona Paz is owned and operated by responded Sulpicio Lines. The two vessels collided in the open sea
within the vicinity of Dumali Point between Marinduque and Oriental Mindoro. All the crewmembers of MV
Doa Paz died, while the two survivors from MT Vector claimed that they were sleeping at the time of the
incident. In BMI Case No. 653-87 after investigation it was found that the MT Vector were at fault and
responsible for its collision with MV Doa Paz. From this, a complaint was filed for damages was filed against
owners of MT Vector and Caltex.

Issue:

Is the charter of a sea vessel liable for damages resulting from collision between the chartered vessel
and a passenger shipment?

Ruling:

No. The charterer has no liability under the Philippine Maritime laws. Caltex and Vector entered into
a contract of affreightment, also known as voyage charter. A Charter party is a contract by which the entire
ship or parts thereof is lent by the owner to another person for a for a specified time or use. A contract of
affreightment, on the other hand, is one by which, the owner of a ship or vessel lends the whole or part of her
to a merchant or other person for transfer of goods, on a particular voyage, for a consideration. In this case, the
general owner the rights still are within the owner. The charterer is free from liability. MT Vector is a common
carrier. The charter party did not convert the common carrier into a private carrier.

27 7
Cl a im S et tl em e n t A nd S ub r oga ti o n: Co n di ti o ns b ef o r e th e I nsu r e d ma y r ec ei v e o n th e
pol icy a f t e r th e l o ss

DELSAN TRANSPORT LINES, INC. v. THE HON. COURT OF APPEALS and AMERICAN HOME
ASSURANCE CORPORATION
G.R. No. 127897
November 15, 2001
De leon, Jr., J.

Facts:

Caltex Philippines (Caltex) entered into a contract of affreightment with Delsan Transport Lines,
Inc.(Delsan), a common carrier, for a period of one year whereby latter agreed to transport Caltex’s industrial
fuel oil from the Batangas-Bataan Refinery to different parts of the country. Delsan took on board its vessel,
MT Maysun 2,277.314 kiloliters of industrial fuel oil of Caltex to be delivered to the Caltex Oil Terminal in
Zamboanga City. The shipment was insured with, American Home Assurance Corporation (AHAC). MT
Maysum set sail from Batangas for Zamboanga City. Unfortunately, the vessel sank near Panay Gulf in the
Visayas taking with it the entire cargo of fuel oil. AHAC paid Caltex the sum of Five Million Ninety-Six Thousand
Six Hundred Thirty-Five Pesos and Fifty-Seven Centavos (P5, 096,635.67) as an indemnity to the vessel which
sank. Exercising its right of subrogation under Article 2207 of the New Civil Code, AHAC demanded of Delsan
the same amount it paid to Caltex. Delsan failed to pay, hence, AHAC filed a complaint for collection of a sum of
money due to its failure to collect from the former. The trial court on the other hand dismissed the complaint,
but, the Court of Appeals reversed the decision.

Issue:

1. Was the payment made by the AHAC to Caltex for the insured value of the lost cargo amounted to an
admission that the vessel was seaworthy, thus precluding any action for recovery against the
petitioner?
2. Was the non-presentation of the marine insurance policy bars the complaint for recovery of sum of
money for lack of cause of action?

Ruling:

1. No. The payment made by the AHAC for the insured value of the lost cargo operates as waiver
of its (private respondent) right to enforce the term of the implied warranty against Caltex under the marine
insurance policy. However, the same cannot be validly interpreted as an automatic admission of the vessel’s
seaworthiness by the AHAC as to foreclose recourse against the Delsan for any liability under its contractual
obligation as a common carrier. The fact of payment grants the private respondent subrogatory right which
enables it to exercise legal remedies that would otherwise be available to Caltex as owner of the lost cargo
against the Delsan common carrier. Article 2207 of the New civil Code provides that:
Art. 2207. If the plaintiff’s property has been insured, and he has received indemnity from the
insurance company for the injury or loss arising out of the wrong or breach of contract
complained of, the insurance company shall be subrogated to the rights of the insured against the
wrongdoer or the person who has violated the contract. If the amount paid by the insurance
company does not fully cover the injury or loss, the aggrieved party shall be entitled to recover
the deficiency from the person causing the loss or injury

2. No. The presentation in evidence of the marine insurance policy is not indispensable in this
case before the insurer may recover from the common carrier the insured value of the lost cargo in the exercise
of its subrogatory right. The subrogation receipt, by itself, is sufficient to establish not only the relationship of
herein private respondent as insurer and Caltex, as the assured shipper of the lost cargo of industrial fuel oil,

27 8
Cl a im S et tl em e n t A nd S ub r oga ti o n: Co n di ti o ns b ef o r e th e I nsu r e d ma y r ec ei v e o n th e
pol icy a f t e r th e l o ss

but also the amount paid to settle the insurance claim. The right of subrogation accrues simply upon payment
by the insurance company of the insurance claim.

27 9
Cl a im S et tl em e n t A nd S ub r oga ti o n: Co n di ti o ns b ef o r e th e I nsu r e d ma y r ec ei v e o n th e
pol icy a f t e r th e l o ss

CEBU SALVAGE CORPORATION v. PHILIPPINE HOME ASSURANCE CORPORATION


G.R. No. 150403
January 25, 2007

Corona, J.

Facts:

Cebu Salvage Corporation and Maria Cristina Chemicals Industries (MCCII) entered into a voyage
charter, a contract of affreightment wherein the ship was leased for a single voyage for the conveyance of
goods, in consideration of the payment of freight, wherein Cebu Salvage was to load silica quartz on board the
M/T Espiritu Santo for transport and discharge. The shipment never reached its destination because the M/T
Espiritu Santo sank, resulting in the total loss of the cargo. MCCII filed a claim with its insurer, Philippine Home
Assurance, which paid the claim in the amount of P211500 and was subrogated to the rights of MCCII.
Thereafter, it filed a case against Cebu Salvage for reimbursement. The RTC ordered Cebu Salvage to pay
MCCII. The CA affirmed the decision of the RTC.

Issue:

Is MCCII liable for its own loss since the voyage charter stipulated that cargo insurance was for the
charterer’s account?

Ruling:

No. This simply meant that the charterer would take care of having the goods insured. It could not
exculpate the carrier from liability for the breach of its contract of carriage. The law, in fact, prohibits it and
condemns it as unjust and contrary to public policy. Moreover, MCCII never dealt with ALS Timber Enterprises
(ALS) and yet Cebu Salvage insists that MCCII should sue ALS for reimbursement for its loss. Certainly, to
permit a common carrier to escape its responsibility for the goods it agreed to transport (by the expedient of
alleging non-ownership of the vessel it employed) would radically derogate from the carrier’s duty of
extraordinary diligence. It would also open the door to collusion between the carrier and the supposed owner
and to the possible shifting of liability from the carrier to one without any financial capability to answer for the
resulting damages.

28 0
Cl a im S et tl em e n t A nd S ub r oga ti o n: Co n di ti o ns b ef o r e th e I nsu r e d ma y r ec ei v e o n th e
pol icy a f t e r th e l o ss

KEPPEL CEBU SHIPYARD, INC. v. PIONEER INSURANCE AND SURETY CORPORATION


G.R. Nos. 180880-81
September 25, 2009

Nachura, J.

Facts:

`Keppel Cebu Shipyard, Inc. (KCSI) and WG&A Jebsens Shipmanagement, Inc. (WG&A) executed a
Shiprepair Agreement wherein KCSI would renovate and reconstruct WG&A’s M/V Superferry 3 using its dry
docking facilities pursuant to its restrictive safety and security rules and regulations. Prior to the execution of
the agreement, Superferry 3 was already insured with Pioneer Insurance. In the course of its repair, Superferry
3 was gutted by fire. Claiming that the extent of the damage was pervasive, WG&A filed an insurance claim
with Pioneer, which paid the insurance claim. WG&A, in turn, executed a Loss and Subrogation Receipt in
favour of Pioneer. Armed with the subrogation receipt, Pioneer tried to collect from KCSI. Pioneer filed a
Request for Arbitration before the Construction Industry Arbitration Commission pursuant to the unheeded
demands from KCSI. The CIAC declared both WG&A and KCSI guilty of negligence. The CIAC ordered KCSI to
pay Pioneer. Both Pioneer and KCSI appealed to the CA. Pioneer argued that there existed a total constructive
loss so that it had to pay WG&A the full amount of the insurance coverage. KCSI insists on the validity of the
limited-liability clause. The CA, however, ordered KCSI to pay Pioneer.

Issue:

Is Pioneer liable to pay the total constructive loss in the full amount of the insurance coverage and, by
operation of law, it was entitled to be subrogated to the rights of WG&A to claim the amount of the loss?

Ruling:

Yes. It cannot be denied that M/V Superferry 3 suffered widespread damage from the fire that
occurred on February 8, 2000, a covered peril under the marine insurance policies obtained by WG&A from
Pioneer. The estimates given by the three disinterested and qualified shipyards show that the damage to the
ship would exceed P270,000,000.00, or of the total value of the policies P360,000,000.00. These estimates
constituted credible and acceptable proof of the extent of the damage sustained by the vessel. It is significant
that these estimates were confirmed by the Adjustment Report dated June 5, 2000 submitted by Richards
Hogg Lindley (Phils.), Inc., and the average adjuster that Pioneer had enlisted to verify and confirm the extent
of the damage. The Adjustment Report verified and confirmed that the damage to the vessel amounted to a
constructive total loss and that the claim for P360,000,000.00 under the policies was compensable. It is also
noteworthy that KCSI did not cross-examine Henson Lim, Director of Richards Hogg, whose affidavit-direct
testimony submitted to the CIAC confirmed that the vessel was a constructive total loss.

Furthermore, The Court ruled as unacceptable the claim of KCSI that there was no ample proof of
payment simply because the person who signed the Receipt appeared to be an employee of Aboitiz Shipping
Corporation. The Loss and Subrogation Receipt issued by WG&A to Pioneer is the best evidence of payment
of the insurance proceeds to the former, and no controverting evidence was presented by KCSI to rebut the
presumed authority of the signatory to receive such payment.

28 1
Cl a im S et tl em e n t A nd S ub r oga ti o n: Co n di ti o ns b ef o r e th e I nsu r e d ma y r ec ei v e o n th e
pol icy a f t e r th e l o ss

EASTERN SHIPPING LINES, INC. v. BPI/MS INSURANCE CORP., & MITSUI SUMITOMO
INSURANCE CO., LTD.
G.R. No. 182864
January 12, 2015

Perez, J.

Facts:

On two separate occasions, BPI/MS and Mitsui shipped on board Eastern Shipping Lines, Inc. (ESLI)’s
vessels 22 and 55 coils of various steel sheets for transportation to and delivery at Manila in favor of consignee
Calamba Steel Center. The shipments were insured with BPI/MS and Mitsui against all risks. Upon arrival at
the port of Manila, the shipments were turned over to Asian Terminals, Inc. for safekeeping. Upon withdrawal
by Calamba Steel, it was found out on both occasions that part of the shipment was damaged and was in bad
order, prompting Calamba Steel to reject the damaged shipment. When ESLI and ATI refused to pay, Calamba
Steel filed an insurance claim for the total amount of the cargo against BPI/MS and Mitsui as cargo insurers. As
a result, BPI/MS and Mitsui became subrogated in place of and with all the rights and favors accorded by law
in favor of Calamba Steel. BPI/MS and Mitsui filed a complaint against ESLI and ATI to recover actual damages
amounting before the Regional Trial Court. The RTC found both the ESLI and ATI liable for the damages
sustained by the two shipments. On appeal, the CA absolved ATI from liability.

Issue:

Is ESLI liable and to what extent is its liability?

Ruling:

Yes. Mere proof of delivery of the goods in good order to a common carrier and of their arrival in bad
order at their destination constitutes a prima facie case of fault or negligence against the carrier. If no adequate
explanation is given as to how the deterioration, loss, or destruction of the goods happened, the transporter
shall be held responsible. From the foregoing, the fault is attributable to ESLI. While no longer an issue, it may
be nonetheless state that ATI was correctly absolved of liability for the damage.

Moreover, a review of the bill of ladings and invoice on the second shipment indicates that the shipper
declared the nature and value of the goods with the corresponding payment of the freight on the bills of lading.
Further, under the caption “description of packages and goods,” it states that the description of the goods to be
transported as “various steel sheet in coil” with a gross weight of 383,532 kilograms (89.510 M3). On the other
hand, the amount of the goods is referred in the invoice, the due execution and genuineness of which has
already been admitted by ESLI, is US$186,906.35 as freight on board with payment of ocean freight of
US$32,736.06 and insurance premium of US$1,813.17. From the foregoing, we rule that the non- limitation of
liability applies in the present case.

28 2
Cl a im S et tl em e n t A nd S ub r oga ti o n: Co n di ti o ns b ef o r e th e I nsu r e d ma y r ec ei v e o n th e
pol icy a f t e r th e l o ss

PHILIPPINE AMERICAN LIFE INSURANCE COMPANY v. COURT OF APPEALS and ELIZA PULIDO
G.R. No. 126223
November 15, 2000

Gonzaga-Reyes, J.

Facts:

Philipine American Life Insurance Company (Philam) received from one Florence Pulido an
application for life insurance in the amount of P100,000.00 which designated her sister, Eliza Pulido, as its
principal beneficiary. Because the insurance applied for was non-medical, petitioner did not require a medical
examination and issued a policy. Later, Philam received Eliza’s claim, which declared that the insured, Florence
Pulido, died of acute pneumonia on September 10, 1991. However, Philam withheld payment on the ground
that the policy claimed under was void from the start for having been procured in fraud. According to Philam,
it came to know about the death of Florence in 1988 even before the application of the insurance was made
by virtue of their investigation, even before the claim of Eliza. Following the filing by Eliza of her claim, Philam
caused another investigation respecting the subject policy. Pursuant to the findings of this second
investigation, Philam stood by its initial decision to treat the policy as void and not to honor the claim. Thus,
this prompted Eliza to enlist the services of counsel in reiterating her claim for death benefits. However, Philam
still refused to make payment.

Issue:

Is there a finding of fraud in the obtainment of Florence Pulidos insurance policy?

Ruling:

No. A duly-registered death certificate is considered a public document and the entries found therein
are presumed correct, unless the party who contests its accuracy can produce positive evidence establishing
otherwise. Philam’s contention that the death certificate is suspect because Dr. Gutierrez was not present
when Florence Pulido died, and knew of Florences death only through Ramon Piganto, does not merit a
conclusion of fraud. No motive was imputed to Dr. Gutierrez for seeking to perpetuate a falsity in public
records. Petitioner was likewise unable to make out any clear motive as to why Ramon Piganto would
purposely lie. Mere allegations of fraud could not substitute for the full and convincing evidence that is
required to prove it. A failure to do so would leave intact the presumption of good faith and regularity in the
performance of public duties, which was the basis of both respondent court and the trial court in finding the
date of Florence Pulidos death to be as plaintiff-private respondent maintained.

28 3
Cl a im S et tl em e n t A nd S ub r oga ti o n: Co n di ti o ns b ef o r e th e I nsu r e d ma y r ec ei v e o n th e
pol icy a f t e r th e l o ss

PRUDENTIAL GUARANTEE and ASSURANCE, INC. v. EQUINOX LAND CORPORATION


G.R. Nos. 152505-06
September 13, 2007

Sandoval-Gutierrez, J.

Facts:

Equinox Land Corporation (Equinox) decided to construct five additional floors to its existing
building, the Eastgate Centre. It then sent invitations to bid to various building contractors. Four building
contractors, including J’Marc Construction & Development Corporation (J’Marc), responded. Finding the bid
of J’Marc to be the most advantageous, Equinox offered the construction project to it. Upon acceptance, two
days later, Equinox formally awarded to J’Marc the contract to build the extension. Thereafter, J’Marc
submitted to Equinox two (2) bonds, namely: (1) a surety bond issued by Prudential Guarantee and Assurance,
Inc. (Prudential), to guarantee the unliquidated portion of the advance payment payable to J’Marc; and (2) a
performance bond likewise issued by Prudential to guarantee J’Marc’s faithful performance of its obligations
under the construction agreement. Equinox and J’Marc signed the contract and related documents. Under the
terms of the contract, J’Marc would supply all the labor, materials, tools, equipment, and supervision required
to complete the project. J’Marc did not adhere to the terms of the contract. It failed to finish the required work,
only 19% of the total project was finished.

Issue:

Is Prudential free from liability claiming that only J’Marc is solidarily liable?

Ruling:

NO. It is not disputed that Prudential entered into a suretyship contract with J’Marc. Section 175 of the
Insurance Code defines a suretyship as "a contract or agreement whereby a party, called the suretyship,
guarantees the performance by another party, called the principal or obligor, of an obligation or undertaking
in favor of a third party, called the obligee. It includes official recognizances, stipulations, bonds, or
undertakings issued under Act 5368, as amended." Corollarily, Article 2047 of the Civil Code provides that
suretyship arises upon the solidary binding of a person deemed the surety with the principal debtor for the
purpose of fulfilling an obligation. In Security Pacific Assurance Corporation v. Tria-Infante, the Court reiterated
the rule that while a contract of surety is secondary only to a valid principal obligation, the surety’s liability to
the creditor is said to be direct, primary, and absolute. In other words, the surety is directly and equally bound
with the principal. Thus, Prudential is barred from disclaiming that its liability with J’Marc is solidary.

28 4
Cl a im S et tl em e n t A nd S ub r oga ti o n: Co n di ti o ns b ef o r e th e I nsu r e d ma y r ec ei v e o n th e
pol icy a f t e r th e l o ss

ABOITIZ SHIPPING CORPORATION v. INSURANCE COMPANY OF NORTH AMERICA


G.R. No. 168402
August 6, 2008

Reyes, R.T., J.

Facts:

MSAS Cargo International Limited and/or Associated and/or Subsidiary Companies (MSAS) procured
a marine insurance policy from respondent Insurance Company of North America (ICNA) UK Limited of
London. The insurance was for a transhipment of certain wooden work tools and workbenches purchased for
the consignee Science Teaching Improvement Project. ICNA issued an "all-risk" open marine policy. The cargo,
packed inside one container van, was shipped "freight prepaid" from Germany. Thereafter, the cargo was
received by Aboitiz Shipping Corporation (Aboitiz). The bill of lading issued by Aboitiz contained the notation
"grounded outside warehouse." The container van was stripped and transferred to another crate/container
van. When the shipment arrived, in the Stripping Report, Aboitiz's checker noted that the crates were slightly
broken or cracked at the bottom. Thus, the cargo was withdrawn by the representative of the consignee, STIP
and delivered to Don Bosco Technical High School. The parcel was received by a certain Mr. Bernhard Willig,
he informed Aboitiz that the cargo sustained water damage. However, it was found that the container van and
other cargoes stuffed there were completely dry and showed no sign of wetness. In a letter, Willig informed
Aboitiz of the damage noticed upon opening of the cargo. The letter concluded that apparently, the damage
was caused by water entering through the broken parts of the crate.

The consignee contacted the Philippine office of ICNA for insurance claims. The Claimsmen
Adjustment Corporation conducted an ocular inspection and survey of the damage. CAC reported to ICNA that
the goods sustained water damage, molds, and corrosion which were discovered upon delivery to consignee.
CAC noted that the shipment was placed outside the warehouse of Pier No. 4, North Harbor, Manila when it
was delivered. Aboitiz refused to settle the claim. ICNA paid the amount of P280,176.92 to consignee. A
subrogation receipt was duly signed by Willig. ICNA formally advised Aboitiz of the claim and subrogation
receipt executed in its favor. Despite followups, however, no reply was received from Aboitiz.

Issue:

Is ICNA the real party-in-interest that possesses the right of subrogation to claim reimbursement
from petitioner Aboitiz?

Ruling:

Yes. A foreign corporation not licensed to do business in the Philippines is not absolutely incapacitated
from filing a suit in local courts. ICNA's cause of action is founded on it being subrogated to the rights of the
consignee of the damaged shipment. The right of subrogation springs from Article 2207 of the Civil Code,
which states:
Article 2207. If the plaintiff's property has been insured, and he has received indemnity from the
insurance company for the injury or loss arising out of the wrong or breach of contract
complained of, the insurance company shall be subrogated to the rights of the insured against the
wrongdoer or the person who has violated the contract. If the amount paid by the insurance
company does not fully cover the injury or loss, the aggrieved party shall be entitled to recover
the deficiency from the person causing the loss or injury. (Emphasis added)

28 5
Cl a im S et tl em e n t A nd S ub r oga ti o n: Co n di ti o ns b ef o r e th e I nsu r e d ma y r ec ei v e o n th e
pol icy a f t e r th e l o ss

Upon payment to the consignee of indemnity for damage to the insured goods, ICNA's entitlement to
subrogation equipped it with a cause of action against petitioner in case of a contractual breach or negligence.

This right of subrogation, however, has its limitations. First, both the insurer and the consignee are
bound by the contractual stipulations under the bill of lading. Second, the insurer can be subrogated only to
the rights as the insured may have against the wrongdoer. If by its own acts after receiving payment from the
insurer, the insured releases the wrongdoer who caused the loss from liability, the insurer loses its claim
against the latter. Furthermore, the giving of notice of loss or injury is a condition precedent to the action for
loss or injury or the right to enforce the carrier's liability. Circumstances peculiar to this case lead us to
conclude that the notice requirement was complied with. Under the Code of Commerce, the notice of claim
must be made within twenty four (24) hours from receipt of the cargo if the damage is not apparent from the
outside of the package. For damages that are visible from the outside of the package, the claim must be made
immediately. To recapitulate, we have found that ICNA, as subrogee of the consignee, is the real party in
interest to institute the claim for damages against petitioner; and pro hac vice that a valid notice of claim was
made by ICNA.

28 6
Cl a im S et tl em e n t A nd S ub r oga ti o n: Co n di ti o ns b ef o r e th e I nsu r e d ma y r ec ei v e o n th e
pol icy a f t e r th e l o ss

MALAYAN INSURANCE CO., INC. v. RODELIO ALBERTO and ENRICO ALBERTO REYES
G.R. No. 194320
February 1, 2012

Velasco, Jr., J.

Facts:

An accident occurred at the corner of EDSA and Ayala Avenue, Makati City, involving four (4) vehicles,
to wit: (1) a Nissan Bus operated by Aladdin Transit with plate number NYS 381; (2) an Isuzu Tanker with
plate number PLR 684; (3) a Fuzo Cargo Truck with plate number PDL 297; and (4) a Mitsubishi Galant with
plate number TLM 732. Previously, Malayan Insurance issued Car Insurance Policy No. PV-025-00220 in favor
of First Malayan Leasing and Finance Corporation (the assured), insuring the aforementioned Mitsubishi
Galant against third party liability, own damage and theft, among others. Having insured the vehicle against
such risks, Malayan Insurance claimed in its Complaint that it paid the damages sustained by the assured.
Maintaining that it has been subrogated to the rights and interests of the assured by operation of law upon its
payment to the latter, Malayan Insurance sent several demand letters to Rodelio Alberto (Alberto) and Enrico
Alberto Reyes (Reyes), the registered owner and the driver, respectively, of the Fuzo Cargo Truck, requiring
them to pay the amount it had paid to the assured. Alberto and Reyes refused to settle their liability. The trial
court ruled in favor of Malayan Insurance, which was reversed by the Court of Appeals. Malayan Insurance
contends that there was a valid subrogation in the instant case, as evidenced by the claim check voucher and
the Release of Claim and Subrogation Receipt presented by it before the trial court. Respondents, however,
claim that the documents presented by Malayan Insurance do not indicate certain important details that
would show proper subrogation

ISSUE:

Is the subrogation of Malayan Insurance impaired and/or deficient?

RULING:

Yes. As noted by Malayan Insurance, Alberto and Reyes had all the opportunity, but failed to object to
the presentation of its evidence. Thus, and as we have mentioned earlier, Alberto and Reyes are deemed to
have waived their right to make an objection. It contemplates full substitution such that it places the party
subrogated in the shoes of the creditor, and he may use all means that the creditor could employ to enforce
payment. We have held that payment by the insurer to the insured operates as an equitable assignment to the
insurer of all the remedies that the insured may have against the third party whose negligence or wrongful act
caused the loss. The right of subrogation is not dependent upon, nor does it grow out of, any privity of contract.
It accrues simply upon payment by the insurance company of the insurance claim. The doctrine of subrogation
has its roots in equity. It is designed to promote and to accomplish justice; and is the mode that equity adopts
to compel the ultimate payment of a debt by one who, in justice, equity, and good conscience, ought to pay

28 7
Cl a im S et tl em e n t A nd S ub r oga ti o n: Co n di ti o ns b ef o r e th e I nsu r e d ma y r ec ei v e o n th e
pol icy a f t e r th e l o ss

UNITED MERCHANTS CORPORATION v. COUNTRY BANKERS INSURANCE CORPORATION


G.R. No. 198588
11 July 2012

Carpio, J.

Facts:

United Merchants Corporation (UMC) leased a warehouse where it assembled and stored its
products. UMC’s General Manager insured UMC’s stocks in trade of Christmas lights against fire with Country
Bankers Insurance Corporation (CBIC). UMC and CBIC executed Endorsement F/96-154 and Fire Invoice No.
16583A to form part of the Insurance Policy. Endorsement F/96-154 provides that UMC’s stocks in trade were
insured against additional perils, to wit: "typhoon, flood, ext. cover, and full earthquake." The sum insured was
also increased. CBIC issued Endorsement F/96-157 where the name of the assured was changed from Alfredo
Tan to UMC. Thereafter, a fire gutted the warehouse rented by UMC. CBIC designated CRM Adjustment
Corporation (CRM) to investigate and evaluate UMC’s loss by reason of the fire. CBIC’s reinsurer, Central
Surety, likewise requested the National Bureau of Investigation (NBI) to conduct a parallel investigation. UMC,
through CRM, submitted to CBIC its Sworn Statement of Formal Claim, with proofs of its loss. UMC demanded
for at least fifty percent (50%) payment of its claim from CBIC. UMC received CBIC’s letter rejecting UMC’s
claim due to breach of Condition No. 15 of the Insurance Policy. CBIC alleged that UMC’s claim was fraudulent
because UMC’s Statement of Inventory showed that it had no stocks in trade and that UMC’s suspicious
purchases for the year 1996 did not even amount to ₱25,000,000.00. UMC’s GIS and Financial Reports further
revealed that it had insufficient capital, which meant UMC could not afford the alleged ₱50,000,000.00 worth
of stocks in trade.

Issue:

Is UMC entitled to claim from CBIC the full coverage of its fire insurance policy?

Ruling:

No. In the present case, as proof of its loss of stocks in trade amounting to ₱50,000,000.00, UMC
submitted its Sworn Statement of Formal Claim together with the following documents: (1) letters of credit
and invoices for raw materials, Christmas lights and cartons purchased; (2) charges for assembling the
Christmas lights; and (3) delivery receipts of the raw materials. However, the charges for assembling the
Christmas lights and delivery receipts could not support its insurance claim. The Insurance Policy provides
that CBIC agreed to insure UMC’s stocks in trade. UMC defined stock in trade as tangible personal property
kept for sale or traffic. Applying UMC’s definition, only the letters of credit and invoices for raw materials,
Christmas lights and cartons may be considered.

It has long been settled that a false and material statement made with an intent to deceive or defraud
voids an insurance policy. The most liberal human judgment cannot attribute such difference to mere
innocent error in estimating or counting but to a deliberate intent to demand from insurance companies’
payment for indemnity of goods not existing at the time of the fire. This constitutes the so-called "fraudulent
claim" which, by express agreement between the insurers and the insured, is a ground for the exemption of
insurers from civil liability.

28 8
Cl a im S et tl em e n t A nd S ub r oga ti o n: Co n di ti o ns b ef o r e th e I nsu r e d ma y r ec ei v e o n th e
pol icy a f t e r th e l o ss

STRONGHOLD INSURANCE COMPANY, INCORPORATED v. INTERPACIFIC CONTAINER


SERVICES AND GLORIA DEE CHONG
G.R. No. 194328
July 01, 2015

Perez, J.

Facts:

Gloria Dee Chong is the owner of a Fuso truck. The vehicle was insured by Stronghold Insurance
Company a comprehensive motor car insurance policy undertook to indemnify the insured against loss or
damage to the car and death or injury caused to third persons by reason of accident. While the policy was in
effect, the vehicle figured in an accident resulting in the death of four persons while seriously injuring three
others. Two vehicles were also heavily damaged as a result of the accident. Pursuant to the provisions of the
insurance contract, Chong filed a claim for the recovery of the proceeds of her policy. Stronghold denied the
claim saying that the accident happened because the driver of the insured vehicle was intoxicated and such
intoxication legally avoided the insurance policy of the Chong. Chong alleged that there was no sufficient basis
on the Stronghold’s claim and prompted the respondent to file a collection of money.

Issue:

Is the Chong entitled for the insurance claim?

Ruling:

Yes. Stronghold failed to prove that at the time of the accident the truck driver was intoxicated
thereby precluding the Chong in the insurance claim. Contrary to the claim of the Stronghold, it miserably
failed to prove the fact of intoxication during the trial. Aside from the Medico Legal Certificate and the
Pagpapatunay, which were omitted of evidentiary value because of the dubious circumstances under which
they were obtained, the Stronghold did not adduce other proof to justify the avoidance of the policy. It must
be emphasized that the RTC doubted the authenticity of the Medico Legal Certificate because of the attendant
alteration and tampering on the face of the document. In adopting the findings of the trial court, the appellate
court reiterated the evidentiary rule that the party alleging violation of the provision of the contract bears the
burden of proof to prove the same. What further dampens Stronghold's position is the absence of the crucial
fact of intoxication in the blotter report which officially documented the incident. Entries in police records
made by a police officer in the performance of the duty especially enjoined by law are prima facie evidence of
the fact therein stated, and their probative value may be substantiated or nullified by other competent
evidence.

28 9
Cl a im S et tl em e n t A nd S ub r oga ti o n: Co n di ti o ns b ef o r e th e I nsu r e d ma y r ec ei v e o n th e
pol icy a f t e r th e l o ss

BANK OF THE PHILIPPINE ISLANDS AND FGU INSURANCE CORPORATION (PRESENTLY


KNOWN AS BPI/MS INSURANCE CORPORATION v. YOLANDA LAINGO
G.R. No. 205206
March 16, 2016

Carpio, J.

Facts:

Rheozel Liango, the son of Yolanda Liango died due to a vehicular accident. Yolanda, instructed the
family’s personal secretary, Alice to go to BPI, Davao branch to inquire about the savings account of Rheozel
because Yolanda want to use the amount for Rheozel’s burial expenses. Alice was able to withdraw from
Rheozel’s account. Two years later, the sister of Rheozel found the Personal Accident Insurance Coverage
Certificate issued by FGU insurance. Rhealyn immediately informed Yolanda. Thereafter, Yolanda sent two
letters to BPI FGU Insurance requesting them to process her claim as beneficiary of Rheozel’s insurance policy
but the insurance company denied the claim stating that she should have filed the claim within three calendar
moths from the death of Rheozel as stated in the Accident Certificate of Insurance. Aggrieved, Yolanda then
filed a specific Performance with Damages And attorney’s Fees with the Regional Trial Court. In its decision
the court decided the case in favor of BPI et al. It was ruled that the prescriptive period of 90 days shall
commence from the time of death of the insured and not from the knowledge of the beneficiary. Since the
insurance claim was filed more than 90 days from the death of the insured, the case must be dismissed. The
CA reversed the ruling of the trial court stating that Yolanda could not be expected to do an obligation which
she did not know existed. The appellate court added that Yolandawas not a party to the insurance contract
hence she could not be bound by the 90 day stipulation.

Issue:

Is Yolanda entitled for the insurance claim despite failure to file a claim within three calendar months
from the death of her son?

Ruling:

Yes. BPI as agent of FGU insurance company failed to notify Yolanda that she is a beneficiary hence the
90 day prescriptive period is not applicable. This case involves the doctrine of representation that notice to the
agent is notice to the principal. Yolanda claims that she had no knowledge that Rheozel was covered by an
insurance policy against disability or death. She attributed the fault to BPI and FGU insurance for failure to file
the notice of insurance claim because when she withdraws from BPI, she was not notified about the insurance
coverage. The Court agreed with Yolanda’s contention. Furthermore, Rheozel was one of those who availed of
the Offered savings account with life and disability of BPI. It was a marketing strategy to entice customers to
invest their money with the added benefit of an insurance policy. With this regard BPI tied with FGU insurance
as its partner hence, BPI acted as agent of FGU insurance with respect to the insurance feature of its own
marketed product. The law provides that an agent is one who binds himself to render some service or to do
something in representation of another.
BPI and FGU Insurance shall bear the loss and must compensate Laingo for the actual damages
suffered by her family plus attorney's fees. Likewise, FGU Insurance has the obligation to pay the insurance
proceeds of Rheozel's personal accident insurance coverage to Laingo, as Rheozel's named beneficiary.

29 0
Cl a im S et tl em e n t A nd S ub r oga ti o n: N o tic e a nd P r oo f o f l oss

FINMAN GENERAL ASSURANCE CORPORATION v. COURT OF APPEALS and USIPHIL


INCORPORATED
G.R. No. 138737
July 12, 2001

Kapunan, J.

Facts:

Usiphil Incorporated(Usiphil) obtained a fire insurance policy from Finman General Assurance
Corporation (then doing business under the name Summa Insurance Corporation) covering certain
properties, e.g., office, furniture, fixtures, shop machinery and other trade equipment. Under Policy No. F3100
issued to Usiphil, Finman undertook to indemnify Usiphil for any damage to or loss of said properties arising
from fire. Usiphil filed with Finman an insurance claim amounting to P987, 126.11 for the loss of the insured
properties due to fire. Finman appointed Adjuster H.H. Bayne to undertake the valuation and adjustment of
the loss. H.H. Bayne then required Usiphil to file a formal claim and submit proof of loss. Usiphil submitted its
Sworn Statement of Loss and Formal Claim and the Proof of Loss signed by its Accounting Manager Pedro
Palallos and countersigned by H.H. Bayne’s Adjuster F.C. Medina, in compliance with what is required of it.
Despite repeated demands by Usiphil, Finman refused to pay the insurance claim because the former failed to
comply with Policy Condition No. 13 regarding the submission of certain documents to prove the loss.

Issue:

Is the disallowance of Usiphil’s claim justified by its failure to submit the required documents in
accordance with Policy Condition No. 13?

Ruling:

Yes. Usiphil had substantially complied with Policy Condition No. 13. A perusal of the records shows
that Usiphil, after the occurrence of the fire, immediately notified Finman thereof. Thereafter, Usiphil
submitted the following documents: (1) Sworn Statement of Loss and Formal Claim and; (2) Proof of Loss. The
submission of these documents, to the Court’s mind, constitutes substantial compliance with the above
provision. Indeed, as regards the submission of documents to prove loss, substantial, not strict as urged by
Finman, compliance with the requirements will always be deemed sufficient. As correctly held by the appellate
court: xxx But, even assuming that plaintiff-appellee indeed failed to submit certain required documents as
proof of loss per Section 13, such violation was waived by the insurer Summa when it signed the document
marked Exhibit E, a breakdown of the amount due to plaintiff-appellee as of February 1985 on the insurance
claim. By such act, defendant-appellant acknowledged its liability under the insurance policy.

29 1
Cl a im S et tl em e n t A nd S ub r oga ti o n: N o tic e a nd P r oo f o f l oss

COUNTRY BANKERS INSURANCE CORPORATION v. LIANGA BAY AND COMMUNITY MULTI-


PURPOSE COOPERATIVE, INC.
G.R. No. 136914
January 25, 2002

De Leon, Jr., J.

Facts:

Country Banker’s Insurance Corp. (CBIC) insured the building of Lianga Bay and Community Multi-
Purpose Corp., Inc. against fire, loss, damage, or liability during the period starting June 20, 1990 for the sum
of Php.200,000.00. On July 1, 1989 at about 12:40 in the morning a fire occurred. The Lianga filed the insurance
claim but the petition denied the same on the ground that the building was set on fire by two NPA rebels and
that such loss was an excepted risk under par.6 of the conditions of the insurance policy that the insurance
does not cover any loss or damage occasioned by among others, mutiny, riot, military or any uprising. Thus,
Lianga filed an action for recovery of loss, damage or liability against CBIC and the Trial Court ordered the
latter to pay the full value of the insurance.

Issue:

Is Country banker correct that the cause of the loss was an excepted risk under the terms of the fire
insurance policy?

Ruling:

No. Where a risk is excepted by the terms of a policy which insures against other perils or hazards,
loss from such a risk constitutes a defense which the insurer may urge, since it has not assumed that risk, and
from this it follows that an insurer seeking to defeat a claim because of an exception or limitation in the policy
has the burden of proving that the loss comes within the purview of the exception or limitation set up. If a proof
is made of a loss apparently within a contract of insurance, the burden is upon the insurer to prove that the
loss arose from a cause of loss which is excepted or for which it is not liable, or from a cause which limits its
liability. Stated elsewise, since the petitioner in this case is defending on the ground of non-coverage and
relying upon an exemption or exception clause in the fire insurance policy, it has the burden of proving the
facts upon which such excepted risk is based, by a preponderance of evidence.

29 2
Cl a im S et tl em e n t A nd S ub r oga ti o n: N o tic e a nd P r oo f o f l oss

FGU INSURANCE CORPORATION v. THE COURT OF APPEALS, et al.


G.R. No. 137775
March 31, 2005

Chico-Nazario, J.

Facts:

Anco Enterprises Company (ANCO), a partnership between Ang Gui and Co To, was engaged
in the shipping business. It owned the M/T ANCO tugboat and the D/B Lucio barge which were
operated as common carriers. Since the D/B Lucio had no engine of its own, it could not maneuver
by itself and had to be towed by a tugboat for it to move from one place to another. San Miguel
Corporation (SMC) shipped from Cebu, on board the D/B Lucio, for towage by M/T ANCO 40,000
cases of beers. When the barge and tugboat arrived at San Jose, Antique, the clouds over the area
were dark and the waves were already big. The arrastre workers unloading the cargoes of SMC on
board the D/B Lucio complained about their difficulty in unloading the cargoes. SMCs District Sales
Supervisor requested ANCOs representative to transfer the barge to a safer place. Only 10,790 cases
of beer were discharged into the custody of the arrastre operator. Later that night, the crew of D/B
Lucio abandoned the vessel because the barges rope attached to the wharf was cut off by the big
waves. At around midnight, the barge run aground and was broken and the cargoes of beer in the
barge were swept away. As a result, ANCO failed to deliver to SMCs consignee 29,210 cases, hence,
SMCs claim against ANCO amounted to P1,346,197.00.

Thereafter, SMC filed a second amended impleading the surviving partner, Co To and the
Estate of Ang Gui. The substituted defendants adopted the original answer with counterclaim of
ANCO, it claimed however that it had an agreement with SMC that ANCO would not be liable for any
losses or damages resulting to the cargoes by reason of fortuitous event. ANCO further asserted that
there was an agreement between them and SMC to insure the cargoes in order to recover indemnity
in case of loss. Pursuant to that agreement, the cargoes to the extent of 20,000 cases was insured with
FGU Insurance Corporation (FGU) for the total amount of P858,500.00 per Marine Insurance Policy
No. 29591.

Issue:

Is FGU liable under the insurance contract considering the circumstances surrounding the
loss of the cargoes?

Ruling:

No. One of the purposes for taking out insurance is to protect the insured against the
consequences of his own negligence and that of his agents. Thus, it is a basic rule in insurance that
the carelessness and negligence of the insured or his agents constitute no defense on the part of the
insurer. This rule however presupposes that the loss has occurred due to causes which could not
have been prevented by the insured, despite the exercise of due diligence. The question now is
whether there is a certain degree of negligence on the part of the insured or his agents that will
deprive him the right to recover under the insurance contract. However, to what extent such
negligence must go in order to exonerate the insurer from liability must be evaluated in light of the
circumstances surrounding each case. When evidence show that the insured’s negligence or
recklessness is so gross as to be sufficient to constitute a willful act, the insurer must be exonerated.
According to the Court, while mistake and negligence of the master or crew are incident to navigation

29 3
Cl a im S et tl em e n t A nd S ub r oga ti o n: N o tic e a nd P r oo f o f l oss

and constitute a part of the perils that the insurer is obliged to incur, such negligence or recklessness
must not be of such gross character as to amount to misconduct or wrongful acts; otherwise, such
negligence shall release the insurer from liability under the insurance contract.

29 4
Cl a im S et tl em e n t A nd S ub r oga ti o n: N o tic e a nd P r oo f o f l oss

PHILIPPINE CHARTER INSURANCE CORPORATION v. CHEMOIL LIGHTERAGE CORPORATION


G.R. No. 136888
June 29, 2005

Chico-Nazario, J.

Facts:

Samkyung Chemical Company, Ltd., based in Ulsan, South Korea, shipped 62.06 metric tons of the
liquid chemical DIOCTYL PHTHALATE (DOP) on board MT TACHIBANA which was valued at US$90,201.57
and another 436.70 metric tons of DOP valued at US$634,724.89 to the Philippines. The consignee was Plastic
Group Phils., Inc. (PGP) in Manila. PGP insured the cargo with Philippine Charter Insurance Corporation (PCIC)
against all risks. The insurance was under Marine Policies for P31,757,969.19 and P4,514,881.00. The ocean
tanker MT TACHIBANA unloaded the cargo to tanker barge of Chemoil Lighterage Corporation (CLC), which
shall transport the same to Del Pan Bridge in Pasig River. The tanker barge would unload the cargo to tanker
trucks, also owned by the CLC, and haul it by land to PGPs storage tanks in Calamba, Laguna. Upon inspection
by PGP, the samples taken from the shipment showed discoloration from yellowish to amber, demonstrating
that it was damaged, as DOP is colorless and water clear. PGP then sent a letter to the petitioner where it
formally made an insurance claim for the loss it sustained due to the contamination. Thus, the PCIC paid PGP
the amount of P5,000,000.00 as full and final payment for the loss. PGP issued a Subrogation Receipt in favour
of PCIC.

Issue:

Is the notice of claim filed within the required period?

Ruling:

No. Both courts held that, indeed, a telephone call was made by Alfredo Chan to Encarnacion
Abastillas, informing the latter of the contamination. However, nothing in the trial courts decision stated that
the notice of claim was relayed or filed with the respondent-carrier immediately or within a period of twenty-
four hours from the time the goods were received. The Court of Appeals made the same finding. Having
examined the entire records of the case, the Court do not find a shred of evidence that will precisely and
ultimately point to the conclusion that the notice of claim was timely relayed or filed. There is no evidence to
confirm that the notice of claim was filed within the period provided for under Article 366 of the Code of
Commerce. PCIC contention proceeds from a false presupposition that the notice of claim was timely filed.

29 5
Cl a im S et tl em e n t A nd S ub r oga ti o n: N o tic e a nd P r oo f o f l oss

ABOITIZ SHIPPING CORPORATION v. COURT OF APPEALS, et al.


G.R. Nos. 121833, 130752 & 137801
17 October 2008

Tinga, J.

Facts:

In G.R. No. 121833 Malayan Insurance Company, Inc. (Malayan) filed five separate actions against
several defendants for the collection of the amounts of the cargoes allegedly paid by Malayan under various
marine cargo policies issued to the insurance claimants. In the five consolidated cases, Malayan sought the
recovery of amounts totaling P639,862.02. Aboitiz Shipping Corporation (Aboitiz) claimed that M/V P. Aboitiz
was seaworthy, that it exercised extraordinary diligence and that the loss was caused by a fortuitous event.
After trial on the merits, the RTC rendered a Decision adjudging Aboitiz liable on the money claims. The Court
of Appeals affirmed the RTC decision. It disregarded Aboitizs argument that the sinking of the vessel was
caused by a force majeure.

In G.R. No. 130752 Asia Traders Insurance Corporation (Asia Traders) and Allied Guarantee Insurance
Corporation (Allied) filed separate actions for damages against Aboitiz to recover by way of subrogation the
value of the cargoes insured by them and lost in the sinking of the vessel M/V P. Aboitiz. Aboitiz reiterated the
defense of force majeure. However, the trial court rendered a decision ordering Aboitiz to pay damages.
Aboitiz sought reconsideration, arguing that the sinking of the M/V P. Aboitiz was caused by a typhoon and
should have applied the real and hypothecary doctrine in limiting the monetary award in favor of the
claimants. The trial court denied Aboitizs motion for reconsideration.

In G.R. No. 137801, Equitable Insurance Corporation (Equitable) filed an action for damages against
Aboitiz to recover by way of subrogation the value of the cargoes insured by Equitable that were lost in the
sinking of M/V P. Aboitiz.

All these complaints were brought before the court to recover from Aboitiz due to capsize of M/VP.
Aboitiz.

Issue:

Can Aboitiz avail limited liability on the basis of the real and hypothecary doctrine of maritime law?

Ruling:

No. The limited liability rule is embodied in Articles 587, 590 and 837 under Book III of the Code of
Commerce. These articles precisely intend to limit the liability of the shipowner or agent to the value of the
vessel, its appurtenances and freightage earned in the voyage, provided that the owner or agent abandons the
vessel. When the vessel is totally lost in which case there is no vessel to abandon, abandonment is not required.
Because of such total loss the liability of the shipowner or agent for damages is extinguished. However, despite
the total loss of the vessel, its insurance answers for the damages for which a shipowner or agent may be held
liable. The instant petitions provide another occasion for the Court to reiterate the well settled doctrine of the
real and hypothecary nature of maritime law. As a general rule, a ship owners liability is merely co-extensive
with his interest in the vessel, except where actual fault is attributable to the shipowner. Thus, as an exception
to the limited liability doctrine, a shipowner or ship agent may be held liable for damages when the sinking of
the vessel is attributable to the actual fault or negligence of the shipowner or its failure to ensure the

29 6
Cl a im S et tl em e n t A nd S ub r oga ti o n: N o tic e a nd P r oo f o f l oss

seaworthiness of the vessel. The instant petitions cannot be spared from the application of the exception to
the doctrine of limited liability in view of the unanimous findings of the courts below that both Aboitiz and the
crew failed to ensure the seaworthiness of the M/V P. Aboitiz.

29 7
Cl a im S et tl em e n t A nd S ub r oga ti o n: N o tic e a nd P r oo f o f l oss

MITSUBISHI MOTORS PHILIPPINES SALARIED EMPLOYEES UNION (MMPSEU) v. MITSUBISHI


MOTORS PHILIPPINES
G.R. NO. 175773
June 17, 2013

Del Castillo, J.

Facts:

Mitsubishi Motors Philippines Salaried Employees Union (MMPSEU) and Mitsubishi Motors
Philippines’s Collective Bargaining Agreement (CBA) provides for the hospitalization insurance benefits for
the covered dependents. When the CBA the parties executed another CBA incorporating the same provisions
on dependents’ hospitalization insurance benefits but in the increased amount of ₱50,000.00. On separate
occasions, three members of MMPSEU, namely, Ernesto Calida (Calida), Hermie Juan Oabel (Oabel) and
Jocelyn Martin (Martin), filed claims for reimbursement of hospitalization expenses of their dependents.
MMPC paid only a portion of their hospitalization insurance claims, not the full amount.

In the case of Calida, his wife, Lanie, was confined at Sto. Tomas University Hospital and incurred
medical expenses amounting to ₱29,967.10. MMPC only paid ₱12,148.63. It refused to give to Calida the
difference between the amount of medical expenses of ₱27,427.10 which he claimed to be entitled to under
the CBA and the ₱12,148.63 which MMPC directly paid to the hospital. In the case of Martin, his father, Jose,
was admitted at The Medical City and incurred medical expenses amounting to ₱9,101.30. MMPC only paid
₱288.40, after deducting from the total medical expenses the amount paid by MEDICard and the ₱316.90
discount given by the hospital. Claiming that under the CBA, they are entitled to hospital benefits amounting
to ₱27,427.10, ₱6,769.35 and ₱8,123.80, respectively, which should not be reduced by the amounts paid by
MEDICard and by Prosper, Calida, Oabel and Martin asked for reimbursement from MMPC. However, MMPC
denied the claims contending that double insurance would result. This prompted the MMPSEU President to
write the MMPC President demanding full payment of the hospitalization benefits. The case was referred to
Voluntary Arbitration for resolution of the issue involving the interpretation of the subject CBA provision.

Issue:

Is recovery from both the CBA and separate health cards not prohibited in the absence of any specific
provision in the CBA?

Ruling:

No. the collateral source rule applies in order to place the responsibility for losses on the party causing
them. Its application is justified so that "'the wrongdoer should not benefit from the expenditures made by the
injured party or take advantage of contracts or other relations that may exist between the injured party and
third persons." Thus, it finds no application to cases involving no-fault insurances under which the insured is
indemnified for losses by insurance companies, regardless of who was at fault in the incident generating the
losses. Here, it is clear that MMPC is a no-fault insurer. Hence, it cannot be obliged to pay the hospitalization
expenses of the dependents of its employees which had already been paid by separate health insurance
providers of said dependents. The conditions set forth in the CBA provision indicate an intention to limit
MMPC’s liability only to actual expenses incurred by the employees’ dependents, that is, excluding the amounts
paid by dependents’ other health insurance providers.

Since the subject CBA provision is an insurance contract, the rights and obligations of the parties must
be determined in accordance with the general principles of insurance law. Being in the nature of a non-life

29 8
Cl a im S et tl em e n t A nd S ub r oga ti o n: N o tic e a nd P r oo f o f l oss

insurance contract and essentially a contract of indemnity, the CBA provision obligates MMPC to indemnify
the covered employees’ medical expenses incurred by their dependents but only up to the extent of the
expenses actually incurred. This is consistent with the principle of indemnity which proscribes the insured
from recovering greater than the loss.

29 9
Cl a im S et tl em e n t A nd S ub r oga ti o n: Ac ti on by t he I ns ur e r

CATHAY INSURANCE CO., INC., et al v. COURT OF APPEALS and EMILIA CHAN LUGAY
G.R. No. 85624
June 5, 1989

Griño-Aquino, J.

Facts:

Cathay Insurance Co. issued fire insurance policies for the total sum of P4,000,000 to the Cebu Filipino
Press of Cebu City. The fire policies described the insured property as "stocks of printing materials, papers and
general merchandise usual to the Assured's trade" stored in a one-storey building of strong materials housing
the Cebu Filipina Press. The co-insurers were indicated in each of the policies. All, except one policy
(Paramount's), were renewals of earlier policies issued for the same property. On December 18, 1981, the
Cebu Filipina Press was razed by electrical fire together with all the stocks and merchandise stored in the
premises.

Mrs. Lugay, owner and operator of the printing press, submitted sworn Statements of Loss Formal
Claims to the insurers. She claimed a total loss of P4, 595,00. The insurance companies denied liability, alleging
violation of certain conditions of the policy, misdeclaration, and arson. Eight years after Cebu Filipina Press
was destroyed by fire in broad daylight, the owner is still waiting to collect the proceeds of seven (7) fire
policies which the Cathay Insurance Co. sold to her.

Issue:

Does the unreasonable delay in the processing and payment of insurance claim entitle Mrs. Lugay to
collect interests?

Ruling:

YES. Unreasonable delay in the processing and payment of the insurance claims entitles the assured
to collect interests at the rate of twice the ceiling prescribed by the Monetary Board for the duration of delay.
The Cathay's contention that the charging of double interest was improper because no unreasonable delay in
the processing of the fire claim was proven, is refuted by the trial court' explicit finding that "there was a delay
that was not reasonable in processing the claim and doing payments.” Under Section 244, a prima facie
evidence of unreasonable delay in payment of the claim is created by the failure of the insurer to pay the claim
within the time fixed in both Section 242 and 243 of the Insurance Code.

As provided in Section 244 also, by reason of the delay and consequent filing of this suit by the insured,
the insurers "shall be adjudged to pay damages which shall consist of attorney's fees and other expenses
incurred by the insured." In view of the not insubstantial value of the Mrs. Lugay’s claims and the considerable
time and effort expended by them and their counsel in prosecuting these claims for the past eight (8) years,
the Court holds that attorney's fees were properly awarded to Mrs. Lugay.

30 0
Cl a im S et tl em e n t A nd S ub r oga ti o n: D el a y in Pa ym e nt of I ns ura nc e Pr o ce e ds

ZENITH INSURANCE CORPORATION v. COURT OF APPEALS and LAWRENCE FERNANDEZ


G.R. No. 85296
May 14, 1990

Medialdea, J

Facts:

On January 25, 1983, Lawrence Fernandez insured his car for "own damage" with Zenith Insurance
Corporation. On July 6, 1983, the car figured in an accident and suffered actual damages in the amount of
P3,640. After allegedly being given a runaround by Zenith for two months, Fernandez filed a complaint with
the Regional Trial Court of Cebu for sum of money and damages resulting from the refusal of Zenith to pay the
amount claimed. Aside from actual damages and interests, Fernandez also prayed for moral damages in the
amount of P10,000.00, exemplary damages of P5,000.00, attorney's fees of P3,000.00and litigation expenses
of P3,000.00. Zenith filed an answer alleging that it offered to pay the claim of Fernandez pursuant to the terms
and conditions of the contract which, Fernandez rejected. On June 4, 1986, a decision was rendered by the trial
court in favor of Fernandez.

Issue:

Is Fernandez entitled to claim moral damages, exemplary damages and attorney’s fees?

Ruling:

The purpose of moral damages is essentially indemnity or reparation, not punishment or correction.
Moral damages are emphatically not intended to enrich a complainant at the expense of a defendant, they are
awarded only to enable the injured party to obtain means, diversions or amusements that will serve to
alleviate the moral suffering he has undergone by reason of the defendant's culpable action. While it is true
that no proof of pecuniary loss is necessary in order that moral damages may be adjudicated, the assessment
of which is left to the discretion of the court according to the circumstances of each case, it is equally true that
in awarding moral damages in case of breach of contract, there must be a showing that the breach was wanton
and deliberately injurious or the one responsible acted fraudently or in bad faith.

In the instant case, Fernandez was given a "run-around" for two months, which is the basis for the
award of the damages granted under the Insurance Code for unreasonable delay in the payment of the claim.
However, the act of Zenith of delaying payment for two months cannot be considered as so wanton or
malevolent to justify an award of P20,000.00 as moral damages, taking into consideration also the fact that the
actual damage on the car was only P3,460. There was no total disclaimer by Fernandez. The reason for Zenith's
failure to indemnify Fernandez within the two-month period was that the parties could not come to an
agreement as regards the amount of the actual damage on the car. The amount of P10,000.00 prayed for by
Fernandez as moral damages is equitable.

On the other hand, exemplary or corrective damages are imposed by way of example or correction
for the public good (Art. 2229, New Civil Code of the Philippines). Exemplary damages were not awarded as
the insurance company had not acted in wanton, oppressive or malevolent manner.

30 1
Cl a im S et tl em e n t A nd S ub r oga ti o n: D el a y in Pa ym e nt of I ns ura nc e Pr o ce e ds

FINMAN GENERAL ASSURANCE CORPORATION v. COURT OF APPEALS and USIPHIL


INCORPORATED
G.R. No. 138737
July 12, 2001

Kapunan, J.

Facts:

Usiphil, Inc. obtained a fire insurance policy from Finman General Assurance Corp. then doing
business in the name of Summa Insurance Corp.). In 1982, Usiphil filed with Finman an insurance
claim for the loss of the insured properties due to fire. Despite several demands by Usiphil, Finman
refused to pay the insurance claim on the ground that the same could not be allowed because Usiphil
failed to comply with a condition in the policy regarding the submission of certain documents to
prove the loss. Thus, Usiphil was constraint to file a complaint against Finman for the unpaid
insurance claim. The trial court ruled in favor of Usiphil, and ordered Finman to pay Usiphil the
amount of the insurance proceeds plus a 24% interest rate per annum until the judgment proceeds
is fully paid. Finman argued that since there was no express finding of unreasonably withheld or
denied the payment of the subject insurance claim, then the award of 24% per annum is not proper.

Issue:

Is the 24% interest rate per annum awarded by the lower courts is legal?

Ruling:

A perusal of the records shows that Usiphil, after the occurrence of the fire, immediately notified
Finman. Thereafter, Usiphil submitted the following documents: (1) Sworn Statement of Loss and
Formal Claim and; (2) Proof of Loss. The submission of these documents, to the Courts constitutes
substantial compliance with the provision. Indeed, as regards the submission of documents to prove
loss, substantial, not strict as urged by Finman, compliance with the requirements will always be
deemed sufficient

Notably, under Section 244, a prima facie evidence of unreasonable delay in payment of the claim
is created by the failure of the insurer to pay the claim within the time fixed in both Sections 243 and
244. Further, Section 29 of the policy itself provides for the payment of such interest.The policy itself
obliges Finman to pay the insurance claim within thirty days after proof of loss and ascertainment of
the loss made in an agreement between Usiphil and Finman. In this case, Finman and Usiphil signed
the agreement indicating that the amount due to Usiphil was P842,683.40 on April 2, 1985. Finman
thus had until May 2, 1985 to pay Usiphil insurance. For its failure to do so, the Finman should
pay, inter alia, 24% interest per annum.

30 2
Cl a im S et tl em e n t A nd S ub r oga ti o n: F ra ud ul e nt Cl a i ms by t h e I nsu r ed

UY HU & CO. v. THE PRUDENTIAL ASSURANCE CO., LTD.


G.R. No. 27778
December 16, 1927

Johns, J.

Facts:

On April 20, 1926, Prudential Assurance Co. Ltd undertook to and did insure against loss and damage
by fire the property, goods, wares and merchandise of Uy Ho and Co. for the sum of P30,000. On May 10, 1926,
and while the policy was in full force and effect, the property was destroyed by fire without the fault or
negligence of Uy Ho and Co. That in accord with the terms and conditions of the policy, Uy Ho and Co. notified
the Prudential Assurance of the fire and of its loss, and requested payment of the P30,000, the full amount of
the policy, and at the same time submitted evidence to verify its claim, but that Prudential, without any legal
or just ground, refused to pay the claim or any part of it.

Prudential Assurance Co. Ltd. alleges that in the policy in question, it was agreed that in the event of
loss, should Uy Ho. Co make a fraudulent claim or any false declaration or use any fraudulent means or devices
to obtain payment for its loss, the policy should become null and void. After the fire, Uy Ho and Co did present
a claim under oath of its manager for P30,000, the alleged amount of its loss. That said claim was false and
fraudulent, in that it was therein represented that the value of merchandise at the time of the fire was
P32,523.30, whereas in truth and in fact a large part of the merchandise claimed and represented in Uy Ho and
Co’s proof of loss was not in the building at the time of the fire, and that the value of the merchandise which
was actually consumed or damaged by the fire was a very small part of the claim.

Issue:

Is the Prudential Assurance Co Ltd. As insurer liable to pay the loss incurred by Uy Ho and Co?

Ruling:

Where a fire insurance policy provides that "If the claim be in any respect fraudulent, or if any false
declaration be made or used in support thereof, or if any fraudulent means or devices are used by the Insured
or anyone acting on his behalf to obtain any benefit under this Policy," and the evidence is conclusive that the
proof of claim which the insured submitted was false and fraudulent both as to the kind, quality and amount
of the goods and their value destroyed by the fire, such a proof of claim is a bar against the insured to recover
on the policy even for the amount of his actual loss.

While it is true that a small portion of the merchandise might have been consumed, and the evidence
of its existence completely destroyed by the fire, yet in the very nature of things, a large portion of it would not
be destroyed, and some evidence would be left by which the amount, kind and quality of it could be
substantially ascertained and determined.
Photography is an exact science. Witnesses pro and con may testify falsely, but a photograph of a scene
is not a false witness, and is conclusive evidence of the actual facts appearing on the photograph.

Based upon the oral evidence of the defendant, together with the photographs in question, which
convincingly show the actual conditions in the bodegas immediately after the fire, we are clearly of the opinion
that plaintiff’s claim is false and fraudulent within the terms and definitions of the policy, and that the value of
the merchandise destroyed by the fire would not exceed P5,000.

30 3
Cl a im S et tl em e n t A nd S ub r oga ti o n: F ra ud ul e nt Cl a i ms by t h e I nsu r ed

Although much latitude should be given to the insured in presenting his proof of claim as to the value
of his loss, in particular as to the price, kind and quality of the property destroyed, yet where the proof is
conclusive, as in this case, that the insured made a claim for a large amount of property which was never in the
bodegas at the time of the fire and for a much larger amount of property than was actually in the bodegas, it
makes the whole claim false and fraudulent, the legal effect of which is to bar plaintiff from the recovery of the
amount of its actual loss.

30 4
Cl a im S et tl em e n t A nd S ub r oga ti o n: B ri n gi ng a n Act io n a nd P r esc ri pt i on of Ac ti o n

ALFONSO G. LOPEZ v. FILIPINAS COMPAÑIA DE SEGUROS


G.R. No. L-19613
April 30, 1966

Regala, J.

Facts:

Lopez applied with the defendant company for the insurance of his properties: Biederman truck
tractor and a Winter Weils trailer from loss or damage in the amount of P20,000.00 and P10,000.00,
respectively. During the application, the company inquired Lopez has any company insured any car or vehicle,
increased your premium renewal, refusal to renew insurance.

Lopez answered in negative but the truth was that the American International Underwriters of the
Philippines (AIU) had already declined similar application for insurance by Lopez with respect of the said
vehicles. The defendant issued to Lopez two Commercial Vehicle Comprehensive Policies covering the
properties. The vehicles mentioned figured in an accident resulting in the total loss of the tractor and partial
damage to the trailer. Plaintiff demand upon the defendant for the payment to him the total amt. of damages
resulting from the accident. On April 28, 1960, defendant rejected the claim on the ground of concealment of
a material fact: that the insured property previously been declined insurance by another company. On May 27,
1960, the plaintiff filed with the Office of the Insurance Commissioner a complaint against the said company.
As suggested, the plaintiff was willing to submit his claim to arbitration but was contested by the defendant
since "the claim of the plaintiff cannot be resolved by arbitration, as recourse to arbitration referred to in the
policy contract, envisioned only differences or disputes, 'with respect to the amount of the company's liability,'
and not to cases where the company does not admit its liability to the insured. With this rejection, the plaintiff
filed his complaint with the CFI of Manila.

Issue:

Is the complaint filed by Lopez with the Office of the Insurance Comm. a commencement of an "action
or suit"within the meaning and intent of general condition?

Ruling:

The terms "action" and "suit" are synonymous. Moreover, it is clear that the determinative or
operative fact which converts a claim into an "action or suit" is the filing of the same with a "court or justice."
Filed elsewhere, as with some other body or office not a court of justice, the claim may not properly be
categorized under either term.

An "action or suit" is essentially "for the enforcement or protection of a right, or the prevention or
redress of a wrong." (Rule 2, Sec. 1, Rules of Court). There is nothing in the Insurance Law, Act No. 2427, as
amended, nor in any of its allied Legislations, which empowers the Insurance Commissioner to adjudicate on
disputes relating to an insurance company's liability to an insured under a policy issued by the former to the
latter. The validity of an insured's claim under a specific policy, its amount, and all such other matters as might
involve the interpretation and construction of the insurance policy, are issues which only a regular court of
justice may resolve and settle. Consequently, the complaint filed by the appellant herein with the Office of the
Insurance Commission could not have been an "action or suit."

30 5
Cl a im S et tl em e n t A nd S ub r oga ti o n: B ri n gi ng a n Act io n a nd P r esc ri pt i on of Ac ti o n

SUN INSURANCE OFFICE, LTD v. COURT OF APPEALS and EMILIO TAN


G.R. No. 89741
March 13, 1991

Paras, J.

Facts:

Emilio Tan took from Sun Insurance Office a P300,000.00 property insurance policy to cover his
interest in the electrical supply store of his brother. Four days after the issuance of the policy, the building was
burned including the insured store. On August 20, 1983, he filed his claim for fire loss with Sun Insurance
Office, but on February 29, 1984, Sun Insurance Office wrote Tan denying the latter’s claim. On April 3, 1984,
Tan wrote Sun Insurance Office, seeking reconsideration of the denial of his claim. Sun Insurance Office
answered the letter, advising Tan’s counsel that the Insurer’s denial of Tan’s claim remained unchanged.

Issues:

(1) Is the filing of a motion for reconsideration interrupts the 12 months prescriptive period to contest
the denial of the insurance claim?
(2) Is the rejection of the claim deemed final only if it contains words to the effect that the denial is final?

Ruling:

No. In this case, Condition 27 of the Insurance Policy of the parties reads: “ Action or suit clause If a
claim be made and rejected and an action or suit be not commenced either in the Insurance Commission or in
any court of competent jurisdiction within twelve (12) months from receipt of notice of such rejection, or in
case of arbitration taking place as provided herein, within twelve (12) months after due notice of the award
made by the arbitrator or arbitrators or umpire, then the claim shall for all purposes be deemed to have been
abandoned and shall not thereafter be recoverable hereunder.” As the terms are very clear and free from any
doubt or ambiguity whatsoever, it must be taken and understood in its plain, ordinary and popular sense. Tan,
in his letter addressed to Sun Insurance Office dated April 3, 1984, admitted that he received a copy of the
letter of rejection on April 2, 1984. Thus, the 12-monthprescriptive period started to run from the said date of
April 2, 1984, for such is the plain meaning and intention of Section 27 of the insurance policy. The condition
contained in an insurance policy that claims must be presented within one year after rejection is not merely a
procedural requirement but an important matter essential to a prompt settlement of claims against insurance
companies as it demands that insurance suits be brought by the insured while the evidence as to the origin
and cause of destruction have not yet disappeared. It is apparent that Section 27 of the insurance policy was
stipulated pursuant to Section 63 of the Insurance Code, which states that: Sec. 63. A condition, stipulation or
agreement in any policy of insurance, limiting the time for commencing an action thereunder to a period of
less than one year from the time when the cause of action accrues, is void.

The insured’s cause of action or his right to file a claim either in the Insurance Commission or in a
court of competent jurisdiction commences from the time of the denial of his claim by the Insurer, either
expressly or impliedly. But the rejection referred to should be construed as the rejection in the first instance
(i.e. at the first occasion or for the first time), not rejection conveyed in a resolution of a petition for
reconsideration. Thus, to allow the filing of a motion for reconsideration to suspend the running of the
prescriptive period of twelve months, a whole new body of rules on the matter should be promulgated so as
to avoid any conflict that may be brought by it.

30 6
Cl a im S et tl em e n t A nd S ub r oga ti o n: B ri n gi ng a n Act io n a nd P r esc ri pt i on of Ac ti o n

No. The Eagle Star case cited by Tan to defend his theory that the rejection of the claim shall be deemed
final only of it contains words to the effect that the denial is final is inapplicable in the instant case. Final
rejection or denial cannot be taken to mean the rejection of a petition for reconsideration. The Insurance policy
in the Eagle Star case provides that the insured should file his claim, first, with the carrier and then with the
insurer. The final rejection being referred to in said case is the rejection by the insurance company.

30 7
Cl a im S et tl em e n t A nd S ub r oga ti o n: B ri n gi ng a n Act io n a nd P r esc ri pt i on of Ac ti o n

JACQUELINE JIMENEZ VDA. DE GABRIEL v. COURT OF APPEALS and FORTUNE INSURANCE &
SURETY COMPANY, INC
G.R. No. 103883
November 14, 1996

Vitug, J.

Facts:

Marcelino Gabriel was employed by Emerald Construction & Development Corporation at its
construction project in Iraq. He was covered by a personal accident insurance in the amount of P100,000.00
under a group policy procured from Fortune Insurance & Surety Company (Fortune Insurance for brevity)by
Emerald Construction for its overseas workers. The insured risk was for bodily injury caused by violent
accidental external and visible means which injury would solely and independently of any other cause result
in death or disability. On 22 May 1982, within the life of the policy, Gabriel died in Iraq. On 12 July 1983,
Emerald Construction reported Gabriel’s death to Fortune Insurance by telephone. Among the documents
thereafter submitted to Fortune Insurance were a copy of the death certificate issued by the Ministry of Health
of the Republic of Iraq which stated that an autopsy report by the National Bureau of Investigation was
conducted to the effect that due to advanced state of postmortem decomposition, the cause of death of Gabriel
could not be determined. Because of this development Fortune Insurance ultimately denied the claim of
Emerald Construction on the ground of prescription. Gabriel’s widow, Jacqueline Jimenez, went to the to the
lower court. In her complaint against Emerald Construction and Fortune Insurance, she averred that her
husband died of electrocution while in the performance of his work. Fortune Insurance alleged that since both
the death certificate issued by the Iraqi Ministry of Health and the autopsy report of the NBI failed to disclose
the cause of Gabriel’s death, it denied liability under the policy. In addition, Fortune Insurance raised the
defense of prescription, invoking Section 384 of the Insurance Code.

Issue:

Is the Jacqueline Jimenez Vda. de Gabriel’s claim against Fortune Insurance denied on the ground of
prescription?

Ruling:

Yes. Section 384 of the Insurance Code provides: Sec. 384. Any person having any claim upon the
policy issued pursuant to this chapter shall, without any unnecessary delay, present to the insurance company
concerned a written notice of claim setting forth the nature, extent and duration of the injuries sustained as
certified by a duly licensed physician. Notice of claim must be filed within six months from date of the accident,
otherwise, the claim shall be deemed waived. Action or suit for recovery of damage due to loss or injury must
be brought, in proper cases, with the Commissioner or the Courts within one year from denial of the claim,
otherwise, the claimants right of action shall prescribe. The notice of death was given to Fortune Insurance,
concededly, more than a year after the death of vda. de Gabriel’s husband. Fortune Insurance, in invoking
prescription, was not referring to the one-year period from the denial of the claim within which to file an action
against an insurer but obviously to the written notice of claim that had to be submitted within six months from
the time of the accident. Vda. de Gabriel argues that Fortune Insurance must be deemed to have waived its
right to show that the cause of death is an excepted peril, by failing to have its answers duly verified. It is true
that a matter of which a written request for admission is made shall be deemed impliedly admitted unless,
within a period designated in the request, which shall not be less than 10 days after service thereof, or within
such further time as the court may allow on motion and notice, the party to whom the request is directed
serves upon the party requesting the admission a sworn statement either denying specifically the matters of

30 8
Cl a im S et tl em e n t A nd S ub r oga ti o n: B ri n gi ng a n Act io n a nd P r esc ri pt i on of Ac ti o n

which an admission is requested or setting forth in detail the reasons why he cannot truthfully either admit or
deny those matters; however, the verification, like in most cases required by the rules of procedure, is a formal,
not jurisdictional, requirement, and mainly intended to secure an assurance that matters which are alleged
are done in good faith or are true and correct and not of mere speculation. When circumstances warrant, the
court may simply order the correction of unverified pleadings or act on it and waive strict compliance with the
rules in order that the ends of justice may thereby be served. In the case of answers to written requests for
admission particularly, the court can allow the party making the admission, whether made expressly or
deemed to have been made impliedly, to withdraw or amend it upon such terms as may be just. The insurance
policy expressly provided that to be compensable, the injury or death should be caused by violent accidental
external and visible means.

In attempting to prove the cause of her husband’s death, all that vda. de Gabriel could submit were a
letter sent to her by her husband’s co-worker, stating that Gabriel died when he tried to haul water out of a
tank while its submerged motor was still functioning, and vda. de Gabriel’s sworn affidavit. The said affidavit,
however, suffers from procedural infirmity as it was not even testified to or identified by vda. de Gabriel
herself. This affidavit therefore is a mere hearsay under the law. In like manner, the letter allegedly written by
the deceased’s co-worker which was never identified to in court by the supposed author, suffers from the same
defect as the affidavit of vda. de Gabriel. Not one of the other documents submitted, to wit, the POEA decision,
the death certificate issued by the Ministry of Health of Iraq and the NBI autopsy report, could give any
probative value to vda. de Gabriel’s claim. The POEA decision did not make any categorical holding on the
specific cause of Gabriel’s death. In summary, evidence is utterly wanting to establish that the insured suffered
from an accidental death, the risk covered by the policy.

30 9
Cl a im S et tl em e n t A nd S ub r oga ti o n: B ri n gi ng a n Act io n a nd P r esc ri pt i on of Ac ti o n

TRAVELLERS INSURANCE & SURETY CORPORATION v. COURT OF APPEALS and VICENTE


MENDOZA
G.R. No. 82036
May 22, 1997

Hermosisima, Jr. J.

Facts:

A 78-year old woman named Feliza Vineza de Mendoza was on her way to hear mass at the
Tayuman Cathedral. While walking along Tayuman , she was bumped by a taxi that was running
fast. Several persons witnessed the accident. After the bumping incident, the old woman was seen
sprawled on the pavement. The old woman brought her to the Mary Johnston Hospital in Tondo. Later
on, the old woman died and caused by traumatic shock as a result of the severe injuries she sustained
.

During the investigation, Armando Abellon, the registered owner of the taxi certified that the
vehicle was driven by one Rodrigo Dumlao and he already absconded.

Vicente Mendoza filed a complaint for damages against Armando Abellon as the owner of the
taxi and Rodrigo Dumlao as the driver of the taxicab that bumped Mendoza’s mother. Subsequently,
Mendoza amended his complaint to include petitioner as the compulsory insurer of the said taxicab.

Issue:

Is the driver, insurer and owner of taxicab solidarily liable to Vicente Mendoza due to the accident that
caused death of his mother?

Ruling:

Yes. Where the contract provides for indemnity against liability to third persons, then third persons
to whom the insured is liable can sue the insurer. Where the contract is for indemnity against actual loss or
payment, then third persons cannot proceed against the insurer, the contract being solely to reimburse the
insured for liability actually discharged by him thru payment to third persons, said third persons' recourse
being thus limited to the insured alone. But in the case at bar, there was no contract shown. The cause of action
of Mendoza against the driver and owner of taxicab is based on torts and quasi-delicts while the cause of action
of Mendoza against the insurance company is based on contract. Even assuming arguendo that there was such
a contract, Mendoza’s cause of action cannot prevail because he failed to file the written claim mandated by
the Insurance Code (before it was amended-action must be brought within six months from date of the
accident); after amendment - "action or suit for recovery of damage due to loss or injury must be brought in
proper cases, with the Commissioner or the Courts within one year from denial of the claim, otherwise the
claimant's right of action shall prescribe." Thus, Mendoza is deemed, under this legal provision, to have waived
his rights as against the insurance company.

31 0
Su b ro ga t io n : R ig ht of S ub r oga ti o n

FIREMAN'S FUND INSURANCE COMPANY and FIRESTONE TIRE AND RUBBER COMPANY OF
THE PHILIPPINES v. JAMILA & COMPANY, INC. and FIRST QUEZON CITY INSURANCE CO., INC
G.R. No. L-27427
April 7, 1976

Aquino, J.

Facts:

Jamila or the Veterans Philippine Scouts Security Agency contracted to supply security guards to
Firestone. Jamila assumed responsibility for the acts of its security guards. First Quezon City Insurance Co., Inc.
executed a bond in the sum of P20k to guarantee Jamila's obligations under that contract. On May 18, 1963,
properties of Firestone valued at P11925 were lost allegedly due to the acts of its employees who connived
with Jamila's security guard. Fireman's Fund, as insurer, paid to Firestone the amount of the loss and is now
subrogated to Firestone's right to get reimbursement from Jamila. Jamila and its surety, First Quezon City
Insurance Co., Inc., failed to pay the amount of the loss in spite of repeated demands.

Fireman's Fund and Firestone Tire and Rubber Co instituted this complaint against Jamila for the
recovery of the sum of P11,925.00 plus interest, damages and attorney's fees. Jamila moved to dismiss the
complaint on the ground of lack of cause of action, complaint did not allege that Firestone, pursuant to the
contractual stipulation quoted in the complaint, had investigated the loss and that Jamila was represented in
the investigation, and Jamila did not consent to the subrogation of Fireman's Fund to Firestone's right to get
reimbursement from Jamila and its surety.

Issues:

1) Is the complaint of Fireman's Fund as subrogee states a cause of action against Jamila?
2) Is Jamila obliged to reimburse the amount to Fireman’s Fund Insurance?

Ruling:

Firestone and Jamila entered into a "contract of guard services" on June 1, ‘65. The allegation was
uncalled for because it is not found in the complaint and so created confusion which did not exist. No copy of
the contract was annexed to the complaint. The confusing statement was an obvious error since it was
expressly alleged in the complaint that the loss occurred on May 18, ‘63. The fact that such an error was
committed is another instance substantiating the observation that F&F's counsel had not exercised due care
in the presentation of his case.

Firestone is really a nominal party in this case as it had already been indemnified for the loss which it
had sustained. It joined as a party-plaintiff in order to help Fireman's Fund to recover the amount of the loss
from Jamila and First Quezon City Insurance Co., Inc. Firestone had tacitly assigned to Fireman's Fund its cause
of action against Jamila for breach of contract. Sufficient ultimate facts are alleged in the complaint to sustain
that cause of action.

Fireman's Fund's action against Jamila is squarely sanctioned by article 2207. As the insurer,
Fireman's Fund is entitled to go after the person or entity that violated its contractual commitment to answer
for the loss insured against. F&F in alleging in their complaint that Fireman's Fund "became a party in interest
in this case by virtue of a subrogation right given in its favor by" Firestone were not relying on the novation by
change of creditors

31 1
Su b ro ga t io n : R ig ht of S ub r oga ti o n

Article 2207 is a restatement of a settled principle of American jurisprudence. Subrogation has been
referred to as the doctrine of substitution. It is an arm of equity that may guide or even force one to pay a debt
for which an obligation was incurred but which was in whole or in part paid by another.

Subrogation is founded on principles of justice and equity, and its operation is governed by principles
of equity. It rests on the principle that substantial justice should be attained regardless of form, that is, its basis
is the doing of complete, essential, and perfect justice between all the parties without regard to form.
Subrogation is a normal incident of indemnity insurance. Upon payment of the loss, the insurer is entitled to
be subrogated pro tanto to any right of action which the insured may have against the third person whose
negligence or wrongful act caused the loss.

The right of subrogation is of the highest equity. The loss in the first instance is that of the insured but
after reimbursement or compensation, it becomes the loss of insurer. Although many policies including
policies in the standard form, now provide for subrogation, and thus determine the rights of the insurer in this
respect, the equitable right of subrogation as the legal effect of payment inures to the insurer without any
formal assignment or any express stipulationto that effect in the policy.

Stated otherwise, when the insurance company pays for the loss, such payment operates as an
equitable assignment to the insurer of the property and all remedies which the insured may have for the
recovery thereof. That right is not dependent upon, nor does it grow out of, any privity of contract, or upon
written assignment of claim, and payment to the insured makes the insurer an assignee in equity.

31 2
Su b ro ga t io n : R ig ht of S ub r oga ti o n

MALAYAN INSURANCE CO., INC v. COURT OF APPEALS, MARTIN C. VALLEJOS, SIO CHOY, SAN
LEON RICE MILL, INC. and PANGASINAN TRANSPORTATION CO., INC,
G.R. No. L-3641
September 26, 1988

Padilla, J.

Facts:

Malayan Insurance Co. issued a car comprehensive policy in favor Sio Choy covering a jeep. The
insurance coverage was for “own damage” not to exceed P600 and “third party liability” amounting to
P20,000. While to policy was in force, the jeep, while driven by Campollo, collided with a bus operated by
PANTRANCO, causing damage to the insured jeep, injuries to the driver and respondent Vallejos, who was one
of the jeepney passenger. Vallejos filed an action for damages against Sio Choy, Malayan, and PANTRANCO.
PANTRANCO’s posed defenses that the jeep was running at an excessive speed, and that the bus stopped at
the shoulder of the highway to avoid the jeep. Sio Choy failed to observe ordinary diligence in the selection and
supervision of its employees. Sio Choy claimed that the accident was solely imputable to PANTRANCO. Sio
Choy later filed a cross-claim against Malayan, claiming that it had already paid Vallejos P5,000 for
hospitalization and as insurer, Malayan should reimburse Sio Choy for the expenses he incurred. Malayan, in
turn, filed a third-party claim against San Leon Rice Mill for the reason that its employee was driving the jeep
at the time of the accident. Malayan alleged that San Leon was liable for the acts of its employee under Art 2180
New Civil Code.

Issue:

Can Sio Choy, San Leon and Malayan be held solidarily liable to Vallejos?

Ruling:

While it is true that where the insurance contract provides for indemnity against liability to third
persons, such third persons can directly sue the insurer, however, the direct liability of the insurer under
indemnity contracts against third party liability does not mean that the insurer can be held solidarily liable
with the insured and/or the other parties found at fault. The liability of the insurer is based on contract; that
of the insured is based on tort.

In the case at bar, Malayan as insurer of Sio Choy, is liable to Vallejos, but it cannot, as incorrectly held
by the trial court, be made "solidarily" liable with the two principal tortfeasors namely Sio Choy and San Leon
Rice Mill, Inc. For if Malayan was solidarily liable with said two (2) respondents by reason of the indemnity
contract against third party liability-under which an insurer can be directly sued by a third party — this will
result in a violation of the principles underlying solidary obligation and insurance contracts.

In the case at bar, the trial court held Malayan together with Sio Choy and San Leon Rice Mills Inc.
solidarily liable to respondent Vallejos for a total amount of P29,103.00, with the qualification that Malayan's
liability is only up to P20,000.00. In the context of a solidary obligation, Malayan may be compelled by Vallejos
to pay the entire obligation of P29,013.00, notwithstanding the qualification made by the trial court. But, how
can Malayan be obliged to pay the entire obligation when the amount stated in its insurance policy with Sio
Choy for indemnity against third party liability is only P20,000.00? Moreover, the qualification made in the
decision of the trial court to the effect that Malayan is sentenced to pay up to P20,000.00 only when the
obligation to pay P29,103.00 is made solidary, is an evident breach of the concept of a solidary obligation. Thus,

31 3
Su b ro ga t io n : R ig ht of S ub r oga ti o n

the Court holds that the trial court erred in holding Malayan, solidarily liable with Sio Choy and San Leon Rice
Mill, Inc. to Vallejos.

31 4
Su b ro ga t io n : R ig ht of S ub r oga ti o n

PAN MALAYAN INSURANCE CORPORATION v. COURT OF APPEALS, ERLINDA FABIE AND HER
UNKNOWN DRIVER
G.R. No. 81026
April 3, 1990

Cortes, J.

Facts:

Canlubang Automotive Resources Corp. obtained from Pan Malayan a motor vehicle
insurance policy for its Mitsubishi Colt Lancer. While the policy was still in effect, the insured car was
allegedly hit by a pick-up owned by Erlinda Fabie but driven by another person. The car suffered
damages in the amount of P42 000. Pan Malayan defrayed the cost of repair of the insured car. It
then demanded reimbursement from Fabie and her driver of said amount, but to no avail. Pan
Malayan filed a complaint for damages with the RTC of Makati against Fabie and the driver. It averred
that the damages caused to the insured car were settled under the “own damage” coverage of the
insurance policy. Fabie filed a motion to dismiss alleging that Pan Malayan had no cause of action
since the “won damage” clause of the policy precluded subrogation under Art. 2207 of the Civil Code.
They contended that indemnification under said article is on the assumption that there was no
wrongdoer or no third party at fault. The RTC dismissed Pan Malayan complaint and ruled that
payment under the “own damage” clause was an admission by the insurer that the damage was
caused by the assured and/or its representatives. CA affirmed but on different ground. Applying the
ejusdem generis rule, CA held that Section III-I of the policy, which was the basis for the settlement
of the claim against insurance, did not cover damage arising from collision or overturning due to the
negligence of third parties as one of the insurable risks.

Issue:

Is the Pan Malayan subrogated to the rights of Canlubang against the driver and his employer?

Ruling:

Article 2207 of the Civil Code is founded on the well-settled principle of subrogation. If the
insured property is destroyed or damaged through the fault or negligence of a party other than the
assured, then the insurer, upon payment to the assured, will be subrogated to the rights of the
assured to recover from the wrongdoer to the extent that the insurer has been obligated to pay.
Payment by the insurer to the assured operates as an equitable assignment to the former of all
remedies which the latter may have against the third party whose negligence or wrongful act caused
the loss. The right of subrogation is not dependent upon, nor does it grow out of, any privity of
contract or upon written assignment of claim. It accrues simply upon payment of the insurance claim
by the insurer.

There are a few recognized exceptions to this rule. For instance, if the assured by his own act
releases the wrongdoer or third party liable for the loss or damage, from liability, the insurer's right
of subrogation is defeated. Similarly, where the insurer pays the assured the value of the lost goods
without notifying the carrier who has in good faith settled the assured's claim for loss, the settlement
is binding on both the assured and the insurer, and the latter cannot bring an action against the
carrier on his right of subrogation. And where the insurer pays the assured for a loss which is not a
risk covered by the policy, thereby effecting "voluntary payment", the former has no right of
subrogation against the third party liable for the loss.

31 5
Su b ro ga t io n : R ig ht of S ub r oga ti o n

It is a basic rule in the interpretation of contracts that the terms of a contract are to be
construed according to the sense and meaning of the terms which the parties thereto have used. In
the case of property insurance policies, the evident intention of the contracting parties, i.e., the
insurer and the assured, determine the import of the various terms and provisions embodied in the
policy. It is only when the terms of the policy are ambiguous, equivocal or uncertain, such that the
parties themselves disagree about the meaning of particular provisions that the courts will intervene.
In such an event, the policy will be construed by the courts liberally in favor of the assured and strictly
against the insurer.
In this case, it cannot be said that the meaning given by Pan Malayan and Canlubang to the phrase "by
accidental collision or overturning" found in the first paint of sub-paragraph (a) is untenable. Although the
terms "accident" or "accidental" as used in insurance contracts have not acquired a technical meaning, the
Court has on several occasions defined these terms to mean that which takes place "without one's foresight or
expectation, an event that proceeds from an unknown cause, or is an unusual effect of a known cause and,
therefore, not expected.” Certainly, it cannot be inferred from jurisprudence that these terms, without
qualification, exclude events resulting in damage or loss due to the fault, recklessness or negligence of third
parties. The concept "accident" is not necessarily synonymous with the concept of "no fault". It may be utilized
simply to distinguish intentional or malicious acts from negligent or careless acts of man.

For even if under the above circumstances Pan Malayan could not be deemed subrogated to the rights
of its assured under Article 2207 of the Civil Code, Pan Malayan would still have a cause of action against Febie.
The insurer who may have no rights of subrogation due to "voluntary" payment may nevertheless recover
from the third party responsible for the damage to the insured property under Article 1236 of the Civil Code.

31 6
Su b ro ga t io n : R ig ht of S ub r oga ti o n

THE PHILIPPINE AMERICAN GENERAL INSURANCE COMPANY, INC. v. COURT OF APPEALS and
FELMAN SHIPPING LINES
G.R. No. 116940
June 11, 1997

Bellosillo, J.

Facts:

On 6 July 1983 Coca-Cola Bottlers Philippines, Inc., loaded on board "MV Asilda," a vessel owned and
operated by respondent Felman Shipping Lines, 7,500 cases of 1-liter Coca-Cola softdrink bottles to be
transported from Zamboanga City to Cebu City for consignee Coca-Cola Bottlers Philippines, Inc., Cebu. The
shipment was insured with petitioner Philippine American General Insurance Co., Inc.

In a joint statement, the Captain as well as the chief mate of the vessel confirmed that the weather was
fine when "MV Asilda" left the port of Zamboanga at 8 p.m. on 6 July. The ship captain stated that around 4 a.m.
of 7 July 1983 he was awakened by the officer on duty to inform him that the vessel had hit a floating log. At
that time he noticed that the weather had deteriorated with strong southeast winds inducing big waves. After
30 minutes, he observed that the vessel was listing slightly to starboard and would not correct itself despite
the heavy rolling and pitching. He then ordered his crew to shift the cargo from starboard to portside until the
vessel was balanced. At about 7 a.m., the master of the vessel stopped the engine because the vessel was listing
dangerously to portside. He ordered his crew to shift the cargo back to starboard (right). The shifting of cargo
took about an hour after which he rang the engine room to resume full speed

At around 8:45 a.m., the vessel suddenly listed to portside and before the captain could decide on his
next move, some of the cargo on deck were thrown overboard and seawater entered the engine room and
cargo holds of the vessel. At that instance, the master of the vessel ordered his crew to abandon ship. MV Asilda
capsized and sank in the waters of Zamboanga del Norte bringing down her entire cargo with her including
the subject 7,500 cases of 1-liter Coca-Cola softdrink bottles. The Ship Captain ascribed the sinking to the entry
of seawater through a hole in the hull caused by the vessel's collision with a partially submerged log.

On 15 July 1983 the consignee Coca-Cola Bottlers Philippines, Inc., Cebu plant, filed a claim with
respondent FELMAN for recovery of damages. FELMAN denied the claim thus prompting the consignee to file
an insurance claim with PHILAMGEN which paid its claim of P755,250.00. Claiming its right of subrogation,
PHILAMGEN sought recourse against respondent FELMA which disclaimed any liability for the loss.
Consequently, on 29 November 1983 PHILAMGEN sued the shipowner for sum of money and damages,
alleging that the total loss of cargo was due to the vessel’s unseaworthiness as she was put to sea in an unstable
condition. FELMAN, on the other hand, filed a motion to dismiss contending that there was no right of
subrogation in favor of PHILAMGEN since it had abandoned all its rights, interests and ownership over the
vessel together with her freight and appurtenances for the purpose of limiting and extinguishing its liability
under Art. 587 of the Code of Commerce.

Issue:

Is the concept of limited liability under Article 587 of the Code of Commerce should apply?

Ruling:

It is generally held that in every marine insurance policy the assured impliedly warrants to
the assurer that the vessel is seaworthy and such warranty is as much a term of the contract as if

31 7
Su b ro ga t io n : R ig ht of S ub r oga ti o n

expressly written on the face of the policy. Thus Sec. 113 of the Insurance Code provides that in every
marine insurance upon a ship or freight, or freightage, or upon anything which is the subject of
marine insurance, a warranty is implied that the ship is seaworthy. Under Sec. 114, a ship is
seaworthy when reasonably fit to perform the service, and to
encounter the ordinary perils of the voyage, contemplated by the parties to the policy. Thus it
becomes the obligation of the cargo owner to look for a reliable common carrier which keeps
its vessels in seaworthy condition. He may have no control over the vessel but he has full control in
the selection of the common carrier that will transport his goods. He also has full discretion in the
choice of assurer that will underwrite a particular venture.

The is no need to belabor the alleged breach of warranty of seaworthiness by the assured as
painstakingly pointed out by FELMAN to stress that subrogation will not work in this case. In policies
where the law will generally imply a warranty of seaworthiness, it can only be excluded by terms in
writing in the policy in the clearest language. And where the policy stipulates that the seaworthiness
of the vessel as between the assured and the assurer is admitted, the question of seaworthiness
cannot be raised by the assurer without showing concealment or misrepresentation by the assured.

The result of the admission of seaworthiness by the assurer PHILAMGEN may mean one or
two things: (a) that the warranty of the seaworthiness is to be taken as fulfilled; or, (b) that the risk
of unseaworthiness is assumed by the insurance company. The insertion of such waiver clauses in
cargo policies is in recognition of the realistic fact that cargo owners cannot control the state of the
vessel. Thus it can be said that with such categorical waiver, PHILAMGEN has accepted the risk of
unseaworthiness so that if the ship should sink by unseaworthiness, as what occurred in this case,
PHILAMGEN is liable.

Payment by the assurer to the assured operates as an equitable assignment to the assurer of
all the remedies which the assured may have against the third party whose
negligence or wrongful act caused the loss. The right of subrogation is not dependent upon, nor does
it grow out of any privity of contract or upon payment by the insurance company of the insurance
claim. It accrues simply upon payment by the insurance company of the insurance claim.

The doctrine of subrogation has its roots in equity. It is designed to promote and to
accomplish justice and is the mode which equity adopts to compel the ultimate payment of a debt by
one who in justice, equity and good conscience ought to pay. Therefore, the payment made by
PHILAMGEN to Coca-Cola Bottlers Philippines, Inc., gave the former the right to bring an action as
subrogee against FELMAN. Having failed to rebut the presumption of fault, the liability of FELMAN
for the loss of the 7,500 cases of 1-liter Coca-Cola softdrink bottles is inevitable.

31 8
Su b ro ga t io n : R ig ht of S ub r oga ti o n

WALLEM PHILIPPINES SHIPPING INC. and SEACOAST MARITIME CORPORATION v.


PRUDENTIAL GUARANTEE & ASSURANCE INC. and COURT OF APPEAL
G.R. No. 152158
February 7, 2003

Mendoza, J.

Facts:

Prudential Guarantee & Assurance Inc. brought an action for damages and attorney’s fees against
Wallem Philippines Shipping, Inc. and Seacoast Maritime Corporation (Seacoast) where Prudential sought the
recovery of the sum of ₱995,677.00, representing the amount it had paid to its insured, General Milling
Corporation, for alleged shortage incurred in the shipment of "Indian Toasted Soyabean Extraction Meal,
Yellow,". In its answer, Wallem denied liability for damage or loss to the shipment. To prove its claim for
indemnity, Prudential presented two witnesses, namely Josephine Suarez and Alfredo Cunanan. Josephine
Suarez, the claims processor of Prudential, testified that she received a claim from GMC in connection with its
shipment which arrived on board M/V Gao Yang. Upon receipt of the claim and its supporting papers, she
referred the same to Tan-Gatue Adjustment Company, Inc., which submitted a report. Upon her
recommendation, Prudential paid GMC the sum of ₱995, 677.09, as evidenced by receipts and a voucher. GMC
then issued a subrogation receipt to Prudential, which in turn sent a demand letter to Wallem. The trial court
ruled that private respondent Prudential failed to prove by clear, convincing, and competent evidence that
there was a shortage in the shipment.

Issue:

Is Wallem liable because Prudential subrogated to the rights of General Milling Corporation?

Ruling:

No. Wallem cannot be held liable because of the failure of Prudential to present the contract of
insurance or a copy thereof. Prudential claims that it is subrogated to the rights of GMC pursuant to their
insurance contract. For this purpose, it submitted a subrogation receipt and a marine cargo risk note. However,
this is not sufficient. As GMC’s subrogee, Prudential can exercise only those rights granted to GMC under the
insurance contract. The contract of insurance must be presented in evidence to indicate the extent of its
coverage. The insurance contract has not been presented. The subrogation receipt may nevertheless be used
to establish the relationship between the petitioner and the consignee and the amount paid to settle the claim.
However, the best evidence of the insurance contract was its original copy, which was presumably in the
possession of Wallem itself. Failure to present this original (or even a copy of it), for reasons the Court cannot
comprehend, must prove fatal to this petition.

31 9

You might also like